You are on page 1of 160

‫ م‬8:35 2021/‫‏‬9/‫‏‬11 https://t.

me/USMLEWorldStep2CK UWorld STEP2 SIM Form 1

1 Item 1 of 40 ABC 0.25

2
Mark
Question Id: 6840 Full Screen Tutorial Lab Values Notes Calculator Reverse Color Text Zoom Settings
Previous Next
3

4 A 57-year-old previously healthy man has a positive fecal occult blood test result and mild iron deficiency anemia. Colonoscopy reveals a mass in
5 the ascending colon. Histopathology is consistent with adenocarcinoma. Imaging studies reveal no metastatic disease. The patient undergoes
6 right hemicolectomy with ileocolonic anastomosis. The postoperative course is complicated by prolonged ileus, but recovery of bowel function is
7 expected in 7-10 days. Parenteral nutrition is started through a central venous catheter. BMI is 26 kg/m2. This patient is at greatest risk of
8 developing which of the following complications of nutritional therapy?
9
 A. Bloodstream infection (30%)
10

11 B. Cholelithiasis (42%)
12
C. Fluid overload (3%)
13

14 D. Hypoglycemic episodes (3%)


15
E. Refeeding syndrome (19%)
16

17

18

19 30%
20
Correct  Answered correctly  01 min, 02 secs
Time Spent  08/15/2021
Last Updated

21

22

23 Explanation
24

25
Parenteral nutrition (PN) is a means of providing total nutrition, including carbohydrates, amino acids, lipids, electrolytes, vitamins, and minerals,

26
to patients who are unable to receive enteral nutrition. Due to the high osmotic load of PN, it must be administered through a central venous
catheter when given for >48 hours. The greatest risk related to PN is central line-associated bloodstream infection (CLABSI).
27

28 Factors that increase the risk of CLABSI in patients receiving PN include poor patient hygiene, severity of patient illness, and duration of PN
29 therapy. Common bacterial organisms that cause CLABSI in patients receiving PN include coagulase-negative staphylococci and
30 Staphylococcus aureus as well as gram-negative organisms (eg, Klebsiella pneumoniae, Pseudomonas aeruginosa). Fungal CLABSI due to
31 Candida species is also common.
32
(Choice B) Cholestasis is common in patients receiving PN and may increase the risk of cholelithiasis. However, cholestasis is typically not seen
33
until patients have been on PN for >2 weeks. This patient with an expected 7-10 day course of PN therapy is at very low risk for cholelithiasis.
34
(Choice C) PN requires the administration of a significant volume of fluid, so fluid overload is a potential complication. However, accurate
35
recording of volume intake and output can usually prevent this complication.
36

37 (Choice D) Hypoglycemic episodes are rare in patients receiving PN. Hyperglycemic episodes are more common. Patients receiving PN are
38 approximately twice as likely to experience hyperglycemia as those receiving the same nutritional intake through the enteral route.
39
(Choice E) Refeeding syndrome is a serious complication that can result from rapid electrolyte shifts when nutrition is reintroduced to
40
malnourished patients. This patient is not significantly malnourished and is at low risk for refeeding syndrome.

Educational objective:
Parenteral nutrition can provide total nutrition directly into the bloodstream of patients who are unable to receive enteral nutrition. Bloodstream
infection (bacterial or fungal) is a common and serious complication.

References
Epidemiology of bloodstream infection associated with parenteral nutrition.

(http://www.ncbi.nlm.nih.gov/pubmed/19084152)

Surgery Gastrointestinal & Nutrition Total parenteral nutrition


Subject System Topic

https://t.me/USMLEWorldStep2CK

REVIEW 0
Feedback End Block

https://www.uworld.com/ClientApp/v15/apps/qbanktestinterface/index.html#/launchtest/7281878/nbme/229400736/3/1 1/1
‫ م‬8:36 2021/‫‏‬9/‫‏‬11 https://t.me/USMLEWorldStep2CK UWorld STEP2 SIM Form 1

1 Item 2 of 40 ABC 0.25

2
Mark
Question Id: 6850 Full Screen Tutorial Lab Values Notes Calculator Reverse Color Text Zoom Settings
Previous Next
3

4 A 32-year-old man is brought to the emergency department due to progressive confusion and lethargy over the past several hours. The patient
5 has no significant medical history but he has been depressed since a recent breakup with his girlfriend. He works at an automotive repair shop,
6 and one of his coworkers reported seeing the patient consume antifreeze prior to symptom onset. Blood pressure is 110/66 mm Hg, pulse is
7 110/min, and respirations are 24/min. The patient is somnolent and responds only to painful stimuli. Pupils are equal and reactive bilaterally.
8 Funduscopy is normal. Lung fields are clear to auscultation and heart sounds are normal. Bilateral costovertebral angles are tender to
9 percussion. Bladder catheterization yields a small amount of bloody urine. Which of the following is most likely to be seen on this patient's arterial
10 blood gas analysis?
11

12 pH pO2 pCO2 HCO3


13
 A. 7.25 95 53 27 (4%)
14

15  B. 7.28 98 24 15 (91%)
16 C. 7.42 97 41 23 (1%)
17
D. 7.51 110 28 18 (2%)
18

19 E. 7.52 110 47 32 (1%)


20

21

22
Incorrect 91%
 12 secs 07/18/2021
23 Correct answer  Answered correctly Time Spent  Last Updated
24 B
25

26
Explanation
27

28

29 Toxic alcohols
30
Toxicity Clinical features Laboratory results
31

32 Slurred speech High osmolar gap


Alcohol
33 Unsteady gait Increased anion gap metabolic
ketoacidosis
34 Altered mentation acidosis (ketosis)
35
Visual blurring, central
36
High osmolar gap
Methanol scotomata
37
Increased anion gap metabolic
ingestion Afferent pupillary defect
acidosis
38 Altered mentation
39
High osmolar gap
40
Flank pain Increased anion gap metabolic
Ethylene glycol
Hematuria, oliguria acidosis
ingestion
Cranial nerve palsies, tetany Calcium oxalate crystals in
urine

Isopropyl CNS depression High osmolar gap


alcohol Disconjugate gaze No increased anion gap or
ingestion Absent ciliary reflex metabolic acidosis

CNS = central nervous system.

This patient's acute toxicity following ingestion of automotive antifreeze raises concern for possible ethylene glycol poisoning. Ethylene glycol
is often ingested intentionally as an ethanol substitute or as a mode of attempted suicide. Following ingestion, ethylene glycol causes inebriation
and sedation, similar to ethyl alcohol. Metabolism via alcohol and aldehyde dehydrogenases produces glycolic acid, which is subsequently
converted to oxalic acid. Oxalic acid can bind to calcium, leading to hypocalcemia and precipitation of calcium oxalate crystals in the urine.

Renal toxicity from ethylene glycol is characterized by acute kidney injury (due to direct nephrotoxicity of glycolate and obstruction of tubules by
calcium oxalate crystals) with flank pain and hematuria (as noted in this patient) as well as oliguria. Patients typically have severe metabolic
acidosis with an increased anion gap, very low bicarbonate level, and compensatory drop in pCO2. The osmolal gap (measured plasma
osmolality – calculated plasma osmolality, which is often estimated by [(2 x plasma Na) + (glucose/18) + (blood urea nitrogen/2.8)]) is typically
elevated as well.

(Choice A) A low pH with an elevated arterial pCO2 is consistent with respiratory acidosis, which may be seen in hypoventilation and
emphysema.

(Choice C) This blood gas analysis is normal, which would be unlikely with ingestion of ethylene glycol.

(Choice D) Hyperventilation causes a decrease in pCO2 and an increase in the pH (respiratory alkalosis).

(Choice E) Metabolic alkalosis is defined by an increase in HCO3 and an increase in the arterial pH. Common causes include vomiting and
diuretic use.

Educational objective:
Ethylene glycol toxicity causes mental status changes associated with precipitation of calcium oxalate crystals in the urine, acute kidney injury, and
severe anion gap metabolic acidosis.

References
Challenges in the diagnosis of ethylene glycol poisoning.

(http://www.ncbi.nlm.nih.gov/pubmed/24215789)

Medicine Endocrine, Diabetes & Metabolism Toxic alcohols


Subject System Topic

https://t.me/USMLEWorldStep2CK

REVIEW 0
Feedback End Block

https://www.uworld.com/ClientApp/v15/apps/qbanktestinterface/index.html#/launchtest/7281878/nbme/229400736/3/1 1/1
‫ م‬8:36 2021/‫‏‬9/‫‏‬11 https://t.me/USMLEWorldStep2CK UWorld STEP2 SIM Form 1

1 Item 3 of 40 ABC 0.25

2
Mark
Question Id: 6851 Full Screen Tutorial Lab Values Notes Calculator Reverse Color Text Zoom Settings
Previous Next
3

4 A 32-year-old primigravida at 36 weeks gestation comes to the emergency department due to irregular contractions for the past 3 hours. The
5 contractions began after the patient went for a walk this afternoon and have not stopped even after resting. They are not painful and occur every
6 7-15 minutes. She has had no vaginal bleeding or leakage of fluid. Fetal movement is normal. The patient's pregnancy has been uncomplicated.
7 Group B Streptococcus screening earlier this week was positive. Blood pressure is 124/78 mm Hg and pulse is 80/min. BMI is 28 kg/m2. The
8 abdomen is soft, nontender, and has no palpable contractions. Fundal height is at 36 cm. The cervix is closed on digital cervical examination.
9 Nonstress test monitoring is shown in the exhibit. Which of the following is the best next step in management of this patient?
10
A. Biophysical profile (18%)
11

12 B. Contraction stress test (3%)


13
 C. Reassurance only (72%)
14

15 D. Scalp stimulation (0%)


16
E. Umbilical artery Doppler (3%)
17

18  F. Vibroacoustic stimulation (1%)

19

20

21 Incorrect 72%
 04 secs 04/25/2021
22 Correct answer  Answered correctly Time Spent  Last Updated
C
23

24

25
Explanation
26

27 Antepartum fetal surveillance is performed to assess fetal status and identify those at risk for hypoxemia and subsequent fetal demise.
28 Testing occurs in patients with maternal or fetal comorbidities who have an increased risk of fetal complications (eg, hypertension, diabetes
29 mellitus, fetal growth restriction) and in those with acute conditions, such as this patient with preterm contractions (ie, irregular uterine contractions
30 and no cervical change).
31
A common antepartum fetal surveillance method is the nonstress test (NST), which evaluates the fetal heart rate (FHR) pattern. The FHR
32
pattern varies based on the autonomic response to various stimuli, including oxygenation status, blood pressure, and CNS abnormalities. A
33
reactive NST—a NST in which, during a 20-minute interval, there are ≥2 FHR accelerations that peak at ≥15 beats per minute above baseline
34 and last ≥15 seconds—indicates adequate fetal oxygenation and uteroplacental blood flow. Therefore, patients (such as this one) who have a
35 reactive NST require no additional fetal monitoring and can continue routine prenatal care.
36
(Choices A and B) A biophysical profile or contraction stress test may be used in further evaluation of patients with an uncomplicated pregnancy
37
and nonreactive NST to determine the risk of fetal hypoxemia and demise. Because patients with a reactive NST have a high negative predictive
38
value for fetal compromise, no additional testing is required.
39

40 (Choices D and F) Fetal scalp stimulation and vibroacoustic stimulation are used in patients with nonreactive NSTs (ie, no accelerations) to
provoke fetal movement, which helps determine whether the nonreactivity is due to a physiologic (eg, fetal sleep cycle) or pathophysiologic (eg,
acidosis) cause.

(Choice E) An umbilical artery Doppler ultrasound is performed in growth-restricted fetuses (<10% estimated fetal weight) to evaluate for
placental insufficiency and progressive fetal hypoxemia. It is not used in patients with an uncomplicated pregnancy and reactive NST.

Educational objective:
A nonstress test (NST) is reactive if, in 20 minutes, there are ≥2 heart rate accelerations that peak at ≥15 beats per minute above the baseline and
last ≥15 seconds. A reactive NST indicates adequate fetal oxygenation and requires no additional fetal monitoring.

References
Antenatal fetal assessment: contraction stress test, nonstress test, vibroacoustic stimulation, amniotic fluid volume, biophysical profile, and
modified biophysical profile—an overview.

(http://www.ncbi.nlm.nih.gov/pubmed/18652922)

Obstetrics & Gynecology Pregnancy, Childbirth & Puerperium Fetal heart tracing
Subject System Topic

https://t.me/USMLEWorldStep2CK

REVIEW 0
Feedback End Block

https://www.uworld.com/ClientApp/v15/apps/qbanktestinterface/index.html#/launchtest/7281878/nbme/229400736/3/1 1/1
‫ م‬8:36 2021/‫‏‬9/‫‏‬11 https://t.me/USMLEWorldStep2CK UWorld STEP2 SIM Form 1

1 Item 4 of 40 ABC 0.25

2
Mark
Question Id: 6855 Full Screen Tutorial Lab Values Notes Calculator Reverse Color Text Zoom Settings
Previous Next
3

4 A 43-year-old man comes to the emergency department due to 2 days of fevers and a lesion on his right foot. The initial lesion was erythematous
5 and edematous, but it quickly developed into a bulla surrounded by erythema. Eventually, the bulla ruptured, leaving a painless ulcer with a black
6 center. The patient has a history of non-Hodgkin lymphoma and recently underwent chemotherapy. Which of the following is the best treatment
7 for this patient’s foot lesion?
8

9
A. Salvage chemotherapy (2%)

10 B. Antiviral therapy (3%)


11
C. Corticosteroids (6%)
12

13 D. Antistaphylococcal antibiotic (33%)


14
 E. Antipseudomonal antibiotic (54%)
15

16

17

18 54% 02 secs 07/30/2021


19
Correct  Answered correctly  Time Spent  Last Updated

20

21

22 Explanation
23

24 Given that this patient just recently received chemotherapy for the treatment of non-Hodgkin lymphoma, he is in an immunocompromised state
25
and is therefore very susceptible to bacterial infections, especially with gram-negative organisms such as Pseudomonas aeruginosa.

26 The lesion described on his foot is characteristic of ecthyma gangrenosum, which is commonly associated with P aeruginosa bacteremia.
27 Typically, the lesions of ecthyma gangrenosum progress very rapidly from a small erythematous macule to larger, nontender nodules with
28 necrosis. Ecthyma gangrenosum is caused by bacterial invasion through the media and adventitia of blood vessels. As a result, appropriate
29 treatment consists of antibiotic therapy that covers P aeruginosa.
30

31 (Choice A) This patient’s foot lesion is not consistent with recurrence of his non-Hodgkin lymphoma, but rather a consequence of his
32 immunocompromised state. Therefore, salvage chemotherapy would not be beneficial.
33

34 (Choice B) Because ecthyma gangrenosum results from P aeruginosa bacteremia, antipseudomonal antibiotics, not antiviral medications, are the
35
appropriate treatment.

36
(Choice C) This patient is already in an immunocompromised state, and corticosteroids would likely worsen his current condition.
37

38
(Choice D) The presence of ecthyma gangrenosum usually signifies the presence of P aeruginosa bacteremia as opposed to infection with
39
staphylococcal organisms.
40

Educational objective:
Ecthyma gangrenosum is a disorder associated with Pseudomonas aeruginosa bacteremia and is characterized by rapidly progressive skin
lesions that develop into nontender nodules with central necrosis. It can be seen in patients who are immunocompromised, and treatment should
consist of antipseudomonal antibiotics.

Medicine Rheumatology/Orthopedics & Sports Pseudomonas


Subject System Topic

https://t.me/USMLEWorldStep2CK

REVIEW 0
Feedback End Block

https://www.uworld.com/ClientApp/v15/apps/qbanktestinterface/index.html#/launchtest/7281878/nbme/229400736/3/1 1/1
‫ م‬8:37 2021/‫‏‬9/‫‏‬11 https://t.me/USMLEWorldStep2CK UWorld STEP2 SIM Form 1

1 Item 5 of 40 ABC 0.25

2
Mark
Question Id: 6857 Full Screen Tutorial Lab Values Notes Calculator Reverse Color Text Zoom Settings
Previous Next
3

4 A 68-year-old man comes to the emergency department 2 hours after a syncopal episode at home. The patient woke up during the night to use
5 the restroom but felt weak and dizzy while walking to it. According to his wife, he passed out just as he reached the restroom and was
6 unconscious for about a minute. The patient states he wakes up 1-2 times every night to urinate, for which he recently began taking a new
7 medication. He has had no prior syncope, and medical history includes hypertension and gout. Supine blood pressure is 126/72 mm Hg and
8 pulse is 82/min. Physical examination shows brisk carotid pulses with no bruit. Jugular venous pulsations are present 2 cm above the sternal
9 angle. Cardiac impulse is sustained but nondisplaced, and there is an S4. No murmurs are present. Lungs are clear to auscultation. Extremities
10 are without edema and pulses are normal. ECG reveals sinus rhythm with findings of left ventricular hypertrophy. Which of the following is the
11 most likely cause of this patient's syncope?
12
A. Decreased circulating blood volume (12%)
13

14 B. Degeneration of autonomic nerve fibers (6%)


15
C. Hypersensitivity of carotid sinus (4%)
16

17  D. Micturition triggered vagal overactivity (4%)


18 E. Peripheral vasodilation (67%)

19
F. Transient cardiac arrhythmia (4%)
20

21

22

23 Incorrect 67%
24 Correct answer  Answered correctly  02 secs
Time Spent  04/20/2021
Last Updated
E
25

26

27 Explanation
28

29

30
Medication-induced orthostasis
31 Mechanism Examples
32
Terazosin, prazosin, doxazosin
33
α-1 blockade–mediated vasodilation Antipsychotics (eg, risperidone)
34
Antihistamines, TCAs
35

36 ACE inhibitors & ARBs


37 Dihydropyridine CCBs
Other vasodilation
38 Hydralazine, nitrates
39
Phosphodiesterase inhibitors

40 Diuretics
Volume depletion
SGLT-2 inhibitors

Beta blockers
Sympathetic blockade
Clonidine

ARBs = angiotensin II receptor blockers; CCBs = calcium channel blockers; SGLT = sodium-glucose cotransporter; TCAs = tricyclic antidepressants.

This patient who lost consciousness shortly after getting up from bed most likely experienced orthostatic syncope, which usually results from
disruption of one or more components of the baroreceptor response. Normally, standing from a seated or supine position causes a rapid drop
in blood pressure that triggers an increase in sympathetic drive, causing vasoconstriction to increase peripheral resistance and increased heart
rate and contractility to aid cardiac output. These changes facilitate a corrective increase in blood pressure to prevent a drop in cerebral perfusion
and its associated symptoms (eg, lightheadedness, syncope).

Common disruptions in the baroreceptor response that can lead to orthostatic syncope include:

Medications can impair vasoconstriction; these include agents that block the sympathetic vasoconstrictive response directly (ie, alpha-1
receptor blockers) and, less commonly, antihypertensive agents that cause vasodilation via other mechanisms (eg, ACE inhibitors,
hydralazine). This patient with frequent nocturnal urination likely recently started an alpha-1 receptor blocker (eg, terazosin) for benign
prostatic hyperplasia, a common scenario leading to orthostatic symptoms.

Some disease processes (eg, Parkinson disease, diabetes mellitus) can cause degeneration of autonomic nerve fibers and impair the
sympathetic baroreceptor response. Such a factor is unlikely in this patient as autonomic damage is typically a late effect and rarely a
presenting disease manifestation (Choice B).

Hypovolemia can prevent an appropriate baroreceptor-mediated increase in blood pressure. It is an unlikely contributor in this patient with
normal jugular venous pressure (Choice A).

(Choice C) Carotid sinus hypersensitivity can cause syncope triggered by tactile stimulus of the carotid sinus. The syncope is not orthostatic in
nature and usually occurs during activities such as shaving or putting on a collared shirt.

(Choice D) Micturition syncope is a form of vasovagal syncope triggered by the increase in vagal tone that occurs with urination; it is unlikely in
this patient as his symptoms started before he began urinating.

(Choice F) Paroxysmal ventricular tachycardia is a potential cause of syncope in patients with ischemic heart disease or severe systolic
dysfunction (concentric left ventricular hypertrophy, evidenced by an S4, is not a significant risk factor). The syncope is usually sudden and rarely
preceded by warning symptoms (eg, weakness, dizziness).

Educational objective:
Orthostatic syncope usually involves a disruption of the baroreceptor response, including medication impairment of vasoconstriction, disease-
induced degeneration of autonomic nerve fibers, or hypovolemia. Recent initiation of an alpha-1 receptor blocker (eg, terazosin) for benign
prostatic hyperplasia is a common scenario that leads to orthostatic symptoms.

References
Orthostatic syncope.

(http://www.ncbi.nlm.nih.gov/pubmed/30725970)

Medicine Cardiovascular System Syncope


Subject System Topic

REVIEW 0
Feedback End Block

https://www.uworld.com/ClientApp/v15/apps/qbanktestinterface/index.html#/launchtest/7281878/nbme/229400736/3/1 1/1
‫ م‬8:37 2021/‫‏‬9/‫‏‬11 https://t.me/USMLEWorldStep2CK UWorld STEP2 SIM Form 1

1 Item 6 of 40 ABC 0.25

2
Mark
Question Id: 6862 Full Screen Tutorial Lab Values Notes Calculator Reverse Color Text Zoom Settings
Previous Next
3

4 A 37-year-old Caucasian male presents to the ER with severe weakness and dizziness. He has had two episodes of syncope over the last eight
5 hours. For the past week, he has been having vague mid-chest discomfort and left-sided neck pain. He returned from a hiking trip to Connecticut
6 two weeks ago, where he had a sore throat and dry cough. He does not recall any tick bites. His father died of a heart attack at 54 years old. On
7 physical exam, his lungs are clear to auscultation bilaterally. He has thready pulses over both radial arteries that disappear with deep inspiration.
8 Which of the following is the most likely diagnosis?
9
A. Acute myocardial infarction (0%)
10

11 B. Hypertrophic cardiomyopathy (4%)


12
 C. Cardiac tamponade (35%)
13

14 D. Acute myocarditis (42%)


15
E. Subacute bacterial endocarditis (5%)
16

17  F. Aortic disease (10%)

18

19

20 Incorrect 35%
21 Correct answer  Answered correctly  02 secs
Time Spent  07/30/2021
Last Updated
C
22

23

24
Explanation
25

26 The patient described has most likely developed cardiac tamponade as a result of viral pericarditis given his recent history of sore throat and dry
27 cough. Pericarditis can cause retrosternal chest pain that radiates to the left arm and shoulder, and may account for the symptoms that occurred
28 the week before the patient presented to the hospital. His current symptoms of weakness, dizziness, and syncope are best explained by the
29 severely depressed cardiac output that occurs in patients with cardiac tamponade.
30

31 Cardiac tamponade produces a decrease in the cardiac output because the pressure exerted on the heart by the pericardial fluid is greater than
32 the venous pressure that fills the right atrium during diastole. Cardiac filling in this setting can only occur during deep inhalation when the negative
33 intrathoracic pressure allows for a pressure difference between the vena cava and the right atrium. Pulsus paradoxus is a typical feature of
34 pericardial tamponade, and is defined as a large decrease in the systolic blood pressure on inspiration. It is demonstrated in this case by the loss
35 of a palpable radial pulse during inspiration.
36

37
(Choice A) Acute myocardial infarction can cause chest pain that radiates to the left shoulder and neck, but pulsus paradoxus is not a common
finding.
38

39
(Choice B) Hypertrophic cardiomyopathy may present with dyspnea, chest pain, and syncope. Patients are at an increased risk of arrhythmias
40
and sudden cardiac death, which typically affects young patients following strenuous exercise.

(Choice D) Acute myocarditis can present with chest pain and signs of cardiogenic shock, but it is not associated with pulsus paradoxus. The
most common cause of infectious myocarditis and pericarditis is coxsackievirus.

(Choice E) Subacute bacterial endocarditis typically occurs in patients with a preexisting valvular defect. It presents with fevers, a new or
worsening heart murmur, and other associated findings, such as splinter hemorrhages, Osler nodes, and Janeway lesions.

(Choice F) Aortic disease would present with a murmur of aortic stenosis or regurgitation, and may lead to pulmonary edema.

Educational objective:
Patients with chest pain, signs of decreased cardiac output, and pulsus paradoxus following a viral infection most likely have cardiac tamponade
resulting from acute pericarditis. In this setting, right atrial filling is impaired, which results in an increase in the systemic venous pressure.
However, the lungs remain clear to auscultation because there is no backup of blood into the pulmonary circulation.

Medicine Cardiovascular System Cardiac tamponade


Subject System Topic

https://t.me/USMLEWorldStep2CK

REVIEW 0
Feedback End Block

https://www.uworld.com/ClientApp/v15/apps/qbanktestinterface/index.html#/launchtest/7281878/nbme/229400736/3/1 1/1
‫ م‬8:37 2021/‫‏‬9/‫‏‬11 https://t.me/USMLEWorldStep2CK UWorld STEP2 SIM Form 1

1 Item 7 of 40 ABC 0.25

2
Mark
Question Id: 6865 Full Screen Tutorial Lab Values Notes Calculator Reverse Color Text Zoom Settings
Previous Next
3

4 A 36-year-old woman is brought to the emergency department by her husband due to 2 days of confusion and agitation. He reports that last night
5 she did not sleep at all and was constantly wandering around the house. He also noticed twitching of her right arm this morning. On physical
6 examination, temperature is 38.1 C (100.7 F), blood pressure is 120/80 mm Hg, pulse is 90/min, and respirations are 16/min. She is not oriented
7 to time or place. Her neck is supple. There are no skin or joint abnormalities. Electroencephalogram shows prominent high-amplitude slow
8 waves over the left temporal and frontal lobes. Which of the following patterns is most likely on cerebrospinal fluid analysis?
9

10 WBC count %neutrophils Glucose, mg/dL RBC count Protein, mg/dL


11
 A. 125 5 65 700 90 (39%)
12

13 B. 1,100 80 12 200 300 (8%)


14
C. 70 50 160 5 70 (24%)
15

16
D. 120 10 7 100 380 (14%)

17 E. 5 60 80 3 128 (12%)

18

19

20
Incorrect 39%
21

22
Correct answer  Answered correctly  02 secs
Time Spent  07/30/2021
Last Updated
A
23

24
Explanation
25

26
Herpes simplex virus (HSV) encephalitis is a common and severe form of viral encephalitis. Patients present with fever, headache, seizures,
27
confusion and stupor that occurs over the course of a few days. CT, MR imaging, and electroencephalography typically demonstrate
28
abnormalities in the frontotemporal region of the brain. Patients may also demonstrate symptoms of temporal lobe pathology such as temporal-
29
based seizures, anosmia, gustatory hallucinations, and bizarre or psychotic behavior.
30

31
Cerebrospinal fluid (CSF) findings in HSV encephalitis are typically nonspecific; however, classic findings are lymphocytic pleocytosis, elevated
32
protein, elevated red blood cell (RBC) count, and normal glucose. The CSF RBC elevation is the result of hemorrhagic destruction of the
33 frontotemporal lobes.
34

35 (Choice B) This pattern of low glucose, elevated protein, and neutrophilic pleocytosis is most consistent with acute bacterial meningitis.
36

37 (Choice D) This pattern of markedly low glucose, elevated protein, and lymphocytic pleocytosis is most consistent with tuberculous meningitis.
38

39 Educational objective:
40 The characteristic cerebrospinal fluid findings in herpes simplex virus encephalitis are lymphocytic pleocytosis, an increased number of
erythrocytes, and elevated protein.

Medicine Nervous System HSV infection


Subject System Topic

https://t.me/USMLEWorldStep2CK

REVIEW 0
Feedback End Block

https://www.uworld.com/ClientApp/v15/apps/qbanktestinterface/index.html#/launchtest/7281878/nbme/229400736/3/1 1/1
‫ م‬8:37 2021/‫‏‬9/‫‏‬11 https://t.me/USMLEWorldStep2CK UWorld STEP2 SIM Form 1

1 Item 8 of 40 ABC 0.25

2
Mark
Question Id: 6869 Full Screen Tutorial Lab Values Notes Calculator Reverse Color Text Zoom Settings
Previous Next
3

4 A 56-year-old man comes to the office due to progressive dyspnea on exertion for 6 months. In the past, he was able to walk several blocks to
5 work, but now he becomes breathless even with daily activities such as shopping for groceries or walking his dog. He also has a nonproductive
6 cough but no fever, chest pain, or orthopnea. The patient has a history of hypertension and his brother died of liver cirrhosis. He has smoked a
7 pack of cigarettes daily for 30 years but is trying to quit. He does not use alcohol or illicit drugs. The patient used to teach in a primary school but
8 stopped 2 months ago due to shortness of breath. Blood pressure is 130/80 mm Hg, pulse is 82/min and regular, and respirations are 18/min.
9 Pulse oximetry shows 92% on room air. BMI is 34 kg/m2. Estimated jugular venous pressure is normal. Lung auscultation reveals bibasilar fine
10 crackles, and heart sounds are normal with no murmurs. There is no lower extremity edema or skin rash. ECG shows no abnormalities.
11 Spirometry findings are as follows.
12
FEV1 65% of predicted
13

14
FVC 58% of predicted

15 FEV1/FVC 85%
16

17 Which of the following is the most likely pathophysiology of this patient's condition?
18

19  A. Alveolar hypoventilation (8%)


20
B. Chronic airway inflammation (7%)
21

22
C. Destruction and dilation of the terminal airways (18%)

23 D. Diffuse, reversible bronchoconstriction (0%)


24
E. Increased pulmonary capillary wedge pressure (4%)
25

26  F. Lung stiffening due to fibrosis (60%)


27

28

29
Incorrect 60%
30 Correct answer  Answered correctly  03 secs
Time Spent  08/21/2021
Last Updated
31 F
32

33
Explanation
34

35
This patient likely has interstitial lung disease (ILD), which is characterized by pulmonary fibrosis that leads to stiffening of the lungs and
36
decreased lung compliance. Progressive dyspnea and nonproductive cough are the most common presenting symptoms. Auscultation typically
37
reveals fine bibasilar "Velcro-like" crackles, and physical examination in advanced disease may show digital clubbing and signs of cor pulmonale
38
(eg, lower extremity edema, hepatomegaly). Chest x-ray reveals reticular or nodular opacities, and spirometry demonstrates restrictive physiology
39 with decreased (<80% of predicted) FEV1, FVC, and total lung capacity. As both FEV1 and FVC are decreased somewhat proportionally, the
40 FEV1/FVC ratio remains normal or may be increased.

The causes of ILD include chronic dust exposure, drug toxicity, and connective tissue disease (eg, sarcoidosis, rheumatoid arthritis). Idiopathic
disease is common and typically occurs in patients age >50, but ILD due to systemic disease can occur in younger patients. More than half of
patients with ILD have a significant smoking history.

(Choice A) Alveolar hypoventilation is the mechanism of hypoxemia and dyspnea in patients with extrinsic restrictive lung disease (eg, obesity
hypoventilation syndrome, neuromuscular weakness). Spirometry findings are similar to those of ILD, but the diffusion capacity is normal (in
contrast to a low diffusion capacity in ILD). Cough and bibasilar crackles are typically absent.

(Choices B and C) Chronic inflammation of the conducting airways and hyperexpansion of the lungs due to alveolar wall destruction characterize
the chronic bronchitis and emphysematous components of chronic obstructive pulmonary disease, respectively. Spirometry findings demonstrate
an obstructive pattern with decreased FEV1, normal FVC, and decreased FEV1/FVC ratio.

(Choice D) Reversible bronchoconstriction characterizes asthma. In the setting of active symptoms, spirometry testing shows obstruction
(decreased FEV1 and FEV1/FVC ratio) that responds to bronchodilators.

(Choice E) Increased pulmonary capillary wedge pressure (reflecting elevated left atrial pressures) occurs in congestive heart failure. In addition
to dyspnea and cough, patients typically have orthopnea, lower extremity swelling, and elevated jugular venous pressure, which are absent in this
patient. Bibasilar crackles are typically present but sound "wet" compared with the "Velcro-like" crackles of ILD.

Educational objective:
Interstitial lung disease is characterized by progressive pulmonary fibrosis that causes decreased lung compliance. Spirometry typically shows
decreased total lung capacity, FVC, and FEV1 and a normal FEV1/FVC ratio.

References
Interstitial lung diseases: an epidemiological overview.

(http://www.ncbi.nlm.nih.gov/pubmed/11816822)

Medicine Pulmonary & Critical Care Interstitial lung disease


Subject System Topic

https://t.me/USMLEWorldStep2CK

REVIEW 0
Feedback End Block

https://www.uworld.com/ClientApp/v15/apps/qbanktestinterface/index.html#/launchtest/7281878/nbme/229400736/3/1 1/1
‫ م‬8:38 2021/‫‏‬9/‫‏‬11 https://t.me/USMLEWorldStep2CK UWorld STEP2 SIM Form 1

1 Item 9 of 40 ABC 0.25

2
Mark
Question Id: 6870 Full Screen Tutorial Lab Values Notes Calculator Reverse Color Text Zoom Settings
Previous Next
3

4 A 70-year-old man comes to the office with pain in his lower extremities. It is most severe in the right posterior thigh, although both legs are
5 affected. He first noticed the pain 2 years ago. Initially, the patient would feel the pain after walking 6 or 7 holes of golf. However, he now has to
6 stop walking after 1 or 2 holes. The pain is much worse when the patient walks downhill but does not occur when he walks uphill. It improves with
7 rest, especially when he sits in a recliner with his legs up. Medical history is notable for diet-controlled type 2 diabetes mellitus, hypertension, and
8 hypercholesterolemia. Current medications include aspirin, lisinopril, amlodipine, and atorvastatin. The patient has a 25-pack-year smoking
9 history but quit 20 years ago. Blood pressure is 130/78 mm Hg, pulse is 78/min, and respirations are 16/min. BMI is 32 kg/m2. Lower extremity
10 pulses are full and symmetric, and there are no trophic changes of the hair or nails. Neurologic examination shows absent ankle reflexes and 2+
11 knee reflexes. Plantar reflexes are downgoing bilaterally. Tuning fork testing shows decreased vibration sensation at both ankles. Muscle
12
strength is normal bilaterally. Which of the following is most likely to establish the cause of this patient's pain?

13
A. Ankle brachial index (13%)
14

15 B. Bone densitometry (DEXA scan) (0%)


16
 C. MRI of the lumbosacral spine (79%)
17

18
D. Nerve conduction studies (5%)

19 E. X-rays of the hip joints (1%)



20

21

22
Incorrect 79%
23

24
Correct answer  Answered correctly  02 secs
Time Spent  04/23/2021
Last Updated
C
25

26

27
Explanation

28
This patient's symptoms are consistent with lumbar spinal stenosis, a common cause of back and lower extremity pain in patients age >60.
29
Spinal stenosis is most commonly caused by degenerative osteoarthritis of the spine (spondylosis), although other contributing factors may
30
include bulging of the intervertebral discs and hypertrophy of the ligamentum flavum. Narrowing of the lumbar spinal canal leads to mechanical
31
compression of the nerve roots, leading to pain in the back and lower extremities. Weakness and sensory symptoms can occur as well.
32

33 The classic symptom of spinal stenosis is neurogenic claudication (pseudoclaudication). Neurogenic claudication is characterized by lower
34 extremity pain with extension of the spine (eg, walking, prolonged standing). Spine flexion (eg, leaning forward, walking uphill) generally relieves
35
the pain. X-rays of the spine can identify degenerative changes suggesting the diagnosis, but MRI is more definitive. MRI can confirm narrowing

36
of the lumbar canal and compression of lumbar nerve roots, and is useful to guide surgical intervention in patients who fail conservative
management.
37

38 (Choice A) Ankle brachial index is used to evaluate suspected lower extremity peripheral arterial disease. This patient has risk factors for
39 atherosclerosis, but he has normal distal pulses with no trophic changes. Also, his symptoms are posture-dependent and exacerbated by spine
40 extension (eg, walking downhill), suggesting neurogenic rather than vascular claudication.

(Choice B) Osteoporosis (diagnosed by bone densitometry) can be complicated by spinal compression fracture. The back pain associated with
compression fractures typically starts following minor trauma (eg, ground-level fall) and decreases over time.

(Choice D) Nerve conduction studies are useful to diagnose peripheral neuropathy when the diagnosis is uncertain. This patient may have some
diabetic neuropathy (decreased vibration sensation at the ankles); however, this would not cause posture-dependent lower extremity pain.

(Choice E) Hip osteoarthritis can cause pain with activity, but it would be exacerbated whether walking uphill or downhill.

Educational objective:
Lumbar spinal stenosis is a common cause of back and lower extremity pain in patients age >60. It manifests as neurogenic claudication, which is
characterized by lower extremity pain with extension of the spine during walking or prolonged standing. MRI is used for definitive diagnosis.

References
Diagnosis of lumbar spinal stenosis: An updated systematic review of the accuracy of diagnostic tests.

(http://www.ncbi.nlm.nih.gov/pubmed/23385136)

Medicine Rheumatology/Orthopedics & Sports Spinal stenosis


Subject System Topic

https://t.me/USMLEWorldStep2CK

REVIEW 0
Feedback End Block

https://www.uworld.com/ClientApp/v15/apps/qbanktestinterface/index.html#/launchtest/7281878/nbme/229400736/3/1 1/1
‫ م‬8:38 2021/‫‏‬9/‫‏‬11 https://t.me/USMLEWorldStep2CK UWorld STEP2 SIM Form 1

1 Item 10 of 40 ABC 0.25

2
Mark
Question Id: 6873 Previous Next Full Screen Tutorial Lab Values Notes Calculator Reverse Color Text Zoom Settings
3
4 A 63-year-old man comes to the office due to fatigue, headaches, and easy bruising. His medical history is significant for chronic obstructive
5 pulmonary disease and stable coronary artery disease. The patient's blood pressure is 180/120 mm Hg and pulse is 80/min. Examination shows
6 increased pigmentation in the palmar creases and in a periumbilical scar from a hernia repair surgery. The patient also has scattered lower
7 extremity ecchymoses. Neuromuscular examination shows symmetric proximal muscle weakness. Laboratory results are as follows:
8
Complete blood count
9

10 Hemoglobin 14.5 g/dL


11
Leukocytes 6,000/mm3
12
Platelets 360,000/mm3
13
14
Serum chemistry
15
16 Sodium 143 mEq/L
17
Potassium 3.0 mEq/L
18
19 Chloride 98 mEq/L
20 Bicarbonate 29 mEq/L
21
Blood urea nitrogen 8 mg/dL
22

23 Creatinine 0.6 mg/dL


24
25 CT scan of the chest reveals a 3-cm mediastinal mass. Which of the following is most likely being overproduced by the mass in this patient?
26
A. Amino acid derivative (2%)
27
28 B. Arachidonic acid derivative (2%)
29
 C. Polypeptide hormone (48%)
30

31  D. Steroid hormone (45%)


32
E. Sterol vitamin (0%)
33

34
35

36 Incorrect 48%
 03
secs 06/25/2021
37
Correct answer
C
 Answered correctly Time Spent  Last Updated

38

39
40 Explanation

Common causes of Cushing syndrome

Etiology Pathologic findings

Cushing disease (ie, pituitary ACTH


ACTH- hypersecretion)
Bilateral hyperplasia involving the zona fasciculata & reticularis
dependent Ectopic ACTH syndrome (eg,
paraneoplastic ACTH secretion)

Benign tumors are typically round, well-circumscribed lesions with cells


similar to normal zona fasciculata
ACTH- Malignant tumors are often irregular and poorly demarcated with
Adrenal adenoma or carcinoma
independent considerable cellular pleomorphism
Atrophy of the zona fasciculata & reticularis in the uninvolved cortex
regions

This patient has hypertension and hyperpigmentation associated with a mediastinal mass. These features suggest a pulmonary malignancy with
ectopic corticotropin (ACTH) production and resulting Cushing syndrome. ACTH is a polypeptide hormone produced by the cleavage of
proopiomelanocortin (POMC). Cleavage of POMC also yields melanocyte-stimulating hormone (MSH) and other smaller peptides. Both ACTH
and MSH can cause hyperpigmentation by binding MSH receptors. The pigmentation may be diffuse or limited to sun-exposed areas, scars,
palmar creases, or oral mucosa.

Patients with ectopic production of ACTH often have manifestations of severe hypercortisolism, including wide purple striae, fatigue, easy bruising,
proximal muscle weakness, and central obesity (although the hypermetabolic state associated with the malignancy may cause weight loss
instead). Other abnormalities may include hyperglycemia, hypokalemia, and hypertension (due to the partial mineralocorticoid activity of cortisol).
Small cell lung cancer is the most common cause of paraneoplastic Cushing syndrome. In addition, some neuroendocrine tumors (eg, bronchial
carcinoids, medullary thyroid cancer) can produce ACTH or, very rarely, corticotropin-releasing hormone.

(Choice A) Overproduction of amino acid-derived hormones is typically seen in neuroendocrine tumors such as carcinoid (serotonin) and
pheochromocytoma (norepinephrine, epinephrine).

(Choice B) Overproduction of arachidonic acid derivatives (eg, leukotrienes, prostaglandins) is not seen in Cushing syndrome and is not a
common cause of any paraneoplastic syndrome.

(Choice D) Cortisol is overproduced by the adrenal glands in primary Cushing syndrome. The excess cortisol suppresses the production of
ACTH; as a result, hyperpigmentation would not occur.

(Choice E) Excess production of calcitriol (1,25-dihydroxyvitamin D) is responsible for the hypercalcemia associated with certain lymphomas and
granulomatous diseases (eg, sarcoidosis).

Educational objective:
Paraneoplastic Cushing syndrome is most commonly due to ectopic ACTH production by small cell lung carcinoma. Patients may have features
of severe hypercortisolism. Hyperpigmentation is frequently present due to the binding of melanocyte-stimulating hormone (MSH) receptors by
ACTH and MSH.

References
The ectopic ACTH syndrome

REVIEW 0
Feedback End Block

https://www.uworld.com/ClientApp/v15/apps/qbanktestinterface/index.html#/launchtest/7281878/nbme/229400736/3/1 1/1
‫ م‬8:38 2021/‫‏‬9/‫‏‬11 https://t.me/USMLEWorldStep2CK UWorld STEP2 SIM Form 1

1 Item 11 of 40 ABC 0.25

2
Mark
Question Id: 6880 Full Screen Tutorial Lab Values Notes Calculator Reverse Color Text Zoom Settings
Previous Next
3

4 A 27-year-old woman, gravida 1 para 0, at 40 weeks gestation comes to the hospital due to painful contractions for the past few hours. The
5 patient has had no leakage of fluid or vaginal bleeding. Fetal movement has been normal. The patient has had an uncomplicated pregnancy.
6 BMI is 42 kg/m2. Vital signs are normal. On admission, the cervix is 5 cm dilated and 90% effaced with the fetal head at −1 station. She received
7 epidural analgesia; on repeat cervical examination, the cervix is 6 cm dilated and 100% effaced with the fetal head at −1 station. Two hours later,
8 the cervix is unchanged. Fetal heart rate monitoring shows a baseline of 130/min, moderate variability, accelerations, and occasional early
9 decelerations. The tocodynamometer shows contractions every 5-7 minutes. Which of the following is the best next step in management of this
10 patient?
11
A. Cesarean delivery (16%)
12

13 B. Continue expectant management (27%)


14
C. Operative vaginal delivery (1%)
15

16  D. Oxytocin infusion (49%)


17
 E. Vaginal misoprostol (5%)
18

19

20
Incorrect 49%
 03
21 secs 09/02/2021
22
Correct answer  Answered correctly Time Spent  Last Updated
D
23

24

25 Explanation
26

27
Disorders of the active phase of labor
28

29
Diagnosis Clinical features Treatment

30 Cervical change slower than expected


Protraction Oxytocin
31 ± Inadequate contractions
32
No cervical change for ≥4 hours with adequate contractions
33
Arrest OR Cesarean delivery
34
No cervical change for ≥6 hours with inadequate contractions
35

36 The first stage of labor begins with the onset of regular contractions and lasts until complete (10 cm) cervical dilation. The first stage is divided
37 into 2 phases:
38
Latent: 0-6 cm, when the cervix dilates slowly and has no expected rate of cervical change
39

40 Active: ≥6-10 cm, when the cervix dilates rapidly with a normal progression of ≥1 cm every 2 hours

This patient's cervix is 6 cm dilated (ie, in the active phase) but has remained unchanged for the past 2 hours, consistent with active phase
protraction: cervical progression of <1 cm every 2 hours. The most common cause of active phase protraction is inadequate contractions.
Ideally, contractions occur every 2-3 minutes, but this patient is contracting only every 5-7 minutes. In patients with active phase protraction, the
best next step in management is oxytocin augmentation, which increases contraction frequency and strength, improving the chances for
spontaneous vaginal delivery compared to expectant management (Choice B).

(Choice A) Cesarean delivery is indicated for active phase arrest, which occurs when a patient is ≥6 cm dilated and lacking cervical change after
4 to 6 hours. If this patient remains 6 cm dilated despite oxytocin augmentation, a cesarean delivery is indicated.

(Choice C) An operative vaginal delivery (eg, forceps-assisted vaginal delivery) is performed during the second stage of labor (10 cm dilation until
fetal delivery) to expedite delivery for category III tracings or maternal exhaustion. This patient is 6 cm dilated.

(Choice E) Vaginal misoprostol is used for cervical ripening (softening and thinning of the cervix) in patients undergoing labor induction. It is not
used for patients in spontaneous active labor.

Educational objective:
The first stage of labor is divided into 2 phases: the latent phase (0-6 cm) and the active phase (≥6-10 cm). Active phase protraction occurs when
cervical dilation proceeds slower than expected (<1 cm every 2 hours). Contraction inadequacy is the most common etiology; management is with
oxytocin augmentation.

References
Preventing the first cesarean delivery: summary of a joint Eunice Kennedy Shriver National Institute of Child Health and Human Development,
Society for Maternal-Fetal Medicine, and American College of Obstetricians and Gynecologists workshop.

(http://www.ncbi.nlm.nih.gov/pubmed/23635639)

Obstetrics & Gynecology Pregnancy, Childbirth & Puerperium Labor management


Subject System Topic

https://t.me/USMLEWorldStep2CK

REVIEW 0
Feedback End Block

https://www.uworld.com/ClientApp/v15/apps/qbanktestinterface/index.html#/launchtest/7281878/nbme/229400736/3/1 1/1
‫ م‬8:38 2021/‫‏‬9/‫‏‬11 https://t.me/USMLEWorldStep2CK UWorld STEP2 SIM Form 1

1 Item 12 of 40 ABC 0.25

2
Mark
Question Id: 6881 Full Screen Tutorial Lab Values Notes Calculator Reverse Color Text Zoom Settings
Previous Next
3

4 A 64-year-old man comes to the emergency department after he had an episode of right hand weakness and slurred speech that lasted 15-20
5 minutes. He has no chest pain, shortness of breath, lightheadedness, syncope, or palpitations. The patient has a long history of hypertension and
6 type 2 diabetes mellitus. A year ago, he had an episode of atrial fibrillation requiring hospitalization, but it resolved spontaneously. Currently, he is
7 comfortable and has no complaints. His blood pressure is 150/100 mm Hg and pulse is irregularly irregular at 110/min. BMI is 34 kg/m2. There
8 are no carotid bruits. Neurologic examination is unremarkable. His serum creatinine is 0.8 mg/dL. Echocardiogram shows left atrial dilation,
9 normal left ventricular ejection fraction, and no major valvular abnormalities. Carotid ultrasound is unremarkable. Which of the following would be
10 most effective in preventing similar neurologic episodes in this patient?
11
A. Amiodarone (3%)
12

13 B. Aspirin (19%)
14
 C. Clopidogrel (5%)
15

16 D. Long-acting nitrates (0%)


17
E. Metoprolol succinate (8%)
18
 F. Rivaroxaban (63%)
19

20 G. Strict blood glucose control (0%)


21

22

23
Incorrect 63%
 02 secs 06/05/2021
24
Correct answer  Answered correctly Time Spent  Last Updated
25 F
26

27
Explanation
28

29
This patient with a self-limited episode of right hand weakness and slurred speech likely had a transient ischemic attack (TIA) due to
30
thromboembolism in the setting of atrial fibrillation (AF) (given his history of a prior episode of AF and his irregularly irregular pulse).
31
Anticoagulant therapy with warfarin or nonvitamin K antagonist oral anticoagulants (NOACs) has been shown to significantly reduce the risk
32
of embolization in patients with AF who are at moderate to high risk of thromboembolic events. Current guidelines recommend the use of
33
CHA2DS2-VASc score for risk assessment in patients with nonvalvular AF. This patient has a CHA2DS2-VASc score >2 (hypertension, diabetes,
34
recent TIA) and should be managed with oral anticoagulation to reduce the long-term risk of systemic embolization.
35
Rivaroxaban is one of the NOACs (along with dabigatran, apixaban, and edoxaban) used to reduce the risk of thromboembolic events in patients
36
with nonvalvular AF. It is not recommended for use in patients with renal failure.
37

38 (Choice A) Amiodarone is occasionally used for maintenance of sinus rhythm in patients with recurrent and symptomatic AF (rhythm-control
39 strategy). Patients with paroxysmal AF who remain in sinus rhythm continue to remain at risk for thromboembolic events and should be managed
40 with anticoagulant therapy.

(Choices B and C) Aspirin monotherapy or the combination of aspirin and clopidogrel is significantly less effective in reducing thromboembolic
risk compared to anticoagulant therapy.

(Choice D) Long-acting nitrates are used for symptom control in patients with refractory angina. They have no role in preventing thromboembolic
events in patients with AF.

(Choice E) Beta blockers (eg, metoprolol) and/or calcium channel blockers are used for ventricular rate control in patients with AF and rapid
ventricular rates. However, regardless of rate control, atrial contraction remains irregular, which increases the risk of clot formation due to
turbulent blood flow.

(Choice G) Strict blood glucose control prevents the microvascular complications of diabetes mellitus; however, it has not been shown to reduce
the risk of recurrent thromboembolic events in patients with AF.

Educational objective:
Warfarin or nonvitamin K antagonist oral anticoagulants (rivaroxaban, dabigatran, apixaban, edoxaban) should be used to reduce the risk of
systemic thromboembolism in patients with atrial fibrillation and moderate to high risk of thromboembolic events (CHA2DS2-VASc score >2).

References
2014 AHA/ACC/HRS guideline for the management of patients with atrial fibrillation: a report of the American College of Cardiology/American
Heart Association Task Force on practice guidelines and the Heart Rhythm Society.

(http://www.ncbi.nlm.nih.gov/pubmed/24682347)

Medicine Cardiovascular System Embolic stroke


Subject System Topic

https://t.me/USMLEWorldStep2CK

REVIEW 0
Feedback End Block

https://www.uworld.com/ClientApp/v15/apps/qbanktestinterface/index.html#/launchtest/7281878/nbme/229400736/3/1 1/1
‫ م‬8:39 2021/‫‏‬9/‫‏‬11 https://t.me/USMLEWorldStep2CK UWorld STEP2 SIM Form 1

1 Item 13 of 40 ABC 0.25

2
Mark
Question Id: 6888 Full Screen Tutorial Lab Values Notes Calculator Reverse Color Text Zoom Settings
Previous Next
3

4 A 79-year-old woman comes to the office due to fatigue, poor appetite, and an unintentional 4-kg (8.8-lb) weight loss. The patient has had 4
5 weeks of nonproductive cough and shortness of breath. Her other medical problems include mild Alzheimer dementia, polymyalgia rheumatica,
6 hypertension, and heart failure with preserved ejection fraction. Temperature is 37.4 C (99.3 F), blood pressure is 148/72 mm Hg, and pulse is
7 88/min. Pulse oximetry is 95% on room air. The patient is thin. There is dullness to percussion and decreased breath sounds in the right lower
8 and middle chest. A bedside portable ultrasound reveals a right-sided pleural effusion. A thoracentesis removes 1.5 L of yellow pleural fluid. The
9 pleural fluid results are as follows:
10
Protein 4.9 g/dL
11

12 Glucose 40 mg/dL
13 Nucleated cell count 1200/mm3
14
Lymphocytes 90%
15

16 Neutrophils 4%
17
Monocytes 6%
18

19
Which of the following is the most likely cause of this patient's pleural effusion?
20

21 A. Chylothorax (16%)

22
B. Congestive heart failure (8%)
23

24 C. Empyema (32%)
25
D. Hypoalbuminemia (3%)
26

27
E. Pulmonary infarction (0%)

28 F. Tuberculosis (37%)

29

30

31
Incorrect 37%
32

33
Correct answer  Answered correctly  04 secs
Time Spent  05/31/2021
Last Updated
F
34

35

36
Explanation

37

38
Light criteria for pleural effusions
39
Transudate Exudate
40

Protein
≤0.5 >0.5
(pleural/serum)

LDH ≤0.6 >0.6


(pleural/serum) Pleural LDH ≤ two-thirds upper limit of normal serum LDH Pleural LDH > two-thirds upper limit of normal serum LDH

Hypoalbuminemia Infection (parapneumonic, TB, fungal, empyema)


Common
(cirrhosis, nephrotic syndrome) Malignancy
causes
Congestive heart failure PE

LDH = lactate dehydrogenase; PE = pulmonary embolism; TB = tuberculosis.

The first step in determining the etiology of a pleural effusion is characterizing the fluid as transudative or exudative. Light's criteria is widely
used for this purpose. Although additional laboratory results are not given in this case, a pleural fluid protein level of 4.9 g/dL is exceedingly high
(normal serum protein is 6-7.8 g/dL). This patient's effusion would therefore most likely qualify as exudative.

Tuberculous effusions are exudative effusions characterized by very high protein levels (always >4 g/dL), lymphocytic leukocytosis, and low
glucose levels (<60 mg/dL). Other classic features include markedly elevated lactate dehydrogenase levels (often >500 U/L) and low pH.
Although this patient does not have traditional social or behavioral risk factors for tuberculosis (TB), latent infection is common, and reactivation is
more likely in elderly patients. In addition, it is quite possible she is on chronic corticosteroids for polymyalgia rheumatica, which would cause
immunosuppression and increase the risk of reactivation.

(Choice A) Chylothorax results from the leakage of chyle into the thoracic space, often from an obstruction of the thoracic duct. It can cause a
lymphocytic predominant effusion, but the fluid is often turbid or milky white (not yellow). With the very high fluid protein and low glucose, TB is far
more likely.

(Choices B and D) Both congestive heart failure and hypoalbuminemia cause transudative effusions characterized by pleural fluid with low
protein and minimal cells. This patient has an exudative effusion with very high protein and a lymphocytic leukocytosis.

(Choice C) An empyema is the presence of frank pus or bacteria (by Gram stain) in the pleural space. However, empyema usually presents with
acute symptoms (fever, pleuritic pain), and pleural fluid shows prominent neutrophilic leukocytosis (often >50,000/mm3). By contrast, this patient's
pleural fluid shows only mild leukocytosis (1,200/mm3) with lymphocytic predominance (90% lymphocytes) and her presentation is subacute,
making TB more likely than empyema.

(Choice E) Pulmonary infarction is typically associated with pulmonary embolus (PE). PE tends to present acutely with chest pain and can cause
bloody pleural effusions. This patient has a subacute presentation with no pain and pleural fluid with low glucose and very high protein
(uncommon in PE or pulmonary infarction).

Educational objective:
Tuberculosis is a common cause of exudative pleural effusions. It is characterized by pleural fluid with very elevated protein (always >4 g/dL),
lymphocytic leukocytosis, and low glucose levels (<60 mg/dL).

Medicine Pulmonary & Critical Care Pleural effusion


Subject System Topic

https://t.me/USMLEWorldStep2CK

REVIEW 0
Feedback End Block

https://www.uworld.com/ClientApp/v15/apps/qbanktestinterface/index.html#/launchtest/7281878/nbme/229400736/3/1 1/1
‫ م‬8:39 2021/‫‏‬9/‫‏‬11 https://t.me/USMLEWorldStep2CK UWorld STEP2 SIM Form 1

1 Item 14 of 40
2
Mark
Question Id: 6890 Previous Next
3

4 A 38-year-old man comes to the office for follow-up. After recently learning that one of his sexual partners tested positive for HIV, the patient is
5 concerned that he may be infected. He has been fatigued lately but otherwise feels well. Vital signs are normal. Scattered, nontender
6 lymphadenopathy is present in the cervical and inguinal chains. The rest of the examination is normal. Rapid HIV testing is positive. Subsequent
7 laboratory results are as follows:
8
Hepatitis panel
9

10 Hepatitis A antibody positive


11
Hepatitis B surface antibody positive
12
Hepatitis B core antibody positive
13

14 Hepatitis B surface antigen negative


15
Hepatitis C virus antibody negative
16

17 CD4 lymphocytes 380/mm3


18
HIV-1 RNA quantification (viral load) 42,000 copies/mL
19
20 Antivaricella immunoglobulin positive
21
Interferon gamma release assay negative
22

23 In addition to starting antiretroviral therapy, which of the following vaccinations are indicated?
24
25 Bacille Live attenuated Pneumococcal Zoster
26 Calmette- influenza
27 Guérin
28
A. Yes Yes Yes Yes (9%)
29
30 B. Yes Yes No Yes (0%)
31 C. Yes No Yes Yes (5%)
32
 D. Yes No Yes No (7%)
33
34 E. No Yes Yes No (39%)
35
 F. No No Yes No (36%)
36

37

38
39 Incorrect 36%
40
Correct answer
F
 Answered correctly  03 secs
Time Spent  04/19/2021
Last Updated

Explanation

Vaccines for adults with HIV


Vaccine Indications

Chronic liver disease (including HBV & HCV)


HAV Men who have sex with men
IV drug users

HBV All patients without documented immunity to HBV

HPV All patients age 11-26

Influenza Annually for all patients (inactivated formulation)

All patients age 11-18


Meningococcus
Large groups living in close proximity (eg, college students,
(serogroups A, C,
military recruits, incarcerated individuals)
W, Y)
Asplenia or complement deficiency

PCV13 once
Pneumococcus
PPSV23 8 weeks later, 5 years later & at age 65

Tetanus,
Tdap once (repeat with each pregnancy in women)
diphtheria &
Td every 10 years
pertussis

Live vaccines (eg, MMR, varicella-zoster) are


contraindicated if CD4+ cell count <200/mm3

HAV = hepatitis A virus; HBV = hepatitis B virus; HCV = hepatitis C virus; HPV = human
papillomavirus; IV = intravenous; MMR = measles, mumps & rubella; PCV13 = 13-valent
pneumococcal conjugate vaccine; PPSV23 = 23-valent pneumococcal polysaccharide
vaccine; Td = tetanus-diphtheria toxoid booster; Tdap = tetanus-diphtheria-acellular
pertussis.

HIV infection causes significant immune dysregulation, which increases the risk of severe infection with a wide range of pathogens. Although
opportunistic infections do not generally occur until cell-mediated immunity is significantly impaired (CD4 <200/mm3), the risk of common infections
(eg, Streptococcus pneumoniae) is significantly increased much earlier in the disease course due to HIV-mediated dysfunction of cytokine
signaling, immune-cell recruitment, and B-cell function.

Therefore, vaccination against common pathogens and screening for specific viral and bacterial infections is warranted for patients with a new
diagnosis of HIV, as follows:

Pneumococcal vaccination is indicated for all patients with HIV. The conjugate vaccine should be given first, followed by the
polysaccharide vaccine after 8 weeks, after 5 years, and at age 65. Patients with low CD4 counts may respond poorly to immunization due
to impaired humoral immunity. For this reason, immunization should be given as early in the disease course as possible.

Seasonal inactivated influenza vaccination is indicated for all patients with HIV due to increased risk of influenza complications (eg,
secondary pneumonia). In contrast, live nasal attenuated influenza immunization is not given to patients with HIV (regardless of CD4
counts) due to risk of systemic spread.

Tuberculosis screening is indicated for all patients with HIV due to elevated risk of tuberculosis reactivation. Either purified protein
derivative (PPD) testing or interferon gamma quantification assay can be used, but PPD anergy (false-negative testing) may occur with low
CD4 counts. Those with positive testing require treatment. Although bacille Calmette-Guérin vaccination offers some immunity to
tuberculosis, it is not recommended for patients with HIV due to the risk of disseminated infection.

Varicella serology is recommended for patients with HIV who have no history of chickenpox or shingles. Those with negative serology who
were born after 1979 should be given zoster vaccination. Live attenuated vaccination should only be administered to those with CD4 counts
>200/mm3 to prevent systemic infection; newer recombinant vaccination may be safely administered to those with lower CD4 counts but the
immunogenic response is often muted. Vaccination is not indicated in this patient with positive antivaricella immunoglobulin titers
(suggesting immunity to varicella-zoster virus).

Educational objective:
Patients with a new diagnosis of HIV require pneumococcal and influenza vaccination. Zoster vaccination is also frequently indicated for those
with no history of the disease and no evidence of immunity. Treatment with prophylactic antibiotics for opportunistic infections is not usually
required until the CD4 count is <200/mm3.

Medicine Allergy & Immunology HIV


Subject System Topic

https://t.me/USMLEWorldStep2CK

REVIEW

https://www.uworld.com/ClientApp/v15/apps/qbanktestinterface/index.html#/launchtest/7281878/nbme/229400736/3/1 1/1
‫ م‬8:39 2021/‫‏‬9/‫‏‬11 https://t.me/USMLEWorldStep2CK UWorld STEP2 SIM Form 1

1 Item 15 of 40
2
Mark
Question Id: 6894 Previous Next
3

5
This item has associated media that may require the use of headphones. Please ensure your system/speaker volume is set to an audible level.

6
A 59-year-old man comes to the office due to headaches. He describes the headaches as diffuse and says that they get worse when he leans
7 forward. He has not had any fever, vomiting, or chest pain but has a chronic cough. The patient was recently diagnosed with small cell lung
8 cancer. He has an extensive smoking history and quit smoking 4 years ago. Blood pressure is 100/60 mm Hg and pulse is 100/min. Physical
9 examination of the neck is shown below. Abdomen is soft and nondistended. There is no peripheral edema. Which of the following would most
10 likely relieve this patient’s symptoms?
11
12 Play Media
13

14
A. Broad-spectrum antibiotics (1%)
15
16
B. High-dose corticosteroids (13%)

17 C. Loop diuretics (18%)


18
D. Pericardiocentesis (22%)
19
20  E. Radiation therapy (43%)
21

22

23
43%
24 Correct  Answered correctly  04 secs
Time Spent  07/30/2021
Last Updated
25

26

27
Explanation
28

29
This patient’s presentation is consistent with superior vena cava (SVC) syndrome, caused by compression of the SVC. Patients with SVC
30
syndrome often have headaches that are worse when leaning forward due to decreased gravitational effects on the blood column. Jugular
31
venous distension is often present, but there is a lack of peripheral edema to suggest cardiac failure. Facial and upper extremity swelling is often
32
prominent from obstruction of venous blood flow. There may be conjunctival congestion as well. Prominent collateral veins may be present on
33 skin examination.
34

35 The most common cause of SVC syndrome is malignancy, either lung cancer (especially small cell lung cancer, which tends to grow centrally
36 rather than peripherally) or lymphoma. Malignancy-related SVC syndrome, especially when it causes respiratory compromise, may require
37 endovenous stenting followed by radiation therapy, although this treatment is primarily palliative. Depending on the type of malignancy and prior
38 treatment course, some patients may benefit from chemotherapy.
39

40 (Choice A) The most common infectious etiology of SVC syndrome is fibrosing mediastinitis from granulomatous infections such as tuberculosis
or histoplasmosis, but this patient’s lung malignancy is the most likely underlying etiology in this case.

(Choice B) High-dose corticosteroids may help relieve SVC syndrome when the cause is a corticosteroid-responsive malignancy like lymphoma.
Corticosteroids are unlikely to be helpful in this case, however.

(Choice C) Diuretic therapy might bring mild improvement to this patient’s symptoms, but would not be as effective as radiation therapy.

(Choice D) Pericardiocentesis would be indicated if this patient’s symptoms were due to a pericardial effusion, but this patient’s impaired venous
flow appears to be localized to the SVC and not the entire venous system.

Educational objective:
Superior vena cava syndrome is a potential complication of lung malignancy, and is manifested by headache, facial swelling, and jugular venous
engorgement without peripheral edema. The primary treatment is radiation therapy as a palliative measure.

Medicine Pulmonary & Critical Care Superior vena cava syndrome


Subject System Topic

https://t.me/USMLEWorldStep2CK

REVIEW

https://www.uworld.com/ClientApp/v15/apps/qbanktestinterface/index.html#/launchtest/7281878/nbme/229400736/3/1 1/1
‫ م‬8:39 2021/‫‏‬9/‫‏‬11 https://t.me/USMLEWorldStep2CK UWorld STEP2 SIM Form 1

1 Item 16 of 40
2
Mark
Question Id: 6899 Previous Next
3

4 An 81-year-old man hospitalized 5 days ago for acute onset of dense right-sided hemiparesis is found to be increasingly lethargic. Over the
5 course of his hospitalization, the weakness in his right arm and leg has slightly improved and his speech has become more intelligible. He was
6 recently started on a soft diet. The patient's other medical problems include hypertension, hypercholesterolemia, and psoriasis. His temperature
7 is 38.9 C (102 F), blood pressure is 100/60 mm Hg, and pulse is 120/min. On chest auscultation, coarse rhonchi are heard over the right lung.
8 Heart sounds are normal. Abdomen is soft and nontender. Extremities have no cyanosis or edema. His leukocyte count is 18,500/mm3,
9 hemoglobin is 9.8 g/dL, and platelet count is 490,000/mm3. A chest x-ray reveals an infiltrate in the posterior segment of the right upper lobe.
10 Which of the following would most likely have prevented this patient's current condition?
11
A. Antibiotic prophylaxis (1%)
12
13  B. Elevation of the head of the bed (50%)
14
C. Frequent use of incentive spirometry (17%)
15
16 D. Nasogastric tube feeding (29%)
17 E. Use of proton pump inhibitor (0%)

18

19
20

21
Incorrect 50% 02 secs 06/19/2021
22
Correct answer
B
 Answered correctly  Time Spent  Last Updated

23

24
25 Explanation
26

27

28

29
30
31

32

33
34

35

36

37

38
39

40

This patient likely has aspiration pneumonia. Conditions that predispose patients to the development of aspiration pneumonia include impaired
consciousness, the presence of tracheal or nasogastric tubes, and impaired cough reflex, which can occur following intubation or a
cerebrovascular accident. Aspiration pneumonia occurs due to aspiration of microbes (commonly anaerobes) of the oral cavity. Measures to
prevent pneumonia in hospitalized patients include oral care, diet modification for patients with dysphagia, and compensatory techniques such
as elevating the head of the bed to 30-45 degrees.

Aspiration pneumonia typically develops over the course of days and causes manifestations such as a low-grade fever, dyspnea, productive
cough, and malaise. The location of the infiltrate is gravity-dependent and differs according to the patient's position at the time of aspiration. In
supine patients, the posterior segments of the upper lobes and superior segments of the lower lobes are most affected, whereas in erect patients
the bases of the lower lobes and the right middle lobe are most affected. Broad-spectrum antibiotics with good anaerobic coverage (eg,
clindamycin) are the mainstay of treatment.

(Choice A) Multiple studies have failed to identify a benefit of prophylactic antibiotics for the prevention of aspiration pneumonia in hospitalized
patients.

(Choice C) Incentive spirometry and deep-breathing techniques are helpful to reduce the risk of postoperative pneumonia in patients who have
undergone thoracic, upper abdominal, or aorta surgery. However, these techniques are not routinely used to prevent aspiration pneumonia.

(Choice D) Jejunal (postpyloric) feeding has been shown to reduce the risk of aspiration. However, simple nasogastric or percutaneous
endoscopic gastrotomy tubes, which are effective methods for providing nutrition to patients with severe dysphagia, predispose to aspiration
pneumonia.

(Choice E) The use of proton pump inhibitors has been strongly correlated to increased risk of pneumonia in multiple settings.

Educational objective:
Aspiration pneumonia commonly occurs in patients with impaired consciousness, the presence of tracheal or nasogastric tubes, or impaired cough
reflex. Measures to prevent aspiration pneumonia include oral care, diet modification for patients with dysphagia, and elevating the head of the
bed to 30-45 degrees.

References
Preventing aspiration pneumonia in older people: do we have the "know-how"?

(http://www.ncbi.nlm.nih.gov/pubmed/24993858)

Medicine Pulmonary & Critical Care Aspiration pneumonia


Subject System Topic

https://t.me/USMLEWorldStep2CK

REVIEW

https://www.uworld.com/ClientApp/v15/apps/qbanktestinterface/index.html#/launchtest/7281878/nbme/229400736/3/1 1/1
‫ م‬8:39 2021/‫‏‬9/‫‏‬11 https://t.me/USMLEWorldStep2CK UWorld STEP2 SIM Form 1

1 Item 17 of 40
2
Mark
Question Id: 6907 Previous Next
3

4 A 43-year-old woman is brought to the emergency department due to 2 days of confusion and lethargy. The patient was diagnosed with hepatitis
5 C infection 10 years ago during an evaluation of elevated liver aminotransferase levels but did not follow up afterward. She drinks a pint of liquor
6 daily and in her 20s used injection drugs. Temperature is 38.2 C (100.8 F), blood pressure is 120/70 mm Hg, pulse is 110/min, and respirations
7 are 20/min. The patient has scleral icterus and scattered spider angiomas. The abdomen is distended with shifting dullness on percussion.
8 There is diffuse abdominal tenderness but no guarding or rebound tenderness. She also has bilateral ankle edema. Laboratory results are as
9 follows:
10
Complete blood count
11
12 Hemoglobin 10.8 g/dL
13 Platelets 90,000/mm3
14
Leukocytes 11,000/mm3
15
16
Serum chemistry
17
18 Sodium 130 mEq/L
19
Blood urea nitrogen 26 mg/dL
20

21
Creatinine 1.5 mg/dL

22

23
Liver function studies

24 Albumin 2.8 g/dL


25
Total bilirubin 4.3 mg/dL
26

27 Alkaline phosphatase 120 U/L


28 Aspartate aminotransferase 87 U/L
29
Alanine aminotransferase 49 U/L
30
31
Coagulation studies
32

33 INR 1.6 (normal 0.8-1.1)


34
Activated partial thromboplastin time 25 sec
35

36 Which of the following is the most likely reason for this patient's current condition?
37
 A. Acute reactivation of viral hepatitis (6%)
38
39 B. Decreased renal clearance of neurotoxins (31%)
40
C. Dysregulated coagulation and fibrinolysis (7%)

 D. Intestinal bacterial translocation (43%)

E. Microscopic bowel perforation (1%)

F. Rapid fall in serum osmolality (10%)

Incorrect 43%
Correct answer  Answered correctly  04 secs
Time Spent  03/27/2021
Last Updated
D

Explanation

Spontaneous bacterial peritonitis


Temperature >37.8 C (100 F)
Abdominal pain/tenderness
Clinical presentation
Altered mental status (abnormal connect-the-numbers test)
Hypotension, hypothermia, paralytic ileus with severe infection

PMNs ≥250/mm3
Positive culture, often gram-negative organisms (eg, Escherichia coli, Klebsiella)
Diagnosis from ascitic fluid
Protein <1 g/dL
SAAG ≥1.1 g/dL

Empiric antibiotics - third-generation cephalosporins (eg, cefotaxime)


Treatment
Fluoroquinolones for SBP prophylaxis

PMN = polymorphonuclear leukocytes; SAAG = serum-ascites albumin gradient; SBP = spontaneous bacterial peritonitis.

This patient with evidence of cirrhosis (eg, ascites, spider angiomas, thrombocytopenia, coagulopathy) has findings concerning for spontaneous
bacterial peritonitis (SBP) as she has fever, ascites, diffuse abdominal pain (a manifestation of peritonitis that can be subtle due to the
presence of ascites), tachycardia, and mental status changes. In SBP, enteric bacteria are thought to translocate across the intestinal wall and
seed ascitic fluid within the peritoneal cavity.

Since SBP is a major cause of morbidity and mortality in patients with cirrhosis, a paracentesis should be performed when it is suspected. The
diagnosis is made when the ascitic fluid neutrophil count is ≥250/mm3. Cultures often show gram-negative organisms but can be negative.
Treatment typically consists of intravenous antibiotics (eg, third-generation cephalosporins, fluoroquinolones).

(Choice A) A patient with acute reactivation of viral hepatitis would have elevated aspartate and alanine aminotransferase levels in the range of
5-20 times the upper limit of normal from baseline. Most patients with chronic hepatitis C without exacerbation have liver enzyme levels that are
either normal or slightly elevated.

(Choice B) This patient has a decreased glomerular filtration rate based on her elevated creatinine of 1.5 mg/dL, possibly due to prerenal
azotemia or a component of hepatorenal syndrome. Confusion and abdominal pain may occur in patients with uremia due to renal failure, but
these symptoms are rare in patients with a blood urea nitrogen level <100 mg/dL. In addition, a decreased glomerular filtration rate would not
cause fever.

(Choice C) Dysregulated coagulation and fibrinolysis are concerning for disseminated intravascular coagulation (DIC), a consumptive
coagulopathy characterized by an elevated INR, activated partial thromboplastin time (aPTT), and fibrin split products with a low serum fibrinogen
and platelet count. This patient's laboratory studies are not consistent with DIC as her aPTT is normal; in addition, the platelet count is usually
lower in DIC.

(Choice E) Microscopic bowel perforation can be seen in acute diverticulitis, which usually presents with fever and pain in the left lower quadrant
of the abdomen.

(Choice F) Acute hyponatremia can cause cognitive changes (eg, coma, seizure), particularly when serum sodium concentrations are <120
mEq/L. This patient's serum sodium (likely hypervolemic hyponatremia in the setting of cirrhosis and ascites) is not low enough to account for her
mental status change, nor would it explain her fever and abdominal pain.

Educational objective:
Spontaneous bacterial peritonitis is a common complication of ascites in which the peritoneal fluid becomes infected by an enteric organism that
translocates across the intestinal wall. The finding on paracentesis of ≥250 neutrophils/mm3 in the ascitic fluid is considered diagnostic.

References
Intestinal permeability in cirrhotic patients with and without spontaneous bacterial peritonitis: is the ring closed?

(http://www.ncbi.nlm.nih.gov/pubmed/19844200)
Intestinal bacterial overgrowth and bacterial translocation in cirrhotic rats with ascites.

(http://www.ncbi.nlm.nih.gov/pubmed/9210626)
Small intestine dysmotility and bacterial overgrowth in cirrhotic patients with spontaneous bacterial peritonitis.

(http://www.ncbi.nlm.nih.gov/pubmed/9794900)

Medicine Gastrointestinal & Nutrition Spontaneous bacterial peritonitis


Subject System Topic

https://t.me/USMLEWorldStep2CK

REVIEW

https://www.uworld.com/ClientApp/v15/apps/qbanktestinterface/index.html#/launchtest/7281878/nbme/229400736/3/1 1/1
‫ م‬8:39 2021/‫‏‬9/‫‏‬11 https://t.me/USMLEWorldStep2CK UWorld STEP2 SIM Form 1

1 Item 18 of 40
2
Mark
Question Id: 6911 Previous Next
3

4 A 69-year-old woman comes to the office due to a rash. The patient has experienced diffuse skin itching for 2 months, starting when she moved
5 into an assisted living facility due to Alzheimer dementia. In the past 5 days, pruritic lesions have developed on her body. There has been no skin
6 pain, fever, or changes in appetite. Other medical problems include hypertension and type 2 diabetes mellitus. Vital signs are normal. Physical
7 examination reveals multiple lesions on the legs, as seen in the exhibit, with similar lesions on the back, axillae, and inguinal folds. The
8 conjunctivae and oral mucosa are normal. Which of the following is the most likely diagnosis of this patient's skin condition?
9
 A. Bullous pemphigoid (78%)
10
11 B. Contact dermatitis (13%)
12
C. Mycosis fungoides (0%)
13

14 D. Pemphigus vulgaris (4%)


15
 E. Staphylococcal scalded skin syndrome (2%)
16

17
18

19 Incorrect 78%
20
Correct answer  Answered correctly  02 secs
Time Spent  05/05/2021
Last Updated
A
21

22

23 Explanation
24
25
Bullous pemphigoid
26

27 Age >60
28 Clinical Pruritic, tense bullae
29
features Rare mucosal involvement

30
Prodrome of eczematous/urticaria-like rash

31 Dementia
Associated
32 Parkinson disease
disorders
33 Depression, bipolar disorder
34
Histology: subepidermal cleavage
35
Diagnosis Immunofluorescence: linear IgG/C3 deposition along basement membrane
36
Serology: autoantibodies to bullous pemphigoid antigens (hemidesmosomes)
37

38 Topical therapies: high-potency corticosteroids


Treatment
39
Systemic therapies: corticosteroid, doxycycline

40
This elderly patient with pruritus and bullae has the typical features of bullous pemphigoid, an autoimmune blistering disease.

Bullous pemphigoid most commonly affects individuals age >60 and has an increased incidence in those with neurologic disorders (eg, Alzheimer
dementia). A prodromal phase characterized by a pruritic, eczematous or urticarial rash may precede the development of bullae by weeks to
months, as in this patient. Bullae are typically 1-3 cm and tense and have an erythematous base; they may rupture to form erosions. Lesion
distribution can be localized or diffuse, involving the trunk, extremities, axillae, and inguinal folds. The associated pruritus can be disabling.

The diagnosis must be confirmed with skin biopsy (ie, light microscopy, direct immunofluorescence) because therapy is often chronic and can
involve agents with significant toxicity (eg, systemic corticosteroids). Biopsy shows subepithelial cleavage and linear deposition of IgG and/or C3
along the basement membrane. Serology for antibodies against bullous pemphigoid antigens (ie, hemidesmosomes) can further support the
diagnosis.

(Choice B) The prodromal rash of bullous pemphigoid can resemble contact dermatitis; however, the vesicles and bullae associated with acute
contact dermatitis are typically present at the onset of the rash, not 2 months later.

(Choice C) Mycosis fungoides, a subtype of cutaneous T-cell lymphoma, presents with patches, plaques, tumors, and generalized erythroderma,
which can mimic the prodromal phase of bullous pemphigoid. However, bullae and erosions are not characteristic.

(Choice D) Pemphigus vulgaris is an autoimmune blistering disease that typically occurs in patients age 40-60 and presents with painful (not
pruritic), flaccid (not tense) blisters that easily rupture to form erosions. Unlike bullous pemphigoid, mucosal (eg, oral) involvement is almost
always present.

(Choice E) Staphylococcal scalded skin syndrome is typically characterized by rapid-onset progressive erythema, flaccid bullae, and
desquamation in an acutely ill child. The chronic course of this older patient's rash is not characteristic.

Educational objective:
Bullous pemphigoid is an autoimmune blistering disorder that typically occurs in patients age >60. It is characterized by a prodromal phase
consisting of a pruritic, eczematous or urticarial rash, followed by formation of tense bullae weeks to months later.

References
Bullous pemphigoid.

(http://www.ncbi.nlm.nih.gov/pubmed/31090818)

Medicine Dermatology Bullous pemphigoid


Subject System Topic

https://t.me/USMLEWorldStep2CK

REVIEW

https://www.uworld.com/ClientApp/v15/apps/qbanktestinterface/index.html#/launchtest/7281878/nbme/229400736/3/1 1/1
‫ م‬8:40 2021/‫‏‬9/‫‏‬11 https://t.me/USMLEWorldStep2CK UWorld STEP2 SIM Form 1

1 Item 19 of 40
2
Mark
Question Id: 6916 Previous Next
3

4 A 62-year-old man comes to the emergency department due to 3 weeks of difficulty walking. Over the past 2 days, he has also had difficulty
5 speaking. His past medical history is notable only for osteoarthritis, for which he takes acetaminophen as needed. Temperature is 37.8 C (100
6 F). His brain MRI findings are shown below.
7
8

10
11
12
13

14

15
16

17
18

19
20

21

22

23

24
25

26

27

28 Which of the following is most likely responsible for this patient’s symptoms?
29
30  A. Glial proliferation (15%)
31 B. Metastases (70%)

32
C. Cerebrospinal fluid obstruction (3%)
33
34 D. Vascular malformations (3%)
35
E. Immune-mediated demyelination (7%)
36

37

38
39 Incorrect 70% 04 secs 07/30/2021
40
Correct answer
B
 Answered correctly  Time Spent  Last Updated

Explanation

This patient has multifocal neurologic symptoms and multiple peripheral enhancing lesions on MRI concerning for brain metastases. Brain
metastases are the most common intracranial tumors in adults. There are several radiographic features suggestive of central nervous system
(CNS) metastases. Contrast-enhanced MRI reveals multiple, well-circumscribed enhancing lesions usually located at the grey-white junction.
These lesions are typically surrounded by large amounts of edema. Over 80% of patients diagnosed with brain metastases already have a known
diagnosis of cancer, but brain metastases can be the presenting lesions. The leading causes of brain metastases are lung cancer, breast cancer,
melanoma, and renal cell carcinoma.

(Choice A) Gliomas, the most common primary CNS tumor in adults, are tumors arising from glial cells. MRI typically shows an enhancing
solitary lesion with irregular borders. Multiple, circumscribed lesions, as seen in this patient, are more consistent with metastases than a primary
brain tumor.

(Choice C) Cerebrospinal fluid outflow obstruction will cause obstructive hydrocephalus and dilation of the ventricles. This patient’s ventricles are
not dilated.

(Choice D) Arteriovenous malformations (AVM) of the brain typically present in adults with hemorrhage, seizure, headache or focal neurologic
signs. AVMs have characteristic dark flow voids and little surrounding edema on MRI. They typically present in persons age 10 to 40.

(Choice E) The most common cause of demyelination in adults is multiple sclerosis. This disease typically presents in people between age 20 to
40 and is more common in women than men. Ovoid plaques in the periventricular region, corpus collosum, and deep white matter are classic MRI
findings.

Educational objective:
Brain metastases are the most common intracranial tumor in adults. They are most often due to melanoma, lung, breast, or renal cancer, and
appear as peripheral, circumscribed, enhancing lesions surrounded by vasogenic edema on MRI.

Medicine Nervous System Brain tumors


Subject System Topic

https://t.me/USMLEWorldStep2CK

REVIEW

https://www.uworld.com/ClientApp/v15/apps/qbanktestinterface/index.html#/launchtest/7281878/nbme/229400736/3/1 1/1
‫ م‬8:49 2021/‫‏‬9/‫‏‬11 https://t.me/USMLEWorldStep2CK UWorld STEP2 SIM Form 1

1 Item 20 of 40 ABC 0.25

2
Mark
Question Id: 6923 Full Screen Tutorial Lab Values Notes Calculator Reverse Color Text Zoom Settings
Previous Next
3

4 A 29-year-old woman, gravida 1 para 0 aborta 1, comes to the office to establish care and to receive preconception counseling. Three years ago,
5 the patient had an elective termination at 16 weeks gestation for fetal anencephaly. Menarche occurred at age 11; menses are regular and last for
6 4-5 days. She has no history of sexually transmitted infections or abnormal Pap tests. The patient has a history of hypothyroidism and depression
7 for which she takes levothyroxine and sertraline. She has no family history of genetic disorders. The patient swims or takes a spinning class 5
8 times a week. She does not use tobacco, alcohol, or illicit drugs. She takes a prenatal vitamin daily. Blood pressure is 120/80 mm Hg. BMI is 24
9 kg/m2. Pelvic examination reveals a small, mobile, retroverted uterus and no adnexal masses. Which of the following should be recommended to
10 decrease the patient's risk of pregnancy complications?
11
 A. Begin folic acid supplementation (79%)
12

13 B. Begin progesterone supplementation (0%)


14
C. Decrease exercise frequency (2%)
15

16 D. Discontinue levothyroxine (0%)


17
E. Discontinue sertraline (3%)
18
 F. No further recommendations (13%)
19

20

21

22 Incorrect 79%
 02 secs 05/04/2021
23 Correct answer  Answered correctly Time Spent  Last Updated
A
24

25

26 Explanation
27

28

29
Neural tube defects

30 Anencephaly
31 Types Encephalocele
32 Spina bifida, myelomeningocele
33
Low folic acid intake
34
Risk Methotrexate, antiepileptics
35
factors Diabetes mellitus
36
Prior pregnancy with neural tube defect
37

38 2nd-trimester ultrasound
Prenatal screening
39
Maternal serum alpha-fetoprotein

40 Average risk: 0.4 mg folic acid daily


Prevention
High risk: 4 mg folic acid daily

The embryonic neural tube closes 3-4 weeks after conception. Neural tube defects (NTDs) (eg, spina bifida, anencephaly) are the result of
incomplete neural tube closure and are typically isolated malformations. Folic acid is involved in the regulation of neural tube closure; women with
folic acid deficiency are at risk of fetal NTDs. All women should begin folic acid supplementation >1 month before conception as the neural tube
closes very early in pregnancy, often before patients are aware of being pregnant. Prenatal vitamins typically have the recommended amount of
folic acid to minimize the risk of NTDs in average-risk patients (0.4 mg daily) (Choice F). High-risk patients (eg, NTD in a prior pregnancy,
antiepileptic medications) require additional folic acid (4 mg daily) supplementation.

(Choice B) Progesterone supplementation decreases the risk of preterm birth in patients with a history of prior spontaneous preterm delivery.
This patient's prior delivery at 16 weeks gestation was induced.

(Choice C) In healthy patients, exercise is recommended as it decreases the risk for pregnancy complications (eg, excessive weight gain, fetal
macrosomia). There is no association between activity level and risk for NTDs.

(Choice D) Discontinuing levothyroxine could cause anovulation and infertility. Levothyroxine is not teratogenic. Untreated hypothyroidism during
pregnancy is associated with an increased risk for complications (eg, preeclampsia, fetal growth restriction).

(Choice E) Discontinuing sertraline leaves the patient at risk for depression during pregnancy and the postpartum period. Sertraline does not
increase the risk for major congenital malformations.

Educational objective:
Supplementation with folic acid decreases the occurrence of fetal neural tube defects. High-risk patients with a history of a prior pregnancy
complicated by a neural tube defect require additional folic acid supplementation.

References
Global birth prevalence of spina bifida by folic acid fortification status: a systematic review and meta-analysis.

(http://www.ncbi.nlm.nih.gov/pubmed/26562127)
Folic acid supplementation for pregnant women and those planning pregnancy: 2015 update.

(http://www.ncbi.nlm.nih.gov/pubmed/26272218)

Obstetrics & Gynecology Pregnancy, Childbirth & Puerperium Neural tube defects
Subject System Topic

https://t.me/USMLEWorldStep2CK

REVIEW 0
Feedback End Block

https://www.uworld.com/ClientApp/v15/apps/qbanktestinterface/index.html#/launchtest/7281878/nbme/229400736/3/1 1/1
‫ م‬8:49 2021/‫‏‬9/‫‏‬11 https://t.me/USMLEWorldStep2CK UWorld STEP2 SIM Form 1

1 Item 21 of 40
2
Mark
Question Id: 6925 Previous Next
3

4 A 56-year-old man comes to the office due to increasing weakness for the past 6 weeks. The patient first noticed difficulty climbing stairs and then
5 developed trouble rising from a chair. Recently, he becomes very tired when using his arms to do housework. Medical history is notable for
6 intermittent joint pains, for which he takes naproxen as needed. The patient works in international finance and traveled to Brazil 2 months ago for
7 business. Temperature is 37.2 C (99 F), blood pressure is 124/76 mm Hg, pulse is 75/min, and respiratory rate is 14/min. Bilateral proximal upper
8 and lower extremity weakness is noted. Distal muscle strength is normal. The remainder of the physical examination shows no abnormalities.
9 Laboratory results are as follows:
10
Total bilirubin 0.2 mg/dL
11
12 Aspartate aminotransferase 250 U/L
13 Alanine aminotransferase 140 U/L
14
Alkaline phosphatase 90 U/L
15
16 Lactate dehydrogenase 665 U/L
17
Creatine kinase 3700 U/L
18

19
Which of the following is the most likely diagnosis?
20

21  A. Amyotrophic lateral sclerosis (0%)


22
B. Guillain-Barré syndrome (2%)
23

24 C. Myasthenia gravis (0%)


25
D. Myotonic dystrophy (1%)
26

27
E. Polymyalgia rheumatica (8%)

28 F. Polymyositis (77%)

29
G. Thyroid myopathy (1%)
30
31 H. Viral hepatitis (5%)
32

33
34
Incorrect 77%
35 Correct answer  Answered correctly  03 secs
Time Spent  05/04/2021
Last Updated
36 F
37

38
Explanation
39

40

Distinguishing features of fibromyalgia, polymyositis


& polymyalgia rheumatica
Clinical features Diagnosis

Young to middle-aged women


Chronic widespread pain ≥3 months of symptoms with widespread pain index
Fibromyalgia Fatigue, impaired concentration or symptom severity score
Tenderness at trigger points (eg, midtrapezius, Normal laboratory studies
costochondral junction)

Elevated muscle enzymes (eg, creatine kinase,


Proximal muscle weakness (eg, increasing difficulty
aldolase, AST)
Polymyositis climbing stairs)
Autoantibodies (anti–Jo-1, ANA)
Pain mild/absent
Biopsy: endomysial infiltrate, patchy necrosis

Age >50
Polymyalgia Systemic signs & symptoms Elevated ESR, C-reactive protein
rheumatica Stiffness > pain in shoulders, hip girdle, neck Rapid improvement with glucocorticoids
Association with giant cell (temporal) arteritis

ANA = antinuclear antibody; AST = aspartate aminotransferase; ESR = erythrocyte sedimentation rate.

This patient with progressive, symmetric proximal muscle weakness has symptoms typical of polymyositis. Polymyositis is usually painless but
may cause mild diffuse myalgias and muscle tenderness in a minority of patients. Muscle enzyme levels (eg, creatine kinase) are elevated, and
autoantibodies (eg, antinuclear antibodies, anti-Jo-1 antibodies) are present in most cases. Polymyositis is similar to dermatomyositis but lacks
the typical skin findings; both may occur independently or as a paraneoplastic manifestation of an underlying malignancy.

Polymyositis is an inflammatory myopathy triggered by unknown, possibly viral, antigens. Muscle biopsy reveals inflammation, patchy necrosis,
and regeneration and fibrosis of muscle fibers. Infiltration of the endomysium by macrophages and CD8+ lymphocytes is typical.

(Choices A and C) Amyotrophic lateral sclerosis is characterized by asymmetric distal rather than proximal weakness. A combination of upper
(eg, hyperreflexia) and lower (eg, muscle atrophy, fasciculations) motor neuron signs is characteristic. Myasthenia gravis causes weakness that
worsens with activity. Symptoms are usually present in facial and distal limb muscles as well as proximal muscles. Muscle enzymes are normal in
these conditions.

(Choice B) Guillain-Barré syndrome is an immune-mediated neuropathy that causes an acute ascending paralysis.

(Choice D) Myotonic dystrophy can be differentiated from polymyositis by distal muscle symptoms, slowed relaxation phase of muscle
contraction, and a familial inheritance pattern.

(Choice E) Polymyalgia rheumatica causes proximal muscle pain without weakness. Inflammatory markers (eg, erythrocyte sedimentation rate)
are elevated, but muscle enzymes are normal.

(Choice G) Hypothyroidism can cause a spectrum of muscle complications ranging from asymptomatic elevation in creatine kinase to myalgias,
proximal myopathy, and rhabdomyolysis. However, most patients will have additional features of hypothyroidism (eg, cold intolerance, abnormal
reflexes).

(Choice H) Aspartate aminotransferase, alanine aminotransferase, and lactate dehydrogenase are elevated in both myocyte and hepatocyte
necrosis. However, this patient's elevated creatine kinase and normal bilirubin suggest a muscle, rather than liver, origin.

Educational objective:
Polymyositis is characterized by symmetric proximal muscle weakness. It is usually painless but may cause mild myalgias. Muscle enzyme levels
(eg, creatine kinase) are elevated. Polymyositis is similar to dermatomyositis but without skin findings.

References
Polymyositis and dermatomyositis: novel insights into the pathogenesis and potential therapeutic targets.

(http://www.ncbi.nlm.nih.gov/pubmed/26175404)
The diagnosis and classification of polymyositis.

(http://www.ncbi.nlm.nih.gov/pubmed/24461380)

Medicine Rheumatology/Orthopedics & Sports Polymyositis and dermatomyositis


Subject System Topic

https://t.me/USMLEWorldStep2CK

REVIEW

https://www.uworld.com/ClientApp/v15/apps/qbanktestinterface/index.html#/launchtest/7281878/nbme/229400736/3/1 1/1
‫ م‬8:50 2021/‫‏‬9/‫‏‬11 https://t.me/USMLEWorldStep2CK UWorld STEP2 SIM Form 1

1 Item 22 of 40
2
Mark
Question Id: 6932 Previous Next
3

4 A 34-year-old woman is brought to the emergency department after being involved in a motor vehicle collision. She was driving home from work
5 when another car hit her on the driver's side at an intersection. The patient was wearing a seat belt, but the air bags did not deploy. She appears
6 restless and diaphoretic and reports chest pain and shortness of breath. Blood pressure is 90/50 mm Hg and pulse is 118/min. Pulse oximetry is
7 88% on room air. The neck veins are distended and the trachea is deviated to the right. Breath sounds are absent on the left side. An immediate
8 intervention, directed at the patient's condition, is performed and leads to improvement in her clinical status. The intervention most likely increased
9 which of the following?
10
 A. End-expiratory pressure (16%)
11
12 B. Intravascular volume (4%)
13
C. Left ventricular contractility (8%)
14

15 D. Systemic vascular resistance (1%)


16
 E. Venous return (68%)
17
18

19
20
Incorrect 68%
21
Correct answer
E
 Answered correctly  02 secs
Time Spent  08/15/2021
Last Updated

22

23

24 Explanation
25

26
Pneumothorax
27

28 Spontaneous pneumothorax Tension pneumothorax


29 Primary: no preceding event or lung disease; often thin,
Associated Life-threatening
30 young men
features Often due to trauma or mechanical ventilation
31 Secondary: underlying lung disease (eg, COPD, CF)
32
Chest pain, dyspnea Same as spontaneous plus:
33 Signs &
↓Breath sounds, ↓chest movement Hemodynamic instability
34 symptoms
Hyperresonant to percussion Tracheal deviation away from affected side
35

36 Same as spontaneous plus:


Visceral pleural line
37 Imaging Contralateral mediastinal shift
Absent lung markings beyond pleural edge
38 Ipsilateral hemidiaphragm flattening
39 Small (≤2 cm): observation & oxygen administration Urgent needle decompression or chest tube
Management
40 Large & stable: needle aspiration or chest tube placement

CF = cystic fibrosis; COPD = chronic obstructive pulmonary disease.

This patient's symptoms and examination findings are most likely due to a left-sided tension pneumothorax (TP), a life-threatening condition that
is due to air within the pleural space and is a common complication of blunt chest trauma. TP develops when injured tissue forms a one-way
valve, allowing air to enter the pleural space but preventing it from escaping naturally. This can increase intrapleural pressure and displace
mediastinal structures, compressing them and the contralateral lung (which leads to tracheal deviation and neck vein distension [due to superior
vena cava compression]).

As a result, TP can lead to the rapid development of hypotension as the high intrathoracic pressure impedes venous return to the right atrium by
compressing the vena cava. In hemodynamically unstable patients, needle decompression (or, if available, direct emergency tube
thoracostomy) should be initiated immediately, followed by placement of a tube thoracostomy. These interventions allow the lung to re-expand
and increase venous return.

(Choice A) In intubated patients, end-expiratory pressure can be increased using positive pressure mechanical ventilation, but this can worsen
TP by further increasing intrapleural pressures and decreasing venous return.

(Choices B, C, and D) TP causes shock due to mechanical obstruction of the vena cava and a drop in right atrial volumes. Consequently, it is
likely ineffective to increase intravascular volume with intravenous fluids or systemic vascular resistance with a vasopressor medication without
decompressing the pneumothorax. Similarly, increasing cardiac contractility with inotropes does not help to resolve the underlying problem of
decreased venous return.

Educational objective:
Pneumothorax should be suspected in any patient with trauma who has decreased breath sounds and hypoxia. Hemodynamic instability suggests
that the pneumothorax is under tension. In such patients, needle decompression (or, if available, direct emergent tube thoracostomy) should be
initiated, immediately followed by placement of a tube thoracostomy.

References
Principles of diagnosis and management of traumatic pneumothorax.

(http://www.ncbi.nlm.nih.gov/pubmed/19561940)
Pneumothorax: from definition to diagnosis and treatment.

(http://www.ncbi.nlm.nih.gov/pubmed/25337391)

Medicine Pulmonary & Critical Care Pneumothorax


Subject System Topic

https://t.me/USMLEWorldStep2CK

REVIEW

https://www.uworld.com/ClientApp/v15/apps/qbanktestinterface/index.html#/launchtest/7281878/nbme/229400736/3/1 1/1
‫ م‬8:50 2021/‫‏‬9/‫‏‬11 https://t.me/USMLEWorldStep2CK UWorld STEP2 SIM Form 1

1 Item 23 of 40
2
Mark
Question Id: 6934 Previous Next
3

4 A 68-year-old man is being evaluated in the recovery unit for sudden-onset shortness of breath. He has a history of ischemic cardiomyopathy with
5 reduced ejection fraction and underwent implantation of a cardioverter-defibrillator. The patient had no complications during the procedure, but in
6 the recovery unit, he began to experience worsening dyspnea. His other medical problems include chronic obstructive pulmonary disease,
7 hypertension, hyperlipidemia, and myocardial infarction. The patient is a former smoker with a 35-pack-year history. Blood pressure is 100/60 mm
8 Hg and pulse is 110/min and regular. Pulse oximetry shows 92% on 10 L/min of oxygen. Cardiac monitor shows sinus tachycardia. A focused
9 physical examination is performed, followed by a chest x-ray, which is shown in the image below.
10
11
12
13

14

15
16

17
18

19
20

21

22

23

24
25

26

27

28

29
30
31

32

33 Which of the following is the most likely cause of this patient's current symptoms?
34

35  A. Acute chronic obstructive pulmonary disease exacerbation (0%)


36
B. Aspiration pneumonitis (0%)
37

38 C. Atelectasis (13%)
39 D. Cardiac tamponade (2%)
40
E. Flash pulmonary edema (4%)

F. Pleural effusion (7%)

G. Pulmonary embolism (5%)

 H. Tension pneumothorax (64%)

Incorrect 64%
Correct answer
H
 Answered correctly  04 secs
Time Spent  08/15/2021
Last Updated

Explanation

The chest x-ray demonstrates a left-sided tension pneumothorax (TP), a life-threatening condition that is likely a complication of this patient's
recent cardiothoracic procedure (cardioverter-defibrillator implantation). TP develops when injured tissue forms a one-way valve, allowing air to
enter the pleural space but preventing it from escaping naturally. The increased intrapleural pressure then displaces mediastinal structures
(blue arrows), compressing them and the contralateral lung, which results in compromised cardiopulmonary function.

TP is characterized by acute severe dyspnea, tachycardia, tachypnea, hypotension, and neck vein distension (due to superior vena cava
compression). If there is diagnostic uncertainty or the patient is hemodynamically stable, a chest x-ray should be obtained and would reveal a
visceral pleural line (red arrows) beyond which no pulmonary vasculature or lung parenchymal markings would be apparent. Ipsilateral
hemidiaphragm flattening may also be present. In hemodynamically unstable patients, needle decompression (or, if available, direct emergency
tube thoracostomy) should be initiated immediately, followed by placement of a tube thoracostomy.

(Choice A) An exacerbation of chronic obstructive pulmonary disease can cause wheezing and diminished breath sounds but would be
uncommon immediately following surgery and would not cause absent radiographic lung markings.

(Choice B) Aspiration pneumonitis can occur after anesthesia due to depression of protective airway reflexes, but imaging typically reveals a
pattern consistent with pulmonary edema or focal consolidation.

(Choice C) When there is atelectasis and collapse of a large portion of the lung, the mediastinal contents shift toward the atelectatic lung with
compensatory hyperinflation of the opposite lung. This patient's x-ray reveals mediastinal shifting toward the right lung, but this is due to the
visible left-sided pneumothorax.

(Choices D and G) Cardiac tamponade and pulmonary embolism can cause similar manifestations (eg, tachycardia, jugular venous distension)
and should be considered in this patient, given his cardiothoracic intervention and hospitalization. However, chest x-rays often demonstrate an
enlarged cardiac silhouette (in cardiac tamponade) or a pleural effusion and atelectasis (in pulmonary embolism), not absent lung markings.

(Choice E) On chest x-ray, flash pulmonary edema would demonstrate pulmonary vascular redistribution with cephalization of the pulmonary
veins and diffuse bilateral interstitial markings.

(Choice F) Pleural effusions frequently blunt the costophrenic angles. Mediastinal structures should not be displaced.

Educational objective:
Tension pneumothorax is a life-threatening condition due to air within the pleural space. On chest x-ray, a visceral pleural line is visible beyond
which no lung markings are apparent. Treatment includes immediate needle decompression or tube thoracostomy.

References
Pneumothorax: from definition to diagnosis and treatment.

(http://www.ncbi.nlm.nih.gov/pubmed/25337391)

Medicine Pulmonary & Critical Care Pneumothorax


Subject System Topic

https://t.me/USMLEWorldStep2CK

REVIEW

https://www.uworld.com/ClientApp/v15/apps/qbanktestinterface/index.html#/launchtest/7281878/nbme/229400736/3/1 1/1
‫ م‬8:50 2021/‫‏‬9/‫‏‬11 https://t.me/USMLEWorldStep2CK UWorld STEP2 SIM Form 1

1 Item 24 of 40
2
Mark
Question Id: 6939 Previous Next
3

4 A 54-year-old man comes to the office due to "feeling weak" for the past 3 months. The patient has had difficulty picking up small objects such as
5 coins and has noted fatigue while typing. His wife reports that he has also had periods of gait instability, including 2 episodes when he nearly fell.
6 There is no bowel or bladder dysfunction. On physical examination, blood pressure is 160/90 mm Hg and pulse is 76/min. Oxygen saturation is
7 96% on room air. There is atrophy of the hand muscles bilaterally. Babinski reflex is upgoing on the right with a hyperactive knee jerk. There is
8 no sensory loss. Laboratory studies reveal a creatine kinase level of 450 U/L. Which of the following is the most likely diagnosis?
9
 A. Amyotrophic lateral sclerosis (81%)
10
11 B. Demyelinating polyradiculoneuropathy (3%)
12
C. Inflammatory myopathy (5%)
13

14 D. Multiple sclerosis (3%)


15
E. Muscular dystrophy (3%)
16

17  F. Thoracic spinal cord compression (2%)

18

19
20 Incorrect 81%
21 Correct answer  Answered correctly  02 secs
Time Spent  05/16/2021
Last Updated
A
22

23

24
Explanation
25

26

27 Amyotrophic lateral sclerosis


28 Upper motor neuron signs
29 Spasticity
30 Hyperreflexia
31 Pathologic reflexes (eg, jaw jerk, extensor plantar reflex)
32 Lower motor neuron signs
Clinical manifestations
33 Atrophy
34 Fasciculations
35 Bulbar symptoms
36 Dysarthria
37 Dysphagia
38
Clinical evaluation
39 Diagnosis EMG
40 MRI of head & spinal cord (to exclude other causes)

Riluzole
Treatment Respiratory support (eg, NIPPV)
Nutritional support (eg, PEG tube)

EMG = electromyography; NIPPV = noninvasive positive pressure ventilation; PEG = percutaneous endoscopic gastrostomy.

Several features of this patient's presentation are concerning for amyotrophic lateral sclerosis (ALS), a progressive disease that results from
anterior horn cell (motor) degeneration. Asymmetric limb weakness is the most common initial presentation. However, because any motor neuron
can be involved, patients may report a variety of symptoms based on the affected regions:

Upper extremity: Difficulty writing, typing, or manipulating small objects (eg, coins)

Lower extremity: Gait disturbances (eg, foot drop, tripping, falling, difficulty climbing stairs)

Bulbar: Dysphagia, dysarthria

Respiratory: Orthopnea, dyspnea on exertion

Examination reveals signs of both lower motor neuron (LMN; eg, muscle atrophy, fasciculations) and upper motor neuron (UMN; eg, increased
muscle tone, hyperreflexia, upgoing Babinski) involvement. Sensation is classically preserved. Laboratory testing may be done to rule out
other potential causes of muscle weakness (eg, thyroid studies, autoantibodies, vitamin B12); in some patients, creatine kinase may be mildly
elevated (<1000 U/L), possibly due to denervation or hypermetabolism in the atrophied muscles. Nerve conduction studies, electromyography,
and neuroimaging (eg, MRI) are also typically obtained to exclude alternate diagnoses.

(Choice B) Chronic inflammatory demyelinating polyradiculoneuropathy is an immune-mediated disease that also causes progressive weakness.
However, because it affects peripheral nerves, sensory loss and LMN signs (eg, hyporeflexia) would be expected.

(Choice C) Inflammatory myopathies (eg, polymyositis) cause weakness and an elevated creatine kinase. However, because these are disorders
of the musculature, hyporeflexia may occur but UMN signs (eg, hyperreflexia, abnormal Babinski reflex) would be unexpected.

(Choice D) Multiple sclerosis (MS) can cause a variety of neurologic deficits but results from lesions in the CNS; therefore, atrophy (a LMN sign)
is typically absent. In addition, MS typically occurs in younger women (age <55) and causes relapsing and remitting symptoms.

(Choice E) Muscular dystrophies can present with weakness and elevated creatine kinase. However, the muscles, not the motor neurons, are
affected; therefore, patients typically have muscle atrophy and hyporeflexia.

(Choice F) Thoracic spinal cord compression occurs most commonly due to malignancies in the vertebral bodies compressing the spinal cord.
Patients typically have significant pain that precedes their motor deficits, sensory deficits (eg, paresthesia), and bowel/bladder dysfunction.
Involvement of the upper limbs is unusual because the hand is mostly innervated by nerves from the cervical spinal cord.

Educational objective:
Amyotrophic lateral sclerosis often presents with asymmetric limb weakness, with examination revealing both upper motor neuron signs (eg,
hyperreflexia, upgoing Babinski) and lower motor neuron signs (eg, fasciculations, atrophy).

Medicine Nervous System Amyotrophic lateral sclerosis


Subject System Topic

https://t.me/USMLEWorldStep2CK

REVIEW

https://www.uworld.com/ClientApp/v15/apps/qbanktestinterface/index.html#/launchtest/7281878/nbme/229400736/3/1 1/1
‫ م‬8:51 2021/‫‏‬9/‫‏‬11 https://t.me/USMLEWorldStep2CK UWorld STEP2 SIM Form 1

1 Item 25 of 40
2
Mark
Question Id: 6960 Previous Next
3

4 A 41-year-old man comes to the office with his wife for evaluation of "odd behavior." According to his wife, over the past several months the
5 patient has had multiple episodes in which he would not answer when spoken to; instead, he would stare directly at her with fumbling hand
6 movements. He becomes responsive after about a minute but appears confused and insists that he has not heard her talk. These episodes are
7 becoming more frequent, and other family members have also noticed him being "forgetful." The patient has no significant medical or psychiatric
8 history and takes no medications. He has a history of injection drug use. The patient does not use tobacco but drinks 2-3 alcoholic beverages
9 daily. Which of the following is the most likely cause of this patient's symptoms?
10
A. Absence seizures (26%)
11
12 B. Complex tic disorder (3%)
13
C. Korsakoff syndrome (13%)
14

15  D. Temporal lobe epilepsy (53%)


16
 E. Transient ischemic attacks (2%)
17
18

19
20
Incorrect 53%
21
Correct answer
D
 Answered correctly  02 secs
Time Spent  09/09/2021
Last Updated

22

23

24 Explanation
25

26
This patient has multiple episodes of unresponsiveness associated with fumbling hand movements, suggesting focal seizures. Focal seizures
originate in a single hemisphere (in contrast to generalized seizures, which originate globally throughout the cerebral cortex) and are
27
subcategorized by level of consciousness during the event.
28

29 Focal seizures with retained awareness (previously known as simple partial seizure): Patients remain aware and interactive. Symptoms
30 correspond to the area of brain involvement. When these seizures result in sensory (eg, olfactory hallucinations) or psychic (eg, fear)
31 symptoms, they are known as auras and may precede other types of seizures.
32
Focal seizures with impaired awareness (previously known as complex partial seizures): Patients appear awake but do not interact
33 appropriately with their environment (eg, blank stare, as in this patient). They frequently engage in repetitive, stereotyped actions known as
34 automatisms (eg, repeating words, gesturing).
35
Focal seizures with impaired awareness are the most common manifestation of temporal lobe epilepsy, the most common type of recurrent
36
seizure disorder. It is characterized by epileptiform discharges that originate within the temporal lobe; associated automatisms often include
37
hand or mouth (eg, lip smacking) movements. Many patients also describe preceding auras. Seizures typically last 30-90 seconds and are often
38
followed by postictal confusion. The diagnosis is made with electroencephalograph, which shows epileptiform discharges over the temporal
39
region.
40
(Choice A) Absence seizures typically occur in children and present with multiple "staring" episodes per day. Seizures last <30 seconds and
postictal states (eg, confusion, lethargy) do not occur.

(Choice B) Tic disorders are characterized by repetitive, stereotyped movements but are not associated with periods of unresponsiveness or
confusion. A patient with a tic disorder typically describes an uncontrollable urge to perform the tic.

(Choice C) Korsakoff syndrome is a late sequalae of thiamine deficiency and causes amnesia and confabulation. However, it typically occurs in
patients with a history of heavy alcohol use or malnutrition and is not associated with stereotyped movements or periods of unresponsiveness,
which suggest a seizure.

(Choice E) Transient ischemic attacks may cause temporary focal symptoms but are typically associated with negative (eg, arm weakness, facial
droop) rather than positive (eg, movement) symptoms. In addition, transient ischemic attacks rarely present with frequent, episodic recurrences,
and this patient's confusion after the episode is more consistent with a post-seizure state.

Educational objective:
Temporal lobe epilepsy is a common cause of focal seizures with impaired consciousness in which patients appear alert but do not interact with
their environment (eg, blank stare). Automatisms (eg, lip smacking, hand movements) and postictal confusion are common and may help
differentiate seizures from other neurologic disorders.

Medicine Nervous System Seizures


Subject System Topic

https://t.me/USMLEWorldStep2CK

REVIEW

https://www.uworld.com/ClientApp/v15/apps/qbanktestinterface/index.html#/launchtest/7281878/nbme/229400736/3/1 1/1
‫ م‬8:51 2021/‫‏‬9/‫‏‬11 https://t.me/USMLEWorldStep2CK UWorld STEP2 SIM Form 1

1 Item 26 of 40
2
Mark
Question Id: 6961 Previous Next
3

4 A 35-year-old primigravida comes to the labor and delivery unit at 36 weeks gestation with abdominal pain and blood-tinged vaginal discharge.
5 The patient had a normal glucose tolerance test at her initial prenatal visit. She was diagnosed with gestational diabetes mellitus at 27 weeks but
6 has not been compliant with diet or insulin therapy. Her family history is significant for type 2 diabetes mellitus, hypertension, coronary artery
7 disease, and stroke in both maternal and paternal relatives. Monitoring shows regular uterine contractions and a normal fetal heart rate. Blood
8 pressure is 135/86 mm Hg and pulse is 100/min. BMI is 40 kg/m2. Cervical examination shows 6 cm dilation and 100% effacement. Her
9 laboratory results on admission are as follows:
10
Hemoglobin 12.4 g/dL
11
12 Sodium 140 mEq/L
13 Potassium 3.6 mEq/L
14
Chloride 112 mEq/L
15
16 Bicarbonate 18 mEq/L
17
Blood urea nitrogen 14 mg/dL
18

19
Creatinine 0.6 mg/dL

20 Glucose 190 mg/dL


21
The patient has a spontaneous vaginal delivery of a boy. Which of the following examination findings would most likely be present in this neonate?
22

23
A. Heart murmur (20%)
24
25 B. Limb defects (9%)
26 C. Myelomeningocele (1%)
27
 D. Respiratory distress (62%)
28

29  E. Small for gestational age (5%)


30
31

32
Incorrect 62%
33 Correct answer  Answered correctly  02 secs
Time Spent  08/15/2021
Last Updated
34 D
35

36
Explanation
37

38
39

40

Women with risk factors for undiagnosed overt diabetes mellitus (eg, obesity, history of gestational diabetes mellitus) should be screened at their
initial prenatal visit. This evaluation in a high-risk population is important as maternal hyperglycemia during embryologic development can lead to
a variety of congenital anomalies, including cardiac (eg, heart murmur), limb (eg, sacral agenesis, caudal regression syndrome), and neural tube
defects (eg, myelomeningocele) (Choices A, B, and C). All women are screened for gestational diabetes mellitus at 24-28 weeks gestation due
to placental hormones causing increased insulin resistance.

Infants of diabetic mothers, regardless whether pre-gestational or gestational, are at increased risk of respiratory distress syndrome (RDS),
preterm delivery, and fetal macrosomia. Maternal hyperglycemia, with resultant fetal hyperinsulinemia, delays cellular maturation.
Consequently, immature pneumocytes in the fetal lungs are unable to produce surfactant, resulting in RDS.

(Choice E) Patients with pre-gestational and gestational diabetes mellitus typically deliver large for gestational age infants due to fetal
hyperinsulinemia (anabolic growth). Pre-gestational diabetes mellitus can also increase the risk for small for gestational age newborns because
long term or poorly controlled diabetes mellitus increases vascular resistance that affects the placenta and restricts fetal growth.

Educational objective:
All forms of poorly controlled diabetes (eg, pre-gestational, gestational) in pregnancy have an increased risk of fetal lung immaturity, preterm
delivery, and macrosomia. Poorly controlled diabetes mellitus during the first trimester (eg, pregestational) has additional risks of cardiac and
central nervous system defects.

References
Delayed appearance of pulmonary maturation markers is associated with poor glucose control in diabetic pregnancies.

(http://www.ncbi.nlm.nih.gov/pubmed/9642613)
A comparison of amniotic fluid fetal pulmonary phospholipids in normal and diabetic pregnancy.

(http://www.ncbi.nlm.nih.gov/pubmed/11967485)

Obstetrics & Gynecology Pregnancy, Childbirth & Puerperium Gestational diabetes


Subject System Topic

https://t.me/USMLEWorldStep2CK

REVIEW

https://www.uworld.com/ClientApp/v15/apps/qbanktestinterface/index.html#/launchtest/7281878/nbme/229400736/3/1 1/1
‫ م‬8:51 2021/‫‏‬9/‫‏‬11 https://t.me/USMLEWorldStep2CK UWorld STEP2 SIM Form 1

1 Item 27 of 40
2
Mark
Question Id: 6962 Previous Next
3

4 A 27-year-old African American woman comes to the office due to left-sided chest pain, nonproductive cough, and dyspnea on exertion for 7
5 days. The chest pain increases with deep breaths and cough. The patient also has had 3 months of bilateral knee pain that is relieved by
6 occasional naproxen use. She reports fatigue and subjective fever on review of systems. She lives in Ohio and has not traveled outside of the
7 United States. She is sexually active. The patient has no other medical problems and takes no other medications. She does not use tobacco,
8 alcohol, or illicit drugs. Her temperature is 38.1 C (100.5 F), blood pressure is 140/90 mm Hg, and pulse is 90/min. Pulse oximetry is 95% on
9 room air. BMI is 24 kg/m2. There is no jugular venous distension. Pulmonary examination reveals decreased breath sounds in the left lower lung
10 field. Heart sounds are muffled. The abdomen is soft and nontender. There is no peripheral edema. Both knees appear swollen, but there is no
11 erythema. Knee range of motion is not restricted. Laboratory results are as follows:
12
Complete blood count
13

14
Hemoglobin 9.6 g/dL

15 Platelets 90,000/mm3
16
Leukocytes 3,800/mm3
17
18
Serum chemistry
19
20 Blood urea nitrogen 24 mg/dL
21 Creatinine 1.9 mg/dL
22

23 Chest x-ray reveals left-sided pleural effusion and enlarged cardiac silhouette. Which of the following is the most likely diagnosis in this patient?
24
25 A. Disseminated gonococcal disease (2%)
26
B. Histoplasmosis (12%)
27

28 C. Reactive arthritis (1%)


29 D. Rheumatoid arthritis (1%)
30
 E. Sarcoidosis (15%)
31

32  F. Systemic lupus erythematosus (66%)


33
G. Tuberculosis (0%)
34

35

36

37 Incorrect 66% 04 secs 04/05/2021


38
Correct answer
F
 Answered correctly  Time Spent  Last Updated

39

40

Explanation

Manifestations of systemic lupus erythematosus


Constitutional: fever, fatigue & weight loss
Symmetric, migratory arthritis
Clinical Skin: butterfly rash & photosensitivity
symptoms Serositis: pleurisy, pericarditis & peritonitis
Thromboembolic events (due to vasculitis & antiphospholipid antibodies)
Neurologic: cognitive dysfunction & seizures

Hemolytic anemia, thrombocytopenia & leukopenia


Hypocomplementemia (C3 & C4)
Laboratory Antibodies:
findings Antinuclear antibodies (sensitive)
Anti-dsDNA & anti-Smith (specific)
Renal involvement: proteinuria & elevated creatinine

dsDNA = double-stranded DNA.

This patient has pleuritic chest pain with a pleural effusion and likely pericardial effusion (muffled heart sounds and enlarged cardiac silhouette)
suggesting serositis, as well as symmetric arthritis, cytopenias, and an elevated creatinine with hypertension (suggesting possible kidney
injury). A systemic inflammatory condition is clearly present. Given this patient's young age and African American heritage, systemic lupus
erythematosus (SLE) is the most likely diagnosis. SLE is diagnosed by the presence of a combination of clinical and serologic markers
(antinuclear antibody, anti-SM antibody, anti-dsDNA antibody, or antiphospholipid antibody).

(Choice A) Disseminated gonococcal infection presents with a migratory polyarthritis, tenosynovitis, and pustular or vesiculopustular skin
lesions. It would not explain the serositis, effusion, or cytopenias.

(Choices B and G) Histoplasmosis (present in Ohio) primarily infects the lungs but can also cause disseminated disease, mostly in
immunocompromised individuals. Hepatosplenomegaly is common, whereas elevated creatinine and 3 months of arthritis are atypical. Similarly,
disseminated tuberculosis (TB) can involve multiple organ systems but would be unusual in an immunocompetent individual; night sweats and
weight loss would be common, and elevated creatinine would not be typical. The constellation of findings in this patient is much more likely to
represent SLE than TB or histoplasmosis.

(Choice C) Reactive arthritis typically presents 1-4 weeks after an enteric or chlamydial infection. Manifestations include an inflammatory
asymmetric arthritis often with conjunctivitis or uveitis. Serositis, pleural effusion, or renal injury would be uncommon.

(Choice D) The knees may be involved in rheumatoid arthritis, but early involvement is more commonly seen in the small joints of the hands and
feet. In addition, serositis and pancytopenia are not typically seen in rheumatoid arthritis.

(Choice E) Sarcoidosis is a common autoimmune disease in young African American women but typically presents with pulmonary (hilar
lymphadenopathy, pulmonary reticular opacities), skin, and ocular (uveitis) manifestations.

Educational objective:
Systemic lupus erythematosus is a multisystem inflammatory condition that commonly causes serositis and a symmetric polyarthritis (knees and
hands most commonly). It is diagnosed by the presence of a combination of clinical and serologic markers.

Medicine Rheumatology/Orthopedics & Sports SLE


Subject System Topic

https://t.me/USMLEWorldStep2CK

REVIEW

https://www.uworld.com/ClientApp/v15/apps/qbanktestinterface/index.html#/launchtest/7281878/nbme/229400736/3/1 1/1
‫ م‬8:51 2021/‫‏‬9/‫‏‬11 https://t.me/USMLEWorldStep2CK UWorld STEP2 SIM Form 1

1 Item 28 of 40
2
Mark
Question Id: 6963 Previous Next
3

4 A 53-year-old man is brought to the emergency department from a local restaurant after he had a sudden onset of retrosternal burning and
5 dizziness. The patient has a history of hyperlipidemia and hypertension that is well-controlled. He has no allergies to medications. On physical
6 examination, he appears uncomfortable but is interactive and speaking in full sentences, with no respiratory distress and no chest pain. Blood
7 pressure is 100/60 mm Hg and pulse is 210/min. Pulse oximetry is 95% on room air. Cardiac examination is notable for significant tachycardia
8 with no audible murmurs; lung examination is unremarkable. His ECG is shown below:
9

10
11
12
13

14

15
16

17
18

19
20
Which of the following medications is most likely to improve this patient’s condition?
21

22
A. Adenosine (25%)
23

24
B. Digoxin (1%)

25 C. Amiodarone (66%)

26
D. Verapamil (2%)
27

28  E. Metoprolol (2%)
29
30
31
Incorrect 66%
32 Correct answer  Answered correctly  05 secs
Time Spent  07/30/2021
Last Updated
33 C
34

35
Explanation
36

37
This patient’s ECG demonstrates a regular, wide-complex tachycardia consistent with monomorphic ventricular tachycardia. The most
38
important first step in the evaluation of this patient is to determine if he is stable or unstable (eg, ongoing ischemic chest pain, hypotension, shock,
39
pulmonary edema, mental status changes).
40
Although this patient is currently stable, his condition could rapidly become unstable. A recommended initial treatment option for stable ventricular
tachycardia is a loading dose of intravenous amiodarone. After his heart rhythm stabilizes, further evaluation is required to determine the
underlying cause of his ventricular tachycardia. Possible etiologies include ischemia, electrolyte abnormalities, hypoxia, drug effects, and heart
failure.

(Choice A) Adenosine is used in the treatment of supraventricular tachycardia. In some cases, supraventricular tachycardia with abberancy may
appear similar to ventricular tachycardia on ECG; however, especially in a patient with risk factors for coronary artery disease (eg, hypertension,
hyperlipidemia), a wide-complex tachycardia should be considered to be ventricular in origin unless it is definitely interpreted as supraventricular
tachycardia with aberrancy (eg, based on comparison with prior ECGs).

(Choice B) Digoxin is sometimes used in the treatment of supraventricular tachyarrhythmias such as atrial fibrillation.

(Choices D and E) Verapamil and metoprolol may be used to treat supraventricular tachycardia if adenosine is not effective in terminating the
rhythm, but they would not be particularly effective in treating ventricular tachycardia.

Educational objective:
A wide-complex tachycardia should be considered to be ventricular in origin unless it is definitively interpreted as supraventricular tachycardia with
aberrancy. In stable patients, a treatment option consists of a loading dose of amiodarone.

Medicine Cardiovascular System Ventricular tachycardia


Subject System Topic

https://t.me/USMLEWorldStep2CK

REVIEW

https://www.uworld.com/ClientApp/v15/apps/qbanktestinterface/index.html#/launchtest/7281878/nbme/229400736/3/1 1/1
‫ م‬8:51 2021/‫‏‬9/‫‏‬11 https://t.me/USMLEWorldStep2CK UWorld STEP2 SIM Form 1

1 Item 29 of 40
2
Mark
Question Id: 20525 Previous Next
3

4 A 6-year-old boy is brought to the urgent care clinic by his mother. The patient appears restless and flushed and coughs frequently. Other than a
5 temperature of 38.3 C (101 F), vital signs are stable. The family recently immigrated from China. The mother speaks Mandarin and a few words
6 of English. The boy speaks some English. Due to technical difficulties, the clinic's telephone interpretation service is inoperable. The physician is
7 a native English speaker but can speak several words of Mandarin. The clinic's medical assistant is a native, fluent Mandarin speaker. Which of
8 the following represents the physician's most appropriate action in caring for this patient?
9
A. Arrange for the patient to be sent to the emergency department (14%)
10
11  B. Ask the Mandarin-speaking medical assistant to serve as interpreter (72%)
12
C. Conduct a focused physical examination only (2%)
13

14 D. Interview the patient and his mother in Mandarin (1%)


15
 E. Obtain a basic history in English directly from the patient (3%)
16

17
F. Reschedule the visit after the interpretation service is restored (6%)

18

19
20 Incorrect 72%
21 Correct answer  Answered correctly  02 secs
Time Spent  07/05/2021
Last Updated
B
22

23

24
Explanation
25

26 When communicating with patients who have limited English proficiency (LEP), providers should use qualified interpreters, either in person or
27 over the phone, who are trained in interpreter skills, ethics, and medical terminology. In this case, communication with the patient's mother, who
28 has LEP, is needed to gather a detailed history, make an informed diagnosis, and care for the patient. Conducting a physical examination only or
29 relying on the child for the history is not sufficient (Choices C and E).
30
Ad hoc interpretation, referring to temporary interpretation by nonqualified individuals (eg, bilingual or partially fluent family members or medical
31
staff), generally is not recommended due to error risk. However, ad hoc interpretation may be necessary in limited situations, including this
32
case, which meet the following conditions:
33
34
the patient has an urgent medical need (ie, presenting with acute symptoms in urgent care setting) and appears ill (eg, restless, coughing,
flushed)
35

36 qualified interpreter services are not reasonably accessible (ie, system is inoperable)
37
delaying care (eg, rescheduling the visit after services are restored) to access interpreter services may result in preventable patient harm
38
(eg, undiagnosed illness leading to rapidly worsening symptoms) (Choice F).
39

40
Therefore, the most fluent staff member should provide ad hoc interpretation in this encounter. The medical assistant, who is a native Mandarin
speaker, is preferred to serve as interpreter over the physician, who has more limited Mandarin fluency (Choice D). Minor children (age <18)
should not serve as interpreters except in true emergencies.

(Choice A) Although this patient should be sent to the emergency department if further assessment determines he is seriously ill, he currently
appears medically stable (ie, normal vital signs). Therefore, he can be evaluated first in the urgent care setting, and arranged for subsequent
outpatient follow-up with qualified interpretation services.

Educational objective:
Ad hoc (ie, informal) interpretation by nonqualified individuals (eg, family members, medical staff, providers) may be necessary in limited
situations, including when a patient has an urgent medical need, interpreter services are not reasonably accessible, and delaying care to access
interpreter services could cause patient harm.

Medicine Social Sciences (Ethics/Legal/Professional) Interpreters


Subject System Topic

https://t.me/USMLEWorldStep2CK

REVIEW

https://www.uworld.com/ClientApp/v15/apps/qbanktestinterface/index.html#/launchtest/7281878/nbme/229400736/3/1 1/1
‫ م‬8:52 2021/‫‏‬9/‫‏‬11 https://t.me/USMLEWorldStep2CK UWorld STEP2 SIM Form 1

1 Item 30 of 40
2
Mark
Question Id: 6966 Previous Next
3

4 A 61-year-old man comes to the office due to episodic abdominal pain, bloating, and chronic diarrhea. The patient has bulky, foul-smelling bowel
5 movements 3 to 4 times a day. He also reports a 7-kg (15-lb) weight loss over the past 2 months. His vital signs are within normal limits. On
6 physical examination, there is mild epigastric tenderness with no guarding or rebound tenderness and normal bowel sounds. Laboratory studies
7 show normal leukocyte count, fasting glucose, liver aminotransferases, and serum lipase. CT scan of the abdomen without contrast is shown
8 below.
9

10
11
12
13

14

15
16

17
18

19
20

21

22

23

24
25

26

27

28

29
30
31

32
Which of the following additional history or findings is most likely to be present in this patient?

33
A. Antiendomysial antibody (3%)
34

35  B. Chronic alcohol consumption (76%)


36
C. Family history of pancreatic cancer (15%)
37

38
D. History of hereditary anemia (0%)

39 E. Stool positive for Giardia antigen (3%)



40
F. Symptom association with dairy products (2%)

Incorrect 76% 07 secs 08/23/2021


Correct answer
B
 Answered correctly  Time Spent  Last Updated

Explanation

This CT scan demonstrates multiple calcifications within the pancreas, which are characteristic of chronic pancreatitis. Dilation of the pancreatic
duct and pancreatic enlargement are also common findings on abdominal CT scan, although advanced disease results in pancreatic atrophy.

This patient's history of bloating and frequent, bulky, foul-smelling bowel movements is consistent with steatorrhea, a manifestation of chronic
pancreatitis caused by impaired secretion of digestive enzymes from the pancreas (eg, normal serum lipase). Malabsorption of micro- and
macronutrients can lead to weight loss and fat-soluble vitamin deficiencies (eg, A, D, E, K). Epigastric abdominal pain and diabetes mellitus
(due to pancreatic endocrine insufficiency) are additional manifestations, though the latter typically occurs with advanced disease. Although
alcohol use and gallstone disease are the 2 most common causes of acute pancreatitis, chronic pancreatitis is usually due to alcohol use.

Lipase levels are not always elevated with chronic pancreatitis, and alcohol use may not always cause liver enzyme elevation; therefore, this
patient's normal laboratory values are not inconsistent with the diagnosis.

(Choice A) Antiendomysial antibodies may be detected in patients with gluten enteropathy (celiac disease), which can present with abdominal
discomfort, malabsorption, and nutrient/vitamin deficiencies; however, pancreatic calcifications are not characteristic.

(Choice C) Certain individuals may have a familial predisposition to developing pancreatic cancer. However, pancreatic cancer typically presents
with obstructive jaundice (conjugated hyperbilirubinemia), epigastric pain, weight loss, and a pancreatic mass on abdominal CT scan.

(Choice D) Certain hereditary anemias (eg, sickle cell, hereditary spherocytosis) may lead to chronic hemolysis, resulting in increased pigmented
gallstone formation. Gallstones can obstruct the ampulla of Vater causing acute pancreatitis, which typically presents with epigastric abdominal
pain, nausea/vomiting, elevated serum lipase, and pancreatic edema/enlargement on CT scan.

(Choice E) Giardia lamblia is a parasitic infection that typically affects high-risk individuals (eg, hikers drinking from streams, child care center
attendees). Patients can develop abdominal discomfort, diarrhea, and weight loss due to malabsorption; however, pancreatic calcifications are not
characteristic.

(Choice F) Lactase deficiency typically presents with abdominal discomfort and osmotic diarrhea after consumption of dairy products.
Steatorrhea, malabsorption, and pancreatic calcifications make chronic pancreatitis much more likely.

Educational objective:
Pancreatic calcifications on abdominal CT scan are characteristic of chronic pancreatitis, which is most commonly caused by chronic alcoholism.
Typical clinical features include steatorrhea, malabsorption, and epigastric abdominal pain.

References
Chronic pancreatitis.

(http://www.ncbi.nlm.nih.gov/pubmed/18092710)

Medicine Gastrointestinal & Nutrition Chronic pancreatitis


Subject System Topic

https://t.me/USMLEWorldStep2CK

REVIEW

https://www.uworld.com/ClientApp/v15/apps/qbanktestinterface/index.html#/launchtest/7281878/nbme/229400736/3/1 1/1
‫ م‬8:52 2021/‫‏‬9/‫‏‬11 https://t.me/USMLEWorldStep2CK UWorld STEP2 SIM Form 1

1 Item 31 of 40
2
Mark
Question Id: 6974 Previous Next
3

4 A 53-year-old man comes to the office due to frequent discomfort in his right foot that limits his daily activities. His medical history is significant for
5 obesity, type 2 diabetes mellitus, hypertension, gout, and obstructive sleep apnea. The patient has a history of cocaine abuse but has not used
6 any illicit drugs in the past 6 months. However, he does acknowledge poor adherence with his prescribed medication regimen. Temperature is
7 37.1 C (98.8 F), blood pressure is 128/76 mm Hg, pulse is 82/min, and respirations are 16/min. On physical examination, the skin on the right foot
8 is thin, shiny, and devoid of hair. There is a small, shallow ulcer on the medial aspect of the right great toe that is without purulent drainage.
9 Which of the following is most likely to diagnose the cause of this patient's foot discomfort?
10
 A. Ankle-brachial index (86%)
11
12 B. Electromyography (4%)
13
C. Serum uric acid level (1%)
14

15 D. Ulcer biopsy (4%)


16
 E. Ulcer swab for culture (3%)
17
18
F. Urine toxicology screen (0%)

19
20

21 Incorrect 86% 03 secs 07/13/2021


22 Correct answer
A
 Answered correctly  Time Spent  Last Updated
23

24
25
Explanation
26

27

28 Ankle-brachial index
29 ABI = SBP of dorsalis pedis or posterior tibial artery ÷ SBP of brachial artery
30
≤0.9 Diagnostic of peripheral artery disease
31

32 0.91-1.3 Normal
33
>1.3 Suggests calcified & uncompressible vessels*
34

35 *Other testing should be considered.

36 ABI = ankle-brachial index; SBP = systolic blood pressure.

37
This patient has multiple risk factors for atherosclerosis including diabetes, hypertension, and obesity. The thin, shiny, hairless skin on his
38
extremities and activity-limiting foot pain (consistent with claudication) suggest peripheral artery disease (PAD). Distal ulcers are common in
39
PAD because poor blood flow precludes proper healing.
40
The ankle-brachial index (ABI) is simple, noninvasive, and diagnostically accurate, making it the initial test of choice in patients with suspected
PAD. It is the ratio of the systolic blood pressure in either the posterior tibial or dorsal pedis artery to the systolic blood pressure in the brachial
artery. A normal ABI is 0.91-1.3; values ≤0.90 are typically diagnostic of PAD, whereas values ≤0.4 indicate severe ischemia. Patients with
diabetes are more likely to have calcified, noncompressible vessels that yield falsely elevated ABI results; however, diabetes does not preclude
testing.

(Choice B) Electromyography can be used to evaluate for neuropathy or myopathy. Poor sensation due to diabetic peripheral neuropathy could
be contributing to this patient's nonhealing ulcer, but diabetic neuropathy can usually be diagnosed on physical examination (eg, symmetric
polyneuropathy with loss of vibratory and/or pain/temperature sensation). Unilateral symptoms suggest PAD rather than diabetic neuropathy
alone.

(Choice C) Uric acid levels are sometimes used to evaluate gout; however, the levels have poor sensitivity and specificity. Gout can cause great
toe pain and gouty tophi if severe, but this patient's physical examination findings and atherosclerotic risk factors are more suggestive of PAD.

(Choice D) Nonhealing skin ulcers can sometimes represent malignancy and warrant biopsy; however, this patient's toe ulcer is highly consistent
with PAD. Should ABI workup be negative and wound care measures fail to resolve the ulcer, skin biopsy may be indicated.

(Choice E) Swabbing of a skin ulcer in the absence of purulent drainage is not helpful because it generally yields growth of normal skin flora.
Poor blood flow, rather than infection, is typically the major hindrance to healing with PAD ulcers.

(Choice F) Cocaine abuse can contribute to peripheral ischemia, but ABI testing is most appropriate to evaluate for suspected PAD in this patient.

Educational objective:
Peripheral arterial disease (PAD) should be suspected in patients with atherosclerotic risk factors and appropriate signs and symptoms (eg, pain;
nonhealing ulcers; skin that is shiny, thin, and hairless). The ankle-brachial index is simple, noninvasive, and diagnostically accurate, making it the
initial test of choice in patients with suspected PAD.

References
Use of ankle-brachial pressure index to predict cardiovascular events and death: a cohort study.

(http://www.ncbi.nlm.nih.gov/pubmed/8973232)

Medicine Cardiovascular System Peripheral vascular disease


Subject System Topic

https://t.me/USMLEWorldStep2CK

REVIEW

https://www.uworld.com/ClientApp/v15/apps/qbanktestinterface/index.html#/launchtest/7281878/nbme/229400736/3/1 1/1
‫ م‬8:52 2021/‫‏‬9/‫‏‬11 https://t.me/USMLEWorldStep2CK UWorld STEP2 SIM Form 1

1 Item 32 of 40
2
Mark
Question Id: 6977 Previous Next
3

4 A 46-year-old woman comes to the office to follow up hypertension. For the last year, her blood pressure has been steadily rising despite dietary
5 modification and exercise. The patient has occasional headaches but otherwise feels well. Blood pressure in the office is 154/94 mm Hg. She is
6 started on chlorthalidone. Two weeks later, the patient is seen in the emergency department due to repeated falls and severe leg cramps. On
7 examination, she has 4/5 muscle strength in her lower extremities, symmetric deep tendon reflexes, and a flexor plantar response bilaterally.
8 There is no sensory loss. After initial stabilization, this patient's symptoms are most likely to improve with which of the following long-term
9 therapies?
10
A. Dexamethasone (7%)
11
12 B. Intravenous immune globulin (3%)
13
C. Labetalol (11%)
14

15 D. Metyrapone (5%)
16
 E. Spironolactone (72%)
17
18

19
20 72%
21
Correct  Answered correctly  02 secs
Time Spent  03/18/2021
Last Updated

22

23

24 Explanation
25

26

27

28

29
30
31

32

33
34

35

36

37

38
39

40 This patient's weakness and leg cramps after initiation of a thiazide diuretic suggest significant hypokalemia. These symptoms in the setting of
persistent hypertension are consistent with primary hyperaldosteronism (PH). Serum potassium in PH may be normal at baseline but can fall
rapidly after initiation of diuretics. Other findings may include metabolic alkalosis and mild hypernatremia (143-147 mEq/L). The best screening
test for PH is an early-morning plasma aldosterone concentration (PAC) to plasma renin activity (PRA) ratio; elevated aldosterone (>15 ng/dL) with
a PAC/PRA ratio >20 suggests PH.

Patients with PH should undergo CT imaging of the adrenal glands to identify whether it is due to bilateral adrenal hyperplasia (50%-60%) or an
aldosterone-producing adrenal adenoma (40%-50%). If PH is due to a unilateral adenoma, adrenalectomy can be curative. Patients with bilateral
adrenal hyperplasia should be managed with aldosterone antagonists (eg, spironolactone, eplerenone).

(Choice A) This patient's PH is not related to an autoimmune condition, and glucocorticoids are unlikely to be of benefit.

(Choice B) Intravenous immune globulin is used in the treatment of Guillain-Barré syndrome, but this is unlikely to be the cause of this patient's
symptoms given the symmetric deep tendon reflexes and rather mild weakness.

(Choice C) Labetalol may rapidly bring this patient's blood pressure under control, but it will not be effective in resolving her hypokalemia, which
is causing muscle cramps and weakness.

(Choice D) Metyrapone is an inhibitor of cortisol synthesis. It is used in patients with Cushing syndrome who are waiting for definitive surgical
treatment.

Educational objective:
Primary hyperaldosteronism is most often due to bilateral adrenal hyperplasia or an aldosterone-producing adrenal adenoma. Adrenalectomy is
recommended for an adenoma, whereas aldosterone antagonists are recommended for bilateral hyperplasia.

References
Outcomes of drug-based and surgical treatments for primary aldosteronism.

(http://www.ncbi.nlm.nih.gov/pubmed/25908468)

Medicine Endocrine, Diabetes & Metabolism Hyperaldosteronism


Subject System Topic

https://t.me/USMLEWorldStep2CK

REVIEW

https://www.uworld.com/ClientApp/v15/apps/qbanktestinterface/index.html#/launchtest/7281878/nbme/229400736/3/1 1/1
‫ م‬8:52 2021/‫‏‬9/‫‏‬11 https://t.me/USMLEWorldStep2CK UWorld STEP2 SIM Form 1

1 Item 33 of 40
2
Mark
Question Id: 6994 Previous Next
3

4 A 72-year-old man is brought to the emergency department with a 2-day history of progressive lethargy. According to his wife, he has not been
5 eating much recently and has had 2 episodes of vomiting over the last 12 hours. His wife also reports that in the last 3 weeks he has been
6 experiencing pain in his right shoulder radiating to the medial aspect of the right upper arm and axilla. Medical history is significant for chronic
7 obstructive pulmonary disease, hypertension, and peptic ulcer disease. The patient's current medications include inhaled fluticasone, inhaled
8 albuterol/ipratropium, candesartan, and hydrochlorothiazide. Blood pressure is 110/80 mm Hg and pulse is 120/min and regular. On physical
9 examination, he is severely lethargic and responds only to strong, painful stimuli. Laboratory results are as follows:
10
Complete blood count
11
12 Hemoglobin 14.5 g/dL
13

14 Serum chemistry
15 Sodium 142 mEq/L
16
Potassium 4.4 mEq/L
17
18 Blood urea nitrogen 31 mg/dL
19
Creatinine 1.6 mg/dL
20

21
14.1
Calcium
mg/dL
22

23 Glucose 105 mg/dL


24
25 Blood, plasma, and serum
26
Phosphorus, inorganic,
27 2.8 mg/dL
serum
28

29
Which of the following is the most likely cause of this patient's current condition?
30
31 A. Adrenal insufficiency (0%)
32
B. Hyperthyroidism (0%)
33
34  C. Malignancy (62%)
35
D. Paget disease of bone (3%)
36

37  E. Primary hyperparathyroidism (28%)


38 F. Thiazide diuretic use (5%)
39

40

Incorrect 62%
Correct answer  Answered correctly  03 secs
Time Spent  05/30/2021
Last Updated
C

Explanation

Hypercalcemia of malignancy
PTHrP secretion (80% of cases): Squamous cell, renal/bladder, gynecological malignancy,
breast
Causes 1,25(OH) vitamin D: Lymphoma
Bone metastasis: Breast, multiple myeloma, lymphoma

Mild to moderate: Nausea, fatigue, constipation, ileus


Severe (>12 mg/dL): Confusion, weakness, delirium, coma
Clinical features
Tumor related: Anorexia/cachexia, local pain, mass effects

Serum calcium often severely elevated


Elevated PTHrP, suppressed PTH
Diagnosis
Localization, identification of primary tumor

PTH = parathyroid hormone; PTHrP = parathyroid hormone-related peptide.

This patient has severe hypercalcemia with significant neurologic symptoms (lethargy, vomiting). Hypercalcemia of this degree is most likely
explained by hypercalcemia of malignancy (HoM) due to an undiagnosed lung cancer, especially in light of this patient's localized pain and
underlying chronic obstructive pulmonary disease. HoM can cause very rapid rises in calcium, producing more pronounced symptoms than
chronic hypercalcemia of similar severity. It is especially common in non-small cell lung cancer and suggests advanced disease with a poor
prognosis. HoM is usually due to secretion of parathyroid hormone-related protein (PTHrP), which acts on parathyroid hormone (PTH)
receptors, leading to hypophosphatemia in addition to hypercalcemia. Other potential mechanisms of HoM include overproduction of 1,25-
dihydroxyvitamin D, bone metastasis, or, rarely, secretion of PTH or interleukin-6.

Causes of hypercalcemia can be categorized as parathyroid hormone (PTH)-dependent (high-normal or increased PTH) or independent
(low/suppressed PTH). HoM due to production of PTHrP suppresses PTH. PTHrP and PTH can both be directly assayed; an elevated PTHrP
along with a suppressed PTH would be diagnostic for HoM.

(Choice A) Acute adrenal insufficiency causes severe hypotension associated with weakness, abdominal pain, anorexia, and mental status
changes. Laboratory findings would include hyponatremia and hyperkalemia. Calcium levels are usually normal.

(Choices B, E, and F) Hyperthyroidism can cause hypercalcemia due to an increase in bone turnover. Thiazide diuretics can cause
hypercalcemia due to decreased renal excretion of calcium. Primary hyperparathyroidism causes PTH-dependent hypercalcemia and is usually
due to an autonomously functioning parathyroid adenoma or parathyroid hyperplasia. However, the hypercalcemia in these conditions is typically
modest (<12 mg/dL). Patients are usually asymptomatic or have only mild, nonspecific symptoms (eg, constipation).

(Choice D) Serum calcium is usually normal in patients with Paget disease, although it may be mildly elevated in immobilized patients due to
increased bone resorption.

Educational objective:
Hypercalcemia of malignancy (HoM) can cause rapid rises in calcium, producing severe neurologic symptoms. It is usually due to secretion of
parathyroid hormone-related protein and is especially common in non-small cell lung cancer. Serum calcium levels are generally much higher in
HoM than in primary hyperparathyroidism.

References
Clinical practice. Hypercalcemia associated with cancer.

(http://www.ncbi.nlm.nih.gov/pubmed/15673803)

Medicine Endocrine, Diabetes & Metabolism Hypercalcemia


Subject System Topic

https://t.me/USMLEWorldStep2CK

REVIEW

https://www.uworld.com/ClientApp/v15/apps/qbanktestinterface/index.html#/launchtest/7281878/nbme/229400736/3/1 1/1
‫ م‬8:52 2021/‫‏‬9/‫‏‬11 https://t.me/USMLEWorldStep2CK UWorld STEP2 SIM Form 1

1 Item 34 of 40
2
Mark
Question Id: 6995 Previous Next
3

4 A 45-year-old woman comes to the emergency department due to sudden onset of abdominal pain. The patient had just finished dinner when she
5 started to experience severe upper abdominal pain associated with nausea and vomiting. The pain radiates to her right shoulder. The patient
6 recently had 2 similar episodes with less intense pain that resolved spontaneously within several hours. She has a history of hyperlipidemia and
7 type 2 diabetes mellitus. The patient lost 15 kg (33.1 lb) after undergoing Roux-en-Y gastric bypass surgery 5 months ago. She does not use
8 tobacco, alcohol, or illicit drugs. Temperature is 37.8 C (100 F), blood pressure is 140/92 mm Hg, pulse is 112/min, and respirations are 18/min.
9 Heart sounds are normal without murmurs and lungs are clear to auscultation. Abdominal examination shows a well-healed midline surgical scar.
10 Right upper quadrant tenderness is present. Bowel sounds are active. Stool is guaiac negative. Which of the following is the most likely cause of
11 this patient's current condition?
12
A. Leakage from the anastomosis site (8%)
13

14 B. Mesenteric ischemia (11%)


15
C. Rapid gastric emptying (37%)
16

17  D. Rapid weight loss (37%)


18 E. Reflux of gastric contents (5%)

19
20

21
Incorrect 37%
22

23
Correct answer
D
 Answered correctly  03 secs
Time Spent  04/06/2021
Last Updated

24
25

26 Explanation

27

28

29
30
31

32

33
34

35

36

37

38
39

40

This patient with severe postprandial right upper quadrant (RUQ) pain radiating to the right shoulder following Roux-en-Y gastric bypass (RYGB)
surgery most likely has gallstone disease with possible cholecystitis.

Patients who undergo RYGB have up to a 30%-40% chance of developing symptomatic gallstones. This is due to resulting rapid weight loss,
which promotes gallstone formation, likely from increased bile concentrations of mucin and calcium. Moreover, patients with weight loss-
associated gallstones are significantly more likely to be symptomatic (eg, biliary colic) than those with gallstones due to other causes.
Consequently, prophylactic ursodeoxycholic acid is often administered for 6 months postoperatively to reduce the risk of gallstone development.
Some surgeons recommend that patients with symptomatic gallstones prior to RYGB undergo cholecystectomy at the time of bypass surgery.

(Choice A) Anastomotic leaks are among the most dangerous complications of RYGB, but they typically occur within days, not months, of surgery
and are characterized by fever, tachycardia, tachypnea, and abdominal pain.

(Choice B) Bowel ischemia usually presents acutely following RYGB. It can predispose to late complications such as stomal stenosis, which
would cause bowel obstruction, and marginal ulcers, which can cause perforation or gastrointestinal bleeding. Chronic mesenteric ischemia (due
to nonsurgical causes) may present with postprandial abdominal pain, nausea, and weight loss, but RUQ pain is not typical.

(Choice C) Rapid gastric emptying (dumping syndrome) is a common complication of RYGB, but the most common symptoms include crampy
abdominal pain, vomiting, diarrhea, and vasomotor symptoms (eg, flushing, palpitations).

(Choice E) Reflux of gastric contents may occur due to postoperative stomal stenosis or as a complication of other bariatric surgeries. However,
the most common symptoms are heartburn, aspiration, and cough. RUQ tenderness is not typical.

Educational objective:
Roux-en-Y gastric bypass is a significant risk factor for symptomatic gallstones due to the rapid weight loss that promotes their formation.

References
Complications of bariatric surgery: presentation and emergency management--a review.

(http://www.ncbi.nlm.nih.gov/pubmed/19344551)
Bariatric surgery: a primer.

(http://www.ncbi.nlm.nih.gov/pubmed/20841586)
Early surgical complications after gastric by-pass: a literature review.

(http://www.ncbi.nlm.nih.gov/pubmed/25861076)

Surgery Gastrointestinal & Nutrition Gallstone disease


Subject System Topic

https://t.me/USMLEWorldStep2CK

REVIEW

https://www.uworld.com/ClientApp/v15/apps/qbanktestinterface/index.html#/launchtest/7281878/nbme/229400736/3/1 1/1
‫ م‬8:53 2021/‫‏‬9/‫‏‬11 https://t.me/USMLEWorldStep2CK UWorld STEP2 SIM Form 1

1 Item 35 of 40
2
Mark
Question Id: 6998 Previous Next
3

4 A 34-year-old man comes to the office due to reddish-brown urine for the past 2 days. The patient says the urine is uniformly discolored
5 throughout micturition, and he has had no blood clots. The patient has a 20-pack-year smoking history. Blood pressure is 130/80 mm Hg and
6 pulse is 80/min. BMI is 32 kg/m2. Oropharyngeal mucosa and tonsils are normal. The lung fields are clear to auscultation, and heart sounds are
7 normal. The abdomen is soft with no organomegaly. No costovertebral angle tenderness is present. External genitalia are normal, and the
8 prostate is not enlarged. Urine microscopy reveals numerous red blood cells, many of which are dysmorphic; white blood cells are 1-2/hpf.
9 Serum creatinine is normal. What is the most appropriate next step in management of this patient?
10
A. CT scan of the abdomen without contrast (31%)
11
12  B. Measurement of urinary protein (36%)
13
C. Reassurance and no additional workup (3%)
14

15 D. Urine culture (3%)


16
 E. Urine cytology (24%)
17
18

19
20 Incorrect 36%
21
Correct answer
B
 Answered correctly  02 secs
Time Spent  04/29/2021
Last Updated

22

23

24 Explanation
25

26
Glomerular hematuria Nonglomerular hematuria
27

28 Type of hematuria Microscopic > gross Gross > microscopic


29 Glomerulonephritis Nephrolithiasis
30 Poststreptococcal Cancer (eg, renal, prostate)
31 Common etiologies IgA nephropathy Polycystic kidney disease
32 Basement membrane disorder Infection (eg, cystitis)
33 Alport syndrome Papillary necrosis
34
Nonspecific or no symptoms Dysuria
35
Clinical presentation Nephritic syndrome Urinary obstruction
36
Reddish-brown/cola urine Pink/bright red urine
37

38 Blood & protein Blood but no protein


Urinalysis
39
RBC casts, dysmorphic RBCs Normal-appearing RBCs

40 RBC = red blood cell.

Hematuria can be classified as nonglomerular or glomerular. Normal-appearing blood cells in urine suggest nonglomerular hematuria, indicating
that the bleeding is coming from sources other than the glomerulus, such as other parts of the kidney or more distant organs (eg, ureter, bladder,
urethra). Nonglomerular hematuria is typically pink or bright red.

Glomerular hematuria is a result of glomerulonephritis, or damage caused by an immune-mediated attack on glomerular endothelium.
Damaged glomeruli allow abnormal passage of protein, lipids, and erythrocytes into urine. Glomerular hematuria typically presents with the
following:

Dysmorphic (abnormally shaped/damaged) erythrocytes and red blood cell casts

Proteinuria

Reddish-brown/cola-colored urine, likely reflecting methemoglobin formation due to the acidic urinary pH and prolonged transit time in the
nephron of dysmorphic cells

Clinical manifestations can vary from no symptoms to nephritic syndrome (eg, hematuria, proteinuria, hypertension, edema, acute kidney injury).

This patient's urinalysis findings strongly suggest glomerular hematuria. A 24-hour urinary protein excretion testing must be performed to
confirm and quantify proteinuria. Evaluation with a 1-time dipstick is unreliable because it may not detect nonalbumin proteinuria and because
sensitivity and specificity are highly dependent on urinary concentration. To establish a diagnosis in a patient with suspected glomerulonephritis, a
kidney biopsy is often required.

(Choice A) CT scan is used in the evaluation of nonglomerular hematuria. Noncontrast CT scan is obtained when there is suspicion for kidney
stones, which would cause flank or groin pain. This patient's dysmorphic erythrocytes and reddish-brown urine are typical findings for glomerular
hematuria.

(Choice C) Reassurance is appropriate when hematuria resolves after a benign cause (eg, recent intense exercise, menstrual bleeding) has
been identified. Normal-appearing erythrocytes would be present in the urine and would resolve on subsequent urinalysis.

(Choice D) Although hematuria can be due to a urinary tract infection (UTI), this patient has no other findings suggestive of UTI, such as fever,
costovertebral angle or suprapubic tenderness, significant number of white blood cells on urine microscopy, or nitrites in urine.

(Choice E) Urine cytology is indicated for evaluation of hematuria if malignancy of the urinary tract is suspected. However, malignancy is unlikely
in this young patient, and it would not cause dysmorphic erythrocytes in the urine. In addition, urine cytology is not generally one of the first steps
in evaluation of hematuria.

Educational objective:
Hematuria can be due to glomerular or nonglomerular pathologies. Proteinuria, dysmorphic erythrocytes, and red blood cell casts suggest a
glomerular cause (glomerulonephritis).

References
The investigation of hematuria.

(http://www.ncbi.nlm.nih.gov/pubmed/30642428)

Medicine Renal, Urinary Systems & Electrolytes Hematuria


Subject System Topic

https://t.me/USMLEWorldStep2CK

REVIEW

https://www.uworld.com/ClientApp/v15/apps/qbanktestinterface/index.html#/launchtest/7281878/nbme/229400736/3/1 1/1
‫ م‬8:53 2021/‫‏‬9/‫‏‬11 https://t.me/USMLEWorldStep2CK UWorld STEP2 SIM Form 1

1 Item 36 of 40
2
Mark
Question Id: 7000 Previous Next
3

4 A 35-year-old woman comes to the office due to moderately severe, achy pain in her lower back, hips, and knees for the past several weeks. She
5 has no associated morning stiffness. The patient has end-stage renal disease due to diabetic nephropathy and has been on hemodialysis for the
6 past 2 years. Her other medical problems include type 1 diabetes mellitus diagnosed 20 years ago and sarcoidosis diagnosed 8 years ago.
7 Physical examination shows no obvious bone or joint deformity, and she has full range of motion in all joints. Laboratory results are as follows:
8
Sodium 136 mEq/L
9

10 Potassium 3.9 mEq/L


11
Bicarbonate 23 mEq/L
12
Creatinine 3.1 mg/dL
13

14 Calcium 7.9 mg/dL


15
Phosphorus 6.1 mg/dL
16

17 Albumin 4.1 g/dL


18

19 Which of the following is the most likely histopathological abnormality underlying this patient's current symptoms?
20
A. Autoimmune parathyroid destruction (26%)
21

22 B. Granulomatous infiltration of parathyroid (22%)


23
C. Parathyroid gland atrophy (16%)
24
25  D. Parathyroid glandular hyperplasia (34%)
26 E. Single parathyroid adenoma (1%)

27

28

29
Incorrect 34%
30
31
Correct answer  Answered correctly  02 secs
Time Spent  09/10/2021
Last Updated
D
32

33
34 Explanation

35

36

37

38
39

40

This patient, with hypocalcemia and hyperphosphatemia in the setting of chronic kidney disease (CKD), has a typical presentation of secondary
hyperparathyroidism. In CKD, decreased production of calcitriol (1,25 dihydroxyvitamin D) leads to decreased intestinal absorption of calcium.
Concurrently, as glomerular filtration rate (GFR) decreases, the kidney cannot adequately excrete phosphate, leading to hyperphosphatemia. The
increased serum phosphate binds circulating calcium, forming insoluble calcium phosphate and further lowering serum calcium.

Hypocalcemia and hyperphosphatemia stimulate release of parathyroid hormone (PTH) to maintain normal calcium and phosphorus levels; over
time this results in parathyroid hyperplasia and secondary hyperparathyroidism. The resulting effects on bone, known as renal
osteodystrophy, include pain and increased risk of fracture. In some cases, chronic parathyroid stimulation leads to the development of
autonomously functioning adenomas (tertiary hyperparathyroidism) with very high PTH levels and hypercalcemia.

Management includes a low-phosphate diet and/or phosphate binders (eg, calcium carbonate, calcium acetate, sevelamer) and supplementation
of calcium and vitamin D.

(Choices A and B) Hypoparathyroidism is most commonly caused by direct injury, vascular damage, or inadvertent removal during thyroid
surgery. Less common causes include autoimmune injury or infiltrative diseases (eg, hemochromatosis, granulomatous diseases).
Hypoparathyroidism can cause low calcium and elevated phosphate levels, but this patient's renal dysfunction due to diabetic nephropathy makes
secondary hyperparathyroidism more likely.

(Choices C and E) Primary hyperparathyroidism is most commonly caused by a parathyroid adenoma (which may cause atrophy of the
surrounding normal parathyroid tissue). Less common causes include multiglandular hyperplasia and parathyroid carcinoma. Patients with
primary hyperparathyroidism would present with hypercalcemia and hypophosphatemia.

Educational objective:
An elevated phosphate level and a low calcium level in the setting of chronic kidney disease are characteristic of secondary hyperparathyroidism.
The resulting elevation in parathyroid hormone can cause renal osteodystrophy with associated bone pain.

References
Chronic kidney disease and the skeleton.

(http://www.ncbi.nlm.nih.gov/pubmed/26273531)

Medicine Endocrine, Diabetes & Metabolism Hyperparathyroidism


Subject System Topic

https://t.me/USMLEWorldStep2CK

REVIEW

https://www.uworld.com/ClientApp/v15/apps/qbanktestinterface/index.html#/launchtest/7281878/nbme/229400736/3/1 1/1
‫ م‬9:00 2021/‫‏‬9/‫‏‬11 https://t.me/USMLEWorldStep2CK UWorld STEP2 SIM Form 1

1 Item 37 of 40
2
Mark
Question Id: 7014 Previous Next
3

4 A 52-year-old woman comes to the office due to urinary leakage. Over the past several months, she has had frequent urinary dribbling and the
5 leakage is worse at night. The patient has no dysuria, fever, or constipation. Her medical history is significant for longstanding type 1 diabetes
6 mellitus and recurrent urinary tract infections. She takes multiple daily injections of insulin. Her blood pressure is 140/90 mm Hg and pulse is
7 90/min. Abdominal examination shows some fullness on palpation and increased dullness to percussion in the suprapubic region. She also has
8 symmetrically decreased vibratory sensation over both ankles. Bladder catheterization produces a postvoid residual volume of 300 mL. Urinalysis
9 shows no leukocyte esterase or nitrites. Which of the following is the most appropriate recommendation for this patient?
10
A. Beta-adrenergic agonist (1%)
11
12  B. Cholinergic agonist (72%)
13
C. Midurethral sling procedure (1%)
14

15 D. Muscarinic antagonist (15%)


16
 E. Pelvic floor exercises (4%)
17
18
F. Tricyclic antidepressant (3%)

19
20

21 Incorrect 72% 03 secs 05/11/2021


22 Correct answer
B
 Answered correctly  Time Spent  Last Updated
23

24
25
Explanation
26

27

28 Differential diagnosis of urinary incontinence


29 Etiology Symptoms
30
↓ Urethral sphincter tone
31 Stress Leakage with coughing, sneezing, lifting
Urethral hypermobility
32

33 Urge Detrusor overactivity Sudden, overwhelming urge to urinate


34
Impaired detrusor contractility
35 Overflow Incomplete emptying & persistent involuntary dribbling
Bladder outlet obstruction
36

37 This patient has a neurogenic bladder due to longstanding diabetes mellitus. Peripheral neuropathy can affect the S2-S4 nerves and cause
38 bladder voiding dysfunction due to detrusor underactivity. Overflow incontinence is the classic sequelae marked by urinary frequency, nocturia,
39 and frequent leakage of small volumes of urine. The average adult bladder has a capacity of 400-500 mL of urine; those with neurogenic bladder
40 can have high postvoid residual urine volumes (>150 mL in women, >50 mL in men).

Nonpharmacological techniques, including suprapubic pressure, timed voids, and double voiding (attempting standing void after sitting void), are
sometimes helpful. Because voiding is mediated through the parasympathetic system, cholinergic agents (eg, bethanechol) may aid bladder
contraction and urethral relaxation in patients with a neurogenic bladder. Severe cases may require intermittent self-catheterization, which is
preferable to an indwelling catheter as the latter increases the risk of urinary tract infections.

(Choices A and D) Muscarinic antagonists and beta agonists (eg, mirabegron) are used to treat urge incontinence (detrusor overactivity) by
relaxing the bladder. These drugs may worsen urinary retention in patients with a neurogenic bladder who already have a detrusor hypofunction.

(Choices C and E) Pelvic floor exercises (eg, Kegel) are recommended for patients with stress urinary incontinence (SUI) to strengthen the
muscles around the bladder and bladder neck. Pelvic floor exercises have no value in overflow incontinence. The best long-term treatment for
SUI is a midurethral sling procedure, which applies pressure to the bladder neck and may aggravate a neurogenic bladder due to bladder outlet
obstruction.

(Choice F) Tricyclic antidepressants have significant anticholinergic properties. Therefore, they may worsen urinary retention by blocking both
bladder contraction and bladder neck relaxation.

Educational objective:
Overflow incontinence may result from a neurogenic bladder and is treated with augmented voiding techniques and cholinergic agonists. Severe
cases are treated with bladder catheterization.

References
ACOG Practice Bulletin No. 155: Urinary Incontinence in Women.

(http://www.ncbi.nlm.nih.gov/pubmed/26488524)

Obstetrics & Gynecology Renal, Urinary Systems & Electrolytes Urinary incontinence
Subject System Topic

https://t.me/USMLEWorldStep2CK

REVIEW

https://www.uworld.com/ClientApp/v15/apps/qbanktestinterface/index.html#/launchtest/7281878/nbme/229400736/3/1 1/1
‫ م‬9:02 2021/‫‏‬9/‫‏‬11 https://t.me/USMLEWorldStep2CK UWorld STEP2 SIM Form 1

1 Item 38 of 40
2
Mark
Question Id: 7015 Previous Next
3

5
This item has associated media that may require the use of headphones. Please ensure your system/speaker volume is set to an audible level.

6
A 72-year-old man comes to the office for a routine health maintenance visit. He reports that he feels well overall. The patient walks 2-3 miles/day
7 without shortness of breath or chest pain. He takes no medications aside from a multivitamin daily. Blood pressure is 142/80 mm Hg and pulse is
8 75/min. Cardiac auscultation findings at the right upper sternal border are heard in the exhibit. Which of the following is the most likely cause of
9 this physical examination finding?
10
11 Play Media
12
13
 A. Calcification of the valvular leaflets (77%)
14

15 B. Fibrin-platelet vegetations attached to the valve (16%)


16 C. Giant cell accumulation in the aortic wall (1%)
17
D. Lipid accumulation in the arterial media (2%)
18

19  E. Rupture of the chordae tendineae (2%)


20

21

22
Incorrect 77% 02 secs 07/12/2021
23 Correct answer
A
 Answered correctly  Time Spent  Last Updated
24
25

26
Explanation
27

28

29 Aortic stenosis
30
Calcific disease (most common, age >70)
31
Causes Congenital bicuspid valve (younger patients)
32
Rheumatic heart disease (common worldwide but rare in developed countries)
33
34 Dyspnea on exertion, decreased exercise tolerance
35 Clinical Angina pectoris (severe AS)
36 manifestations Syncope (severe AS)
37
Heart failure (severe AS)

38 Crescendo-decrescendo systolic murmur best heard at RUSB with radiation to carotids &
Physical
39 axillae
examination
40 Slow-rising (delayed) & weak carotid pulse

Management Aortic valve replacement for severe AS causing symptoms or LV systolic dysfunction

AS = aortic stenosis; LV = left ventricular; RUSB = right upper sternal border.

This patient has a crescendo-decrescendo systolic ejection murmur auscultated at the right upper sternal border, which is classic for aortic
stenosis (AS). Age-related calcification of the aortic valve leaflets is the most common cause of AS in countries where rheumatic heart
disease is rare (eg, the United States); the presentation is typically age >70 in patients with a normal trileaflet aortic valve, but can be substantially
earlier (eg, age 50) in patients with a congenital bicuspid aortic valve. Patients with AS typically remain asymptomatic (such as this patient) for a
long time and gradually develop symptoms (eg, exertional dyspnea, angina, syncope) as the stenosis becomes severe. The pulse pressure
also begins to narrow as AS becomes severe.

Other typical features of an AS murmur include a soft and delayed S2 and radiation to the carotids. The timing of the peak intensity of the murmur
correlates with the severity of AS; a late-peaking murmur is indicative of severe disease because the contracting left ventricle needs more time to
generate adequate pressure to overcome the severely stenotic valve.

(Choices B and E) Fibrin-platelet valvular vegetations can occur with infective endocarditis, typically leading to a regurgitant, rather than a
stenotic, murmur. The systolic murmur of mitral regurgitation (or tricuspid regurgitation) differs from AS in that it is holosystolic rather than
crescendo-decrescendo. In addition, mitral regurgitation is typically best heard at the cardiac apex. Chordae tendineae rupture can occur
spontaneously and cause mitral (or tricuspid) regurgitation. No chordae tendineae are attached to the aortic valve.

(Choice C) Giant cell accumulation in the aorta and some of its large branches occurs in temporal arteritis (also known as giant cell arteritis). The
typical presentation includes fever, headache, jaw claudication, and vision loss; the heart valves are not typically involved and no murmur is
expected.

(Choice D) Lipid accumulation in the arterial media underlies the development of atherosclerosis. The early stages of aortic valve calcification
involve a process similar to that of atherosclerosis, but the lipid accumulation is within the aortic valve leaflets, not the arterial wall.

Educational objective:
Aortic stenosis causes a crescendo-decrescendo systolic ejection murmur best heard at the right upper sternal border. Age-related calcification of
the aortic valve leaflets is the most common cause of aortic stenosis in countries where rheumatic heart disease is rare (eg, the United States).

Medicine Cardiovascular System Aortic stenosis


Subject System Topic

https://t.me/USMLEWorldStep2CK

REVIEW

https://www.uworld.com/ClientApp/v15/apps/qbanktestinterface/index.html#/launchtest/7281878/nbme/229400736/3/1 1/1
‫ م‬9:02 2021/‫‏‬9/‫‏‬11 https://t.me/USMLEWorldStep2CK UWorld STEP2 SIM Form 1

1 Item 39 of 40
2
Mark
Question Id: 7009 Previous Next
3

4 A 54-year-old man comes to the emergency department due to 2 episodes of coffee-ground emesis since yesterday. The patient has had episodic
5 nausea and abdominal discomfort for the past several months. He consumes a pint of vodka daily. Upper endoscopy reveals nonbleeding
6 esophageal varices, diffuse gastric erythema, and a small duodenal ulcer. The patient is treated with intravenous fluids and a proton pump
7 inhibitor. He has no further episodes of hematemesis in the hospital, but the next day he becomes lethargic. Blood pressure is 108/66 mm Hg
8 and pulse is 98/min. The patient requires repeated physical stimuli to remain awake. Physical examination shows muscle wasting, gynecomastia,
9 and several spider angiomas. The abdomen is distended with a protruding umbilicus. He has mild pitting edema of the bilateral lower
10 extremities. Laboratory results are as follows:
11
Complete blood count
12
13 Hematocrit 32%
14 Mean corpuscular volume 102 µm3
15
Leukocytes 8,000/mm3
16

17
Serum chemistry
18

19 Sodium 136 mEq/L


20
Potassium 3.8 mEq/L
21
Creatinine 0.8 mg/dL
22

23
Liver function studies
24
25 Albumin 3.1 g/dL
26
Total bilirubin 2.0 mg/dL
27

28 Alkaline phosphatase 120 U/L


29 Aspartate aminotransferase 62 U/L
30
Alanine aminotransferase 24 U/L
31

32
Coagulation studies
33
34 INR 1.4
35
Diagnostic paracentesis shows a peritoneal fluid leukocyte count of 400/mm3 with 90% neutrophils and an albumin of 1.1 g/dL. Which of the
36
following is the best next step in management of this patient?
37

38 A. Administer intravenous furosemide (6%)


39
 B. Initiate broad-spectrum antibiotics (67%)
40

C. Initiate noncardioselective beta blocker (8%)

D. Perform large-volume paracentesis (10%)

 E. Perform transjugular intrahepatic portosystemic shunting (7%)

Incorrect 67%
Correct answer  Answered correctly  04 secs
Time Spent  03/29/2021
Last Updated
B

Explanation

Spontaneous bacterial peritonitis


Temperature >37.8 C (100 F)
Abdominal pain/tenderness
Clinical presentation
Altered mental status (abnormal connect-the-numbers test)
Hypotension, hypothermia, paralytic ileus with severe infection

PMNs >250/mm3
Positive culture, often gram-negative organisms (eg, Escherichia coli, Klebsiella)
Diagnosis from ascitic fluid
Protein <1 g/dL
SAAG >1.1 g/dL

Empiric antibiotics - third-generation cephalosporins (eg, cefotaxime)


Treatment
Fluoroquinolones for SBP prophylaxis

PMN = polymorphonuclear leukocytes; SAAG = serum-ascites albumin gradient; SBP = spontaneous bacterial peritonitis.

This patient with likely alcoholic cirrhosis (eg, ascites, spider angiomas, varices) has new mental status changes and an ascitic fluid neutrophil
count >250/mm3, concerning for spontaneous bacterial peritonitis (SBP). A serum-ascites albumin gradient >1.1 g/dL (3.1 − 1.1 = 2.0 g/dL)
suggests that the ascites is due to portal hypertension. The low level of protein in the ascitic fluid increases the risk of developing SBP due to the
low concentration of opsonins.

Because SBP is a major source of morbidity and mortality in patients with cirrhosis, there should be a high suspicion for it. Cultures are often
negative but can grow either gram-negative rods or gram-positive cocci, so broad-spectrum antibiotics (eg, third-generation cephalosporins,
fluoroquinolones) are required for treatment.

(Choice A) Diuretics can be used to manage ascites by combining spironolactone and furosemide; however, diuretics would not usually be
administered to a patient who is acutely ill as they can lower blood pressure and make the patient worse.

(Choice C) Noncardioselective beta blockers (eg, propranolol, nadolol) are used in patients with cirrhosis who have esophageal varices to
decrease portal vein blood flow to prevent the progression of varices and the risk of variceal bleeding. They are usually permanently discontinued
after a patient has SBP due to an increased rate of mortality.

(Choice D) Large-volume paracentesis (eg, therapeutic paracentesis) is performed in patients with symptomatic ascites (eg, shortness of breath,
abdominal pain) due to a large volume of ascitic fluid.

(Choice E) Transjugular intrahepatic portosystemic shunting (TIPS) is performed when a patient has ascites that does not respond to medical
therapy (eg, diuretics) or has ongoing active or recurrent bleeding from varices even after appropriate treatment with upper endoscopy. TIPS is
associated with portosystemic encephalopathy in up to 35% of patients.

Educational objective:
Spontaneous bacterial peritonitis should be suspected in patients with fever, abdominal pain, and encephalopathy with evidence of cirrhosis (eg,
ascites). The finding on paracentesis of >250 neutrophils/mm3 in the ascitic fluid is considered diagnostic. Initial treatment consists of broad-
spectrum antibiotics while awaiting culture results.

References
Introduction to the revised American Association for the Study of Liver Diseases Practice Guideline management of adult patients with ascites
due to cirrhosis 2012.

(http://www.ncbi.nlm.nih.gov/pubmed/23463403)

Medicine Gastrointestinal & Nutrition Spontaneous bacterial peritonitis


Subject System Topic

https://t.me/USMLEWorldStep2CK

REVIEW

https://www.uworld.com/ClientApp/v15/apps/qbanktestinterface/index.html#/launchtest/7281878/nbme/229400736/3/1 1/1
‫ م‬9:02 2021/‫‏‬9/‫‏‬11 https://t.me/USMLEWorldStep2CK UWorld STEP2 SIM Form 1

1 Item 40 of 40
2
Mark
Question Id: 20464 Previous Next
3

4 A randomized controlled single-blind study is conducted to evaluate the effectiveness of 2 different massage interventions, classic soft tissue
5 mobilization and dynamic soft tissue mobilization, compared to no intervention. Forty-five healthy men age 18-35 are randomized (ratio 1:1:1) to
6 classic intervention, dynamic intervention, or no intervention (ie, control) groups. An analysis of variance (ANOVA) is used to determine whether
7 the change in hip flexion angle (HFA) differs between groups. Change in HFA is defined as the difference between preintervention and
8 postintervention HFA measurements. Pairwise post hoc comparisons are conducted to determine whether change in HFA differs between groups
9 at a 5% significance level. Study results reveal an overall significant difference between the groups (F = 5.03, p < 0.01). Results from the pairwise
10 comparisons are shown below.
11
Hip flexion angle measurements
12
13 Difference
95% confidence
14 Comparison in HFA
interval
15 change
16 Dynamic intervention vs No intervention (control) 4.74 1.15 to 8.33
17
Classic intervention vs No intervention (control) 1.34 −1.90 to 4.58
18

19 Dynamic intervention vs Classic intervention 3.40 0.16 to 6.64


20
Based on these findings, which of the following conclusions is justified?
21

22
A. All differences in HFA change between groups are statistically significant (3%)
23

24  B. HFA change in the dynamic intervention group is significantly different from HFA change in either the classic intervention or control
25 group (39%)
26 C. Only the difference in HFA change between the dynamic intervention and control groups is statistically significant (45%)
27
D. The classic intervention group differs significantly in HFA change from the control group (5%)
28

29  E. There is no statistically significant difference in HFA change between any specific groups (5%)
30
31

32
Incorrect 39% 16 secs 04/02/2021
33 Correct answer
B
 Answered correctly  Time Spent  Last Updated
34

35

36
Explanation
37

38 Analysis of variance (ANOVA) compares the means of ≥3 independent groups based on the F test (comparing the variability between these
39 groups to the variability within these groups). However, a statistically significant ANOVA (ie, if the p-value corresponding to the F test is below
40 the significance level, usually 0.05) indicates only that at least 1 group mean is different from the rest; it is necessary to conduct additional tests
(ie, group comparisons) to determine which group means differ. Different approaches are available to conduct group comparisons, including
calculating confidence intervals (CIs) for the difference in group means.

In this study, ANOVA is used to determine whether the change in hip flexion angle (HFA) differs between 3 groups: classic soft tissue mobilization
(CSTM), dynamic soft tissue mobilization (DSTM), and a control. ANOVA is significant (p < 0.01), so at least 1 group mean is different from the
rest (ie, at least 2 group means are different from each other) (Choice E).

Additional comparisons (ie, pairwise post hoc comparisons) are conducted to identify which group means are different. These comparisons do not
provide a p-value; instead, they provide CIs for the differences in 2 group means. A difference in group means equal to 0 indicates that the group
means are equal. Therefore, when comparing the difference in group means:

A CI that includes 0 indicates that there is no difference in group means.

A CI that excludes 0 indicates that there is a difference in group means.

Only the CIs that compare DSTM versus CSTM (ie, CI: 0.16 to 6.64) and DSTM versus control (ie, CI: 1.15 to 8.33) exclude 0, so HFA change in
the DSTM group is significantly different from HFA change in either the CSTM or control group (Choices A and C).

(Choice D) The CI that compares CSTM versus control (ie, CI: −1.90 to 4.58) includes 0, so there is no significant difference in HFA change
between the CSTM and control groups.

Educational objective:
Analysis of variance (ANOVA) is used to compare the means of independent groups; it determines only whether at least 2 group means differ.
Group comparisons are follow-up tests used to determine which group means differ. A confidence interval for the difference in group means that
includes 0 indicates no significant difference in group means, and vice versa.

Medicine Biostatistics & Epidemiology Anova


Subject System Topic

https://t.me/USMLEWorldStep2CK

REVIEW

https://www.uworld.com/ClientApp/v15/apps/qbanktestinterface/index.html#/launchtest/7281878/nbme/229400736/3/1 1/1
‫ م‬9:03 2021/‫‏‬9/‫‏‬11 https://t.me/USMLEWorldStep2CK UWorld STEP2 SIM Form 1

1 Item 1 of 40
2
Mark
Question Id: 6843 Previous Next
3

4 A 70-year-old resident of a nursing home is brought to the emergency department due to increased lethargy and decreased oral intake for one
5 day. According to the paramedics, he had an episode of vomiting last night and a mild cough and shortness of breath earlier today. The patient's
6 medical history includes diabetes mellitus type 2, benign prostatic hyperplasia, and Parkinson disease with cognitive impairment. Temperature is
7 38.9 C (102 F), blood pressure is 70/60 mm Hg, and pulse is 120/min. On examination, extremities are warm to the touch and there are bronchial
8 breath sounds over the right lung base. The abdomen is soft, nontender, and nondistended. Fingerstick blood glucose is 154 mg/dL and arterial
9 blood gas findings on room air are as follows:
10
pH 7.24
11
12 PaCO2 30 mm Hg
13 PaO2 85 mm Hg
14

15 Which of the following best explains this patient's arterial blood gas findings?
16
 A. Alveolar ventilation/perfusion mismatch (17%)
17
18 B. Impaired renal bicarbonate reabsorption (11%)
19
C. Increased gastrointestinal chloride loss (4%)
20

21 D. Increased hepatic fatty acid metabolism (1%)


22 E. Increased tissue metabolic acid production (64%)

23

24
25
Incorrect 64%
26

27
Correct answer  Answered correctly  07 secs
Time Spent  03/27/2021
Last Updated
E
28

29
30 Explanation

31
The patient's arterial blood gas shows an acidemia (pH <7.4) with a low PaCO2 (<40 mm Hg), suggesting a primary metabolic acidosis (rather
32
than a primary respiratory acidosis, in which the PaCO2 would be elevated). In this patient with fever, tachycardia, hypotension, and bronchial
33
breath sounds in the right lung base, the most likely cause is septic shock due to pneumonia (possibly aspiration pneumonia given the history of
34
vomiting). Septic shock causes hypotension and a hypermetabolic state; insufficient oxygen delivery to meet the metabolic demands of the
35
peripheral tissues results in increased anaerobic metabolism from the cells, leading to a buildup of lactic acid.
36

37 Lactic acidosis is the most common cause of a metabolic acidosis in patients requiring hospitalization, as sepsis is common. In patients
38
presenting to the hospital, an elevated lactic acid level due to sepsis can be concerning for a poor prognosis unless appropriate treatment is begun
quickly; such treatment includes intravenous volume resuscitation with crystalloid, possible vasopressor therapy, and management of any
39
underlying infection with appropriate antibiotics.
40
(Choice A) Alveolar ventilation/perfusion mismatch can be seen in pneumonia when the airways are compromised by alveolar filling. Impaired
ventilation can cause a respiratory acidosis with CO2 retention. However, this patient's PaCO2 is depressed, which is likely due to a respiratory
compensation for his metabolic acidosis.

(Choice B) Type II renal tubular acidosis is due to impaired renal bicarbonate reabsorption and is usually associated with hypokalemia and a non-
anion gap metabolic acidosis. It would not explain this patient's fever, hypotension, tachycardia, or crackles.

(Choice C) Vomiting generally results in the loss of chloride and hydrochloric acid, leading to a hypochloremic metabolic alkalosis. In addition,
one episode of vomiting is also unlikely to lead to significant metabolic derangements.

(Choice D) Increased hepatic fatty acid metabolism is seen in the fasting state. This generally results in a mild ketosis but is unlikely to cause a
significant metabolic derangement such as that seen in this septic patient.

Educational objective:
Patients with septic shock can develop impaired and insufficient oxygen delivery to the peripheral tissues, causing anaerobic metabolism. This
leads to an increase in production of lactic acid, which can produce significant metabolic acidosis.

References
Lactic acidosis in sepsis: it's not all anaerobic: implications for diagnosis and management.

(http://www.ncbi.nlm.nih.gov/pubmed/26378980)

Medicine Pulmonary & Critical Care Sepsis


Subject System Topic

https://t.me/USMLEWorldStep2CK

REVIEW

https://www.uworld.com/ClientApp/v15/apps/qbanktestinterface/index.html#/launchtest/7281878/nbme/229400737/3/1 1/1
‫ م‬9:03 2021/‫‏‬9/‫‏‬11 https://t.me/USMLEWorldStep2CK UWorld STEP2 SIM Form 1

1 Item 2 of 40
2
Mark
Question Id: 6846 Previous Next
3

4 A 38-year-old man comes to the emergency department due to increased fatigue over the last week. Recently, he has become easily short of
5 breath while walking, and he often develops a choking sensation shortly after falling asleep in a supine position. His past medical history is
6 insignificant except for an upper respiratory tract infection 2 weeks ago. He has not had any chest pain, palpitations, or loss of consciousness.
7 His blood pressure is 110/70 mm Hg and pulse is 100/min. Physical examination reveals bibasilar crackles in his lungs with pitting edema in both
8 lower extremities. On palpation of his precordium, the point of maximum impulse is felt in the sixth intercostal space along the anterior axillary
9 line. Which of the following is most likely to be found in this patient?
10
 A. Mitral valve prolapse (5%)
11
12  B. Dilated cardiomyopathy (70%)
13
C. Restrictive cardiomyopathy (4%)
14

15 D. Hypertrophic obstructive cardiomyopathy (5%)


16
E. Constrictive pericarditis (13%)
17
18

19
20
Incorrect 70%
21
Correct answer
B
 Answered correctly  02 secs
Time Spent  07/30/2021
Last Updated

22

23

24 Explanation
25

26
This patient most likely has myocarditis, which can lead to the development of dilated cardiomyopathy. He clearly has signs and symptoms of
heart failure as evidenced by his shortness of breath and paroxysmal nocturnal dyspnea. Physical examination reveals bibasilar crackles, pitting
27
edema, and displacement of the point of maximal impulse. Given this patient’s young age and history of a preceding upper respiratory tract
28
infection, his heart failure is most likely caused by myocarditis, which, in developed nations, is commonly caused by viruses such as Coxsackie B.
29
30
(Choice A) Mitral valve prolapse is typically a congenital condition, not a sequela of myocarditis.
31

32
(Choice C) Restrictive cardiomyopathy is often caused by myocardial infiltrative diseases such as amyloidosis, sarcoidosis, or hemochromatosis.
33
34
(Choice D) Hypertrophic obstructive cardiomyopathy is a genetic disease that causes hypertrophy of the interventricular septum; it is not
35
associated with myocarditis.
36

37 (Choice E) Constrictive pericarditis, a rare result of acute pericarditis, is due to pericardial sac scarring and loss of elasticity. This patient’s
38 symptoms, however, are more consistent with myocarditis than pericarditis, which typically presents with sharp, pleuritic chest pain (improved by
39 leaning forward) and a friction rub.
40
Educational objective:
Myocarditis should be suspected in young patients with a recent viral illness who present with heart failure, chest pain, or arrhythmias. Most
patients will have at least partial recovery of myocardial function, but some will develop dilated cardiomyopathy.

Medicine Cardiovascular System Myocarditis


Subject System Topic

https://t.me/USMLEWorldStep2CK

REVIEW

https://www.uworld.com/ClientApp/v15/apps/qbanktestinterface/index.html#/launchtest/7281878/nbme/229400737/3/1 1/1
‫ م‬9:03 2021/‫‏‬9/‫‏‬11 https://t.me/USMLEWorldStep2CK UWorld STEP2 SIM Form 1

1 Item 3 of 40
2
Mark
Question Id: 6847 Previous Next
3

4 A 36-year-old man comes to the office due to bilateral skin lesions over the knees and elbows for the past several years. The patient says the
5 lesions have enlarged progressively, which he attributes to excessive kneeling in his work as a plumber. The lesions are not painful but he has
6 had occasional itching. The patient has used several moisturizers, which provided only temporary relief. He has no other medical problems and
7 takes no medications. He smokes a pack of cigarettes a day and drinks 1 or 2 cans of beer daily. Temperature is 37 C (98.7 F), blood pressure is
8 130/86 mm Hg, and pulse is 74/min. Skin examination findings of his knees are shown below.
9

10
11
12
13

14

15
16

17
18

19
20

21

22

23

24
25

26

27

28

29
30
31

32

33
Which of the following is the most likely cause of this patient's skin lesions?
34

35
 A. Atopic dermatitis (0%)
36

37
B. Dyshidrotic eczema (1%)

38 C. Frictional dermatitis (5%)


39
D. Lichen simplex chronicus (2%)
40
E. Mycosis fungoides (0%)

F. Nummular eczema (0%)

 G. Plaque psoriasis (88%)

H. Seborrheic dermatitis (0%)

I. Tinea corporis (0%)

Incorrect 88%
Correct answer  Answered correctly  05 secs
Time Spent  03/31/2021
Last Updated
G

Explanation

This patient's rash is characteristic of psoriasis. Psoriasis is a chronic skin condition characterized by hyperkeratosis, leading to well-demarcated
erythematous plaques with a white or silver scale, primarily on the extensor surfaces (eg, knees, elbows). Additional complications of psoriasis
include nail changes (eg, pitting, onycholysis), ocular inflammation (eg, conjunctivitis, uveitis), and psoriatic arthritis. Factors that can lead to
worsening of symptoms include local skin trauma (Koebner phenomenon), infections, withdrawal from systemic glucocorticoids, and certain
medications (eg, antimalarials, indomethacin, propranolol).

The diagnosis of psoriasis is typically made by history and physical examination, but a skin biopsy may be necessary in difficult cases. Initial
treatment for plaque psoriasis includes topical high-potency glucocorticoids or vitamin D derivatives (eg, calcipotriene).

(Choices A, B, C, and F) Atopic dermatitis (eczema) in adults typically presents with an erythematous, scaly, pruritic rash. It is usually located on
the flexural surfaces of the arms, as opposed to the extensor surfaces in psoriasis. Nummular (or discoid) eczema is a variant characterized by
scattered, round eczematous plaques primarily on the back and extremities. Dyshidrotic eczema (pompholyx) presents with pruritic vesicles and
erythema on the palms and soles. Chronic irritation from physical friction can also lead to eczematous symptoms, usually on the fingers and
hands.

(Choice D) Lichen simplex chronicus (neurodermatitis) is characterized by thickened excoriated plaques due to persistent scratching and
rubbing. It is associated with anxiety disorders and typically occurs in areas that are easy to reach (eg, arms, legs, neck).

(Choice E) Cutaneous T cell lymphoma, sometimes referred to as mycosis fungoides, can present with a variety of appearances but most
commonly is seen as scaly, pruritic patches or plaques.

(Choice H) Seborrheic dermatitis causes a scaly, oily, erythematous rash affecting the skinfolds around the nose, eyebrows, and ears.
Involvement of the scalp is commonly referred to as dandruff.

(Choice I) Tinea corporis is a dermatophyte infection that causes a ring of erythema and scaling with central clearing as opposed to the silvery,
scaly lesion seen in this patient.

Educational objective:
Psoriasis is characterized by well-demarcated erythematous plaques with a white or silver scale, primarily on the extensor surfaces (eg, knees,
elbows). The diagnosis is typically based on clinical findings, but biopsy is occasionally necessary. Initial treatment includes topical high-potency
glucocorticoids or vitamin D derivatives.

References
Psoriasis.

(http://www.ncbi.nlm.nih.gov/pubmed/23668525)

Medicine Dermatology Psoriasis


Subject System Topic

https://t.me/USMLEWorldStep2CK

REVIEW

https://www.uworld.com/ClientApp/v15/apps/qbanktestinterface/index.html#/launchtest/7281878/nbme/229400737/3/1 1/1
‫ م‬9:03 2021/‫‏‬9/‫‏‬11 https://t.me/USMLEWorldStep2CK UWorld STEP2 SIM Form 1

1 Item 4 of 40
2
Mark
Question Id: 6858 Previous Next
3

4 A 64-year-old man is brought to the office by his wife because she has noticed yellowish discoloration of his eyes and skin for the past 3 months.
5 He has not had any nausea or vomiting, but has noted decreased appetite and abdominal discomfort recently. Temperature is 37.1 C (98.8 F),
6 blood pressure is 150/90 mm Hg, pulse is 90/min, and respirations are 16/min. BMI is 18 kg/m2. Physical examination reveals an underweight
7 white male in no acute distress. There is some fullness in the right upper abdominal quadrant. Laboratory findings are the following:
8
Total bilirubin 12.5 mg/dL
9

10 Asparatate aminotransferase 95 U/L


11
Alanine aminotransferase 65 U/L
12
Alkaline phosphatase 350 U/L
13

14 Blood urea nitrogen 14 mg/dL


15
Creatinine 1.2 mg/dL
16

17
Which of the following is the most likely primary site for this patient’s pathology?
18

19 A. Liver parenchyma (12%)



20
B. Duodenum (1%)
21

22  C. Pancreas (64%)
23
D. Gallbladder (20%)
24
25 E. Circulating red blood cells (1%)
26 F. Bone marrow (0%)
27

28

29
30
Incorrect 64% 03 secs 07/30/2021
31
Correct answer
C
 Answered correctly  Time Spent  Last Updated

32

33
34 Explanation
35
This patient with jaundice, decreased appetite with weight loss (given his low BMI), abdominal discomfort (without prominent abdominal pain),
36
and fullness in the right upper quadrant of his abdomen likely has pancreatic cancer. His elevated alkaline phosphatase and total bilirubin
37
suggest a cholestatic pattern of jaundice, which could be caused by a pancreatic mass obstructing the common bile duct. Diagnosis should be
38
confirmed with abdominal imaging.
39

40
(Choice A) The aminotransferases would typically be much more elevated if the primary source of pathology were the liver parenchyma. Liver
masses are most likely due to metastases rather than to primary cancers arising from the liver parenchyma.

(Choice B) A mass in the duodenum could cause jaundice similar to pancreatic cancer by blocking the flow of bile into the duodenal papilla;
however, this would be much less likely than pancreatic cancer.

(Choice D) Compared to pancreatic neoplasms, gallbladder neoplasms are much rarer; they would also be less likely to present initially with
jaundice because bile can still pass through the common bile duct.

(Choice E) Hemolysis can cause hyperbilirubinemia, but it would not cause elevation of alkaline phosphatase.

(Choice F) Disorders of the bone marrow can cause ineffective erythropoiesis with early breakdown of red blood cells, but alkaline phosphatase
would not be elevated.

Educational objective:
Jaundice, weight loss, and vague abdominal discomfort are common presenting symptoms of pancreatic cancer. Elevation of bilirubin and alkaline
phosphatase implies a cholestatic cause of jaundice, which can be caused by blockage of the common bile duct by a pancreatic mass.

Medicine Gastrointestinal & Nutrition Pancreatic cancer


Subject System Topic

https://t.me/USMLEWorldStep2CK

REVIEW

https://www.uworld.com/ClientApp/v15/apps/qbanktestinterface/index.html#/launchtest/7281878/nbme/229400737/3/1 1/1
‫ م‬9:03 2021/‫‏‬9/‫‏‬11 https://t.me/USMLEWorldStep2CK UWorld STEP2 SIM Form 1

1 Item 5 of 40
2
Mark
Question Id: 6860 Previous Next
3

4 A 17-year-old girl is brought to the emergency department by her older sister who found the patient writhing on the floor with severe abdominal
5 pain. Since then, the patient has had several episodes of coffee-ground emesis and 2 episodes of dark green diarrhea. She has major
6 depressive disorder and was hospitalized a month ago for a suicide attempt. The patient also has anemia and chronic back pain from a motor
7 vehicle collision a year ago. She is sexually active and uses condoms inconsistently. Her last menstrual period was 3 weeks ago. Temperature is
8 37 C (98.6 F), blood pressure is 80/50 mm Hg, pulse is 120/min, and respirations are 28/min. On physical examination, she is lethargic and
9 appears uncomfortable. There are multiple healing linear scars along the left forearm. Capillary refill is 5 seconds. The abdomen is soft but
10 diffusely tender to palpation. Which of the following is the most likely diagnosis in this patient?
11
 A. Acetaminophen overdose (21%)
12
13 B. Acute appendicitis (0%)
14
 C. Acute iron poisoning (67%)
15
16 D. Narcotic overdose (0%)
17 E. Pelvic inflammatory disease (0%)
18
F. Ruptured ectopic pregnancy (3%)
19
20 G. Tricyclic antidepressant overdose (5%)
21

22

23
Incorrect 67%
24 Correct answer  Answered correctly  02 secs
Time Spent  03/28/2021
Last Updated
25 C
26

27
Explanation
28

29
30 Acute iron poisoning
31
Abdominal pain, hematemesis, diarrhea
32
Clinical features Shock
33
Liver necrosis
34

35 Anion gap metabolic acidosis


36 Diagnostic findings Elevated serum iron
37 Radiopaque pills on abdominal x-ray
38 Deferoxamine
Treatment
39 Whole bowel irrigation
40
This adolescent with a history of depression and suicidality has abdominal pain, coffee-ground emesis (ie, hematemesis), and diarrhea, all early
signs of acute iron poisoning that are likely due to intentional overdose. Shortly after ingestion, iron directly injures the gastrointestinal mucosa
by free radical production and lipid peroxidation. This injury leads to upper (eg, hematemesis) and lower (eg, melena) gastrointestinal bleeding;
stools may also be green/black from disintegrating iron tablets.

Tachycardia and hypotension occur with hypovolemic shock (gastrointestinal losses), distributive shock (vasodilation), and/or cardiogenic shock
(myocardial injury). Increased lactic acid production due to poor perfusion contributes to anion gap metabolic acidosis. The respiratory rate
may increase to compensate for this acidosis. Iron exposure through the portal circulation and high metabolic activity of hepatocytes make the
liver particularly prone to injury, and hepatic necrosis can develop 1-2 days after ingestion.

Liver failure and shock are the most common causes of death. Patients who survive may experience bowel obstruction due to gastrointestinal
scarring weeks later.

(Choice A) Acetaminophen toxicity can cause nausea, vomiting, and hepatotoxicity; gastrointestinal bleeding is uncommon.

(Choice B) Acute appendicitis classically presents with anorexia, vomiting, and right lower quadrant pain. Appendiceal rupture may present with
diffuse abdominal tenderness and signs of septic shock. In both cases, fever would be expected, and upper gastrointestinal bleeding would not
occur.

(Choice D) Clinical findings of narcotic overdose can include lethargy and hypotension. However, respiratory depression and bradycardia would
be expected; this patient is tachypneic and tachycardic.

(Choice E) This patient with inconsistent condom use is at risk for sexually transmitted illnesses and complications such as pelvic inflammatory
disease (PID). PID can cause lower abdominal pain, abnormal uterine bleeding, and vaginal discharge but not hematemesis.

(Choice F) Ectopic pregnancy should be considered in a sexually active adolescent who has abdominal pain and vomiting. However, symptoms
typically occur 6-8 weeks after the last menstrual period, and ectopic pregnancy does not cause gastrointestinal bleeding.

(Choice G) Tricyclic antidepressant overdose causes anticholinergic effects such as hyperthermia, flushing, tachycardia, hypotension, and altered
mental status. Decreased gastric motility, rather than diarrhea, would be expected.

Educational objective:
Acute iron poisoning initially presents with symptoms of direct injury to the gastrointestinal tract (eg, hematemesis) and is followed by shock and
anion gap metabolic acidosis. Hepatic necrosis and bowel scarring/obstruction may also occur.

References
Iron overdose epidemiology, clinical features and iron concentration-effect relationships: the UK experience 2008-2017.

(http://www.ncbi.nlm.nih.gov/pubmed/29587543)

Pediatrics Gastrointestinal & Nutrition Iron poisoning


Subject System Topic

https://t.me/USMLEWorldStep2CK

REVIEW

https://www.uworld.com/ClientApp/v15/apps/qbanktestinterface/index.html#/launchtest/7281878/nbme/229400737/3/1 1/1
‫ م‬9:04 2021/‫‏‬9/‫‏‬11 https://t.me/USMLEWorldStep2CK UWorld STEP2 SIM Form 1

1 Item 6 of 40
2
Mark
Question Id: 6861 Previous Next
3

4 A 64-year-old woman comes to the office due to 8 weeks of increasing back pain. Her pain is no longer relieved with over-the-counter analgesics.
5 She also reports intermittent fever and night sweats but no chills. The patient has end-stage renal disease due to chronic glomerulonephritis and
6 has been on continuous ambulatory peritoneal dialysis for 2 years. She smoked cigarettes for 30 years but quit 18 years ago. Temperature is
7 38.4 C (101.1 F), blood pressure is 130/86 mm Hg, pulse is 96/min, and respirations are 14/min. Pulse oximetry is 94%. BMI is 19 kg/m2. There
8 is no scleral icterus. Pulmonary examination reveals scattered crackles over the left mid-upper lung. Marked tenderness is noted over the L3 and
9 L4 vertebral areas. Laboratory results are as follows:
10
Complete blood count
11
12 Hemoglobin 8.2 g/dL
13 Mean corpuscular volume 86 µm3
14
Platelets 425,000/mm3
15
16 Leukocytes 6,800/mm3
17
18 Immunologic studies
19
Erythrocyte sedimentation rate 110 mm/hr
20

21
Chest imaging reveals a cavitary infiltrate in the left upper lung. Imaging of the spine shows collapse of the L3 and L4 vertebral bodies. Which of
22
the following is the most likely diagnosis in this patient?
23

24  A. Metastatic lung cancer (18%)


25
B. Multiple myeloma (12%)
26

27 C. Osteoporotic compression fracture (4%)


28
D. Secondary hyperparathyroidism (4%)
29
30  E. Tuberculosis (61%)

31

32

33 Incorrect 61% 04 secs 06/26/2021


34 Correct answer
E
 Answered correctly  Time Spent  Last Updated
35

36

37 Explanation
38

39 This patient with back pain has a subacute illness with intermittent fevers, radiographic evidence of pulmonary and spinal disease, and
40 laboratory results showing normocytic anemia and likely chronic inflammation (elevated erythrocyte sedimentation rate and thrombocytosis)
suggesting disseminated tuberculosis (TB). Although she has no apparent social risk factors (eg, homelessness, group living, international
travel) for TB, chronic kidney disease (CKD) is one of several immunosuppressed states (eg, diabetes mellitus, HIV, hematologic malignancies,
chronic immunosuppressant medication) that increase the risk for Mycobacterium tuberculosis reactivation.

M tuberculosis preferentially infects the lung apices due to higher oxygen tensions and slower lymphatic outflow (allowing for greater
accumulation of organisms). Dissemination occurs hematologically and may affect a wide range of organ systems, including the lymph nodes
(cervical, mediastinal, and hilar), liver, bone, and central nervous system. Reactivation TB (in the setting of immunosuppression) typically
presents with subacute or chronic symptoms such as fever, weight loss (possibly reflected in this patient's low BMI), night sweats, and fatigue.
Pulmonary TB can cause cough (often with hemoptysis), along with cavitary lesions, infiltrates, and pleural effusions seen on chest x-ray.
Skeletal TB can present as spondylitis (with back pain and potential vertebral fractures), arthritis, or osteomyelitis.

(Choice A) Lung cancer typically presents with pulmonary symptoms (eg, cough, hemoptysis, dyspnea, chest pain). Although this patient has a
history of significant tobacco use, her abstinence for >15 years decreases the risk of lung cancer (by up to 90%). In addition, even though lung
cancer often metastasizes to the spine, subacute fever is not a typical feature of this disease.

(Choice B) Multiple myeloma (MM) commonly causes bone pain, anemia, renal insufficiency, fatigue, and hypercalcemia. However, fevers are
rare in the absence of acute infection. This patient has subacute fevers and a pulmonary infiltrate, making MM less likely than TB.

(Choice C) Osteoporosis may cause compression fractures in elderly patients. However, this patient has many other concurrent manifestations
of TB that make osteoporosis an unlikely cause of her compression fracture.

(Choice D) Secondary hyperparathyroidism can cause fractures and bone disease in patients with CKD. It is not associated with pulmonary
infiltrates and subacute fevers.

Educational objective:
Active tuberculosis (TB) typically presents with subacute or chronic symptoms (eg, fever, weight loss, night sweats, fatigue, cough). TB may
spread hematogenously to the lymph nodes, liver, bones, and central nervous system.

Medicine Infectious Diseases Tuberculosis


Subject System Topic

https://t.me/USMLEWorldStep2CK

REVIEW

https://www.uworld.com/ClientApp/v15/apps/qbanktestinterface/index.html#/launchtest/7281878/nbme/229400737/3/1 1/1
‫ م‬9:04 2021/‫‏‬9/‫‏‬11 https://t.me/USMLEWorldStep2CK UWorld STEP2 SIM Form 1

1 Item 7 of 40
2
Mark
Question Id: 6868 Previous Next
3

4 A 69-year-old man comes to the emergency department due to 2 episodes of dizziness over the past week. The patient has had no chest pain or
5 shortness of breath, but he reports losing 5 kg (11 lb) over the past 2 months. Other medical conditions include mild cognitive impairment and
6 osteoarthritis of his right knee. The patient eats plenty of vegetables and fruit and takes a multivitamin daily. His appetite has been stable
7 recently. The patient is a former smoker and does not use alcohol or illicit drugs. He lives with his wife and is independent in his activities of daily
8 living. Temperature is 37.1 C (98.8 F), blood pressure is 170/90 mm Hg, and pulse is 142/min and irregularly irregular. On examination, he has
9 flat neck veins while sitting and the lungs are clear to auscultation. There is no peripheral edema. Which of the following is the best initial test for
10 this patient?
11
 A. 24-hour urine cortisol excretion (1%)
12
13 B. Carotid sinus massage (7%)
14
C. Exercise stress testing (2%)
15
16 D. Renin/aldosterone ratio (5%)
17 E. Tilt table testing (3%)
18
 F. TSH and free T4 (71%)
19
20 G. Urine metanephrines (7%)
21

22

23
Incorrect 71%
24 Correct answer  Answered correctly  02 secs
Time Spent  04/26/2021
Last Updated
25 F
26

27
Explanation
28

29
30 Atrial fibrillation
31
Atrial remodeling due to:
32
Age-related myocardial changes
33
Atrial enlargement from heart disease (eg, HTN, MS)
34 Pathogenesis
Foci of rapid electrical activity:
35
Commonly originate in pulmonary veins
36
Alcohol intake & ↑ sympathetic drive may contribute
37

38
Asymptomatic or palpitations ± dyspnea, fatigue
Clinical
Irregularly irregular pulse
39 presentation
May present with thromboembolic event (eg, stroke)
40
ECG shows variable R-R intervals & no P waves
Diagnosis
Continuous ECG often needed for paroxysmal disease

HTN = hypertension; MS = mitral stenosis.

This patient's rapid and irregular pulse is most likely due to atrial fibrillation, which is the likely cause of his dizziness episodes over the past
week. In addition, unintentional weight loss and hypertension with wide pulse pressure are suggestive of hyperthyroidism, a common
trigger of atrial fibrillation. The wide pulse pressure (eg, 170 − 90 = 80 mm Hg) in hyperthyroidism results from peripheral vasodilation that
decreases diastolic blood pressure and elevated cardiac contractility and stroke volume that increase pulse pressure.

Testing for hyperthyroidism with serum TSH and free T4 levels should be performed in all patients with new-onset atrial fibrillation, especially
those with suggestive signs and symptoms such as in this patient. Restoration of a euthyroid state often resolves the atrial fibrillation.

(Choice A) Measurement of 24-hour urine cortisol excretion evaluates for cortisol excess (ie, Cushing syndrome). However, excess cortisol
typically causes weight gain rather than weight loss.

(Choice B) Carotid sinus massage can be considered for initial management of a rapid and regular tachycardia (eg, atrioventricular nodal
reentrant tachycardia) as it can be both diagnostic and therapeutic. It is less helpful for atrial fibrillation, possibly due to decreased effectiveness
against multiple, irregular atrial impulses. In addition, it would not treat the underlying cause of this patient's tachyarrhythmia.

(Choice C) Exercise stress testing is used to evaluate for coronary artery disease (CAD). However, the contribution of CAD to atrial fibrillation is
mostly via acute myocardial infarction or ischemic heart failure, and there is no evidence of either in this patient (eg, no chest pain, peripheral
edema, or jugular venous distension).

(Choice D) Calculation of the renin/aldosterone ratio is useful if primary hyperaldosteronism (ie, Conn syndrome) is suspected.
Hyperaldosteronism causes hypertension but typically with elevations in both diastolic and systolic blood pressure (ie, pulse pressure is not
widened) and it would not explain this patient's weight loss.

(Choice E) Tilt table testing is occasionally used in the workup of vasovagal syncope, which is unlikely to be the cause of this patient's dizziness
given his weight loss and atrial fibrillation.

(Choice G) Plasma or urine metanephrines are elevated with pheochromocytoma, which can cause hypertension and palpitations. However,
most affected patients also have episodic headaches and diaphoresis, and elevations in both diastolic and systolic blood pressure are typical (ie,
pulse pressure is not widened).

Educational objective:
Hyperthyroidism should always be considered in the evaluation of patients with new-onset atrial fibrillation, particularly if other signs or symptoms
are present. Serum TSH and free T4 are the appropriate initial diagnostic tests.

References
Hyperthyroidism and risk of atrial fibrillation or flutter: a population-based study.

(http://www.ncbi.nlm.nih.gov/pubmed/15302638)

Medicine Endocrine, Diabetes & Metabolism Atrial fibrillation


Subject System Topic

https://t.me/USMLEWorldStep2CK

REVIEW

https://www.uworld.com/ClientApp/v15/apps/qbanktestinterface/index.html#/launchtest/7281878/nbme/229400737/3/1 1/1
‫ م‬9:04 2021/‫‏‬9/‫‏‬11 https://t.me/USMLEWorldStep2CK UWorld STEP2 SIM Form 1

1 Item 8 of 40
2
Mark
Question Id: 6871 Previous Next
3

4 A 42-year-old woman comes to the office due to right knee pain for the past 3 days. She was carrying a heavy backpack on a muddy trail when
5 she tripped, twisting her knee. The patient immediately felt sharp pain in the knee but was able to continue hiking. The next morning, she had
6 swelling at the knee and significant medial knee pain with stairs and squatting. The patient is able to fully extend the knee but has a sensation
7 when she walks that the knee is catching, and the pain prevents her from doing her normal daily activities. Medical history is notable for mild
8 obesity, chronic anxiety, and recurrent carpal tunnel syndrome. She does not take any medications and does not use tobacco, alcohol, or illicit
9 drugs. Which of the following is most likely to be found on examination of this patient's knee?
10
 A. Anterior laxity of the tibia relative to the femur (4%)
11
12 B. Excessive valgus deviation with lower leg abduction (12%)
13
 C. Painful click during knee extension and external rotation (70%)
14

15 D. Posterior bulge behind the knee (0%)


16
E. Tenderness at the lateral knee during flexion and extension (0%)
17
18
F. Tenderness at the medial knee below the joint line (12%)

19
20

21 Incorrect 70% 03 secs 06/25/2021


22 Correct answer
C
 Answered correctly  Time Spent  Last Updated
23

24
25
Explanation
26

27

28 Provocative maneuvers for meniscal tear


29 Patient standing on affected leg with knee flexed 20 degrees
30 Thessaly test Internal & external rotation of the knee
31 Positive: pain, catching, or palpable crepitus
32
Knee held in internal & then external rotation
33
McMurray test Flex & extend the knee while palpating the knee joint
34
Positive: pain; audible or palpable click
35

36 This patient has acute knee pain associated with swelling and a catching sensation suggesting a meniscal tear. Meniscal tears commonly result
37 from twisting injuries of the knee during strenuous activity on uneven ground. They can also be seen following minimal trauma in older patients
38 due to chronic degeneration of the cartilage. The pain is usually immediate but severity is variable. Effusions are common but develop slowly
39 and may not be apparent until the next day. Other characteristic symptoms include a sensation of instability and exacerbation of pain with
40 squatting.

Examination of the knee in a meniscal tear will often show tenderness at the joint line. Patients may also have palpable locking, catching, or
crepitus when the joint is rotated or extended while under load (eg, Thessaly test, McMurray test). Diagnosis is best confirmed with MRI, which
effectively visualizes the soft tissues of the knee. Initial treatment should begin with conservative measures (eg, rest, decreased weight bearing),
but many patients will require surgery.

(Choice A) Anterior laxity of the tibia is seen in tears of the anterior cruciate ligament. This injury typically causes rapid-onset hemarthrosis that is
apparent within minutes to hours. This patient's slow-onset effusion and catching with ambulation are more characteristic of meniscus injury.

(Choice B) Valgus instability is seen in tears of the medial collateral ligament. These tears can be associated with a concurrent meniscal injury
but are usually caused by a blow to the lateral knee.

(Choice D) A popliteal (Baker) cyst is enlargement of the gastrocnemius or semimembranosus bursa due to extrusion of synovial fluid posteriorly
from the knee joint. It is typically seen in patients with preexisting joint disease (eg, osteoarthritis).

(Choices E and F) Iliotibial band syndrome is an overuse injury characterized by poorly localized pain at the lateral knee. Examination shows
tenderness at the lateral femoral condyle during flexion and extension. Pes anserinus pain syndrome is an overuse injury characterized by
localized pain and tenderness over the anteromedial tibia, just below the joint line. These conditions typically do not arise suddenly or cause
catching sensations.

Educational objective:
Meniscal tears can result from an acute, twisting injury of the knee or from minimal trauma in older patients with chronic degeneration of the
cartilage. Typical features include joint line tenderness, a slow-onset effusion, sensation of instability, and locking or catching when the joint is
rotated or extended while under load.

References
Evaluating acutely injured patients for internal derangement of the knee.

(http://www.ncbi.nlm.nih.gov/pubmed/22335264)

Surgery Rheumatology/Orthopedics & Sports Knee trauma


Subject System Topic

https://t.me/USMLEWorldStep2CK

REVIEW

https://www.uworld.com/ClientApp/v15/apps/qbanktestinterface/index.html#/launchtest/7281878/nbme/229400737/3/1 1/1
‫ م‬9:04 2021/‫‏‬9/‫‏‬11 https://t.me/USMLEWorldStep2CK UWorld STEP2 SIM Form 1

1 Item 9 of 40
2
Mark
Question Id: 6877 Previous Next
3

4 A 63-year-old Caucasian male is hospitalized after he had a single generalized tonic-clonic seizure. An MRI of his brain reveals a mass in the
5 right frontal lobe, and a subsequent biopsy of the mass is consistent with an astrocytoma. Which of the following tumor characteristics is the best
6 predictor of this patient’s prognosis?
7
8  A. Size of the tumor (13%)

9 B. Metastatic disease (20%)


10
 C. Degree of anaplasia (39%)
11
12 D. Location of the tumor (15%)
13
E. Vascular invasion (11%)
14

15
16

17 Incorrect 39% 02 secs 07/30/2021


18
Correct answer  Answered correctly  Time Spent  Last Updated
C
19
20

21 Explanation
22

23 New-onset seizures are a common presentation of brain tumors. Astrocytomas are the most common brain tumors in adults, and represent a
24 subcategory of gliomas, which include all brain tumors of glial cell origin. Other gliomas include oligodendrogliomas and ependymomas. The
25
most important prognostic factors for astrocytomas include patient age, functional status, and tumor grade. Low-grade astrocytomas are classified

26
as grade I or II. They typically have increased cellular proliferation and atypia, but lack necrosis, mitoses, and neovascularity. Grade III
astrocytomas are also known as anaplastic astrocytomas, and are characterized by an increased number of mitoses. Glioblastoma multiforme
27
(GBM) is a grade IV astrocytoma and has the worst prognosis of all. Neovascularity or necrosis is characteristically present in GBMs. Although
28
some of these high-grade astrocytomas arise de novo, others are believed to result from the progression of lower grade astrocytomas.
29
30
(Choice A) The size of the tumor may impact the symptoms that the patient experiences, but overall mortality is not changed.
31

32
(Choice B) Most patients with astrocytomas die from either the primary tumor or intracranial metastases. Distant metastases are fairly rare.
33
34
(Choice D) The location of the tumor may affect the patient’s functionality and symptoms, but overall survival is usually not affected.
35

36 (Choice E) Neovascularity is a negative prognostic factor since it is seen in patients with GBM, but patients with vascular invasion do not
37 necessarily have a worse prognosis.
38

39 Educational objective:
40 New-onset seizures are a common presentation of brain tumors, the most common of which in adults are astrocytomas. The prognosis of
astrocytomas is most affected by tumor grade, with increased atypia, mitoses, neovascularity, or necrosis conveying a worse prognosis.

Medicine Nervous System Brain tumors


Subject System Topic

https://t.me/USMLEWorldStep2CK

REVIEW

https://www.uworld.com/ClientApp/v15/apps/qbanktestinterface/index.html#/launchtest/7281878/nbme/229400737/3/1 1/1
‫ م‬9:04 2021/‫‏‬9/‫‏‬11 https://t.me/USMLEWorldStep2CK UWorld STEP2 SIM Form 1

1 Item 10 of 40
2
Mark
Question Id: 6884 Previous Next
3

4 A 67-year-old man comes to the office concerned about several swollen lymph nodes in his axilla. The lymph nodes are nonpainful but have been
5 present >3 weeks. He has had no recent fevers, chills, or weight loss. The patient is an investment banker and is careful with his health, noting
6 that he exercises daily and eats organic foods when possible. He does not use tobacco, alcohol, or illicit drugs. He has been married to his wife
7 for 45 years. Temperature is 36.9 C (98.2 F), blood pressure is 156/92 mm Hg, and pulse is 68/min. He is fit and well nourished. There are
8 several enlarged, nontender, mobile lymph nodes in his bilateral cervical and axillary chains. Chest examination is normal. With deep exhalation,
9 a spleen tip is palpable under the left lower ribs. Laboratory results are as follows:
10
Complete blood count
11
12 Hemoglobin 8.5 g/dL
13 Mean corpuscular volume 89 µm3
14
Platelets 90,000/mm3
15
16 Leukocytes 56,800/mm3
17
Neutrophils 8%
18

19
Lymphocytes 88%

20 Monocytes 4%
21

22 Serum chemistry
23
Blood urea nitrogen 12 mg/dL
24
25 Creatinine 1.1 mg/dL
26

27 This patient would benefit most from therapy directed against which of the following?
28
 A. BCR-ABL tyrosine kinase (39%)
29
30  B. CD20 cell surface antigen (49%)
31
C. Epidermal growth factor receptor (2%)
32

33 D. Interleukin-1 receptor (3%)


34 E. Tumor necrosis factor-alpha (4%)
35

36

37
Incorrect 49%
38

39
Correct answer
B
 Answered correctly  03 secs
Time Spent  04/19/2021
Last Updated

40

Explanation

This patient presents with several manifestations of chronic lymphocytic leukemia (CLL) including multi-chain lymphadenopathy,
splenomegaly (palpable spleen tip), mild cytopenias (anemia, thrombocytopenia), and a marked lymphocytic predominant leukocytosis.
Diagnosis can be confirmed with peripheral blood smear (smudge cells) and flow cytometry. Patients with CLL have a median survival of 10
years, and treatment is not beneficial if a patient is asymptomatic. Progressive bone marrow failure (causing cytopenias), massive
lymphadenopathy/splenomegaly, or the presence of severe B symptoms (eg, night sweats, weight loss) are all indications for treatment.

Rituximab, a monoclonal antibody against the CD20 antigen expressed on B lymphocytes, is a common first-line therapy for systemic
manifestations of CLL (often used in combination with fludarabine). Patients on treatment for CLL are at higher risk of infections, which are one of
the leading causes of morbidity and mortality in CLL patients.

(Choice A) Imatinib inhibits the product of the BCR-ABL tyrosine kinase fusion gene in chronic myelogenous leukemia (CML). CML presents with
a marked leukocytosis of predominantly neutrophil (not lymphocyte) lineage.

(Choice C) Monoclonal antibodies directed against epidermal growth factor receptors are used in the treatment of many cancers, including
colorectal, head and neck, and certain types of lung cancer. They are not used in the treatment of CLL.

(Choice D) Interleukin-1 receptor antagonists are useful in the treatment of rheumatoid arthritis.

(Choice E) Tumor necrosis factor-alpha inhibitors are useful in the treatment of inflammatory diseases (eg, rheumatoid arthritis, Crohn disease).

Educational objective:
Patients with chronic lymphocytic leukemia are often asymptomatic at diagnosis. Median survival is 10 years and treatment is not necessary until
advanced symptoms occur. Monoclonal antibodies directed against the CD20 antigen are often part of first-line treatment for systemic disease.

Medicine Hematology & Oncology Chronic lymphocytic leukemia


Subject System Topic

https://t.me/USMLEWorldStep2CK

REVIEW

https://www.uworld.com/ClientApp/v15/apps/qbanktestinterface/index.html#/launchtest/7281878/nbme/229400737/3/1 1/1
‫ م‬9:04 2021/‫‏‬9/‫‏‬11 https://t.me/USMLEWorldStep2CK UWorld STEP2 SIM Form 1

1 Item 11 of 40
2
Mark
Question Id: 6891 Previous Next
3

4 A 39-year-old man is brought to the emergency department due to right arm weakness and difficulty walking since this morning. According to his
5 wife, the patient has not felt well for the past 2 weeks and has noted fatigue, joint pains, and feeling "out of sorts." He has a history of untreated
6 hepatitis C infection. Temperature is 38 C (100.5 F), blood pressure is 120/65 mm Hg, and pulse is 105/min and regular. On funduscopic
7 examination, several small areas of hemorrhage are seen bilaterally. Skin examination is unremarkable. There is decreased muscle strength in
8 the right upper and lower extremities along with hyperreflexia. Which of the following would be most helpful in establishing the diagnosis?
9
 A. Antinuclear antibodies (19%)
10
11  B. Blood culture (32%)
12
C. Bone marrow biopsy (3%)
13

14 D. Carotid ultrasound study (18%)


15
E. Lumbar puncture (26%)
16

17
18

19 Incorrect 32%
20
Correct answer  Answered correctly  02 secs
Time Spent  04/15/2021
Last Updated
B
21

22

23 Explanation
24
25
Infective endocarditis—modified Duke criteria
26

27 Major criteria
28 Blood culture positive for typical microorganism
29
(eg, Staphylococcus aureus, Enterococcus,

30
viridans streptococci)
Echocardiogram showing valvular vegetation
31
Minor criteria
32
Predisposing cardiac lesion
33
Diagnostic criteria Intravenous drug use
34
for IE Temperature >38 C (100.4 F)
35
Embolic phenomena
36
Immunologic phenomena (eg, glomerulonephritis)
37
Positive blood culture not meeting above criteria
38
Definite IE
39 2 major OR 1 major + 3 minor criteria
40 Possible IE
1 major + 1 minor OR 3 minor criteria

Fever (>90%)
Heart murmur (85%)
Petechiae (≤50%)
Clinical findings Subungual splinter hemorrhages (<50%)
(frequency) Osler nodes, Janeway lesions (<50%)
Neurologic phenomena (embolic) (≤40%)
Splenomegaly (≤30%)
Roth spots (retinal hemorrhage) (<5%)

IE = infective endocarditis.

This patient with acute right-sided hemiparesis consistent with stroke also has subacute systemic symptoms, low-grade fever, and bilateral small
retinal hemorrhages. This presentation raises concern for infective endocarditis (IE) with septic emboli to the eye and brain. The presence of
hepatitis C virus (HCV) infection suggests the IE might have been caused by injection drug use, the most common cause of HCV transmission
in the United States.

IE often presents with subacute fever, nonspecific symptoms (eg, malaise, fatigue, arthralgia, myalgia), and/or complications related to septic
emboli to the brain (eg, embolic stroke, brain abscess) or other organs (eg, vertebral osteomyelitis, septic arthritis, kidney infarction).
Examination frequently reveals new cardiac murmur and signs of embolism (eg, petechiae, splinter hemorrhage). Roth spots, small hemorrhagic
lesions on the retina with white centers, are highly suggestive of IE but are seen in only approximately 2% of cases.

Patients with suspected IE require 3 sets of blood cultures from 3 different venipuncture sites to establish a microbiologic diagnosis and an
echocardiogram to evaluate for vegetations/abscess and to assess valve function. This patient with hemiparesis also requires brain imaging (eg,
contrast-enhanced MRI).

(Choice A) Antinuclear antibodies can help diagnose systemic lupus erythematosus (SLE), which often causes stroke due to vasculitis or
antiphospholipid antibodies. However, SLE is associated with cotton-wool spots on the retina, not hemorrhagic lesions. In addition, this patient
has 3 or 4 minor modified Duke criteria (fever >38 C [100.4 F], Roth spots [immunologic phenomena], embolism, possible injection drug use) and
requires evaluation for IE.

(Choice C) Essential thrombocytosis can cause stroke and transient eye symptoms; diagnosis can be made with bone marrow biopsy. However,
low-grade fever, hemorrhagic retinal lesions, and possible injection drug use make IE far more likely.

(Choice D) Carotid atherosclerosis is a common cause of embolic stroke and cotton-wool retinal lesions but is usually seen in adults of advanced
age who have risk factors (eg, diabetes mellitus, hypertension, hypercholesterolemia). Low-grade fever and hemorrhagic retinal lesions would be
uncommon.

(Choice E) Lumbar puncture can confirm the diagnosis of meningitis, which typically presents with headache, neck stiffness, and fever.
Hemiparesis would be atypical. Brain imaging is required prior to lumbar puncture in patients with focal neurologic signs.

Educational objective:
Infective endocarditis often presents with subacute or chronic nonspecific symptoms and/or complications due to septic embolism (eg, stroke,
brain abscess). The diagnosis generally is confirmed with 3 sets of blood cultures and an echocardiogram.

References
Brain abscess.

(http://www.ncbi.nlm.nih.gov/pubmed/25075836)
Brain abscess: current management.

(http://www.ncbi.nlm.nih.gov/pubmed/24174804)

Medicine Nervous System Embolic stroke


Subject System Topic

https://t.me/USMLEWorldStep2CK

REVIEW

https://www.uworld.com/ClientApp/v15/apps/qbanktestinterface/index.html#/launchtest/7281878/nbme/229400737/3/1 1/1
‫ م‬9:05 2021/‫‏‬9/‫‏‬11 https://t.me/USMLEWorldStep2CK UWorld STEP2 SIM Form 1

1 Item 12 of 40
2
Mark
Question Id: 6892 Previous Next
3

4 A 64-year-old man who is being treated for community-acquired pneumonia develops watery diarrhea. He has been having 7-8 bowel movements
5 daily. He now feels weak and nauseated, and has had 1 episode of vomiting over the last 12 hours. His temperature is 37.8 C (100.4 F), blood
6 pressure is 100/60 mm Hg, and pulse is 95/min. Physical examination reveals dry mucous membranes, poor skin turgor, and crackles at the right
7 lung base. His abdomen is soft without tenderness. There is no peripheral edema. His laboratory findings are as follows:
8
Serum sodium 122 mEq/L
9

10 Serum potassium 3.1 mEq/L


11
Chloride 100 mEq/L
12
Bicarbonate 18 mEq/L
13

14 Blood urea nitrogen (BUN) 56 mg/dL


15
Serum creatinine 1.6 mg/dL
16

17 Blood glucose 90 mg/dL


18 Urine sodium 8 mEq/L
19
20 Which of the following is the most likely cause of this patient's hyponatremia?
21

22  A. High plasma osmolality (2%)


23
B. Inappropriate antidiuretic hormone secretion (14%)
24
25  C. Intravascular volume depletion (66%)

26 D. Left ventricular dysfunction (0%)


27
E. Renal tubular necrosis (15%)
28

29
30
31 Incorrect 66%
32
Correct answer
C
 Answered correctly  05 secs
Time Spent  06/18/2021
Last Updated

33
34

35 Explanation
36

37

38

39

40

Hyponatremia can be classified according to the patient's volume status - hypovolemic, euvolemic, or hypervolemic. This patient's history
(vomiting and diarrhea), examination findings (poor skin turgor and dry mucous membranes), and laboratory results (azotemia) suggest that he is
likely hypovolemic due to extrarenal loss of sodium and water. Hypovolemia decreases tissue perfusion, which stimulates antidiuretic hormone
(ADH) secretion and activates the renin-angiotensin-aldosterone system. As a result, extrarenal fluid losses are associated with decreased urine
output and a urine sodium level <20 mEq/L.

Hypovolemic hyponatremia can be corrected in most cases with infusion of isotonic saline. The saline itself will raise serum sodium to a small
degree as it has a higher sodium concentration than the patient's serum. And later, as the volume is replaced, ADH secretion falls and urine
excretion of free water increases. This effect may raise serum sodium levels rapidly and the patient should be monitored closely to avoid
neurologic complications.

(Choice A) Serum osmolality can be calculated as follows:

Serum osmolality = (2 x Na) + (BUN / 2.8) + (Glucose / 18)

An increased serum osmolality (normal is 275-295 mOsm/kg H2O) may be due to severe hyperglycemia, advanced renal failure, or alcohol
intoxication. However, this patient has a decreased serum osmolality [(2 x 122) + (56/2.8) + (90/18)] of 269 mOsm/kg H2O.

(Choice B) The syndrome of inappropriate ADH secretion results in euvolemic hyponatremia due to the retention of free water by the kidneys.

(Choice D) Left ventricular dysfunction can cause hypervolemic hypotonic hyponatremia. Despite the increased plasma and extracellular
volumes, the carotid baroreceptors sense low pressure - as they do with hypovolemic hyponatremia - due to the low cardiac output (decreased
effective circulating volume). This patient has no findings of hypervolemia (eg, peripheral edema, jugular venous distention).

(Choice E) Renal tubular necrosis may be seen in patients with severe hypotension and impaired renal perfusion. It is characterized by
hypervolemic hyponatremia, usually with urine sodium >20 mEq/L.

Educational objective:
Extrarenal fluid losses can result in hypovolemic hyponatremia due to secretion of antidiuretic hormone and retention of free water. Patients have
a decreased serum osmolality, decreased urine sodium concentration (<20 mEq/L), and clinical findings of volume depletion.

References
Diagnosis and management of hyponatraemia in hospitalised patients.

(http://www.ncbi.nlm.nih.gov/pubmed/19769706)

Medicine Renal, Urinary Systems & Electrolytes Hyponatremia


Subject System Topic

https://t.me/USMLEWorldStep2CK

REVIEW

https://www.uworld.com/ClientApp/v15/apps/qbanktestinterface/index.html#/launchtest/7281878/nbme/229400737/3/1 1/1
‫ م‬9:05 2021/‫‏‬9/‫‏‬11 https://t.me/USMLEWorldStep2CK UWorld STEP2 SIM Form 1

1 Item 13 of 40
2
Mark
Question Id: 6896 Previous Next
3

4 A 7-year-old girl with sickle cell disease is brought to the emergency department due to 2 days of decreased appetite and sleepiness. She refused
5 to get out of bed this morning. The patient had an episode of acute chest syndrome 2 years ago that required hospitalization and exchange
6 transfusion. She has no dyspnea, extremity pain, or headache. Her temperature is 39.4 C (103 F), blood pressure is 70/30 mm Hg, pulse is
7 150/min, and respirations are 22/min. On physical examination, the patient is asleep initially but arouses and responds to questions appropriately.
8 Her chest is clear to auscultation and the abdomen is soft and nontender. Passive movements of her extremities are painless. There is no nuchal
9 rigidity. Scattered petechiae are seen on both legs. Which of the following pathogens is most likely to be cultured from this patient's blood?
10
 A. Escherichia coli (8%)
11
12 B. Pseudomonas aeruginosa (3%)
13
C. Salmonella enteritidis (19%)
14

15 D. Staphylococcus aureus (14%)


16
 E. Streptococcus pneumoniae (54%)
17
18

19
20
Incorrect 54%
21
Correct answer
E
 Answered correctly  02 secs
Time Spent  04/07/2021
Last Updated

22

23

24 Explanation
25

26
Infections in sickle cell disease
27

28 Infection Etiology
29 Pneumonia Streptococcus pneumoniae
30
Staphylococcus aureus
31 Osteomyelitis/septic arthritis
Salmonella species
32

33 Streptococcus pneumoniae
Bacteremia/sepsis
34 Haemophilus influenzae type B
35
Meningitis Streptococcus pneumoniae
36

37 Sickle cell disease (SCD) is typically associated with a complete loss of splenic function (ie, functional asplenia) by age 4 due to splenic
38 infarctions from sickled erythrocytes. Loss of splenic function dramatically increases the risk of fulminant bacterial infection because circulating
39 bacteria are primarily destroyed via opsonization followed by splenic macrophage destruction; this is particularly true of encapsulated bacteria
40
such as Streptococcus pneumoniae.

Although immunization and prophylactic penicillin have reduced the rate of pneumococcal sepsis significantly, S pneumoniae remains a leading
cause of bacteremia in patients with SCD. Common presenting manifestations include acute-onset fever, hypotension, tachypnea, and lethargy,
with rapid progression to severe sepsis. Although petechiae are more commonly associated with Neisseria meningitidis infection, they can occur
with any cause of disseminated intravascular coagulation (DIC), including sepsis due to pneumococcal bacteremia; asplenic patients with
pneumococcal infections are also more likely to develop purpura fulminans (characterized by DIC, tissue thrombosis, and skin hemorrhage and
necrosis), reflecting infection severity.

In patients with SCD, S pneumoniae–associated bacteremia, sepsis, pneumonia, and meningitis are associated with a very high mortality. For
patients with SCD who develop a fever, empiric antibiotics should be initiated rapidly, even if the source of infection is not yet identified. A third-
generation cephalosporin (eg, ceftriaxone) is generally used to ensure pneumococcus coverage; vancomycin should be added if there is concern
for sepsis (eg, hemodynamic instability, as in this patient) or meningitis.

(Choice A) Escherichia coli and other Enterobacteriaceae (eg, Klebsiella) can cause sepsis in patients with SCD but are much less common than
pneumococcus.

(Choice B) Pseudomonas aeruginosa can cause superinfection of leg ulcers in patients with vascular insufficiency due to SCD. P aeruginosa is
also a common cause of bacteremia in patients with burn wounds and neutropenia but does not commonly cause isolated bacteremia or sepsis.

(Choices C and D) Salmonella species and Staphylococcus aureus are the 2 most common causes of osteomyelitis in patients with SCD.
However, this patient has no signs of bone infection. In the absence of osteoarticular infection, pneumococcus is the most likely cause of
bacteremia/sepsis in a patient with SCD.

Educational objective:
Children with sickle cell disease are susceptible to splenic infarction, which predisposes them to infection with encapsulated organisms,
particularly Streptococcus pneumoniae. These patients should receive the pneumococcal vaccine and penicillin prophylaxis to lower the risk of
serious pneumococcal infections.

References
Invasive infections and sickle-cell disease

(http://www.ncbi.nlm.nih.gov/pubmed/20797638)
Bacteremia in children with sickle hemoglobinopathies

(http://www.ncbi.nlm.nih.gov/pubmed/22215095)

Pediatrics Hematology & Oncology Sickle cell


Subject System Topic

https://t.me/USMLEWorldStep2CK

REVIEW

https://www.uworld.com/ClientApp/v15/apps/qbanktestinterface/index.html#/launchtest/7281878/nbme/229400737/3/1 1/1
‫ م‬9:05 2021/‫‏‬9/‫‏‬11 https://t.me/USMLEWorldStep2CK UWorld STEP2 SIM Form 1

1 Item 14 of 40
2
Mark
Question Id: 6903 Previous Next
3

4 A 35-year-old woman comes to the office for a routine physical examination. The patient reports occasional palpitations but has no other chronic
5 medical conditions. She has a fractured right upper molar tooth and is due to have an extraction next week. She has no significant medical
6 history but has been told that she has "a murmur." The patient's only medication is a daily oral contraceptive. She has a sedentary lifestyle and
7 reports no dyspnea on routine activity. She developed hives following use of trimethoprim-sulfamethoxazole. The patient does not use tobacco,
8 alcohol, or illicit drugs. Blood pressure is 120/70 mm Hg and pulse is 70/min. Physical examination shows no jugular venous distension in the
9 upright position. Normal breath sounds are heard on chest auscultation. A late systolic murmur heard over the apex prolongs with the Valsalva
10 maneuver. There is no lower extremity edema, and dorsalis pedis and radial pulses are easily palpable. Which of the following is recommended
11 for this patient prior to the dental procedure?
12
 A. Intravenous ampicillin (0%)
13

14 B. Intravenous cefazolin (0%)


15
 C. No antibiotics (100%)
16

17 D. Oral amoxicillin (0%)


18 E. Oral azithromycin (0%)
19
F. Surveillance blood cultures (0%)
20

21

22

23 Incorrect
24 Correct answer
C
Collecting Statistics  03 secs
Time Spent  08/20/2021
Last Updated
25

26

27 Explanation
28

29
30
High-risk conditions for infective endocarditis

31 Prosthetic heart valve


32 Previous infective endocarditis
33 Structural valve abnormality in transplanted heart
34 Unrepaired cyanotic congenital heart disease
35 Repaired congenital heart disease with residual defect
36
This patient who is scheduled to have a dental extraction has a late systolic murmur over the cardiac apex that prolongs (occurs earlier in systole)
37
with Valsalva maneuver (decreased left ventricular blood volume), consistent with mitral valve prolapse (MVP).
38

39 MVP is a common underlying cardiac lesion among patients who develop infective endocarditis (IE). However, current clinical guidelines
40
recommend that no antibiotic prophylaxis be given to patients with MVP prior to invasive procedures: Although the prevalence of MVP in the
general population is relatively high (~4%), the absolute risk of IE in patients with MVP is actually quite low (~1 in 5,725 patient-years).
Furthermore, patients with MVP generally develop IE due to transient bacteremia during normal daily activities (eg, brushing or flossing teeth)
rather than from invasive procedures; and antibiotic prophylaxis has not been shown to decrease episodes of IE.

In contrast, patients with prosthetic heart valves, prosthetic cardiac grafts, or certain congenital cyanotic heart diseases are considered high risk
for adverse outcomes from IE; therefore, they should receive antibiotic prophylaxis prior to invasive procedures associated with transient
bacteremia, including:

Dental work involving gingival penetration, including routine dental cleaning (Streptococcus viridians)
Respiratory procedures that penetrate the mucosa (S viridians)
Skin and soft tissue procedures in areas of active infection (Staphylococcal species)
Gastrointestinal and genitourinary procedures in areas of active infection (Enterococcus)

Prophylaxis for gastrointestinal and genitourinary procedures that do not involve an active infection (eg, endoscopy) do not generally require
prophylaxis due to a low chance of significant bacteremia.

(Choices A, B, D, and E) Ampicillin, cefazolin, amoxicillin, and azithromycin (for penicillin-allergic patients) are suitable antibiotic choices for
prevention of IE in high-risk patients undergoing dental manipulation. However, antibiotic prophylaxis is not indicated in patients with MVP.

(Choice F) All dental procedures that cause significant manipulation of gingival tissue or the periapical region of the teeth can cause transient
bacteremia. However, surveillance blood cultures do not play a role in the prevention of IE.

Educational objective:
Antibiotic prophylaxis for infective endocarditis (IE) is recommended only in patients with high risk of adverse outcomes from IE (eg, prosthetic
heart valve, previous IE). It is not recommended in patients with mitral valve prolapse, other acquired valvular dysfunction (eg, rheumatic fever),
or relatively low-risk congenital heart disease (eg, atrial septal defect, bicuspid aortic valve).

References
Infective Endocarditis in Adults: Diagnosis, Antimicrobial Therapy, and Management of Complications: A Scientific Statement for Healthcare
Professionals From the American Heart Association

(http://www.ncbi.nlm.nih.gov/pubmed/26373316)

Medicine Infectious Diseases Endocarditis


Subject System Topic

https://t.me/USMLEWorldStep2CK

REVIEW

https://www.uworld.com/ClientApp/v15/apps/qbanktestinterface/index.html#/launchtest/7281878/nbme/229400737/3/1 1/1
‫ م‬9:05 2021/‫‏‬9/‫‏‬11 https://t.me/USMLEWorldStep2CK UWorld STEP2 SIM Form 1

1 Item 15 of 40
2
Mark
Question Id: 6910 Previous Next
3

4 At a local laboratory, the normal reference range for serum free T4 levels is 5-12 µg/dL. Free T4 levels are measured in 2 patient samples, one
5 from a population of healthy patients without thyroid disease and one from a population of patients with hyperthyroidism. If the laboratory
6 increases the upper limit of normal serum free T4 levels to 13 µg/dL, how would the sensitivity and specificity of free T4 level measurements for
7 detecting hyperthyroidism in the samples change?
8

9 Sensitivity Specificity
10
 A. ↑ ↑ (1%)
11
12 B. ↑ ↓ (20%)
13
C. ↑ No change (2%)
14

15  D. ↓ ↑ (67%)

16 E. ↓ ↓ (1%)
17
F. ↓ No change (2%)
18

19 G. No change ↑ (1%)
20
H. No change ↓ (0%)
21

22 I. No change No change (2%)


23

24
25
Incorrect 67%
26 Correct answer  Answered correctly  02 secs
Time Spent  08/29/2021
Last Updated
27
D

28

29
Explanation
30
31

32

33
34

35

36

37

38

39

40

The cutoff value of a quantitative diagnostic test determines whether a given result is interpreted as positive or negative. There is often overlap
between the serum values of "healthy" and "diseased" populations: At the cutoff of the top image above, the right tail of the "healthy" curve
represents false positives (FPs) and the left tail of the "diseased" curve represents false negatives (FNs). Therefore, in such instances, a cutoff
value that correctly categorizes all individuals in both populations cannot be chosen.

Sensitivity represents the ability of a test to correctly identify those with a given disease; it is calculated as the number of patients correctly testing
positive (TP) divided by the total number of patients with the disease: Sensitivity = TP / (TP + FN). Specificity represents the ability of a test to
correctly identify those without a given disease; it is calculated as the number of patients correctly testing negative (TN) divided by the total
number of patients without the disease: Specificity = TN / (TN + FP).

Changing the cutoff point to a higher value (shift to the right) would cause fewer individuals with the disease to have a positive test result (↓TP),
so sensitivity would decrease because the numerator TP has decreased; the denominator (TP + FN), which represents all patients with the
disease, does not change with the change in cutoff. Fewer individuals without the disease would also test positive (↑TN), so specificity would
increase because the numerator TN has increased; the denominator (TN + FP), which represents all patients without the disease, does not
change with the change in cutoff.

Changing the cutoff point to a lower value (shift to the left) would cause more patients with the disease to test positive (↑TP), increasing the
sensitivity of the test. However, as a consequence, more patients without the disease would also test positive (↓TN), resulting in decreased
specificity.

Educational objective:
There is always a trade-off between the sensitivity and specificity of a diagnostic test. Typically, as the sensitivity increases the specificity
decreases, and vice versa. The cutoff value of a quantitative diagnostic test determines whether a given result is interpreted as positive or
negative. Increasing the cutoff point typically causes fewer patients with the disease to test positive, decreasing test sensitivity. Fewer patients
without the disease will also test positive, resulting in a decreased number of false positives and increased specificity.

Medicine Biostatistics & Epidemiology Sensitivity, specificity, NPV, PPV


Subject System Topic

https://t.me/USMLEWorldStep2CK

REVIEW

https://www.uworld.com/ClientApp/v15/apps/qbanktestinterface/index.html#/launchtest/7281878/nbme/229400737/3/1 1/1
‫ م‬9:05 2021/‫‏‬9/‫‏‬11 https://t.me/USMLEWorldStep2CK UWorld STEP2 SIM Form 1

1 Item 16 of 40
2
Mark
Question Id: 6915 Previous Next
3

4 A 20-year-old man is brought to the hospital due to severe agitation. He has a diagnosis of schizophrenia and severe intellectual disability. He is
5 totally dependent on his mother for activities of daily living. The patient has been experiencing auditory hallucinations, which he claims are the
6 voices of his neighbors saying that he is "going to hurt little children." On admission, he is uncooperative and physically combative. Blood
7 pressure is 140/90 mm Hg and pulse is 100/min. The patient requires 3 doses of haloperidol in the emergency department. Later that day, he
8 appears less agitated but has muscle stiffness. His head and neck are turned to the left side, and he has difficulty straightening his neck. Which
9 of the following is the best treatment for this patient's current condition?
10
 A. Dantrolene (13%)
11
12  B. Diphenhydramine (79%)
13
C. Lorazepam (6%)
14

15 D. Olanzapine (0%)
16
E. Quetiapine (0%)
17
18

19
20
Incorrect 79%
21
Correct answer
B
 Answered correctly  02 secs
Time Spent  07/17/2021
Last Updated

22

23

24 Explanation
25

26
Antipsychotic extrapyramidal effects Pharmacotherapy*
27

28 Acute Sudden, sustained contraction of the neck, mouth, Benztropine


29
dystonia tongue & eye muscles Diphenhydramine

30 Beta blocker (propranolol)


31 Akathisia Subjective restlessness, inability to sit still Benzodiazepine (lorazepam)
32 Benztropine
33
Benztropine
34 Parkinsonism Gradual-onset tremor, rigidity & bradykinesia
Amantadine
35

36 Gradual onset after prolonged therapy (>6


Tardive Valbenazine
37 months): dyskinesia of the mouth, face, trunk &
dyskinesia Deutetrabenazine
38
extremities

39 *Management may include reducing the dose or switching to another antipsychotic, depending on the clinical scenario.
40
Haloperidol is a high-potency, first-generation antipsychotic medication frequently used to treat dangerous agitation. This patient's abrupt onset
of increased muscle stiffness and neck muscle contraction (torticollis) is consistent with acute dystonia, a type of extrapyramidal symptom
(EPS) due to dopamine receptor D2 antagonism by haloperidol. Other examples of EPS include akathisia, parkinsonism, and tardive dyskinesia.

This patient is at a higher risk for acute dystonias due to his young age, male sex, and the use of a high-potency, first-generation antipsychotic.
Diphenhydramine and anticholinergic medications (eg, benztropine) are the treatment of choice for acute dystonic reactions and are often
administered intramuscularly for rapid effect. Laryngeal dystonias, which appear less frequently, can obstruct the airway and be life-threatening.

(Choice A) Dantrolene is a skeletal muscle relaxant used in the treatment of neuroleptic malignant syndrome; this rare reaction to D2 antagonists
is characterized by high fevers, diffuse rigidity, mental status changes, and autonomic instability.

(Choice C) Lorazepam is a benzodiazepine that is not as effective as diphenhydramine or benztropine for reversing an acute dystonic reaction.
Lorazepam can be more helpful, however, for treating akathisia.

(Choices D and E) Olanzapine and quetiapine are second-generation antipsychotics. Both have a lower incidence of extrapyramidal symptoms
than haloperidol and can be considered for subsequent use in treating this patient's psychosis. However, the patient's acute dystonia needs to be
treated first before considering whether an alternate antipsychotic is needed.

Educational objective:
Acute dystonia is a type of extrapyramidal symptom and a common side effect of antipsychotic medications, particularly high-potency, first-
generation antipsychotics such as haloperidol. Treatment consists of intramuscular benztropine or diphenhydramine.

References
Treatment of movement disorder emergencies.

(http://www.ncbi.nlm.nih.gov/pubmed/24356784)
Haloperidol for psychosis-induced aggression or agitation (rapid tranquillisation).

(http://www.ncbi.nlm.nih.gov/pubmed/28758203)

Psychiatry Psychiatric/Behavioral & Substance Abuse Antipsychotics


Subject System Topic

https://t.me/USMLEWorldStep2CK

REVIEW

https://www.uworld.com/ClientApp/v15/apps/qbanktestinterface/index.html#/launchtest/7281878/nbme/229400737/3/1 1/1
‫ م‬9:05 2021/‫‏‬9/‫‏‬11 https://t.me/USMLEWorldStep2CK UWorld STEP2 SIM Form 1

1 Item 17 of 40
2
Mark
Question Id: 6920 Previous Next
3

4 A 56-year-old woman comes to the office with abdominal discomfort, frequent bloating, and urinary urgency. She has been treated for presumed
5 urinary tract infections twice in the last month, but her symptoms have not resolved. Her last menstrual period was at age 51. The patient has a
6 history of asthma and has been using albuterol more frequently due to mild dyspnea. Her appendix was removed at age 16; she has had no other
7 surgeries. She does not use tobacco, alcohol, or illicit drugs. Blood pressure is 144/92 mm Hg and pulse is 86/min. BMI is 31 kg/m2. Pelvic
8 examination reveals right-sided adnexal fullness and firm nodularity along the rectovaginal septum. Laboratory results are as follows:
9
Hemoglobin 11.1 mg/dL
10
11 Platelets 225,000/mm3
12 Blood urea nitrogen 21 mg/dL
13
Creatinine 1.2 mg/dL
14

15 Chest x-ray reveals a small pleural effusion on the right. Which of the following is the most likely diagnosis in this patient?
16

17  A. Choriocarcinoma (7%)
18
B. Endometrioma (6%)
19
20  C. Epithelial ovarian carcinoma (61%)
21 D. Granulosa cell tumor (2%)
22
E. Mucinous cystadenoma (7%)
23

24 F. Ovarian fibroma (13%)


25

26

27
Incorrect 61%
28 Correct answer
C
 Answered correctly  03 secs
Time Spent  03/31/2021
Last Updated
29
30
31
Explanation
32

33
34 Epithelial ovarian carcinoma
35
Asymptomatic: incidental adnexal mass
36
Clinical presentation Subacute: pelvic/abdominal pain, bloating, early satiety
37
Acute: dyspnea, obstipation/constipation, abdominal distension
38

39
Family history

40
Genetic mutations (BRCA1, BRCA2)
Age ≥50
Risk factors Hormone replacement therapy
Endometriosis
Infertility
Early menarche/late menopause

Oral contraceptives
Protective factors Multiparity
Breastfeeding

Laboratory findings ↑ CA-125

Solid, complex mass


Ultrasound findings Thick septations
Ascites

Surgical exploration
Management
± Chemotherapy

Epithelial ovarian cancer is the most common subtype of ovarian cancer and often occurs in postmenopausal patients. Early in the disease
course, vague symptoms (eg, bloating, urinary urgency) predominate as the growing adnexal mass exerts pressure on the surrounding pelvic
organs (eg, bladder). Because these symptoms are nonspecific, they are often overlooked or dismissed; as a result, the cancer proliferates and
extends beyond the ovarian capsule, spreading throughout the peritoneal cavity. This causes worsening symptoms (eg, abdominal distension,
constipation/obstipation), leading to the eventual but often delayed diagnosis of advanced disease.

Symptoms of advanced disease are based on the location of metastases and development of ascites. This patient has disease throughout the
pelvis, resulting in rectovaginal nodularity and urinary symptoms. The fixed adnexal mass can cause ureteral obstruction, leading to
hydronephrosis and elevated serum creatinine (eg, renal insufficiency). When disease spreads past the diaphragm, patients can develop pleural
effusions (ie, dyspnea) due to either metastatic spread to the pleura or passive diffusion of ascitic fluid across the diaphragm.

(Choice A) Although metastatic choriocarcinoma can cause a pleural effusion, gestational trophoblastic disease is associated with pregnancy (eg,
hydatidiform mole) and does not occur in postmenopausal patients.

(Choice B) Endometriosis can cause urinary symptoms, nodularity of the rectovaginal septum, and an adnexal mass (eg, endometrioma).
Because this condition results from proliferation and cyclic shedding of ectopic endometrial tissue (ie, menses), it is not active after menopause
and therefore unlikely in this patient.

(Choice D) Granulosa cell tumors typically present with a large adnexal mass and symptoms of hyperestrogenism (eg, precocious puberty,
endometrial hyperplasia, postmenopausal bleeding). This patient has no symptoms of increased estrogen secretion.

(Choice E) Mucinous cystadenomas are benign ovarian neoplasms that can grow to large sizes (up to 20 cm) and are often diagnosed when
patients have bulk symptoms (eg, pelvic pressure, urinary frequency/urgency). Benign cystadenomas are less likely in patients with rectovaginal
nodularity and pleural effusion (ie, metastasis).

(Choice F) Ovarian fibromas are benign tumors that may present with Meigs syndrome (ascites and/or pleural effusion). It is unlikely in this
patient with rectovaginal nodularity, which indicates metastatic implants from a malignant process.

Educational objective:
Epithelial ovarian cancer is often diagnosed at an advanced stage, when the disease has extended past the adnexa, causing symptoms more
severe than the initial vague and nonspecific ones. Rectovaginal nodularity combined with a pleural effusion are signs of metastatic spread.

References
Morphological subtypes of ovarian carcinoma: a review with emphasis on new developments and pathogenesis.

(http://www.ncbi.nlm.nih.gov/pubmed/21716157)

Obstetrics & Gynecology Female Reproductive System & Breast Ovarian cancer
Subject System Topic

https://t.me/USMLEWorldStep2CK

REVIEW

https://www.uworld.com/ClientApp/v15/apps/qbanktestinterface/index.html#/launchtest/7281878/nbme/229400737/3/1 1/1
‫ م‬9:06 2021/‫‏‬9/‫‏‬11 https://t.me/USMLEWorldStep2CK UWorld STEP2 SIM Form 1

1 Item 18 of 40
2
Mark
Question Id: 6922 Previous Next
3

4 A 65-year-old woman comes to the emergency department with 24 hours of abdominal pain, nausea, and 2 episodes of vomiting. She describes
5 her pain as "coming in waves." The patient has never had similar symptoms, and her current medical issues include atrial fibrillation, nonischemic
6 cardiomyopathy, hypertension, and type 2 diabetes mellitus. She underwent an abdominal hysterectomy for endometrial hyperplasia 15 years ago
7 and takes multiple medications, including some over-the-counter products. Temperature is 37.2 C (99 F), blood pressure is 139/88 mm Hg, pulse
8 is 98/min, and respiratory rate is 16/min. Lungs are clear to auscultation and percussion. The abdomen is distended, tympanic, and moderately
9 tender. There is no rigidity or rebound, and bowel sounds are active. Abdominal x-ray is shown below.
10
11
12
13

14

15
16

17
18

19
20

21

22

23

24
25

26

27

28

29
30
31

32

33
34

35 This patient's condition is most strongly associated with which of the following?
36

37  A. Diabetic autonomic neuropathy (7%)


38
B. Digoxin toxicity (3%)
39
C. Hypokalemia (2%)
40

D. Peptic ulcer disease (0%)

 E. Previous abdominal surgery (84%)

F. Recent antibiotic use (1%)

Incorrect 84% 03 secs 05/07/2021


Correct answer
E
 Answered correctly  Time Spent  Last Updated

Explanation

This patient's presentation is most consistent with small bowel obstruction (SBO). A history of prior abdominal or pelvic surgery is the most
important risk factor for SBO in high-income countries due to adhesion development. Other conditions that increase risk include abdominal or
inguinal hernias, intestinal inflammation (eg, Crohn disease), malignancy, and a history of abdominal radiation. Common signs and symptoms
include moderate to severe colicky abdominal pain, vomiting, failure to pass flatus or stool (obstipation), abdominal distension, and diffuse
tenderness. Increased pain with hyperactive bowel sounds is sometimes noted during a peristaltic rush, although as the SBO progresses sounds
may become diminished and disappear altogether.

Abdominal x-rays commonly reveal dilated loops of the small bowel (black arrows) with multiple air-fluid levels. CT scans can help identify a
transition point and provide evidence of underlying complications. Most patients with SBO can be managed initially with conservative measures
(eg, bowel rest, nasogastric suction, correction of metabolic derangements). However, those with evidence of complications (eg, fever,
hemodynamic instability) warrant urgent surgical exploration.

(Choice A) Diabetic autonomic neuropathy can result in gastroparesis and enteropathy. However, gastroparesis commonly presents with early
satiety and vomiting, and enteropathy with nocturnal diarrhea and fecal incontinence. Neither would present acutely nor demonstrate dilated loops
of the small bowel on x-ray.

(Choice B) Digoxin toxicity frequently presents with cardiac, neurologic, and gastrointestinal symptoms (eg, nausea, vomiting, abdominal pain),
but abdominal distension and obstipation would not be present.

(Choice C) Electrolyte abnormalities such as hypokalemia can lead to paralytic ileus, which presents similarly to SBO. However, paralytic ileus is
more likely to present with decreased bowel sounds and constant pain instead of colicky pain. Radiography is also more likely to demonstrate a
dilated colon but less commonly shows a transition point or air-fluid levels.

(Choice D) Peptic ulcer disease does not usually cause abnormal findings on plain radiography unless perforation occurs, in which case free air
will be seen on an upright or decubitus film.

(Choice F) Recent antibiotic use can lead to Clostridium difficile colitis. Plain abdominal films are typically normal unless toxic megacolon
develops, in which case there is profound dilation of the colon.

Educational objective:
Small bowel obstruction commonly presents with moderate to severe colicky abdominal pain, vomiting, obstipation, distension, and diffuse
tenderness. Prior abdominal or pelvic surgery is an important risk factor, but conditions including hernias, intestinal inflammation, malignancy, and
prior radiation also increase the risk.

References
Adhesive small bowel adhesions obstruction: evolutions in diagnosis, management and prevention.

(http://www.ncbi.nlm.nih.gov/pubmed/27022449)
Adult small bowel obstruction.

(http://www.ncbi.nlm.nih.gov/pubmed/23758299)

Surgery Gastrointestinal & Nutrition Bowel obstruction


Subject System Topic

https://t.me/USMLEWorldStep2CK

REVIEW

https://www.uworld.com/ClientApp/v15/apps/qbanktestinterface/index.html#/launchtest/7281878/nbme/229400737/3/1 1/1
‫ م‬9:06 2021/‫‏‬9/‫‏‬11 https://t.me/USMLEWorldStep2CK UWorld STEP2 SIM Form 1

1 Item 19 of 40
2
Mark
Question Id: 6926 Previous Next
3

4 A 63-year-old man comes to the office due to shortness of breath on exertion over the last 6 months. The patient describes his symptoms as
5 progressive, causing him to decrease his outside activities recently. He has no dyspnea at rest. During evaluation, Doppler flow tracings across
6 his aortic and mitral valves are acquired from an apical window. The patient's ECG and flow velocity tracings are presented above normal tracings
7 in the exhibit below.
8

10
11
12
13

14

15
16

17
18

19
20

21

22

23

24
25

26

27
This patient most likely suffers from which of the following?
28

29  A. Aortic regurgitation (40%)


30
 B. Aortic stenosis (57%)
31

32 C. Mitral regurgitation (1%)


33
D. Mitral stenosis (0%)
34

35
E. Mitral valve prolapse (0%)

36

37

38 Incorrect 57%
39 Correct answer  Answered correctly  02 secs
Time Spent  04/15/2021
Last Updated
B
40

Explanation

Doppler echocardiography is useful for detecting the presence and severity of valvular regurgitation or stenosis. The ultrasound beam records
blood flow and velocity across the heart valves:

Blood flow toward the transducer is recorded above the baseline.


Blood flow away from the transducer is recorded below the baseline.

Using the apical window (with the transducer placed at the cardiac apex), aortic flow appears as a shift below the baseline and mitral flow
appears above the baseline.

In patients with valvular stenosis (ie, smaller cross-sectional valve area), obstruction to blood flow causes flow velocity to increase in order to
maintain overall forward blood flow across the valve. This increased flow velocity, detected as a murmur on auscultation, can be measured
during Doppler echocardiography.

This patient's graph, taken from an apical window, depicts a more pronounced trough (below the baseline) in blood flow velocity across the aortic
valve compared to normal; given that blood flow across the aortic valve is away from the transducer in the apical window, this more pronounced
trough reflects higher flow velocity across the valve, consistent with obstruction to forward flow due to valvular aortic stenosis.

(Choice A) Aortic regurgitation refers to retrograde blood flow from the aorta to the left ventricle during diastole. With the apical window, blood
flow would be toward the transducer and appear as a shift above (rather than below) the baseline.

(Choices C and E) Mitral regurgitation can occur by itself or in association with mitral valve prolapse (seen on echocardiography as billowing of a
portion of the mitral leaflets). Mitral regurgitation causes backflow of blood from the left ventricle into the left atrium. Recorded from the apical
position, it would be identified as velocity below the baseline on the graph.

(Choice D) Mitral stenosis causes an increase in blood flow velocity across the mitral valve. The graph does not depict any change in blood flow
velocity across the patient's mitral valve.

Educational objective:
Doppler echocardiography uses the ultrasound beam to record blood flow and velocity across the heart valves. The presence of valvular stenosis
or regurgitation changes the flow velocity across the valves; this change can be measured and quantified in patients with valvular heart disease.

References
2014 AHA/ACC guideline for the management of patients with valvular heart disease: a report of the American College of
Cardiology/American Heart Association Task Force on Practice Guidelines.

(http://www.ncbi.nlm.nih.gov/pubmed/24603191)

Medicine Cardiovascular System Aortic stenosis


Subject System Topic

https://t.me/USMLEWorldStep2CK

REVIEW

https://www.uworld.com/ClientApp/v15/apps/qbanktestinterface/index.html#/launchtest/7281878/nbme/229400737/3/1 1/1
‫ م‬9:06 2021/‫‏‬9/‫‏‬11 https://t.me/USMLEWorldStep2CK UWorld STEP2 SIM Form 1

1 Item 20 of 40
2
Mark
Question Id: 6929 Previous Next
3

4 A 42-year-old woman comes to the emergency department due to severe epigastric pain for the past 3 hours. The pain is constant and deep and
5 radiates to her back. She also has nausea and has had 3 episodes of vomiting since the pain started. The patient has a history of hyperlipidemia
6 and takes simvastatin. She drinks 1 or 2 glasses of wine on social occasions but does not use tobacco or recreational drugs. Temperature is 38.2
7 C (100.8 F), blood pressure is 140/90 mm Hg, and pulse is 110/min. BMI is 31 kg/m2. Abdominal examination shows voluntary guarding and
8 tenderness. There is no rebound tenderness and bowel sounds are normal. Laboratory results are as follows.
9
Leukocytes 14,000/mm3
10
11
Liver function studies
12
13 Total bilirubin 1.0 mg/dL
14 Alkaline phosphatase 90 U/L
15
Aspartate aminotransferase 37 U/L
16

17 Alanine aminotransferase 79 U/L


18
Serum lipase normal
19
20 Which of the following is the most likely cause of this patient's abdominal pain?
21

22  A. Acute cholecystitis (51%)

23 B. Acute viral hepatitis (4%)


24
C. Alcoholic pancreatitis (5%)
25

26 D. Common bile duct stricture (5%)


27
E. Helicobacter pylori infection (20%)
28

29 F. Medication adverse effect (9%)


30 G. Pancreatic neoplasm (1%)
31

32

33
34 51% 04 secs 04/04/2021
35
Correct  Answered correctly  Time Spent  Last Updated

36

37

38
Explanation

39
This patient has the classic triad of acute cholecystitis (AC), with constant right upper quadrant or epigastric pain (which can radiate to the
40
right shoulder or back and is often associated with nausea or vomiting), fever, and leukocytosis. Examination may also show guarding due to
peritoneal irritation. AC usually occurs in the setting of cystic duct obstruction due to gallstones. Patients may have preceding episodic periods of
transient pain due to temporary obstruction of the cystic duct by gallstones. Unlike in AC, gallbladder inflammation is not present in biliary colic.
Another, less common form of AC is acalculous cholecystis, which most frequently occurs in critically ill patients.

Common risk factors for the development of gallstones include obesity, female sex, older age (>40 years), and pregnancy. Other risk factors
include rapid weight loss, hemolytic anemias, hypertriglyceridemias, and conditions that result in biliary stasis (eg, diabetes mellitus). Diagnosis is
frequently confirmed with abdominal ultrasound.

(Choice B) Acute viral hepatitis is characterized by marked elevations in serum bilirubin, aminotransferases, and alkaline phosphatase. Physical
examination frequently shows jaundice and hepatomegaly.

(Choice C) Alcoholic pancreatitis presents very similarly to AC. However, this patient's lack of alcohol consumption, liver function test
abnormalities, and normal lipase make this diagnosis much less likely.

(Choice D) Biliary stricture can present with symptoms of biliary colic or obstructive jaundice, which in severe cases may lead to acute cholangitis
(eg, fever, abdominal pain, jaundice) rather than cholecystitis.

(Choice E) Helicobacter pylori is a risk factor for peptic ulcer disease, which can cause postprandial dyspepsia and epigastric pain radiating to the
back. Fever and leukocytosis are not commonly seen unless there is a complication such as perforation, which also presents with hypotension
and peritoneal signs.

(Choice F) Certain medications such as oral contraceptives, ceftriaxone, and octreotide can predispose to the formation of gallstones, but this
patient does not have a prescription history of any of these.

(Choice G) Pancreatic cancer may commonly present with chronic, not acute, epigastric pain in addition to weight loss, anorexia, and
hyperbilirubinemia.

Educational objective:
Acute cholecystitis commonly presents with the triad of constant right upper quadrant or epigastric pain (with radiation to the right shoulder or
back), fever, and leukocytosis. Common risk factors for the development of gallstones include obesity, female sex, older age (>40 years), and
pregnancy.

References
Surgical and nonsurgical management of gallstones.

(http://www.ncbi.nlm.nih.gov/pubmed/24866215)

Surgery Gastrointestinal & Nutrition Cholecystitis


Subject System Topic

https://t.me/USMLEWorldStep2CK

REVIEW

https://www.uworld.com/ClientApp/v15/apps/qbanktestinterface/index.html#/launchtest/7281878/nbme/229400737/3/1 1/1
‫ م‬9:06 2021/‫‏‬9/‫‏‬11 https://t.me/USMLEWorldStep2CK UWorld STEP2 SIM Form 1

1 Item 21 of 40
2
Mark
Question Id: 6935 Previous Next
3

4 A 26-year-old nulligravid woman comes to the office for evaluation of infertility. She and her partner have had intercourse 4 or 5 times a week for
5 the past year in an attempt to become pregnant. The patient's periods have always been irregular, averaging 2 every 6 months, with heavy
6 menstrual bleeding. The last menstrual period was 6 weeks ago and lasted 7 days. She has no headache, visual abnormalities, or nipple
7 discharge. The patient also reports excess body hair that requires frequent shaving or waxing. She has no other medical conditions and has
8 never had any surgery. The patient takes no medications except a multivitamin and does not use tobacco, alcohol, or illicit drugs. She has no
9 history of sexually transmitted infections. The patient's father has type 2 diabetes mellitus. BMI is 33 kg/m2. Blood pressure is 130/80 mm Hg
10 and pulse is 80/min. Physical examination shows thick hair over the upper lip and oily skin. The abdomen is soft and nontender, with no masses.
11 Pelvic examination reveals a normal uterus and no adnexal masses. There is no clitoromegaly. Pubic hair and breast development is Tanner
12
stage 5. Urine pregnancy test is negative. Serum chemistries, TSH, and prolactin levels are normal. The partner's semen analysis is normal.

13
Which of the following is the best next step to improve this patient's chance of conception?

14
 A. Bromocriptine (2%)
15
16 B. Cyclic estrogen with progesterone (12%)
17
C. Hysteroscopic lysis of adhesions (0%)
18

19
D. Intrauterine insemination (1%)

20 E. Spironolactone (2%)
21
F. Tubal flushing (0%)
22

23  G. Weight loss (79%)


24
25

26
Incorrect 79% 02 secs 08/21/2021
27 Correct answer
G
 Answered correctly  Time Spent  Last Updated
28

29
30
Explanation
31

32

33 Polycystic ovary syndrome


34
Androgen excess (eg, acne, male pattern baldness, hirsutism)
35
Oligoovulation or anovulation (eg, menstrual irregularities)
36 Clinical features
Obesity
37
Polycystic ovaries on ultrasound
38
↑ Testosterone levels
39
Pathophysiology ↑ Estrogen levels
40
LH/FSH imbalance

Metabolic syndrome (eg, diabetes, hypertension)


Obstructive sleep apnea
Comorbidities
Nonalcoholic steatohepatitis
Endometrial hyperplasia/cancer

Weight loss (first-line)


Treatment options Oral contraceptives for menstrual regulation
Letrozole for ovulation induction

Polycystic ovary syndrome (PCOS) is a common cause of anovulatory infertility. The typical presentation includes a history of irregular
menses, obesity, and signs of androgen excess (eg, hirsutism). Patients with PCOS are typically obese and have an imbalance of LH and FSH,
resulting in anovulation. The first step in infertility management is weight loss as it can restore normal ovulation through a reduction in adipose
tissue volume and resultant decreased peripheral estrogen conversion. Normalization of estrogen levels resolves the LH/FSH imbalance, allowing
for an LH surge and subsequent ovulation. If lifestyle changes are ineffective, infertility can be managed with ovulation induction agents (eg,
letrozole).

(Choice A) Bromocriptine is used to treat hyperprolactinemia caused by pituitary adenomas, which can cause infertility through anovulation. This
patient has a normal prolactin level and no evidence of galactorrhea.

(Choice B) In patients with PCOS who are not pursuing pregnancy, treatment focus is on preventing uncontrolled endometrial proliferation (eg,
endometrial hyperplasia) due to chronic anovulation. First-line therapy is cyclic estrogen/progestin administration (eg, combined oral contraceptive
pills).

(Choice C) Hysteroscopic lysis of adhesions is the treatment for intrauterine synechiae (eg, Asherman syndrome). Typical patient presentation is
secondary amenorrhea and cyclic pelvic pain after intrauterine trauma (eg, dilation and curettage).

(Choice D) Intrauterine insemination treats male factor infertility in the setting of an abnormal semen analysis.

(Choice E) Spironolactone can be used in patients with PCOS to treat hirsutism by blocking peripheral androgen receptors; it does not induce
ovulation.

(Choice F) Tubal flushing, performed during either a diagnostic hysterosalpingogram or diagnostic laparoscopy, may increase the likelihood of
pregnancy by clearing the fallopian tubes of debris. Tubal blockage is more likely in patients with a history of sexually transmitted infections (eg,
pelvic inflammatory disease).

Educational objective:
Polycystic ovary syndrome may present with infertility, oligomenorrhea (eg, anovulation), hirsutism, and obesity. Weight loss leads to resumption
of normal ovulation and is the first-line therapy for infertility management.

References
Genetic, hormonal and metabolic aspects of PCOS: an update.

(http://www.ncbi.nlm.nih.gov/pubmed/27423183)

Obstetrics & Gynecology Female Reproductive System & Breast Polycystic ovary disease
Subject System Topic

https://t.me/USMLEWorldStep2CK

REVIEW

https://www.uworld.com/ClientApp/v15/apps/qbanktestinterface/index.html#/launchtest/7281878/nbme/229400737/3/1 1/1
‫ م‬9:06 2021/‫‏‬9/‫‏‬11 https://t.me/USMLEWorldStep2CK UWorld STEP2 SIM Form 1

1 Item 22 of 40
2
Mark
Question Id: 6937 Previous Next
3

4 A 62-year-old man comes to the office with 3 weeks of occasional severe dizziness. The dizziness typically occurs either at night when he rolls
5 over in bed or in the morning when he tries to get up. His symptoms consist of 30-40 seconds of a severe spinning sensation. His past medical
6 history is significant for diabetes mellitus type 2 that is treated with oral antihyperglycemia agents. Blood pressure is 160/100 mm Hg and pulse is
7 75/min. Which of the following is most likely to decrease this patient's symptoms?
8

9  A. Head positioning maneuvers (75%)

10 B. Head of bed elevation (6%)


11
C. Hypertension treatment (14%)
12
13 D. Tight blood glucose control (2%)
14
E. Low-dose aspirin (1%)
15
16

17
18 75%
19
Correct  Answered correctly  02 secs
Time Spent  07/30/2021
Last Updated

20

21

22 Explanation
23

24 The initial step in evaluating a patient with dizziness is to classify the patient’s complaints as either vertigo (a sensation of spinning), presyncope
25
(feeling like one is about to faint), or disequilibrium (a sense of imbalance). This patient’s complaint of a spinning sensation categorizes his

26
dizziness as vertigo. Vertigo can be further categorized as either central or peripheral. This patient’s description of intermittent, short episodes of
vertigo is most consistent with a peripheral etiology, and given his history of vertigo that occurs after rolling over in bed, benign paroxysmal
27
positional vertigo (BPPV) is the most likely diagnosis.
28

29 BPPV occurs secondary to the displacement of otoliths from their normal location in the utricle into the adjacent semicircular canals. The
30 diagnosis can be confirmed by performing the Dix-Hallpike maneuver, which consists of placing the head in particular positions in order to
31 observe for the development of nystagmus. The most beneficial treatment for BPPV includes using certain maneuvers to reposition the displaced
32 otoliths, such as the Epley maneuver. Medications are generally not particularly effective.
33
34 (Choice B) BPPV attacks are not precipitated by lying in a supine position, but rather by certain types of head movements. As a result, elevating
35
the head of the bed is unlikely to be beneficial, and will not solve the underlying problem of the displaced otoliths.

36
(Choice C) Although this patient’s blood pressure is elevated at 160/100 mm Hg, it is unlikely the cause of his symptoms and is a separate issue
37
from his presenting complaint.
38

39
(Choice D) This patient would likely benefit from tight blood glucose control in the long term, but abnormal glucose levels are currently not the
40
cause of his symptoms. In addition, hypoglycemia should be avoided because if severe, it can cause cognitive dysfunction and may be
misinterpreted as dizziness.

(Choice E) Cerebellar stroke is a rare cause of vertigo, and this patient’s diabetes and hypertension do put him at risk for cerebrovascular
disease. However, he has no additional neurologic findings and his history is classic for BPPV. While placing him on low-dose aspirin may be
reasonable, it is unlikely to improve his symptoms.

Educational objective:
Episodic dizziness that occurs while rolling over in bed is a classic history for benign paroxysmal positional vertigo (BPPV). The most effective
treatment for BPPV is repositioning the displaced otoliths with techniques such as the Epley maneuver.

Medicine Nervous System Benign paroxysmal positional vertigo


Subject System Topic

https://t.me/USMLEWorldStep2CK

REVIEW

https://www.uworld.com/ClientApp/v15/apps/qbanktestinterface/index.html#/launchtest/7281878/nbme/229400737/3/1 1/1
‫ م‬9:07 2021/‫‏‬9/‫‏‬11 https://t.me/USMLEWorldStep2CK UWorld STEP2 SIM Form 1

1 Item 23 of 40
2
Mark
Question Id: 6941 Previous Next
3

4 A 12-year-old boy is being evaluated after he fell while hiking in the woods. An x-ray of his left arm is shown on the slide below.
5
6

7
8

10
11
12
13

14

15
16

17
18

19
20

21

22

23

24
25

26

27

28

29
30
31

32 Which of the following structures is at a high risk of being injured in this patient?
33
34  A. Brachial artery (24%)

35 B. Median nerve (2%)


36
 C. Radial nerve (69%)
37

38 D. Ulnar nerve (1%)


39 E. Musculocutaneous nerve (1%)
40

Incorrect 69%
Correct answer  Answered correctly  03 secs
Time Spent  07/30/2021
Last Updated
C

Explanation

This patient has a displaced, oblique fracture involving the middle portion of his left humerus (midshaft humeral fracture). The radial nerve
travels in the spiral groove along the dorsal aspect of the humerus, and is particularly prone to damage in such a fracture. A thorough
neurovascular examination should be done in all patients with fractures in their extremities. The radial nerve is assessed by testing the sensation
on the dorsal surface of the hand as well as the strength of the wrist and finger extensors. An injury to the radial nerve may cause wrist drop due
to weakness of the extensor muscles.

(Choice A) Brachial artery injury is rarely caused by a midshaft fracture of the humerus, but a careful physical examination should include
assessment of the peripheral pulses as well as inspection for any rapidly developing hematomas because this injury could potentially be life
threatening.

(Choice B) The median nerve is not usually injured in a midshaft humerus fracture, but is susceptible to injury in fractures that involve the
supracondylar region of the humerus.

(Choice D) The ulnar nerve is usually injured as it courses along the medial epicondyle. Ulnar nerve function is assessed by testing sensation in
the 4th and 5th digits as well as the strength of abduction and adduction of the fingers.

(Choice E) Injury to the musculocutaneous nerve is rare. If it occurs, it may cause weakness of elbow flexion and forearm supination.

Educational objective:
Radial nerve injury is a frequent complication of humeral midshaft fractures, and results in weakness of the wrist and finger extensors as well as
loss of sensation in the dorsum of the hand.

Surgery Rheumatology/Orthopedics & Sports Upper extremity long bone fracture


Subject System Topic

https://t.me/USMLEWorldStep2CK

REVIEW

https://www.uworld.com/ClientApp/v15/apps/qbanktestinterface/index.html#/launchtest/7281878/nbme/229400737/3/1 1/1
‫ م‬9:07 2021/‫‏‬9/‫‏‬11 https://t.me/USMLEWorldStep2CK UWorld STEP2 SIM Form 1

1 Item 24 of 40
2
Mark
Question Id: 6950 Previous Next
3

4 A 63-year-old man comes to the office due to severe itching that interferes with his sleep. He has also noted yellow discoloration of his eyes and
5 recently his skin. His appetite is decreased lately and he has lost 6.8 kg (15 lb) over the past 3 months. His stools are pale. The patient has a
6 history of type 2 diabetes and hypertension. He has smoked a pack of cigarettes daily for the past 30 years and drinks occasionally. Vital signs
7 are within normal limits. Examination shows marked jaundice. Abdominal CT scan shows a mass in the head of the pancreas and multiple liver
8 lesions suspicious for metastasis. Biopsy of the pancreatic lesion shows adenocarcinoma. He is diagnosed with nonresectable metastatic
9 pancreatic cancer. His current total bilirubin level is 15.4 mg/dL. Which of the following would be most effective for palliative symptom control in
10 this patient?
11
 A. Daily antihistamine therapy (8%)
12
13 B. Daily benzodiazepine therapy (0%)
14
 C. Endoscopic stent placement (47%)
15
16 D. Surgical bypass procedure (11%)
17 E. Ursodeoxycholic acid (32%)
18

19
20

21
Incorrect 47% 01 sec 03/21/2021
22
Correct answer
C
 Answered correctly  Time Spent  Last Updated

23

24
25 Explanation
26
This patient has advanced pancreatic adenocarcinoma, which can obstruct the common bile duct (CBD) and cause extrahepatic cholestasis.
27
Patients typically develop jaundice and pruritus due to direct (conjugated) hyperbilirubinemia and are at increased risk for cholangitis. Given the
28
poor overall prognosis in surgically unresectable pancreatic cancer, palliative care is usually preferred.
29
30 Endoscopic stent placement can effectively relieve CBD obstruction in most patients and is less invasive and less risky than surgery. Surgical
31 bypass (eg, anastomosis between the gallbladder or CBD and jejunum) is sometimes considered as a second-line option in patients in whom stent
32 placement would be technically challenging (Choice D). Studies have shown no difference in survival between the 2 options.
33 (Choice A) Antihistamines can relieve widespread pruritus in conditions with increased histamine (eg, urticaria, mastocytosis), but they are less
34 effective in treating pruritus due to cholestasis.
35
(Choice B) Benzodiazepines are not useful for decreasing pruritus due to obstructive jaundice. Some studies have shown that μ-opioid receptor
36
antagonists (eg, naltrexone, naloxone) can relieve pruritus in some patients with cholestasis.
37

38 (Choice E) Patients with pruritus due to intrahepatic cholestasis may benefit from replacing hydrophobic bile salts with hydrophilic bile salts (eg,
39
ursodeoxycholic acid). However, CBD obstruction must be relieved in extrahepatic cholestasis to improve jaundice and pruritus.

40 Educational objective:
Advanced pancreatic cancer can cause jaundice and pruritus by obstructing the common bile duct (extrahepatic cholestasis). Treatment is usually
palliative with endoscopic common bile duct stent placement to relieve the obstruction in most patients.

References
Malignant distal biliary obstruction: a systematic review and meta-analysis of endoscopic and surgical bypass results.

(http://www.ncbi.nlm.nih.gov/pubmed/17157990)

Medicine Gastrointestinal & Nutrition Pancreatic cancer


Subject System Topic

https://t.me/USMLEWorldStep2CK

REVIEW

https://www.uworld.com/ClientApp/v15/apps/qbanktestinterface/index.html#/launchtest/7281878/nbme/229400737/3/1 1/1
‫ م‬9:07 2021/‫‏‬9/‫‏‬11 https://t.me/USMLEWorldStep2CK UWorld STEP2 SIM Form 1

1 Item 25 of 40
2
Mark
Question Id: 6959 Previous Next
3

4 A 65-year-old woman comes to the office due to progressive dyspnea on exertion and dry cough over the last 3 months. The patient has difficulty
5 sleeping flat and has lost 4.5 kg (10 lb) during this period. She has not had chest pain. Her medical history includes hypothyroidism,
6 hypertension, type 2 diabetes, hyperlipidemia, and coronary artery disease. She underwent coronary artery bypass grafting surgery 6 years ago.
7 Three years ago, she had a left mastectomy for breast cancer followed by radiation and chemotherapy. Her temperature is 37 C (98.6 F), blood
8 pressure is 110/80 mm Hg, pulse is 90/min, and respirations are 22/min. On examination, there is no jugular venous distension or
9 lymphadenopathy. There is dullness to percussion and decreased breath sounds over the left lung base, but the rest of the pulmonary
10 examination is normal. The abdomen is soft and nontender without organomegaly. There is no peripheral edema. Chest x-ray reveals a large left
11 pleural effusion. Which of the following is the most likely diagnosis in this patient?
12
 A. Congestive heart failure (7%)
13

14 B. Hypothyroidism (0%)
15
 C. Malignant pleural effusion (87%)
16

17 D. Parapneumonic effusion (4%)


18 E. Pulmonary embolism (0%)
19
20

21
Incorrect 87%
22

23
Correct answer
C
 Answered correctly  02 secs
Time Spent  06/14/2021
Last Updated

24
25

26 Explanation

27
This patient with 3 months of progressive symptoms has a large, nonpainful, unilateral (left-sided) pleural effusion. In the setting of a cancer
28
history and rapid weight loss, these manifestations suggest a malignant pleural effusion. A thoracentesis is required to confirm the
29
diagnosis. Malignant pleural effusions are exudative and marked by fluid with low pH (<7.2), low glucose (<60 mg/dL), and often very high lactate
30
dehydrogenase levels. Cytology of pleural fluid is often low yield for the diagnosis of malignancy. Additional imaging (eg, PET scan, CT scan) is
31
typically required to identify a source to biopsy. The most common causes of malignant pleural effusions are breast cancer and lung cancer.
32

33 (Choice A) Congestive heart failure (CHF) typically causes bilateral, transudative effusions often associated with signs of volume overload (eg,
34 jugular venous distension, lower extremity edema, ascites). CHF can occasionally cause a unilateral effusion, but it tends to be on the right side.
35
With a left unilateral effusion, no volume overload, and rapid weight loss, a malignant cause is far more likely.

36 (Choice B) Hypothyroidism commonly causes symptoms of fatigue, weight gain (not loss), cold intolerance, constipation, and dry skin. It is a rare
37 cause of pleural effusion.
38
(Choice D) Pneumonia often results in parapneumonic effusions. These present acutely (not subacutely) and are marked by pleuritic pain and
39
signs of infection (eg, fever, chills). This patient has a painless effusion, no evidence of infiltrate by chest x-ray, and no signs of infection.
40
(Choice E) A pulmonary embolism (PE) can cause a small, exudative pleural effusion. Such effusions are inflammatory and marked by pain.
With a large, painless pleural effusion and pulmonary symptoms for 3 months, PE is an unlikely cause.

Educational objective:
Malignancy should be suspected in patients with large unilateral pleural effusions without evidence of infection. The most common causes of
malignant pleural effusions are breast cancer and lung cancer.

Medicine Pulmonary & Critical Care Pleural effusion


Subject System Topic

https://t.me/USMLEWorldStep2CK

REVIEW

https://www.uworld.com/ClientApp/v15/apps/qbanktestinterface/index.html#/launchtest/7281878/nbme/229400737/3/1 1/1
‫ م‬9:07 2021/‫‏‬9/‫‏‬11 https://t.me/USMLEWorldStep2CK UWorld STEP2 SIM Form 1

1 Item 26 of 40
2
Mark
Question Id: 6967 Previous Next
3

4 A 3-year-old African American boy is brought to the emergency department with severe lower extremity pain that started 3 hours ago. He points to
5 his lower back and thighs when asked where the pain is worst. He had cough and congestion last week, during which time his oral intake was
6 decreased. Temperature is 37.2 C (99 F), blood pressure is 110/70 mm Hg, and pulse is 140/min. There is prominent tenderness with any
7 palpation of the thighs. No erythema, tenderness, or warmth of the hips or knees is present. Strength is intact throughout the lower extremities.
8 Laboratory results are as follows:
9
Hemoglobin 8.5 g/dL
10
11 Mean corpuscular volume 85 fL
12 Leukocytes 15,500/mm3
13
Platelets 390,000/mm3
14

15 Total bilirubin 2.1 mg/dL


16
Direct bilirubin 0.2 mg/dL
17
18
Lactate dehydrogenase 700 U/L

19
Which of the following is the most likely cause of this patient's back pain?
20

21  A. Bone infection (8%)


22
B. Bone necrosis (11%)
23

24 C. Deep venous thrombosis (0%)


25
 D. Microvascular occlusion (76%)
26

27
E. Muscle inflammation (1%)

28 F. Nerve root compression (0%)


29
30
31
Incorrect 76% 04 secs 07/04/2021
32

33
Correct answer
D
 Answered correctly  Time Spent  Last Updated

34

35

36
Explanation

37

38

39

40

This African-American patient with severe pain in the setting of anemia, leukocytosis, and signs of hemolysis (eg, elevated lactate
dehydrogenase, elevated indirect bilirubin) likely has a vaso-occlusive pain crisis due to sickle cell disease (SCD). SCD pain is due to
microvascular occlusion from sickled red blood cells, leading to local tissue damage, hypoxia, and acidosis. The hypoxia and acidosis lead to
sickling of additional red blood cells that hemolyze, causing elevations of lactate dehydrogenase and indirect bilirubin. Patients with pain crises
may also have leukocytosis due to stress.

Pain crises are the most common reason that patients with SCD seek medical attention. Episodes usually involve severe pain at multiple sites.
Triggers include viral infections, dehydration, cold, and stress. No definitive test exists to confirm the diagnosis of SCD pain crises, and patients
should be treated appropriately based on their subjective reports of pain. Treatment includes hydration, nonsteroidal anti-inflammatory drugs, and
opioids.

(Choice A) Osteomyelitis, like SCD, can present with bony pain and leukocytosis. Patients with osteomyelitis, however, tend to be febrile, and
pain is confined to one location.

(Choice B) Patients with SCD are predisposed to avascular necrosis of the femoral and humeral heads. This progressive destruction of bone
can be bilateral, but the pain is typically chronic rather than acute.

(Choice C) Deep venous thrombosis usually occurs in older adults with hypercoagulability risk factors (eg, malignancy, indwelling catheters,
prolonged immobility). Pain and swelling of the calf is most common unilaterally as bilateral thromboses are rare.

(Choice E) Viral myositis, or inflammation of the muscles, typically causes diffuse myalgias and elevated creatine kinase. Although anemia could
be seen in the setting of viral myelosuppression, the elevated indirect bilirubin suggests hemolysis, which makes SCD more likely.

(Choice F) Spinal nerve root compression typically presents with pain, weakness, and paresthesias in a defined nerve root distribution. Pain is
often reproduced by the straight leg raise test, not by direct palpation of the area of pain.

Educational objective:
Vaso-occlusive pain crises in sickle cell disease cause severe musculoskeletal pain due to microvascular occlusion from sickled red blood cells.

References
Pain site frequency and location in sickle cell disease: the PiSCES project.

(http://www.ncbi.nlm.nih.gov/pubmed/19631468)
Sickle cell disease in children: differentiating osteomyelitis from vaso-occlusive crisis.

(http://www.ncbi.nlm.nih.gov/pubmed/19255393)

Pediatrics Hematology & Oncology Sickle cell


Subject System Topic

https://t.me/USMLEWorldStep2CK

REVIEW

https://www.uworld.com/ClientApp/v15/apps/qbanktestinterface/index.html#/launchtest/7281878/nbme/229400737/3/1 1/1
‫ م‬9:07 2021/‫‏‬9/‫‏‬11 https://t.me/USMLEWorldStep2CK UWorld STEP2 SIM Form 1

1 Item 27 of 40
2
Mark
Question Id: 6971 Previous Next
3

4 A 76-year-old man comes to the office due to increased fatigue over the past 2 months. His exercise tolerance has also decreased. The patient
5 has had no chest pain, nausea, vomiting, dizziness, or syncope. Other medical conditions include hypertension and hyperlipidemia. Blood
6 pressure is 124/79 mm Hg. On examination, the lungs are clear to auscultation and a short midsystolic murmur is heard over the right upper
7 sternal border. ECG is shown in the exhibit. Which of the following is the best initial pharmacotherapy for this patient?
8

9  A. Adenosine (0%)

10 B. Amlodipine (0%)
11
C. Lidocaine (0%)
12
13  D. Metoprolol (100%)
14
E. No additional therapy needed (0%)
15
16

17
18 Incorrect 02 secs 09/10/2021
19
Correct answer
D
Collecting Statistics  Time Spent  Last Updated

20

21

22 Explanation
23

24
25

26

27

28

29
30
31

32

33
34

35

36

37

38

39

40

This patient with 2 months of fatigue and decreased exercise tolerance has an ECG showing a rapid rate (~114/min) with irregularly irregular R-
R intervals and an absence of organized P waves, consistent with atrial fibrillation with rapid ventricular response. The acute management
of atrial fibrillation in a hemodynamically stable patient involves rate control with pharmacologic agents that slow conduction through the
atrioventricular (AV) node.

In the acute setting the ventricular rate should be slowed to <90-100/min, and with chronic management a goal rate of <80/min (or lower if
coronary artery disease is present) is often set. A beta blocker (eg, metoprolol, esmolol) or nonhydropyridine calcium channel blocker (eg,
verapamil, diltiazem) is given initially. If monotherapy is unable to provide adequate rate control, a second agent is then added (eg, diltiazem
added to metoprolol). Due to its positive inotropic effects, digoxin is particularly useful as a second-line agent in patients with left ventricular
systolic dysfunction. Amiodarone has rate control properties (due to inhibition of calcium channels in the AV node) in addition to rhythm control
properties and is sometimes used as a third-line agent for rate control.

Some patients with atrial fibrillation are considered for pharmacologic (eg, amiodarone, dofetilide, flecainide) or electrical cardioversion to sinus
rhythm (ie, rhythm control). Most patients benefit from the initiation of chronic anticoagulation at the time of diagnosis to reduce the risk of
thromboembolic complications (eg, stroke).

(Choice A) Adenosine blocks conduction through the AV node, but its effects are short-lived. The drug is most useful in helping to determine the
specific rhythm present in rapid supraventricular tachycardia or to help terminate atrioventricular nodal reentrant tachycardia, but it is not useful for
rate control of atrial fibrillation.

(Choice B) Dihydropyridine calcium channel blockers (eg, amlodipine, nifedipine) are primarily vasodilators with no significant effect on the AV
node; therefore, they are not useful for rate control of atrial fibrillation.

(Choice C) Lidocaine (which inhibits ventricular depolarization through effects on sodium channels) is sometimes used for rhythm control of
ventricular tachycardia, but it is not useful for atrial arrhythmias.

(Choice E) Without improved ventricular rate control this patient is likely to have ongoing symptoms and is at risk for tachycardia-induced
cardiomyopathy.

Educational objective:
Rate control of atrial fibrillation is accomplished with atrioventricular node blocking agents; a beta blocker (eg, metoprolol) or nondihydropyridine
calcium channel blocker (eg, verapamil, diltiazem) is given initially.

References
Rate control in atrial fibrillation.

(http://www.ncbi.nlm.nih.gov/pubmed/27560277)
Who, when and how to rate control for atrial fibrillation.

(http://www.ncbi.nlm.nih.gov/pubmed/18281823)

Medicine Cardiovascular System Atrial fibrillation


Subject System Topic

https://t.me/USMLEWorldStep2CK

REVIEW

https://www.uworld.com/ClientApp/v15/apps/qbanktestinterface/index.html#/launchtest/7281878/nbme/229400737/3/1 1/1
‫ م‬9:08 2021/‫‏‬9/‫‏‬11 https://t.me/USMLEWorldStep2CK UWorld STEP2 SIM Form 1

1 Item 28 of 40
2
Mark
Question Id: 6973 Previous Next
3

4 A 72-year-old man comes to the clinic after noticing an irregular heartbeat for the past several weeks. He has had no chest pain, shortness of
5 breath, lightheadedness, or syncope. Medical history includes hypertension and mild aortic stenosis. Blood pressure is 138/80 mm Hg and pulse
6 is 70/min. ECG is shown in the exhibit. Which of the following is the most likely diagnosis?
7
8  A. Atrial fibrillation (6%)

9 B. First-degree atrioventricular block (6%)


10
C. Premature atrial contractions (3%)
11
12  D. Second-degree atrioventricular block (79%)
13
E. Wolff-Parkinson-White syndrome (4%)
14

15
16

17 Incorrect 79% 02 secs 06/14/2021


18
Correct answer  Answered correctly  Time Spent  Last Updated
D
19
20

21 Explanation
22

23
Atrioventricular block
24
25 Clinical presentation ECG features Management
26
First degree Asymptomatic PR interval prolongation Observation
27

28
Observation
Mobitz type I second Progressive PR interval lengthening followed by dropped
29
Usually asymptomatic (rarely PPM
degree QRS complex
placement)
30
31 Mobitz type II second Fatigue, lightheadedness, Constant PR interval with randomly dropped QRS
PPM placement
32 degree syncope complexes
33
Third degree Fatigue, lightheadedness,
34 Complete dissociation of P waves & QRS complexes PPM placement
(complete) syncope
35

36 PPM = permanent pacemaker.

37
This patient's ECG shows normal P waves with progressive lengthening of the PR interval followed by a dropped QRS complex, which is
38
representative of Mobitz type I second-degree atrioventricular (AV) block (also known as Wenckebach AV block). Patients with Mobitz type I
39 second-degree AV block typically have an irregular heart rhythm (due to intermittent delay of ventricular contraction), but they are usually
40 asymptomatic. The block occurs at the level of the AV node and rarely progresses to more advanced AV block (ie, Mobitz type II second-degree
AV block, third-degree AV block).

In contrast to Mobitz type I second-degree AV block, Mobitz type II second-degree AV block is recognized by a constant PR interval and randomly
dropped QRS complexes. The block more commonly occurs below the AV node, and affected patients are usually symptomatic, experiencing
fatigue, chest pain, dyspnea, lightheadedness, and/or syncope. Mobitz type II second-degree AV block also creates a risk of sudden cardiac
arrest.

(Choice A) Atrial fibrillation causes an irregular rhythm, but P waves are absent on ECG. Patients may be asymptomatic or may have
palpitations and fatigue (or dyspnea, chest pain, lightheadedness, and/or syncope if the heart rate is rapid).

(Choice B) First-degree AV block involves a constant lengthening of the PR interval with a QRS complex following every P wave (ie, no dropped
QRS complexes). The heart rhythm is regular, and patients are asymptomatic.

(Choice C) Premature atrial contractions are recognized on ECG as P waves that occur earlier than expected and have different morphology
(shape) than a normal sinus P wave. Patients may experience palpitations but are otherwise asymptomatic.

(Choice E) Patients with Wolff-Parkinson-White syndrome have a baseline ECG demonstrating slurred upstroke of the QRS complex. These
patients can develop episodes of AV reentrant tachycardia (which has a regular rhythm and typically demonstrates inverted P waves immediately
following the QRS complexes).

Educational objective:
Mobitz type I second-degree atrioventricular block is recognized on ECG by normal P waves with progressive lengthening of the PR interval until a
QRS complex is dropped. Patients have an irregular heart rhythm but are usually asymptomatic.

Medicine Cardiovascular System Heart block


Subject System Topic

https://t.me/USMLEWorldStep2CK

REVIEW

https://www.uworld.com/ClientApp/v15/apps/qbanktestinterface/index.html#/launchtest/7281878/nbme/229400737/3/1 1/1
‫ م‬9:08 2021/‫‏‬9/‫‏‬11 https://t.me/USMLEWorldStep2CK UWorld STEP2 SIM Form 1

1 Item 29 of 40
2
Mark
Question Id: 6975 Previous Next
3

4 A 14-year-old boy is brought to the office due to fatigue when playing basketball. The patient finds it difficult to play for a long time and to compete
5 with his peers. He has had no shortness of breath, chest pain, or palpitations. His medical history is insignificant, and he received all childhood
6 vaccinations. Academically, he is in the top 20% of his class. On physical examination, the patient has a soft mid-systolic murmur best heard at
7 the left upper sternal border. He also has wide splitting of the second heart sound that does not change with respiration. The rest of the physical
8 examination is unremarkable. Which of the following is the most likely cause of the observed findings?
9
 A. Corrected transposition of the great vessels (1%)
10
11 B. Dynamic obstruction of the left ventricular outflow tract (11%)
12
C. Fixed obstruction of the left ventricular outflow tract (7%)
13

14  D. Left-to-right intracardiac shunting (65%)


15
E. Right-to-left intracardiac shunting (14%)
16

17
18

19 Incorrect 65%
20
Correct answer  Answered correctly  02 secs
Time Spent  03/21/2021
Last Updated
D
21

22

23 Explanation
24
25
This patient has classic physical examination findings of a large atrial septal defect (ASD) with left-to-right shunt. Patients with this congenital

26
heart defect often remain asymptomatic until adulthood but can subsequently develop decreased exercise tolerance (fatigue, dyspnea), atrial
arrhythmias (atrial fibrillation or flutter), stroke due to paradoxical embolization, or cerebral abscess. Eventually, severe pulmonary hypertension
27
and right heart failure can develop.
28

29 Cardiac auscultation in patients with ASD (with large left-to-right shunting and normal pulmonary artery pressure) typically reveals several
30 characteristic findings:
31
1. Wide and fixed splitting of the second heart sound (S2): Resulting from delayed closure of the pulmonic valve due to the enlarged right
32
ventricle's prolonged emptying (widened S2), with no difference between inspiration and expiration (fixed S2)
33
2. Mid-systolic or ejection murmur over the left upper sternal border: Resulting from increased flow across the pulmonic valve. The low-
34 velocity left-to-right shunt flow across the ASD itself does not produce any audible murmur.
35 3. Mid-diastolic rumble: Resulting from increased flow across the tricuspid valve
36
(Choice A) An ASD is often created temporarily in some patients with transposition of the great vessels to promote mixing of the pulmonary and
37
systemic circulation before a more definitive arterial switch procedure can be performed. There is no such history in this patient.
38

39
(Choice B) Dynamic obstruction of the left ventricular outflow tract (LVOT) can be seen in hypertrophic cardiomyopathy (HCM). Physical

40
examination reveals a harsh crescendo-decrescendo systolic murmur heard best at the apex and lower left sternal border. Wide, fixed splitting of
S2 is not present in HCM.

(Choice C) A fixed obstruction of the LVOT is seen in aortic stenosis, which causes a systolic ejection murmur best heard at the right upper
sternal border.

(Choice E) Right-to-left intracardiac shunting can be seen in a variety of congenital heart defects. Although ASD can progress to severe
pulmonary hypertension and right-to-left shunting, this usually does not occur until adulthood and is associated with cyanosis and clubbing (ie,
Eisenmenger syndrome).

Educational objective:
Atrial septal defect (ASD) is characterized by wide, fixed splitting of the second heart sound on physical examination. A systolic murmur is
sometimes heard at the left upper sternal border from increased flow across the pulmonary valve, but there usually is no audible murmur from the
low-velocity left-to-right shunt flow across the ASD.

References
Incidence and clinical features of asymptomatic atrial septal defect in school children diagnosed by heart disease screening.

(http://www.ncbi.nlm.nih.gov/pubmed/12547990)
Atrial septal defects.

(http://www.ncbi.nlm.nih.gov/pubmed/24725467)

Pediatrics Cardiovascular System Atrial septal defect


Subject System Topic

https://t.me/USMLEWorldStep2CK

REVIEW

https://www.uworld.com/ClientApp/v15/apps/qbanktestinterface/index.html#/launchtest/7281878/nbme/229400737/3/1 1/1
‫ م‬9:08 2021/‫‏‬9/‫‏‬11 https://t.me/USMLEWorldStep2CK UWorld STEP2 SIM Form 1

1 Item 30 of 40
2
Mark
Question Id: 6979 Previous Next
3

4 A 5-year-old girl is brought to the office due to a 3-day history of diarrhea and abdominal cramps. She has a history of failure to thrive and
5 recurrent respiratory infections requiring frequent hospitalizations. The patient has been taking antibiotics for 2 weeks to treat bacterial sinusitis
6 and was feeling well until 3 days ago when she started having watery diarrhea. She has 6-8 episodes of diarrhea a day with no blood or mucus in
7 the stool. Temperature is 38.3 C (100.9 F) and pulse is 145/min. Mucous membranes are dry. Lung auscultation reveals bilateral crackles. The
8 abdomen is soft with moderate diffuse tenderness. Bowel sounds are active. Laboratory results are as follows:
9
Leukocytes 24,000/mm3
10
11 Hemoglobin 9.5 g/dL
12 Platelets 340,000/mm3
13

14 A stool smear shows numerous white blood cells. Which of the following is the most likely pathophysiology of this patient's diarrhea?
15
 A. Bacterial invasion of the intestinal mucosa (10%)
16

17 B. Deficiency of pancreatic exoenzymes (7%)


18
 C. Disruption of the colonic microflora (80%)
19
20 D. Granulomatous inflammation of the intestine (0%)
21 E. Mucosal damage due to drug hypersensitivity (0%)
22

23

24
25
Incorrect 80% 02 secs 03/31/2021
26
Correct answer
C
 Answered correctly  Time Spent  Last Updated

27

28

29 Explanation
30
31
Clostridium difficile infection in children
32

33
Recent antibiotics

34
Risk factors Hospitalization
PPI
35

36 Profuse diarrhea (most common)


Clinical presentation
37 Fulminant colitis or toxic megacolon
38
Diagnosis Stool PCR
39

40
Treatment Oral metronidazole or vancomycin

Hand hygiene with soap & water


Contact isolation
Infection control
Sporicidal disinfectants
(eg, bleach)

PCR = polymerase chain reaction; PPI = proton pump inhibitor.

This patient is being treated with systemic antibiotics and now has abdominal pain, diarrhea, low-grade fever, and signs of dehydration
(tachycardia, dry mucous membranes). These findings are consistent with antibiotic-associated diarrhea (AAD). AAD is a common
complication of antibiotic therapy (especially penicillins, cephalosporins, and clindamycin) and results from disruption of the normal enteric
bacterial flora (ie, the gut microbiome). Although most cases of AAD cannot be attributed to a specific organism, the most commonly identified
cause is Clostridium difficile, which is extremely likely in the setting of abdominal tenderness, fecal leukocytes, and frequent hospitalization.
Stool assay for C difficile toxin-producing gene can confirm the diagnosis.

(Choice A) Bacterial invasion of the intestinal mucosa causes dysentery (eg, crampy, mucoid or bloody, low-volume stools). Common causes
include Salmonella or enteroinvasive Escherichia coli. This patient does not have dysentery.

(Choice B) This patient's failure to thrive and recurrent respiratory infections are suspicious for cystic fibrosis. Most patients with this disorder
have exocrine pancreatic insufficiency, which manifests as chronic loose stools due to fat malabsorption. In contrast, this patient's acute diarrhea,
fever, and leukocytosis are more consistent with an infectious colitis.

(Choice D) Noncaseating granulomatous inflammation is characteristic of Crohn disease, which can present similarly to infectious colitis.
However, Crohn disease rarely presents before the second decade of life, whereas AAD is much more common in school-age children.

(Choice E) Hypersensitivity reactions to antibiotics include types I (anaphylaxis) and III (serum sickness) but not diarrhea.

Educational objective:
Antibiotic-associated diarrhea is caused by disruption of normal colonic flora. Clostridium difficile is the most commonly identified cause of
antibiotic-associated diarrhea, presenting with low-grade fever, diarrhea, abdominal pain, and leukocytes in the stool.

References
Clostridium difficile infection in infants and children.

(http://www.ncbi.nlm.nih.gov/pubmed/23277317)
Clostridium difficile infection in children: a comprehensive review.

(http://www.ncbi.nlm.nih.gov/pubmed/23659563)
Clinical practice guidelines for Clostridium difficile infection in adults and children: 2017 update by the Infectious Diseases Society of America
(IDSA) and Society for Healthcare Epidemiology of America (SHEA).

(http://www.ncbi.nlm.nih.gov/pubmed/29562266)

Pediatrics Gastrointestinal & Nutrition Clostridioides difficile infection


Subject System Topic

https://t.me/USMLEWorldStep2CK

REVIEW

https://www.uworld.com/ClientApp/v15/apps/qbanktestinterface/index.html#/launchtest/7281878/nbme/229400737/3/1 1/1
‫ م‬9:08 2021/‫‏‬9/‫‏‬11 https://t.me/USMLEWorldStep2CK UWorld STEP2 SIM Form 1

1 Item 31 of 40
2
Mark
Question Id: 7245 Previous Next
3

4 A 71-year-old man is being evaluated for impaired cognition and memory loss. Approximately a year ago, he began having difficulty balancing his
5 checkbook and preparing meals. About 5 months ago, he became more unsteady on his feet, to the point that he started using a cane, and
6 sustained a fall while walking to the bathroom. During the past month, he has begun isolating himself from family and has appeared sadder than
7 usual. The patient also has had increasing urinary frequency and urgency. He has type 2 diabetes mellitus, hypertension, and high cholesterol.
8 Which of the following is the most likely finding associated with this patient's symptoms?
9
 A. Cerebrovascular disease (52%)
10
11 B. Eosinophilic inclusions in neurons of the substantia nigra (5%)
12
C. Loss of dopamine-producing cells (8%)
13

14 D. Low acetylcholine activity (26%)


15
E. Silver-staining inclusions in neurons of the hippocampus (5%)
16

17
18

19 52%
20
Correct  Answered correctly  01 sec
Time Spent  03/18/2021
Last Updated

21

22

23 Explanation
24
25
Differential diagnosis of dementia subtypes
26

27 Early, insidious short-term memory loss


Alzheimer
28 Language deficits & spatial disorientation
disease
29
Later personality changes

30 Stepwise decline
31 Vascular Early executive dysfunction
32 dementia Cerebral infarction &/or deep white matter
33 changes on neuroimaging
34
Early personality changes
35 Frontotemporal
Apathy, disinhibition & compulsive behavior
36 dementia
Frontotemporal atrophy on neuroimaging
37

38
Visual hallucinations
Dementia with Lewy Spontaneous parkinsonism
39
bodies Fluctuating cognition
40
Rapid eye movement behavior disorder

Ataxia early in disease


Normal pressure
Urinary incontinence
hydrocephalus
Dilated ventricles on neuroimaging

Behavioral changes
Prion disease Rapid progression
Myoclonus &/or seizures

This patient's presentation is consistent with dementia, defined as a progressive cognitive decline in ≥1 domains (eg, language, learning and
memory, executive function). This decline must be severe enough to affect activities of daily living (eg, cleaning, cooking, finances) and
independence. Common causes of dementia include Alzheimer disease, dementia with Lewy bodies, and vascular dementia.

Vascular dementia is due to infarctions in large artery (usually cortical) or smaller artery (subcortical or lacunar) distributions. Patients
characteristically exhibit a stepwise functional deterioration (as seen in this patient with discrete deterioration episodes 1 year ago, 5 months ago,
and 1 month ago); there may be partial improvement in the interim, and the infarctions can also present insidiously. Vascular dementia is
associated with typical atherosclerotic risk factors (eg, smoking, diabetes, hypertension, hyperlipidemia). Clinical findings are determined by
the affected area of the brain: Medial frontal disease can cause executive dysfunction (eg, difficulty balancing checkbook); left parietal disease
can result in aphasia; medial temporal disease can lead to anterograde amnesia; and subcortical vascular disease can cause focal motor findings,
gait disturbances (eg, frequent falls, unsteadiness), urologic findings (as seen in this patient), and personality changes.

(Choice B) Dementia with Lewy bodies is characterized by round, eosinophilic, and cytoplasmic inclusions within the neurons of the substantia
nigra. Patients typically develop the core clinical features of episodic fluctuating cognitive function, visual hallucinations, parkinsonism, and REM
sleep behavior disorder. This patient's absence of visual hallucinations makes this less likely.

(Choice C) Parkinson disease is due to the loss of dopamine-producing cells in the substantia nigra. Patients usually develop the cardinal
findings of tremor, bradykinesia, and rigidity. Dementia typically occurs later in the disease course.

(Choice D) Alzheimer disease is characterized by impaired acetylcholine synthesis. It typically presents insidiously, rather than abruptly, with a
slow and gradual, rather than a stepwise, progression. Neuropsychiatric symptoms (eg, personality changes, apathy, social isolation) occur later
in the course of the disease. Factors suggesting vascular dementia over Alzheimer disease include prominent executive dysfunction, history of
stroke, vascular risk factors, and abrupt onset with stepwise functional deterioration.

(Choice E) Behavioral variant frontotemporal dementia (also known as Pick disease) is characterized by silver-staining cytoplasmic inclusions
within the neurons of the hippocampus. Frontotemporal dementia often presents with early personality change and later memory impairment and
compulsive behavior.

Educational objective:
Vascular dementia is due to large artery (usually cortical) or smaller artery (usually subcortical or lacunar) infarctions. An abrupt onset of
neurologic symptoms, focal motor findings, gait disturbances, urologic symptoms, or personality changes occurs, with the specific symptoms
determined by the location of the infarcts. This decline in function typically proceeds in a stepwise manner, with stability or partial improvement in
the interim.

References
Vascular dementia.

(http://www.ncbi.nlm.nih.gov/pubmed/26595643)

Medicine Nervous System Vascular dementia


Subject System Topic

https://t.me/USMLEWorldStep2CK

REVIEW

https://www.uworld.com/ClientApp/v15/apps/qbanktestinterface/index.html#/launchtest/7281878/nbme/229400737/3/1 1/1
‫ م‬9:08 2021/‫‏‬9/‫‏‬11 https://t.me/USMLEWorldStep2CK UWorld STEP2 SIM Form 1

1 Item 32 of 40
2
Mark
Question Id: 6983 Previous Next
3

4 A 67-year-old man is brought to the emergency department due to dizziness and generalized weakness over the last 24 hours. Two months ago,
5 he was diagnosed with locally invasive bladder cancer and had bilateral nephrostomy tubes placed to relieve ureteral obstruction. The patient
6 recently completed 2 cycles of chemotherapy complicated by nausea with poor oral intake. He is scheduled to have a cystectomy in the near
7 future. He has no other medical issues. Temperature is 37.8 C (100 F), blood pressure is 72/49 mm Hg, pulse is 125/min, and respirations are
8 22/min. The neck is supple with no jugular venous distension. Chest auscultation reveals vesicular breath sounds and normal cardiac sounds.
9 The abdomen is soft, nontender, and nondistended. Neurologic examination shows no focal abnormalities. Extremities are warm to the touch with
10 bounding pulses. Laboratory results are as follows:
11
Complete blood count
12
13 Hemoglobin 9.8 g/dL
14 Platelets 80,000/mm3
15
Leukocytes 2,400/mm3
16

17
Serum chemistry
18

19
Sodium 134 mEq/L

20 Potassium 5.0 mEq/L


21
Bicarbonate 18 mEq/L
22

23 Blood urea nitrogen 30 mg/dL


24 Creatinine 2.9 mg/dL
25

26 Liver function studies


27
Total protein 5.5 g/dL
28

29
Albumin 2.8 g/dL

30
The patient is given 3 L of normal saline and broad-spectrum antibiotics. Three hours later, mean arterial blood pressure is 50 mm Hg and pulse
31
is 120/min. Which of the following is the best next step in management of this patient?
32

33  A. Dobutamine infusion (11%)


34
B. Intravenous 3% saline (2%)
35

36 C. Intravenous albumin (8%)


37 D. Intravenous hydrocortisone (13%)
38
 E. Norepinephrine infusion (58%)
39

40 F. Packed red blood cell transfusion (5%)

Incorrect 58%
Correct answer  Answered correctly  04 secs
Time Spent  09/10/2021
Last Updated
E

Explanation

Patients with immunosuppression (eg, leukopenia following recent chemotherapy) often cannot mount a significant cytokine response to generate
fever (eg, ≥38 C [100.4 F]) in the setting of infection. However, confusion, hypotension, tachycardia, and tachypnea are all concerning for infection
with sepsis.

In sepsis, aggressive early intervention with crystalloid infusion (30 mL/kg over the first 3 hours) and intravenous broad-spectrum antibiotics (within
an hour of presentation) are crucial to reducing morbidity and mortality. However, patients with continued hypotension (mean arterial pressure
[MAP] <60 mm Hg) following 3 L of crystalloid infusion (eg, septic shock) require a vasopressor medication.

The first-line agent for septic shock is norepinephrine, which acts on alpha-1 and beta-1 adrenergic receptors to cause potent peripheral
vasoconstriction and a moderate increase in cardiac output. Titration to achieve a MAP of 60-70 mm Hg and urine output of 0.5 mL/kg/hr is
indicated. The data in support for norepinephrine come from several trials comparing it to other agents, with dopamine use falling out of favor due
to increased arrhythmias and mortality.

(Choice A) Dobutamine acts on beta-1 adrenergic receptors to increase cardiac inotropy and chronotropy. It is the first-line agent for cardiogenic
shock, which also causes hypotension. However, it would cause elevated jugular venous pressure and cold extremities, as opposed to this
patient's warm extremities and bounding pulses (reflecting hyperdynamic shock with decreased peripheral resistance).

(Choice B) Intravenous 3% (hypertonic) saline is reserved for severe hyponatremia (eg, sodium <125 mEq/L with coma or seizure). This
patient's mild hyponatremias, likely due to hypovolemia from his illness, will likely improve with crystalloid infusion.

(Choice C) Crystalloids (eg, normal saline, lactated Ringer's) are the fluid of choice to restore volume quickly; they are as effective as albumin but
are less costly and more readily available.

(Choice D) Intravenous hydrocortisone is sometimes reserved for those with severe sepsis who do not respond to volume resuscitation and
vasopressors.

(Choice F) Blood transfusion is generally reserved for those with hemoglobin levels <7 mg/dL (or <9 mg/dL in the setting of significant cardiac
history); this patient's hemoglobin level is 9.8 mg/dL.

Educational objective:
Patients with sepsis who do not achieve a mean arterial pressure (MAP) >60 mm Hg after adequate fluid repletion (eg, septic shock) require a
vasopressor to improve MAP to goal range. The first-line vasopressor agent for septic shock is norepinephrine, which causes significant
peripheral vasoconstriction and moderate increase in cardiac output.

References
Surviving sepsis campaign: international guidelines for management of severe sepsis and septic shock: 2012.

(http://www.ncbi.nlm.nih.gov/pubmed/23353941)

Medicine Pulmonary & Critical Care Sepsis


Subject System Topic

https://t.me/USMLEWorldStep2CK

REVIEW

https://www.uworld.com/ClientApp/v15/apps/qbanktestinterface/index.html#/launchtest/7281878/nbme/229400737/3/1 1/1
‫ م‬9:08 2021/‫‏‬9/‫‏‬11 https://t.me/USMLEWorldStep2CK UWorld STEP2 SIM Form 1

1 Item 33 of 40
2
Mark
Question Id: 20562 Previous Next
3

4 A group of pediatric cardiologists conducted a case-control study of childhood stroke risk associated with congenital heart disease (CHD). A
5 random sample of children age <20 with ischemic and hemorrhagic strokes were selected from children enrolled in an integrated health care plan.
6 Facility-matched stroke-free controls (3 per case) were also selected. Researchers determined exposure to CHD (diagnosed before stroke) and
7 calculated odds ratios to analyze risk factors for stroke. An excerpt of study results is shown below.
8
Stroke No stroke
9

10 n = 300 3 per case, n = 900


11
CHD No CHD CHD No CHD
12
13 n = 13 n = 287 n=3 n = 897
14 CHD = congenital heart disease.
15
16 Which of the following is closest to the odds ratio of CHD in children who had a stroke compared with children who have not had a stroke?
17
 A. 0.003 (2%)
18

19 B. 0.045 (7%)
20
C. 0.067 (10%)
21

22 D. 2.063 (5%)
23 E. 13.54 (74%)

24
25

26
Incorrect 74%
27

28
Correct answer  Answered correctly  02 secs
Time Spent  06/08/2021
Last Updated
E
29
30
31 Explanation
32
The odds of an event are related to the probability (P) of that event, with odds equal to P / (1 − P). For example, the probability of rolling a single
33
die and getting a 1 is 1/6, but the odds of rolling a 1 are 1/5. The odds ratio (OR) is a measure of association used in case-control studies to
34
compare the odds of exposure in cases (ie, individuals with disease) relative to controls (ie, individuals without disease):
35

36 OR = (odds of exposure in cases) / (odds of exposure in controls)


37
In this question, the disease of interest is stroke, so cases are children who had a stroke, and controls are children who have not had a
38 stroke. The exposure of interest is congenital heart disease (CHD), so the reference category of the exposure is no CHD. Therefore, the OR of
39 interest is the odds of CHD in cases (ie, children who had a stroke) relative to the odds of CHD in controls (ie, children who have not had a
40 stroke).

A shortcut to calculate the OR when a contingency table has a standard format (ie, exposure of interest in the first row of exposure status and
cases in the first column of disease status) is:

OR = (a/c) / (b/d)

Based on the information given, the standard contingency table is:

Stroke + Stroke −

CHD + 13 3

CHD − 287 897

Total 300 900

The odds of CHD in children who had a stroke and children who have not had a stroke are:

Odds of CHD in cases: a/c = 13/287

Odds of CHD in controls: b/d = 3/897

Therefore, the OR of CHD in children who had a stroke compared with children who have not had a stroke is:

OR = (a/c) / (b/d) = (13/287) / (3/897) = 13.54

The OR is >1, which indicates that CHD is a risk factor for stroke in children.

This example calculated the OR of exposure (ie, odds of exposure among cases relative to controls). The OR is sometimes described as the odds
of disease among exposed individuals relative to nonexposed individuals, given by (a/b) / (c/d). Both OR definitions are mathematically equivalent
and reduce to OR = ad / bc.

Educational objective:
The odds ratio (OR) is a measure of association calculated as:

OR = (odds of exposure in cases) / (odds of exposure in controls)

For a correctly formatted contingency table: OR = ad / bc (or other mathematically equivalent equations).

Medicine Biostatistics & Epidemiology Measures of association


Subject System Topic

https://t.me/USMLEWorldStep2CK

REVIEW

https://www.uworld.com/ClientApp/v15/apps/qbanktestinterface/index.html#/launchtest/7281878/nbme/229400737/3/1 1/1
‫ م‬9:09 2021/‫‏‬9/‫‏‬11 https://t.me/USMLEWorldStep2CK UWorld STEP2 SIM Form 1

1 Item 34 of 40
2
Mark
Question Id: 6987 Previous Next
3

4 A 63-year-old man comes to the office due to progressive fatigue over the last 6 months. The patient feels "exhausted" by the end of the day and
5 has lost 4.5 kg (10 lb). He was hospitalized with pneumonia 7 months ago and notes that "everything deteriorated" thereafter. He has no fevers,
6 chills, abdominal pain, or urinary symptoms. Medical history is significant for hypertension treated with lisinopril. He is a lifetime nonsmoker but
7 drinks alcohol socially. Blood pressure is 146/92 mm Hg and pulse is 80/min. The patient is obese. Mucous membranes are pale. Abdominal
8 examination is normal, but rectal examination shows a mildly enlarged, nontender prostate. Laboratory results are as follows:
9
Complete blood count
10
11 Hemoglobin 9.2 g/dL
12 Mean corpuscular volume 86 µm3
13
Platelets 150,000/mm3
14

15 Leukocytes 9,200/mm3
16

17 Serum chemistry
18
Sodium 137 mEq/L
19
20
Potassium 3.9 mEq/L

21 Chloride 102 mEq/L


22
Bicarbonate 24 mEq/L
23

24 Calcium 11.8 mg/dL


25 Blood urea nitrogen 30 mg/dL
26
Creatinine 2.2 mg/dL
27

28 Urinalysis shows no proteinuria or hematuria. Which of the following is the most likely cause of this patient's kidney injury?
29
30  A. Glomerular pathology (12%)
31
B. Lisinopril toxicity (4%)
32

33 C. Renal artery stenosis (7%)


34 D. Renal tubular damage (43%)

35
E. Urinary outflow obstruction (31%)
36

37

38

39 Incorrect 43%
40
Correct answer  Answered correctly  04 secs
Time Spent  05/10/2021
Last Updated
D

Explanation

Multiple myeloma

Pathophysiology Plasma cell neoplasm produces monoclonal paraprotein (immunoglobulin)

Bone pain, fractures


Manifestations Constitutional symptoms (weight loss, fatigue)
Recurrent infections

Normocytic anemia
Renal insufficiency
Laboratory
Hypercalcemia (constipation, muscle weakness)
Monoclonal paraproteinemia (M-spike)

Radiology Osteolytic lesions/osteopenia (osteoclast activation)

This patient presents with constitutional symptoms (fatigue and weight loss), normocytic anemia, renal insufficiency, and hypercalcemia, raising
suspicion for multiple myeloma (MM). MM is a plasma cell malignancy marked by monoclonal protein production (causing a characteristic M-
spike on electrophoresis). Renal insufficiency is common in patients with MM and has many sources. Most often, monoclonal light chains clog
the renal tubules, causing intratubular cast formation and toxicity (myeloma cast nephropathy). Patients with cast nephropathy often have a
normal urinalysis (UA) with little proteinuria (the UA detects albumin but not the monoclonal proteins). Other common causes of renal insufficiency
in MM include amyloidosis and monoclonal immunoglobulin deposition disease (MIDD), both of which cause glomerular injury and nephrotic
syndrome.

Patients with MM are at higher risk of infection due to immune dysregulation and hypogammaglobulinemia. Bacterial upper respiratory and urinary
tract infections are common. This patient likely has had MM for some time; a susceptibility to pneumonia may have been the first manifestation.

(Choice A) Patients with MM can develop renal insufficiency from glomerular pathology (eg, amyloidosis, MIDD). However, this patient's UA
showed no hematuria or proteinuria, making glomerular injury less likely.

(Choice B) Lisinopril can reduce the perfusion pressure of the kidney, causing kidney injury. This most commonly occurs within 3-5 days after
starting the medication. Lisinopril toxicity is unlikely to account for this patient's hypercalcemia, anemia, or long-standing fatigue.

(Choice C) Renal artery stenosis (RAS) is marked by a slow rise in creatinine and is often associated with severe hypertension (>180/120 mm
Hg) and recurrent flash pulmonary edema. RAS does not commonly cause weight loss or hypercalcemia.

(Choice E) Urinary outflow obstruction may present as renal insufficiency with a normal UA and without pain. However, it would not account for
this patient's long-standing constitutional symptoms, anemia, or hypercalcemia. Prostate cancer could cause urinary obstruction but is typically
associated with areas of induration or nodules on examination.

Educational objective:
A patient with a combination of renal failure, hypercalcemia, and anemia should be evaluated for multiple myeloma (MM). Renal insufficiency is a
common complication of MM. Light chain cast nephropathy with resultant renal tubular injury is most often the cause.

Medicine Hematology & Oncology Multiple myeloma


Subject System Topic

https://t.me/USMLEWorldStep2CK

REVIEW

https://www.uworld.com/ClientApp/v15/apps/qbanktestinterface/index.html#/launchtest/7281878/nbme/229400737/3/1 1/1
‫ م‬9:09 2021/‫‏‬9/‫‏‬11 https://t.me/USMLEWorldStep2CK UWorld STEP2 SIM Form 1

1 Item 35 of 40
2
Mark
Question Id: 6997 Previous Next
3

4 A 6-year-old boy is brought to the office by his parents due to difficulties at school. The teacher is concerned that the patient has poor social and
5 verbal skills. He has difficulty making friends with classmates and avoids eye contact with the teacher. The patient also has trouble transitioning
6 between activities and is disruptive in the classroom. He will not stay in his seat, and when redirected, he throws temper tantrums or starts biting
7 his hand. His parents have also noticed that he is anxious and often flaps his arms and misbehaves in new situations that make him
8 uncomfortable. The patient spoke his first word at around age 18 months and began walking at age 2 years. He continues to require speech
9 therapy for poor articulation and disorganized sentence structure. The patient has been tracking along the 50th percentile for height and weight.
10 During the examination, he becomes upset and starts biting his hand. Further testing in this patient is most likely to identify which of the following?
11
 A. Accumulation of ammonia (39%)
12
13 B. Chromosomal trisomy (7%)
14
C. Loss of paternally inherited allele (8%)
15
16 D. MECP2 genetic mutation (16%)
17 E. Trinucleotide repeat expansion (27%)

18

19
20

21
Incorrect 27% 02 secs 05/06/2021
22
Correct answer
E
 Answered correctly  Time Spent  Last Updated

23

24
25 Explanation
26

27
Genetic disorders associated with autism
28

29
Syndrome Etiology Key features*

30 More severe in boys


Fragile X
31 Trinucleotide repeat in FMR1 gene Dysmorphic facies (eg, long face, large ears)
syndrome
32 Macroorchidism
33
Almost exclusively in girls
34 Rett
MECP2 mutation Developmental regression
35 syndrome
Loss of purposeful hand movements
36

37 Tuberous Dermatologic findings (eg, ash-leaf spots)


TSC1/2 mutation
38
sclerosis Seizures

39 Down Dysmorphic facies (eg, upslanting palpebral fissures, epicanthal folds)


Trisomy 21
40 syndrome Congenital heart disease

Prader-Willi Obesity/hyperphagia
Loss of paternal allele on Cr 15q11-q13
syndrome Hypogonadism

Microcephaly
Angelman
Loss of maternal allele on Cr 15q11-q13 Ataxic gait/tremors
syndrome
Happy demeanor, inappropriate laughter

*Developmental delay & variable intellectual disability are common features.

This patient has features of autism spectrum disorder (ASD), including social deficits (eg, poor eye contact, difficulty making friends), motor
stereotypies (eg, hand flapping), and speech delay (eg, first word at age 18 months). It is common for patients with ASD to have an identifiable
genetic abnormality, and fragile X syndrome (FXS) is the most commonly associated inherited syndrome. Therefore, patients with ASD should
undergo FXS testing and chromosomal microarray to detect an underlying genetic cause.

FXS is due to an unstable trinucleotide (CGG) repeat expansion in the FMR1 gene on the X chromosome. This mutation causes
hypermethylation of the gene and impaired transcription of the protein FMRP, which is important for normal cognitive development. Therefore, in
addition to ASD, patients with FXS often have:

Developmental delay (eg, first step at age 2) and intellectual disability

Neuropsychiatric comorbidities, as seen in this child with features of attention deficit hyperactivity disorder (eg, difficulty remaining seated,
trouble transitioning)

Aggression, anxiety, and self-injurious behavior (eg, hand biting)

Typical physical features of FXS (eg, elongated face, large ears, enlarged testes) may not present until adolescence.

(Choice A) Hyperammonemia, as seen in urea cycle disorders, can cause developmental delay and intellectual disability, but vomiting and
encephalopathy (eg, altered mental status) would also be expected.

(Choice B) Down syndrome (ie, trisomy 21) is associated with ASD and intellectual disability; however, the diagnosis is usually made in infancy
due to the characteristic dysmorphic features present at birth.

(Choice C) Prader-Willi syndrome (ie, loss of paternal allele on chromosome 15q11-q13) is a genetic cause of ASD but also presents with obesity
(ie, hyperphagia) and short stature in childhood. This patient's normal weight and height make this condition unlikely.

(Choice D) The MECP2 mutation is responsible for Rett syndrome, which can cause autistic features and sterotypical hand movements.
However, it presents almost exclusively in girls with developmental regression (eg, normal initial development followed by regression in
communication at age 12-18 months) and abnormal gait.

Educational objective:
Fragile X syndrome should be considered in a child with features of autism, particularly if comorbid developmental delay, anxiety, and/or attention
deficit hyperactivity disorder are present. Detection of a trinucleotide (CGG) repeat in the FMR1 gene is diagnostic.

References
Autism symptoms in fragile X syndrome.

(http://www.ncbi.nlm.nih.gov/pubmed/28617074)

Psychiatry Psychiatric/Behavioral & Substance Abuse Fragile x syndrome


Subject System Topic

https://t.me/USMLEWorldStep2CK

REVIEW

https://www.uworld.com/ClientApp/v15/apps/qbanktestinterface/index.html#/launchtest/7281878/nbme/229400737/3/1 1/1
‫ م‬9:09 2021/‫‏‬9/‫‏‬11 https://t.me/USMLEWorldStep2CK UWorld STEP2 SIM Form 1

1 Item 36 of 40
2
Mark
Question Id: 7001 Previous Next
3

4 A 63-year-old woman is brought to the emergency department by her daughter who found the patient wandering through the house the past 2
5 nights in search of the bathroom. The patient's medical history is significant for hypertension and hyperlipidemia, for which she takes lisinopril and
6 atorvastatin. She drinks 1-2 glasses of wine daily and does not use illicit drugs. On examination, the patient is well-groomed and cooperative.
7 Temperature is 36.6 C (97.9 F), blood pressure is 140/90 mm Hg, and heart rate is 85/min. The mucous membranes are moist, the lungs are
8 clear, and the abdomen is soft and nontender. There is no peripheral edema. Laboratory results show leukocytes of 13,000/mm3 and hemoglobin
9 of 12.2 g/dL. Her electrolytes and kidney function tests are normal. Urinalysis shows a white blood cell count of 100/hpf and high leukocyte
10 esterase. The patient is admitted to the hospital and becomes severely agitated overnight. She repeatedly attempts to leave her room, claiming
11 she is "being kept as a prisoner and is late for her flight to Florida." She strikes a nurse attempting to take her blood pressure and repeatedly tries
12
to pull out her intravenous line. Which of the following is the most appropriate next step in management of this patient?

13
 A. Administer haloperidol (69%)
14

15 B. Administer lorazepam (12%)


16
C. Insert bladder catheter (3%)
17
18
D. Reorient the patient and challenge her delusions (8%)

19 E. Start donepezil (0%)


20
F. Use soft restraints (5%)
21

22

23

24
69% 02 secs 06/05/2021
25 Correct  Answered correctly  Time Spent  Last Updated

26

27

28 Explanation
29
30 This patient's acute onset of confusion and severe agitation is consistent with delirium. Delirium is an acute confusional state where symptoms
31 wax and wane, typically caused by an underlying acute medical condition, substance intoxication, or adverse effect of medication. Management of
32 delirium includes identifying and treating the underlying disorder and managing agitation.
33
In this patient, a urinary tract infection is likely to blame. Although antibiotics will help resolve the delirium by treating the underlying medical
34 disorder, her agitation also requires intervention. Mild confusion and agitation can be addressed with nonpharmacological interventions (eg,
35 frequent reorientation, familiar objects and people, a sitter); however, this patient is severely agitated (eg, hitting staff and pulling out lines). Her
36 behavior places her at immediate risk for injury to herself or staff, falling, wandering off, or interfering with needed evaluation and treatment. The
37 use of short-term, low-dose haloperidol is appropriate in this circumstance. Second-generation antipsychotics can also be used.
38
(Choice B) Benzodiazepines such as lorazepam have been shown to increase the incidence of delirium. They are appropriate only for the
39
treatment of delirium secondary to alcohol or benzodiazepine withdrawal, which is unlikely in this patient given her substance use history.
40
(Choice C) This patient is pulling at her intravenous line and could potentially pull at a catheter as well, which could be dangerous and add a
source for possible future infection. If she is incontinent, adult diapers are preferable.

(Choice D) Although frequent reorientation is helpful in cases of mild delirium, it is unlikely to manage adequately this patient's agitated and
combative behavior. Further intervention is necessary for the patient's safety. In addition, challenging delusions and hallucinations may lead to
further agitation and should be avoided.

(Choice E) Donepezil is a cholinesterase inhibitor used in the treatment of dementia. Cholinesterase inhibitors are not effective in treating
delirium.

(Choice F) Restraints can worsen delirium and should be reserved as a last resort when the patient's safety is in jeopardy.

Educational objective:
Management of delirium includes identifying and treating the underlying medical disorder and managing behavioral agitation. Low-dose
haloperidol can be used when severe agitation or psychotic symptoms place the patient at risk for harm.

References
Haloperidol, risperidone, olanzapine, and aripiprazole in the management of delirium: a comparison of efficacy, safety, and side effects.

(http://www.ncbi.nlm.nih.gov/pubmed/25191793)

Medicine Nervous System Delirium


Subject System Topic

https://t.me/USMLEWorldStep2CK

REVIEW

https://www.uworld.com/ClientApp/v15/apps/qbanktestinterface/index.html#/launchtest/7281878/nbme/229400737/3/1 1/1
‫ م‬9:09 2021/‫‏‬9/‫‏‬11 https://t.me/USMLEWorldStep2CK UWorld STEP2 SIM Form 1

1 Item 37 of 40
2
Mark
Question Id: 6841 Previous Next
3

5
The following vignette applies to the next 2 items. The items in the set must be answered in sequential order. Once you click Proceed to Next
Item, you will not be able to add or change an answer.
6

7 A 21-year-old college student is brought to the emergency department by his roommates, who say that he is not acting like himself. The patient
8 has no known medical history or physical symptoms. Over the past month, he has insisted that he is being watched or followed. Last week, he
9 was asked to leave a campus party due to a fight with a fellow student over whether the partygoer was following him. Although the patient is
10 usually a conscientious student, he recently stopped attending classes and said that a professor was putting thoughts in his head and tracking his
11 movements. Temperature is 37.2 C (99 F), blood pressure is 148/88 mm Hg, and pulse is 108/min. On examination, the patient appears nervous
12 and guarded. He refuses to answer questions, saying, "I can't talk about that." The patient repeatedly and quickly glances over his shoulder
13 toward the door. During the examination, he abruptly stands and shouts, "I have to go now!" The physician attempts to calm the patient and
14 reassure him that the hospital staff is trying to help. He pushes the physician aside, attempts to flee the emergency department, and is restrained
15 by 2 security guards. He kicks the walls and screams at the guards as he is escorted back to his room.
16
Item 1 of 2
17 Which of the following is the most appropriate course of action?
18

19  A. Advise security guards to allow the patient to leave the emergency department (3%)
20
B. Apply physical restraints but defer medication until the workup is complete (10%)
21

22 C. Ask the patient's roommates to speak with him while he is restrained by the security guards (2%)
23 D. Escort the patient to a locked seclusion room while additional information is obtained from his roommates (11%)
24
 E. Physically restrain the patient and administer an intramuscular antipsychotic (70%)
25

26

27

28 Incorrect 70%
29 Correct answer
E
 Answered correctly  04 secs
Time Spent  06/14/2021
Last Updated
30
31

32 Explanation
33
34 This patient is acutely psychotic, poses a danger to others, and is at risk of accidental self-injury in his present agitated state. His mental status
35 is unclear because he refuses to answer questions and has attempted to flee the hospital prior to a complete evaluation. He is at high risk of
36 continuing to act violently in response to paranoid delusions. Efforts to calm the patient with verbal techniques are unsuccessful, he has a history
37 of assaultive behavior (ie, recent fight), and he has been physically aggressive with the physician and security guards. To maintain safety, he
38 requires immediate emergency intervention, which should include both physical restraints and medications to calm him (typically an
39
intramuscular antipsychotic, a benzodiazepine, or a combination of the two).

40 Physical restraints should be implemented in a humane manner, ideally by a designated restraint team specifically trained in the procedure.
Restraints should be removed as soon as adequate sedation is achieved through pharmacologic means. Once the patient is calmer, the physician
should continue medical workup and formulate an appropriate treatment plan.

(Choice A) This patient's acutely psychotic, agitated state and unpredictable behavior make him a potential danger to himself and others. He
cannot be allowed to leave the hospital until the evaluation is complete and treatment is established.

(Choice B) As evidenced by his behavior in the emergency department, this patient will likely continue to struggle against physical restraints,
risking physical injury and rhabdomyolysis if the restraints are used for a prolonged period. It is safer and more humane to use medication to
achieve sedation and shorten the time needed for physical restraints.

(Choice C) The patient's roommates should not be asked to intervene because it puts them at risk for harm. In fact, they should be asked to
leave the area for their safety.

(Choice D) Although some psychiatric episodes can be safely deescalated in seclusion or quiet rooms, this patient has already demonstrated
behaviors (eg, kicking the walls) that could result in significant accidental self-injury if left alone in such a room.

Educational objective:
Acutely agitated patients who pose a danger to themselves or others may require physical restraints and emergency administration of sedating
intramuscular medications.

References
Approach to the agitated emergency department patient.

(http://www.ncbi.nlm.nih.gov/pubmed/29395692)
Chemical agents for the sedation of agitated patients in the ED: a systematic review.

(http://www.ncbi.nlm.nih.gov/pubmed/27707527)

Psychiatry Psychiatric/Behavioral & Substance Abuse Psychosis


Subject System Topic

https://t.me/USMLEWorldStep2CK

REVIEW

https://www.uworld.com/ClientApp/v15/apps/qbanktestinterface/index.html#/launchtest/7281878/nbme/229400737/3/1 1/1
‫ م‬9:09 2021/‫‏‬9/‫‏‬11 https://t.me/USMLEWorldStep2CK UWorld STEP2 SIM Form 1

1 Item 38 of 40
2
Mark
Question Id: 6951 Previous Next
3

4 Item 2 of 2
5 The patient is calmer after receiving ziprasidone but is still reluctant to answer questions. Physical restraints are removed, and he allows vital
6 signs and physical examination. Temperature is 37.2 C (99 F), blood pressure is 130/80 mm Hg, and pulse is 104/min. Physical examination
7 shows no abnormalities. ECG reveals sinus tachycardia, and the QTc interval is 505 ms. When the nurse requests a urine sample, the patient
8 throws the cup on the ground. He shouts, "My personal liberties are being violated," and attempts to kick the nurse. Which of the following is the
9 best next step in management of this patient?
10
 A. Additional ziprasidone (5%)
11
12 B. Haloperidol (19%)
13
C. Lithium (0%)
14

15  D. Lorazepam (52%)
16
E. Physical restraints only (21%)
17
18

19
20
Incorrect 52%
21
Correct answer
D
 Answered correctly  12 secs
Time Spent  06/14/2021
Last Updated

22

23

24 Explanation
25

26
Pharmacologic management of acutely psychotic patients who are severely agitated and potentially violent consists of antipsychotics,
benzodiazepines, or a combination of the two (eg, haloperidol plus lorazepam). Although this patient was calmer after receiving an initial dose
27
of ziprasidone, he is now becoming increasingly agitated and requires additional chemical sedation to prevent injury to himself or others.
28

29 Because this patient's QTc interval is >500 ms, medication selection must take into account the risk of QTc prolongation. As a class,
30 antipsychotics possess varying degrees of quinidine-like cardiac effects and can prolong QTc and lead to dysrhythmias, including torsade de
31 pointes. Among second-generation antipsychotics, ziprasidone is most likely to cause QTc prolongation. Given the potential class effect of
32 antipsychotics on the QTc interval, this patient's agitation should be managed with a benzodiazepine, as opposed to additional antipsychotics.
33 Lorazepam is a commonly used benzodiazepine in the treatment of agitation because of its availability in an intramuscular formulation, rapid
34 onset, and relatively safe therapeutic index.
35 (Choices A and B) Benzodiazepines are preferred for the management of acute agitation in patients who have a QTc of ≥500 ms due to the risk
36 of QTc prolongation associated with antipsychotics. Ziprasidone and the intravenous formulation of haloperidol, in particular, carry a significant
37 risk for QTc prolongation.
38
(Choice C) Lithium, a mood stabilizer used primarily in the treatment of bipolar disorder, requires gradual titration to achieve therapeutic blood
39
levels. It is not effective for the treatment of acutely agitated, psychotic patients.
40
(Choice E) This patient would likely struggle against physical restraints, which should be accompanied by chemical sedation to reduce the risk of
injury to both the patient and the staff.

Educational objective:
Medication options for acute psychotic agitation include antipsychotics and benzodiazepines. Benzodiazepines are preferred for the management
of acute agitation in patients who have a QTc ≥500 ms due to the risk of QTc prolongation associated with antipsychotics.

References
QTc prolongation, torsades de pointes, and psychotropic medications.

(http://www.ncbi.nlm.nih.gov/pubmed/23295003)
Management of psychosis and agitation in medical-surgical patients who have or are at risk for prolonged QT interval.

(http://www.ncbi.nlm.nih.gov/pubmed/25226194)

Psychiatry Psychiatric/Behavioral & Substance Abuse Brief psychotic disorder


Subject System Topic

https://t.me/USMLEWorldStep2CK

REVIEW

https://www.uworld.com/ClientApp/v15/apps/qbanktestinterface/index.html#/launchtest/7281878/nbme/229400737/3/1 1/1
‫ م‬9:09 2021/‫‏‬9/‫‏‬11 https://t.me/USMLEWorldStep2CK UWorld STEP2 SIM Form 1

1 Item 39 of 40
2
Mark
Question Id: 6953 Previous Next
3

5
The response options for the next 2 items are the same. Select one answer for each item in the set.

6
Item 1 of 2
7
A 73-year-old woman is hospitalized following an acute anterior wall myocardial infarction. On day 3 of her hospitalization, she develops
8
shortness of breath and chest discomfort. Blood pressure is 80/40 mm Hg and pulse is 120/min. On physical examination, there are crackles at
9
bilateral lung bases and a holosystolic murmur at the left sternal border. Cardiac catheterization findings are given below:
10
11 Oxygen saturation, arterial 90%
12
Oxygen saturation, superior vena cava 64%
13

14 Oxygen saturation, right atrium 65%


15
Oxygen saturation, right ventricle 79%
16
Oxygen saturation, pulmonary artery 80%
17
18 Cardiac output 2.5 L/min (normal: 4-8 L/min)
19
Right atrial pressure 12 mm Hg (normal: 1-6 mm Hg)
20

21 Right ventricular pressure 45/11 mm Hg (normal: 15-30/1-6 mm Hg)


22 Pulmonary artery pressure 47/17 mm Hg (normal: 15-30/6-12 mm Hg)
23
Pulmonary capillary wedge pressure 20 mm Hg (normal: 6-12 mm Hg)
24
25 Which of the following is the most likely diagnosis?
26

27  A. Hypovolemic state (0%)


28
B. Septic shock, hyperdynamic phase (0%)
29
30
C. Left ventricular pump failure (14%)

31 D. Atrial septal defect (1%)


32
 E. Ventricular septal defect (59%)
33
34 F. Acute mitral regurgitation (14%)
35
G. Right ventricular infarction (1%)
36

37
H. Pulmonary embolism (1%)

38 I. Cardiac tamponade (5%)


39

40

Incorrect 59%
Correct answer  Answered correctly  06 secs
Time Spent  08/07/2021
Last Updated
E

Explanation

This patient likely has a new ventricular septal defect (VSD), a complication that can occur due to rupture of the interventricular septum 3-5 days
after a myocardial infarction. Acute hemodynamic instability (eg, hypotension, tachycardia) and a holosystolic murmur heard best at the left
sternal border are classic findings. This complication is usually diagnosed by pulmonary artery catheterization or echocardiography.

Analysis of this patient's oxygen saturation readings reveals that the blood in the right atrium has a 65% oxygenation and the blood in the right
ventricle has a 79% saturation, whereas in normal circumstances these readings should be similar. This gradient in oxygen saturation suggests
that there is oxygenated blood from the left heart flowing into the right heart at the ventricular level, as might occur with VSD. The increased
right-sided heart pressures are due to increased blood flow from the left-to-right shunt. The low cardiac output also stems from this new
defect. Pulmonary capillary wedge pressure (reflecting left atrial pressures) may be elevated.

(Choice A) Hypovolemia can cause hemodynamic instability, but a murmur is not expected. Furthermore, the pulmonary capillary wedge
pressure would be low, not elevated, in hypovolemia.

(Choice B) In septic shock, the pulmonary capillary wedge pressure would be low, as the peripheral blood vessels are dilated causing a relative
intravascular volume deficit.

(Choices C and F) Left ventricular pump failure and acute mitral regurgitation can lead to elevated right-sided pressures, but increased oxygen
saturation of the blood in the right ventricle would not be expected.

(Choice D) Atrial septal defects (ASDs) do not commonly stem from myocardial infarctions. In an ASD, the abnormal oxygen saturation gradient
would be between the superior vena cava and the right atrium.

(Choices G and H) Right ventricular infarction and pulmonary embolism can lead to elevated right atrial pressures but would not change the
right-sided oxygen saturations.

(Choice I) Cardiac tamponade classically causes equilibration of diastolic pressures, which is not seen in these cardiac catheterization findings.

Educational objective:
Ventricular septum rupture is a complication that can occur 3-5 days after a myocardial infarction. Patients present with acute hemodynamic
compromise and a holosystolic murmur at the left sternal border. Cardiac catheterization may show an influx of oxygenated blood from the left
ventricle into the right ventricle, as well as an increase in right-sided heart pressures due to increased flow.

Medicine Cardiovascular System Ventricular septal defect


Subject System Topic

https://t.me/USMLEWorldStep2CK

REVIEW

https://www.uworld.com/ClientApp/v15/apps/qbanktestinterface/index.html#/launchtest/7281878/nbme/229400737/3/1 1/1
‫ م‬9:10 2021/‫‏‬9/‫‏‬11 https://t.me/USMLEWorldStep2CK UWorld STEP2 SIM Form 1

1 Item 40 of 40
2
Mark
Question Id: 6991 Previous Next
3

4 Item 2 of 2
5
A 74-year-old nursing home resident is hospitalized with altered mental status and poor oral intake. On day 3 of her hospitalization, she develops
6
shortness of breath and chest discomfort. Blood pressure is 78/40 mm Hg and pulse is 125/min. On physical examination, the lungs are clear, but
7
a soft holosystolic murmur is heard at the left sternal border. Cardiac catheterization findings are given below:
8

9 Oxygen saturation, arterial 82%


10
Oxygen saturation, superior vena cava 60%
11
12
Oxygen saturation, right atrium 62%

13 Oxygen saturation, right ventricle 62%


14
Oxygen saturation, pulmonary artery 61%
15
16 Cardiac output 2.4 L/min (normal: 4-8 L/min)
17 Right atrial pressure 13 mm Hg (normal: 1-6 mm Hg)
18
Right ventricular pressure 52/12 mm Hg (normal: 15-30/1-6 mm Hg)
19
20 Pulmonary artery pressure 54/16 mm Hg (normal: 15-30/6-12 mm Hg)
21
Pulmonary capillary wedge pressure 6 mm Hg (normal: 6-12 mm Hg)
22

23 Which of the following is the most likely diagnosis?


24
 A. Hypovolemic state (4%)
25

26 B. Septic shock, hyperdynamic phase (3%)


27
C. Left ventricular pump failure (4%)
28

29 D. Atrial septal defect (2%)


30
E. Ventricular septal defect (3%)
31

32
F. Acute mitral regurgitation (5%)

33 G. Right ventricular infarction (12%)


34
 H. Pulmonary embolism (58%)
35

36 I. Cardiac tamponade (4%)


37

38

39
Incorrect 58%
40 Correct answer  Answered correctly  07 secs
Time Spent  08/07/2021
Last Updated
H

Explanation

This patient has chest pain, shortness of breath, hypotension, tachycardia, and hypoxia (as evidenced by her low arterial oxygen saturation), but
her lungs are clear on exam. This clinical situation should raise suspicion for pulmonary embolism, even without viewing the cardiac
catheterization findings. The soft holosystolic murmur at the left sternal border is most likely due to tricuspid regurgitation from elevated right-
sided heart pressures.

The cardiac catheterization reveals a low cardiac output, elevated right atrial pressure, elevated right ventricular pressure, and elevated pulmonary
artery pressure. However, the pulmonary capillary wedge pressure (PCWP), which reflects left atrial pressure, is normal. Taken together, these
values suggest increased resistance between the pulmonary artery and the pulmonary capillaries and left ventricle, which is consistent with
presence of a pulmonary embolism.

(Choice A) Hypovolemia does not cause elevated right heart pressures.

(Choice B) Septic shock should be a consideration in this patient, but is unlikely given the pressure gradient between the pulmonary arterial
pressure and the PCWP.

(Choice C) Left ventricular pump failure can cause elevation of right-sided heart pressures, but this would be accompanied by an elevation of the
PCWP.

(Choice D) An atrial septal defect would cause an increase in oxygenation within the right heart (right atrium and ventricle).

(Choice E) A ventricular septal defect would cause an increase in oxygenation in the right ventricle.

(Choice F) Acute mitral regurgitation would cause elevation of the PCWP.

(Choice G) Right ventricular infarction would not increase the pulmonary artery pressure.

(Choice I) Cardiac tamponade classically causes equilibration of diastolic pressures on cardiac catheterization.

Educational objective:
Pulmonary embolism should be considered in patients with hypoxia, tachypnea, and tachycardia. On cardiac catheterization, right heart pressures
are elevated, as is the gradient between the pulmonary artery and the pulmonary capillary wedge pressure.

Medicine Cardiovascular System Pulmonary embolism


Subject System Topic

https://t.me/USMLEWorldStep2CK

REVIEW

https://www.uworld.com/ClientApp/v15/apps/qbanktestinterface/index.html#/launchtest/7281878/nbme/229400737/3/1 1/1
‫ م‬9:11 2021/‫‏‬9/‫‏‬11 https://t.me/USMLEWorldStep2CK UWorld STEP2 SIM Form 1

1 Item 1 of 40 ABC 0.25

2
Mark
Question Id: 6836 Full Screen Tutorial Lab Values Notes Calculator Reverse Color Text Zoom Settings
Previous Next
3

4 A 63-year-old woman comes to the office with a one-day history of pain in her right hand. She has a history of rheumatoid arthritis that has been
5 well controlled with methotrexate and occasional ibuprofen. The patient's medical history is otherwise unremarkable, and she does not use
6 tobacco, alcohol, or illicit drugs. Temperature is 38.2 C (100.8 F), blood pressure is 140/90 mm Hg, and pulse is 90/min. Musculoskeletal
7 examination shows swelling of the right second metacarpophalangeal joint with guarding and exquisite tenderness to palpation. There are also
8 joint deformities involving the wrists and metacarpophalangeal and proximal interphalangeal joints bilaterally. Which of the following is the most
9 likely cause of this patient's current problem?
10
A. Flare-up of rheumatoid arthritis (33%)
11

12 B. Gram-negative bacterial infection (7%)


13
 C. Gram-positive bacterial infection (57%)
14

15 D. Mycobacterial infection (0%)


16
E. Osteoarthritis (1%)
17

18

19

20
Omitted 57%
21
Correct answer  Answered correctly  01 sec
Time Spent  06/06/2021
Last Updated
C
22

23

24 Explanation
25

26
Septic arthritis
27

28 Abnormal joint: OA, RA, prosthetic joint, gout


29
Age >80

30
Risk factors Diabetes
Intravenous drug use, heavy alcohol use
31
Intraarticular glucocorticoid injections
32

33 Acute monarthritis: hot, swollen, decreased ROM


34 Clinical features Fever
35 Elevated ESR & CRP
36
Blood cultures
37 Diagnosis
Synovial fluid analysis: leukocytosis (>50,000/mm3), Gram stain, culture
38
Gram-positive cocci: vancomycin
39
Gram-negative rod: third-generation cephalosporin
40 Initial treatment
Negative microscopy: vancomycin (+ third-generation cephalosporin if
immunocompromised)

CRP = C-reactive protein; ESR = erythrocyte sedimentation rate; OA = osteoarthritis; RA = rheumatoid arthritis; ROM = range
of motion.

This patient has fever and acute monoarticular arthritis with a background history of rheumatoid arthritis. This presentation is most consistent with
septic arthritis. Septic arthritis is more likely to occur in joints that have pre-existing abnormalities (eg, osteoarthritis, chronic rheumatoid
arthritis). Other risk factors include advanced age, diabetes mellitus, joint surgery or glucocorticoid injection, and intravenous drug use. Septic
arthritis also occurs with increased frequency in prosthetic joints, either due to contamination in the intraoperative period or later hematogenous
seeding.

Given her age and lack of intravenous drug use, this patient's infection is most likely due to gram-positive organisms such as Staphylococcus
aureus. Septic arthritis should be evaluated promptly with arthrocentesis for Gram stain and culture because joint destruction can progress
rapidly. Synovial fluid analysis typically reveals >50,000 leukocytes/mm3 with >90% neutrophils, although cell counts may be lower in infected
prosthetic joints with more indolent organisms (eg, Propionibacterium). Antibiotics should be initiated after joint aspiration is performed, and
surgical joint drainage may be required in severe cases.

(Choice A) Rheumatoid arthritis can occasionally present with an acute monoarthritis with fever resembling septic arthritis. However, flares of
rheumatoid arthritis are usually less abrupt and more likely to be polyarticular.

(Choice B) Gram-negative bacteria (eg, Escherichia coli, Pseudomonas aeruginosa) can cause septic arthritis, but these organisms are typically
seen in patients with a history of intravenous drug use, severe immunosuppression, or advanced age. Neisseria gonorrhea is a common cause of
septic arthritis in sexually active young adults.

(Choice D) Tuberculous and nontuberculous mycobacteria are occasional pathogens in septic arthritis. They typically present with an indolent,
persistent, culture-negative oligo- or monoarthritis. This patient's acute presentation is more consistent with gram-positive septic arthritis.

(Choice E) Osteoarthritis is most often seen in the proximal and distal interphalangeal joints. Metacarpophalangeal joint involvement is unusual.

Educational objective:
Septic arthritis presents with acute monoarthritis and fever and is more likely to occur in joints that have pre-existing abnormalities (eg,
osteoarthritis, rheumatoid arthritis). Gram-positive organisms (eg, Staphylococcus aureus) are most common. Diagnosis requires prompt
arthrocentesis for Gram stain and culture.

References
Clinical management of septic arthritis.

(http://www.ncbi.nlm.nih.gov/pubmed/23591823)
Approach to septic arthritis.

(http://www.ncbi.nlm.nih.gov/pubmed/21916390)

Medicine Rheumatology/Orthopedics & Sports Septic arthritis


Subject System Topic

https://t.me/USMLEWorldStep2CK

REVIEW 0
Feedback End Block

https://www.uworld.com/ClientApp/v15/apps/qbanktestinterface/index.html#/launchtest/7281878/nbme/229400738/3/1 1/1
‫ م‬9:11 2021/‫‏‬9/‫‏‬11 https://t.me/USMLEWorldStep2CK UWorld STEP2 SIM Form 1

1 Item 2 of 40 ABC 0.25

2
Mark
Question Id: 6842 Full Screen Tutorial Lab Values Notes Calculator Reverse Color Text Zoom Settings
Previous Next
3

4 An 18-month-old girl is brought to the emergency department due to bloody stools. Twice this morning, the patient passed large, maroon stools.
5 She has not vomited, has not been fussy, and has been eating normally. The patient has no chronic medical conditions and takes no daily
6 medications. Vital signs are normal. The abdomen is soft and nontender; there is no hepatosplenomegaly. No anal fissures or hemorrhoids are
7 present. This patient's condition is most likely caused by which of the following?
8

9
A. Adenomatous colon polyp (15%)

10 B. Ectopic gastric mucosa (66%)



11
C. IgA vasculitis (2%)
12

13 D. Inflammatory bowel disease (1%)


14
E. Telescoping bowel segments (11%)
15

16

17

18 Omitted 66% 03 secs 07/12/2021


19
Correct answer
B
 Answered correctly  Time Spent  Last Updated

20

21

22 Explanation
23

24
Lower gastrointestinal bleeding in children
25

26 Typical age of
Condition Clinical features Diagnosis
27 presentation
28
Well-appearing
29 Anal fissure Any Anal pain with defecation Anal tear on examination
30 Constipation
31
Diarrhea
32
Infectious colitis Any Abdominal pain Stool culture
33
Fever
34

35 Well-appearing
36 Food protein–induced allergic Normal weight gain Resolution with milk/soy
Infancy
37 proctocolitis Blood-/mucus-streaked elimination diet
38
stools

39 Colicky abdominal pain Abdominal ultrasound


Intussusception Early childhood
40 Currant jelly stools Air enema

Meckel diverticulum Early childhood Painless rectal bleeding Meckel scan

Juvenile polyp School-aged Painless rectal bleeding Colonoscopy

Abdominal pain
Inflammatory Diarrhea
Adolescence Colonoscopy & upper endoscopy
bowel disease Weight loss/failure to
thrive

This young child has sudden-onset, painless gastrointestinal (GI) bleeding with a normal examination and no other systemic features. These
findings are concerning for a Meckel diverticulum, the most common congenital intestinal abnormality.

Meckel diverticulum is a remnant of a persistent vitelline duct and is typically located within 2 feet of the ileocecal valve. The diverticulum often
contains ectopic gastric mucosa, which secretes hydrochloric acid that can lead to ulceration of the adjacent small bowel with subsequent GI
bleeding (ie, red or maroon stools). Presentation is most common at age <2, and bleeding is typically painless and not accompanied by other GI
symptoms (eg, diarrhea, vomiting).

A commonly used diagnostic test is the technetium-99m pertechnetate scan, or Meckel scan, during which the radiotracer localizes to the
ectopic gastric mucosa within the diverticulum. Surgical management is typically required for symptomatic patients.

(Choice A) Adenomatous colon polyps are unlikely in this patient because they usually present in adulthood and are often asymptomatic (but
may cause hematochezia). In contrast, juvenile polyps (which typically present with painless hematochezia at age 2-10) are most commonly
hamartomas and may be considered in this patient if Meckel scan is negative.

(Choice C) IgA vasculitis (Henoch-Schönlein purpura) can cause GI bleeding due to inflamed intestinal vessels. However, this condition is most
common in school-aged children and also causes purpuric rash on the lower extremities, arthralgias, and abdominal pain.

(Choice D) Very early onset inflammatory bowel disease is a rare cause of GI bleeding in young children and is typically accompanied by fever,
weight loss, and abdominal pain, which are not seen in this patient.

(Choice E) Telescoping bowel segments, or intussusception, presents with colicky abdominal pain and currant jelly stools. Although Meckel
diverticulum can serve as a lead point for intussusception, it is unlikely in this well-appearing patient with no fussiness and a normal abdominal
examination.

Educational objective:
Painless gastrointestinal bleeding, particularly in a child age <2, should raise concern for Meckel diverticulum. Pathogenesis involves small bowel
ulceration and bleeding due to the secretion of acid by ectopic gastric mucosa within the diverticulum.

References
Clinical characteristics of Meckel diverticulum in children: a retrospective review of a 15-year single-center experience.

(http://www.ncbi.nlm.nih.gov/pubmed/28796070)

Pediatrics Gastrointestinal & Nutrition Meckel diverticulum


Subject System Topic

https://t.me/USMLEWorldStep2CK

REVIEW 0
Feedback End Block

https://www.uworld.com/ClientApp/v15/apps/qbanktestinterface/index.html#/launchtest/7281878/nbme/229400738/3/1 1/1
‫ م‬9:11 2021/‫‏‬9/‫‏‬11 https://t.me/USMLEWorldStep2CK UWorld STEP2 SIM Form 1

1 Item 3 of 40 ABC 0.25

2
Mark
Question Id: 6844 Full Screen Tutorial Lab Values Notes Calculator Reverse Color Text Zoom Settings
Previous Next
3

4 A 54-year-old woman comes to the emergency department due to progressive fatigue over the last 3 days. She says that she could not get out of
5 bed today due to weakness. She has had mild shortness of breath, fever, and nausea. She has not had any chest pain, abdominal pain, diarrhea,
6 or urinary symptoms. Three years ago, the patient was diagnosed with undifferentiated ductal breast cancer and underwent a left radical
7 mastectomy. Six months ago, she was found to have lytic lesions in her vertebrae and right femur. In addition, she has a single mass in her liver.
8 Her current medications include an antiestrogen medication and pain killers. Her blood pressure is 110/70 mm Hg and pulse is 120/min.
9 Laboratory findings are as follows:
10
Hemoglobin 6.3 g/dL
11

12 Leukocytes 18,000/mm3
13 Platelets 42,000/mm3
14
Mean corpuscular volume 85 fl
15

16 Reticulocyte count 7.6%


17

18 Total bilirubin 2.4 mg/dL


19
Direct bilirubin 0.3 mg/dL
20

21 Aspartate aminotransferase 50 U/L


22
Alanine aminotransferase 64 U/L
23

24
Alkaline phosphatase 160 U/L

25 Lactate dehydrogenase 1200 U/L


26

27
Sodium 138 mEq/L
28

29
Potassium 3.8 mEq/L

30 Bicarbonate 22 mEq/L
31
Chloride 110 mEq/L
32

33 Blood urea nitrogen 31 mg/dL


34 Creatinine 1.8 mg/dL
35

36
INR 3.2 (normal: 0.8-1.2)
37

38
Fibrinogen 125 mg/dL (normal: 200-348 mg/dL)

39
Her complete blood count a month ago was normal. Which of the following is the most likely cause of this patient’s current anemia?
40

 A. Bone marrow infiltration (16%)

B. Anemia of chronic disease (10%)

C. Autoimmune hemolysis (16%)

 D. Microangiopathic hemolytic anemia (55%)

E. Iron deficiency (1%)

F. Impaired folic acid metabolism (0%)

Incorrect 55%
 04 secs 07/30/2021
Correct answer  Answered correctly Time Spent  Last Updated
D

Explanation

This patient likely has disseminated intravascular coagulation (DIC), a disorder that is associated with metastatic breast carcinoma. DIC is the
most common coagulopathy in patients with malignancies, and can be seen in gastric, breast, and lung cancer, among others. This patient’s
laboratory values show thrombocytopenia, decreased fibrinogen, and increased INR, which are indicative of DIC.

The patient's elevated lactate dehydrogenase (LDH), reticulocyte count, and bilirubin levels are consistent with hemolysis resulting from
microangiopathic hemolytic anemia (MAHA), which is commonly seen in association with DIC. MAHA describes nonimmune hemolysis from
erythrocyte fragmentation and can develop in the absence of DIC; schistocytes are seen on peripheral smear.

(Choice A) Bone marrow infiltration by tumor cells would most commonly cause pancytopenia. The laboratory studies in this patient, however,
suggest cellular injury (high LDH and reticulocytosis). She also has leukocytosis, not leukopenia.

(Choice B) Anemia of chronic disease produces a normocytic anemia with a normal reticulocyte count. In addition, this patient's complete blood
count was normal a month ago, making anemia of chronic disease less likely.

(Choice C) Autoimmune destruction of red blood cells results from IgG-induced hemolysis. Although anemia, reticulocytosis, jaundice, and an
elevated LDH are commonly seen, a decreased fibrinogen level and a prolonged INR would not be expected.

(Choice E) Iron deficiency produces a hypochromic, microcytic anemia, and is typically caused by chronic gastrointestinal blood loss in adults. It
does not, however, produce many of the laboratory abnormalities seen in this patient.

(Choice F) Impaired folic acid metabolism produces a megaloblastic anemia and would not be associated with laboratory signs of DIC.

Educational objective:
Malignancy can cause disseminated intravascular coagulopathy, and the anemia seen in patients with DIC is often due to microangiopathic
hemolysis.

Medicine Hematology & Oncology DIC


Subject System Topic

https://t.me/USMLEWorldStep2CK

REVIEW 0
Feedback End Block

https://www.uworld.com/ClientApp/v15/apps/qbanktestinterface/index.html#/launchtest/7281878/nbme/229400738/3/1 1/1
‫ م‬9:12 2021/‫‏‬9/‫‏‬11 https://t.me/USMLEWorldStep2CK UWorld STEP2 SIM Form 1

1 Item 4 of 40 ABC 0.25

2
Mark
Question Id: 6845 Full Screen Tutorial Lab Values Notes Calculator Reverse Color Text Zoom Settings
Previous Next
3

4 A 44-year-old woman comes to the office due to progressive shortness of breath and cough. She can hardly walk a block without having to stop
5 due to dyspnea. The patient recently immigrated to the United States from the Middle East. She has a history of a "heart murmur" and joint pains
6 as an adolescent. She is a lifetime nonsmoker. Blood pressure is 110/70 mm Hg and pulse is 110/min and irregularly irregular. ECG shows atrial
7 fibrillation. An echocardiogram shows thickening of the mitral valve leaflets with commissural fusion and restricted mitral valve opening. Which of
8 the following is the most likely finding on physical examination?
9
A. Early diastolic murmur at the left sternal border (7%)
10

11 B. Fourth heart sound at the cardiac apex (1%)


12
C. Late systolic murmur at the cardiac apex (6%)
13

14  D. Mid-diastolic murmur at the cardiac apex (80%)


15
 E. Presystolic murmur at the cardiac apex (4%)
16

17

18

19 Incorrect 80%
20
Correct answer  Answered correctly  03 secs
Time Spent  07/16/2021
Last Updated
D
21

22

23 Explanation
24

25
This patient's clinical features (progressive dyspnea, cough, history of joint pains and heart murmur) and echocardiogram findings (restricted mitral

26
valve opening) are consistent with rheumatic mitral stenosis. Cardiac auscultation in patients with mitral stenosis indicates a loud first heart
sound, an early diastolic sound after the second heart sound (opening snap), and a characteristic murmur.
27

28 The murmur of mitral stenosis is a low-pitched diastolic rumble, best heard at the apex (using the bell of the stethoscope) with the patient lying
29 on the left side and the breath held in end expiration.
30
When mitral stenosis is mild, the murmur is mostly heard in late diastole.
31
As the stenosis progresses (eg, worsening symptoms in this patient), the diastolic murmur is heard earlier in the cardiac cycle (eg, mid-
32
diastolic murmur) and eventually can be heard immediately after the opening snap.
33
Patients with mitral stenosis who are in sinus rhythm (unlike this patient who is in atrial fibrillation) also have an increased intensity of the diastolic
34
murmur with atrial contraction during late diastole, a phenomenon termed "presystolic accentuation" (Choice E).
35

36 (Choice A) An early diastolic murmur at the left lower sternal border is often heard in patients with aortic regurgitation, whereas the murmur of
37 mitral stenosis is typically best heard at the cardiac apex.
38
(Choice B) The fourth heart sound is produced by the filling of the stiff right or left ventricle during atrial contraction. It would not be expected in
39 patients with atrial fibrillation.
40
(Choice C) A late systolic murmur is typically heard in patients with mitral valve prolapse. Isolated and uncomplicated mitral stenosis does not
cause systolic murmurs.

Educational objective:
Cardiac auscultation in patients with mitral stenosis indicates a loud first heart sound, an early diastolic sound after second heart sound (opening
snap), and a low-pitched diastolic murmur heard best at the cardiac apex.

Medicine Cardiovascular System Mitral stenosis


Subject System Topic

https://t.me/USMLEWorldStep2CK

REVIEW 0
Feedback End Block

https://www.uworld.com/ClientApp/v15/apps/qbanktestinterface/index.html#/launchtest/7281878/nbme/229400738/3/1 1/1
‫ م‬9:12 2021/‫‏‬9/‫‏‬11 https://t.me/USMLEWorldStep2CK UWorld STEP2 SIM Form 1

1 Item 5 of 40 ABC 0.25

2
Mark
Question Id: 6849 Full Screen Tutorial Lab Values Notes Calculator Reverse Color Text Zoom Settings
Previous Next
3

4 A 68-year-old otherwise healthy woman comes to the office for a routine health maintenance visit. She has no current symptoms. The patient
5 walks 3.2 km (2 mi) per day and has a healthy diet. She is not sexually active and does not use alcohol or tobacco. She takes a daily
6 multivitamin. Blood pressure is 125/75 mm Hg and pulse is 78/min. Physical examination is unremarkable. Two years ago, her blood tests
7 showed a total cholesterol level of 157 mg/dL, HDL of 65 mg/dL, and LDL of 70 mg/dL (10-year estimated cardiac risk: 6.4%). A mammogram at
8 that time was also normal. The patient had a normal colonoscopy 5 years ago and a normal Pap test with human papillomavirus cotesting 3 years
9 ago. Which of the following tests is most appropriate for this patient at the present time?
10
A. Colonoscopy (1%)
11

12 B. Lipid profile (21%)


13
 C. Mammogram (73%)
14

15 D. Pap test (1%)


16
 E. TSH level (2%)
17

18

19

20
Incorrect 73%
21
Correct answer  Answered correctly  02 secs
Time Spent  07/13/2021
Last Updated
C
22

23

24 Explanation
25

26
US Preventive Services Task Force
27
recommendations for breast cancer screening*
28

29
Definitely beneficial: Mammogram every 2 years for women age 50-

30
recommended 74

31 Possibly beneficial:
Mammogram in women age 40-49
32 individualize
33
Uncertain benefit Mammogram in women age ≥75
34

35 *For average-risk individuals.


36
Breast cancer screening with mammography is recommended for all women age 50-74. Screening can be considered beginning at age 40,
37
although the prevalence is lower and false-positive rate is higher at this age. The greatest reduction in breast cancer mortality appears to be in
38
women age 60-69. Screening after age 75 appears to be less beneficial.
39
Mammography should be performed at least every 2 years. A more frequent interval (eg, yearly) is advocated by many experts. This patient's
40
last mammogram was 2 years ago, so repeat mammography is indicated at this time.

(Choice A) Colon cancer screening is recommended for patients age 45-70 and may be considered in younger patients with a significant family
history of colorectal cancer. Screening options include high-sensitivity fecal occult blood testing annually, multitarget stool DNA test every 1-3
years, or colonoscopy every 10 years. This patient had a colonoscopy 5 years ago and does not need further screening at this time.

(Choice B) Cholesterol screening with a serum lipid panel is recommended for most adults, with lipid-lowering therapy (eg, statins) indicated for
those with a calculated 10-year risk >7%-10%. Lipid values typically change slowly over time; for average-risk patients in whom a statin is not
indicated, a 3-5–year screening interval is appropriate.

(Choice D) Cervical cancer screening is recommended for women age 21-65. Options include cervical cytology (eg, Pap test) every 3 years, or,
for women age 30-65, high-risk human papillomavirus testing (with or without cytology cotesting) every 5 years. Women who have had adequate
screening can discontinue at age 65.

(Choice E) There is currently insufficient evidence to recommend for or against thyroid disease screening in asymptomatic patients (eg, weight
changes, tachycardia).

Educational objective:
Breast cancer screening with mammography is recommended for all women age 50-74 and can be considered beginning at age 40. Screening
after age 75 appears to be less beneficial. Mammography should be performed at least every 2 years.

References
Screening for breast cancer: recommendation statement.

(http://www.ncbi.nlm.nih.gov/pubmed/27175847)

Medicine Female Reproductive System & Breast Breast cancer


Subject System Topic

https://t.me/USMLEWorldStep2CK

REVIEW 0
Feedback End Block

https://www.uworld.com/ClientApp/v15/apps/qbanktestinterface/index.html#/launchtest/7281878/nbme/229400738/3/1 1/1
‫ م‬9:12 2021/‫‏‬9/‫‏‬11 https://t.me/USMLEWorldStep2CK UWorld STEP2 SIM Form 1

1 Item 6 of 40 ABC 0.25

2
Mark
Question Id: 20753 Full Screen Tutorial Lab Values Notes Calculator Reverse Color Text Zoom Settings
Previous Next
3

4 A randomized controlled trial is conducted to investigate the effect of a sliding hip screw versus cancellous screws on the risk of reoperation after
5 surgery for hip fractures. One thousand patients age ≥50 with a low-energy hip fracture requiring fracture fixation are randomly assigned to
6 receive a sliding hip screw (n = 500) or multiple cancellous screws (n = 500). The primary outcome is rate of hip reoperation within 24 months
7 after initial surgery. The study has an 80% power to detect a difference in reoperation within 24 months between surgical groups of ≥10% at a
8 0.05 significance level. Results show no statistically significant difference between groups (p = 0.18), and researchers conclude that the lack of
9 statistical significance may be due to a type II error. If the study has no participants lost to follow-up, which of the following justifies the
10 researchers' conclusion?
11
A. The method used to assign participants to treatments increased the chance of selection bias (6%)
12

13 B. The number of participants left at the end of the study reduced the power of the study (15%)
14
 C. The real difference between groups was less than expected at the beginning of the study (57%)
15

16  D. The significance level was too high to detect a significant difference between groups (20%)
17

18

19
Incorrect 57%
20 Correct answer  Answered correctly  03 secs
Time Spent  07/11/2021
Last Updated
21
C

22

23
Explanation
24

25
This study comparing 2 surgical groups (n = 500/group) has 80% power to detect a ≥10% difference in reoperation rates at a 5% significance
26 level. It failed to detect a difference between groups (p = 0.18), which researchers attributed to a type II error. The power of a study is the
27 complement of the probability β of a type II error (power = 1 − β):
28
Type II error: failure to detect a difference between groups (ie, an effect) when one truly exists
29

30 Power: ability to detect a difference between groups (ie, an effect) when one truly exists
31
When comparing groups in a study (eg, comparing the distributions of a variable in 2 groups), type II error and power are influenced by 4
32 interrelated factors:
33
Sample size (n): With larger sample sizes, the difference between groups is easier to detect. Therefore, with all other factors fixed, larger
34
sample size = ↑ power, ↓ β. This study had no loss to follow-up, meaning no change in sample size that could explain a type II error
35
(Choice B).
36

37 Significance level (α): The significance level is the threshold used to establish statistical significance. The lower the significance level of a
38 study (eg, 1% vs 5% threshold), the more difficult it is to detect differences that might exist. Therefore, higher significance levels = ↑

39
power, ↓ β; or in other words, a significance level that is too low (not too high) could cause a type II error (Choice D).

40 Variability (eg, standard deviation) of study outcome: In outcomes with smaller variability, the distributions are tighter around their means,
making the difference between groups easier to detect. Therefore, smaller variability = ↑ power, ↓ β. A type II error could have occurred if
the outcome had a larger variability than expected.

Effect size (eg, difference between treatment groups): With larger effect sizes, the difference between groups is easier to detect.
Therefore, larger effect size = ↑ power, ↓ β. A type II error could have occurred if the effect size (ie, real difference between groups) is
smaller (ie, less) than expected.

(Choice A) Random assignment of participants to treatments minimizes selection bias. Also, selection bias has no impact on the probability of a
type II error (ie, failing to detect a difference that exists).

Educational objective:
Type II error describes a study's failure to detect an effect (eg, difference between groups) when one truly exists. The probability of a type II error
is affected by sample size, outcome variability, effect size, and significance level.

Surgery Biostatistics & Epidemiology Power and sample size


Subject System Topic

https://t.me/USMLEWorldStep2CK

REVIEW 0
Feedback End Block

https://www.uworld.com/ClientApp/v15/apps/qbanktestinterface/index.html#/launchtest/7281878/nbme/229400738/3/1 1/1
‫ م‬9:12 2021/‫‏‬9/‫‏‬11 https://t.me/USMLEWorldStep2CK UWorld STEP2 SIM Form 1

1 Item 7 of 40 ABC 0.25

2
Mark
Question Id: 6859 Full Screen Tutorial Lab Values Notes Calculator Reverse Color Text Zoom Settings
Previous Next
3

4 An 86-year-old nursing home resident is brought to the hospital due to a one day history of progressive lethargy and fever. In the emergency
5 department, the patient is found to have a blood pressure of 75/40 mm Hg, pulse of 120/min, and oxygen saturation of 85% on room air. His
6 leukocyte count is 22,000/mm3 with 7% bands, and his urine contains greater than 100 WBC/hpf. The patient is intubated, resuscitated with
7 intravenous fluids, and started on broad-spectrum intravenous antibiotics. Three days later, examination reveals non-blanching erythema on both
8 heels. Which of the following is the best intervention at this point?
9

10  A. Place pillows under both lower legs (82%)

11 B. Avoid frequent patient repositioning (1%)


12
C. Massage both heels twice per day (11%)
13

14 D. Apply low-potency corticosteroid cream (3%)


15
 E. Apply topical hydrogen peroxide (0%)
16

17

18

19 Incorrect 82%
 01 sec 07/30/2021
20
Correct answer  Answered correctly Time Spent  Last Updated
A
21

22

23 Explanation
24

25 This patient’s initial presentation of lethargy, hemodynamic instability, leukocytosis, and pyuria is most consistent with sepsis from a urinary tract
26
infection. The initial treatment geared towards treating his sepsis is appropriate. Pressure ulcers are a frequent complication seen in patients

27
with limited mobility, such as those in the intensive care unit.

28 This patient has nonblanchable erythema on his heels without the development of frank ulcers, suggesting that the process is in its fairly early
29 stages. This finding should serve as a warning sign that the patient is at high risk of developing more severe pressure ulcers, and the most
30 effective treatment at this time is to place pillows under his lower legs to relieve some of the pressure. Further treatments are not indicated at
31 the present time because there is no break in the skin. When a frank ulcer is present, dressings that maintain a moist environment are
32 appropriate.
33

34 (Choice B) Frequent repositioning of the patient may help to prevent the development or worsening of pressure ulcers; it should be encouraged,
35 not avoided.
36
(Choice C) Frequent massage may in fact cause further breakdown of this patient’s skin.
37

38
(Choice D) Corticosteroid cream is not recommended for the treatment of pressure ulcers because it may delay proper wound healing.
39

40
(Choice E) Hydrogen peroxide is not indicated for the treatment of pressure ulcers. If there is concern that the ulcer has become infected, then a
topical antibacterial agent such as silver sulfadiazine is recommended.

Educational objective:
Patients with limited mobility, such as those in the intensive care unit, are prone to the development of pressure ulcers. Relieving pressure on the
lower extremities, which is accomplished by placing pillows under the legs, is effective in preventing pressure ulcers. In cases where a frank ulcer
has developed, a dressing that maintains a moist wound environment should be utilized.

Surgery Rheumatology/Orthopedics & Sports Pressure induced injury


Subject System Topic

https://t.me/USMLEWorldStep2CK

REVIEW 0
Feedback End Block

https://www.uworld.com/ClientApp/v15/apps/qbanktestinterface/index.html#/launchtest/7281878/nbme/229400738/3/1 1/1
‫ م‬9:12 2021/‫‏‬9/‫‏‬11 https://t.me/USMLEWorldStep2CK UWorld STEP2 SIM Form 1

1 Item 8 of 40 ABC 0.25

2
Mark
Question Id: 6874 Full Screen Tutorial Lab Values Notes Calculator Reverse Color Text Zoom Settings
Previous Next
3

4 A 61-year-old smoker is admitted to the hospital for progressive weakness and decreased exercise tolerance. He is found to have a moderate
5 left-sided pleural effusion on chest x-ray. After initial evaluation, a thoracentesis is performed at the 7th intercostal space along the left midaxillary
6 line that drains two liters of yellowish fluid. Two hours after the procedure, the patient complains of dizziness and appears diaphoretic. His
7 systolic blood pressure is 70 mmHg and heart rate is 130/min, regular in rhythm. His oxygen saturation is 91% on 40% oxygen delivered via a
8 Venturi mask. On examination, there is dullness to percussion and absent breath sounds on the left side of his chest. Which of the following is
9 the most likely cause of this patient’s current symptoms?
10
A. Left ventricular outflow obstruction (8%)
11

12  B. Decreased left ventricular preload (64%)


13
C. Obstruction of pulmonary artery blood flow (12%)
14

15 D. Increased left ventricular end-diastolic pressure (10%)


16
 E. Decreased left ventricular afterload (3%)
17

18

19

20 Incorrect 64%
 02 secs 07/30/2021
21
Correct answer  Answered correctly Time Spent  Last Updated
B
22

23

24 Explanation
25

26
This patient’s hypotension is most likely a result from decreased left ventricular preload secondary to a hemothorax. The lack of breath sounds

27
and dullness to percussion on the left side of his chest is probably due to an accumulation of fluid in the pleural space. Since this patient just had
two liters of pleural fluid removed two hours previously, it would be unlikely for the effusion to reaccumulate that rapidly. As a result, this raises the
28
possibility of a left hemothorax occurring as a complication of the initial thoracentesis. The intercostal arteries course along the underside of the
29
ribs, and injury to one of these arteries can result in bleeding that accumulates in the pleural space. The hemothorax is impairing this patient’s
30
ability to breathe, resulting in hypoxia. His hemodynamic instability is likely secondary to intravascular volume depletion, which in turn leads to
31
decreased left ventricular preload. Initial treatment consists of a tube thoracostomy and intravenous fluid resuscitation. A thoracotomy may be
32
necessary if the hemothorax is very large or if the bleeding persists.
33

34
(Choice A) Left ventricular outflow obstruction can lead to heart failure, which could cause hypotension and pulmonary edema if severe.
35
However, left ventricular outflow obstruction should not develop rapidly after a thoracentesis.
36

37 (Choice C) Obstruction of blood flow in the pulmonary artery, such as that seen with a pulmonary embolism, could produce hypoxia and
38 hypotension. While a pulmonary embolism can result in a pleural effusion, it would not be expected to be as large or as rapidly accumulating as
39 that seen in this patient.
40
(Choice D) Increased left ventricular end-diastolic pressure from heart failure can cause hypoxia and hypotension, but it would not be expected to
occur this quickly after a thoracentesis.

(Choice E) A decrease in the left ventricular afterload would cause an increase in cardiac output.

Educational objective:
Hemothorax is a potential complication of a thoracentesis, and it should be suspected in patients who develop a rapid reaccumulation of pleural
effusion, difficulty breathing, and hemodynamic instability shortly after the thoracentesis is performed.

Surgery Pulmonary & Critical Care Pleural effusion


Subject System Topic

https://t.me/USMLEWorldStep2CK

REVIEW 0
Feedback End Block

https://www.uworld.com/ClientApp/v15/apps/qbanktestinterface/index.html#/launchtest/7281878/nbme/229400738/3/1 1/1
‫ م‬9:12 2021/‫‏‬9/‫‏‬11 https://t.me/USMLEWorldStep2CK UWorld STEP2 SIM Form 1

1 Item 9 of 40 ABC 0.25

2
Mark
Question Id: 6905 Full Screen Tutorial Lab Values Notes Calculator Reverse Color Text Zoom Settings
Previous Next
3

4 A 17-year-old boy comes to the emergency department with scrotal pain. He woke up from sleep 2 hours ago with severe, left-sided scrotal pain
5 that radiates to his left groin. The patient vomited once but has had no fever or diarrhea. There was no recent trauma. He is sexually active with
6 multiple partners and does not use condoms. The patient has no chronic medical conditions and takes no daily medications. Temperature is 37.8
7 C (100.0 F), blood pressure is 126/88 mm Hg, and pulse is 110/min. Physical examination shows left testicular swelling and tenderness. The
8 abdomen is soft and nontender with normal bowel sounds. There is no percussion tenderness at the costovertebral angle. Which of the following
9 additional examination findings would most likely warrant urgent surgical evaluation?
10
A. Copious mucopurulent discharge from the urethral meatus (0%)
11

12  B. Lack of testicular elevation with stroking of inner thigh (91%)


13
C. Palpable "bag of worms" scrotal mass more prominent upon standing (1%)
14

15 D. Positive transillumination of hemiscrotum (0%)


16
 E. Relief of pain with manual elevation of the testis (4%)
17

18
F. Transient inguinal bulge while performing Valsalva maneuver (1%)

19

20

21 Incorrect 91%
22 Correct answer  Answered correctly  02 secs
Time Spent  03/28/2021
Last Updated
B
23

24

25
Explanation
26

27

28 Acute scrotal pain


29 Cause Clinical features
30
Severe pain involving entire testis
31
Testicular Profound testicular swelling
32
torsion High-riding testis
33
Cremasteric reflex absent
34

35 Urinary frequency/urgency
36 Epididymitis Induration, swelling & tenderness of epididymis
37 Pyuria, bacteriuria
38 Severe pain from anterior abdomen into scrotum & penis
Fournier
39 Skin edema, crepitus, blisters/bullae
gangrene
40 Urgent surgical evaluation required

This patient has abrupt onset of scrotal pain associated with localized swelling and tenderness. In a young man, this is most likely due to
testicular torsion. The pain may awaken patients at night and is often associated with nausea and vomiting. Testicular torsion is caused by
twisting of the spermatic cord, resulting in compression of the pampiniform plexus of the testicular vein and reduced venous outflow. Testicular
torsion is a surgical emergency because irreversible damage to the testis can occur rapidly due to impaired blood flow.

Due to poor fixation of the testis to the tunica vaginalis during development (bell-clapper deformity), the oval testis may rest in a horizontal plane
within the scrotum as opposed to the normal vertical plane; this predisposes the testicle to twist. The diagnosis of testicular torsion is often made
based on clinical findings. Shortening of the spermatic cord due to twisting can cause the testis to ride high in the scrotum. In addition, the
cremasteric reflex (upward retraction of the testis on pinching or stroking the inner thigh) will be absent in almost all patients. If uncertain, the
diagnosis can be confirmed with Doppler ultrasound showing lack of blood flow in the affected testicle.

(Choices A and E) Mucopurulent discharge from the urethral meatus in association with testicular pain suggests epididymitis. Other common
symptoms include frequency, dysuria, and fever. Pain relief on elevation of the testis (Prehn sign) is also suggestive of epididymitis rather than
torsion, although this technique has low diagnostic utility. Management of epididymitis includes antibiotics, analgesics, and elevation.

(Choice C) Varicocele is characterized by distension of the pampiniform plexus within the spermatic cord. Examination findings are often
described as a "bag of worms" in the scrotum. Varicocele is often asymptomatic but may cause a dull pain that is worsened with standing. It is a
chronic disorder and does not require urgent evaluation.

(Choice D) Transillumination suggests a testicular hydrocele, a collection of fluid between the parietal and visceral tunica vaginalis. It can cause
testicular swelling but usually does not cause acute, severe pain. Hydroceles typically do not require intervention.

(Choice F) An incarcerated inguinal hernia can present with acute groin pain and vomiting. A palpable bulge can be seen, but a transient bulge
that is only present on Valsalva suggests an uncomplicated hernia. Uncomplicated hernias usually do not cause acute pain and can be managed
nonurgently.

Educational objective:
Testicular torsion is a surgical emergency due to possible irreversible damage to the testis. Clinical findings include acute scrotal pain and
tenderness, a high-riding testis, and loss of the cremasteric reflex.

References
Testicular torsion: diagnosis, evaluation, and management.

(http://www.ncbi.nlm.nih.gov/pubmed/24364548)

Surgery Male Reproductive System Testicular torsion


Subject System Topic

https://t.me/USMLEWorldStep2CK

REVIEW 0
Feedback End Block

https://www.uworld.com/ClientApp/v15/apps/qbanktestinterface/index.html#/launchtest/7281878/nbme/229400738/3/1 1/1
‫ م‬9:12 2021/‫‏‬9/‫‏‬11 https://t.me/USMLEWorldStep2CK UWorld STEP2 SIM Form 1

1 Item 10 of 40 ABC 0.25

2
Mark
Question Id: 6908 Full Screen Tutorial Lab Values Notes Calculator Reverse Color Text Zoom Settings
Previous Next
3

4 A 63-year-old man comes to the office for cancer follow-up. The patient was diagnosed with laryngeal cancer 6 months ago and was treated with
5 radiation therapy and platinum-based chemotherapy. He has been feeling better and his strength is increasing. He is eating well without
6 aspiration and his voice has not changed. He reports that his hearing sounds muffled and a high-pitched noise is present in both ears. He has no
7 earache and no dizziness. Otoscopic examination is normal. Which of the following is the most likely mechanism of this patient's hearing loss?
8

9
A. Decreased ossicular mobility (1%)

10 B. Electrolyte abnormalities (0%)


11
C. Eustachian tube dysfunction (0%)
12

13  D. Medication side effect (95%)


14
E. Metastatic spread of cancer (0%)
15

16 F. Middle ear infection (0%)


17 G. Radiation-induced vasculopathy (1%)

18

19

20
Incorrect 95% 02 secs 07/31/2021
21

22
Correct answer
D
 Answered correctly  Time Spent  Last Updated

23

24

25
Explanation

26
This patient with hearing loss and tinnitus likely has ototoxicity due to platinum-based chemotherapy. Platinum-based chemotherapeutic
27
agents (eg, cisplatin, carboplatin) are used to treat a wide variety of both adult and pediatric malignancies. They are commonly used in
28
combination with radiation therapy to treat head and neck squamous cell carcinoma.
29

30 Hearing loss and other neurotoxic effects (eg, peripheral neuropathy, CNS toxicity) are more commonly due to cisplatin than to carboplatin.
31 Ototoxicity typically manifests as irreversible, bilateral, high-frequency sensorineural hearing loss often accompanied by tinnitus and/or
32 imbalance. The exact mechanism of injury is unclear, but it results in neurotoxicity and death of the outer hair cells in the organ of Corti.
33 Ototoxicity is dose-dependent; therefore, monitoring protocols are often put in place (especially for children) to ensure early detection and allow
34 for alteration of the chemotherapeutic regimen if needed. Research on otoprotective strategies is ongoing.
35
(Choice A) Otosclerosis causes decreased ossicular mobility and can present as hearing loss with a normal otoscopic examination. However,
36
hearing loss due to otosclerosis is often asymmetric. It is also hereditary and occurs most often in young women.
37

38 (Choice B) Cisplatin can cause electrolyte abnormalities as a result of its toxic effect on the kidneys. Hearing loss due to cisplatin, however,

39
results from neurotoxicity and damage to the cochlear hair cells.

40 (Choice C) Eustachian tube dysfunction can cause hearing loss. However, hearing loss is typically unilateral and accompanied by pain, popping
sounds, and a middle ear effusion.

(Choice E) Tinnitus and hearing loss can be caused by a primary cerebellopontine angle tumor (eg, vestibular schwannoma) but would be very
unlikely to be caused by metastases, which are usually found in the cerebral cortex. Furthermore, head and neck squamous cell carcinoma rarely
metastasizes to the brain; distant metastases are typically seen in the lungs, bone, liver, or skin.

(Choice F) Chemotherapy can increase susceptibility to many infections. However, a middle ear infection typically presents with ear pain, fever,
and bulging of the tympanic membrane, in addition to hearing loss.

(Choice G) Osteoradionecrosis is a complication of radiation therapy caused by damage to the vasculature. It most commonly affects the
mandible and results in exposure of the bone.

Educational objective:
Ototoxicity is a common side effect of platinum-based chemotherapeutic agents and presents with irreversible, bilateral, high-frequency
sensorineural hearing loss often associated with tinnitus and/or imbalance.

References
Incidence and sites of distant metastases from head and neck cancer.

(http://www.ncbi.nlm.nih.gov/pubmed/11408812)
Mechanisms of cisplatin-induced ototoxicity and prevention.

(http://www.ncbi.nlm.nih.gov/pubmed/31036996)

Medicine Nervous System Head and neck cancers


Subject System Topic

https://t.me/USMLEWorldStep2CK

REVIEW 0
Feedback End Block

https://www.uworld.com/ClientApp/v15/apps/qbanktestinterface/index.html#/launchtest/7281878/nbme/229400738/3/1 1/1
‫ م‬9:13 2021/‫‏‬9/‫‏‬11 https://t.me/USMLEWorldStep2CK UWorld STEP2 SIM Form 1

1 Item 11 of 40
2
Mark
Question Id: 6909 Previous Next
3

4 A 42-year-old African American man comes to the clinic with a 6-month history of progressive weakness, fatigue, and lower extremity swelling. He
5 has not been ill recently. Past medical history is unremarkable, but he has a family history of hypertension, obesity, and heart disease. The
6 patient does not use tobacco, alcohol, or illicit drugs. He does not take any medications and has no allergies. Blood pressure is 165/100 mm Hg,
7 heart rate is 100/min, and respirations are 20/min. The patient's body mass index is 29 kg/m2. Physical examination shows facial puffiness and
8 pitting edema of both lower extremities. A normal S1 and S2 are heard on cardiac examination, and breath sounds are clear. His abdomen is soft
9 and nontender with no palpable masses. Laboratory results are as follows:
10
Sodium 141 mEq/L
11
12 Potassium 4.4 mEq/L
13 Chloride 102 mEq/L
14
Bicarbonate 26 mEq/L
15
16 Blood urea nitrogen 21 mg/dL
17
Creatinine 1 mg/dL
18

19
Glucose 95 mg/dL

20 Albumin 2.2 g/dL


21

22 Anti-nuclear antibody Negative


23
HBsAg Negative
24
25 Hepatitis C virus antibody Negative
26 HIV-1 antibody Negative
27
HIV-2 antibody Negative
28

29
Urinalysis shows 4+ protein. The 24-hour urine protein excretion is 6.5 g. Which of the following is the most likely cause of this patient's
30
condition?
31

32
 A. Focal segmental glomerulosclerosis (73%)
33
34
B. Hypertensive emergency (3%)

35 C. IgA nephropathy (3%)


36
D. Membranoproliferative glomerulonephritis (14%)
37

38 E. Multiple myeloma (1%)


39 F. Rapidly progressive glomerulonephritis (3%)
40

73% 06 secs 08/19/2021


Correct  Answered correctly  Time Spent  Last Updated

Explanation

This patient's edema, proteinuria (>3.5 g/day), and hypoalbuminemia are classic features of nephrotic syndrome (NS). The absence of
hematuria excludes a diagnosis of nephritic syndrome. In adults, the most common cause of NS is focal segmental glomerulosclerosis
(FSGS). Obese individuals, those of African descent, and patients with HIV infection are at increased risk for developing FSGS. Adult patients
with NS usually require a renal biopsy to confirm the diagnosis. The pathognomonic finding is partial sclerosis limited to a segment of the
glomerulus.

All patients with NS are at increased risk for infection (loss of immunoglobulins), thrombosis (loss of antithrombin and plasminogen), and protein
malnutrition. Although this patient's blood urea nitrogen and creatinine levels are normal, the diagnosis of FSGS cannot be ruled out due to renal
insufficiency being a delayed complication of untreated NS.

(Choice B) Hypertensive emergency should be suspected in patients with an abrupt rise in blood pressure and signs of end-organ damage (eg,
retinal hemorrhages, papilledema, nephrosclerosis, encephalopathy). It is likely that this patient's hypertension is secondary to NS as his
symptoms have progressed over 6 months and he does not have additional signs of end-organ injury.

(Choice C) IgA nephropathy can present with nephrotic or nephritic symptoms. The most common clinical feature, however, is gross hematuria
that follows an upper respiratory tract infection.

(Choice D) Membranoproliferative glomerulonephritis is an uncommon cause of NS and nephritic syndrome and is usually seen in younger
patients. It is associated with hepatitis B or C.

(Choice E) Multiple myeloma can cause primary amyloidosis, which is associated with NS. However, FSGS is much more common than multiple
myeloma, particularly in a patient this young.

(Choice F) Rapidly progressive glomerulonephritis (RPGN) presents with nephritic syndrome and rapidly worsening renal function.

Educational objective:
Findings for nephrotic syndrome (NS) include a urine protein excretion of >3.5 g/day, edema, and hypoalbuminemia. Focal segmental
glomerulosclerosis is the most common cause of NS in adults, particularly African Americans.

References
Spectrum of nephrotic syndrome in adults: clinicopathological study from a single center in India

(http://www.ncbi.nlm.nih.gov/pubmed/23438313)

Medicine Renal, Urinary Systems & Electrolytes FSGS


Subject System Topic

https://t.me/USMLEWorldStep2CK

REVIEW

https://www.uworld.com/ClientApp/v15/apps/qbanktestinterface/index.html#/launchtest/7281878/nbme/229400738/3/1 1/1
‫ م‬9:13 2021/‫‏‬9/‫‏‬11 https://t.me/USMLEWorldStep2CK UWorld STEP2 SIM Form 1

1 Item 12 of 40
2
Mark
Question Id: 6912 Previous Next
3

4 A 52-year-old man is brought to the emergency department after he was hit by a car while crossing the street. Initial trauma workup does not
5 reveal any fractures but shows bruising to the extremities, right chest, and upper abdomen. Eight hours after the accident, the patient develops
6 right-sided chest discomfort and shortness of breath. Blood pressure is 130/80 mm Hg, pulse is 102/min, and respirations are 24/min. Pulse
7 oximetry is 90% on room air. Physical examination shows bronchial breath sounds at the right base and normal S1 and S2 heart sounds. The
8 abdomen is soft and nontender. The patient's chest x-ray is shown in the image below:
9

10
11
12
13

14

15
16

17
18

19
20

21

22

23

24
25

26

27

28

29
30
31

32
Which of the following is the most likely diagnosis?
33
34
A. Aortic rupture (8%)
35

36  B. Aspiration pneumonitis (2%)


37 C. Diaphragmatic rupture (0%)
38
D. Hemothorax (3%)
39

40 E. Hospital-acquired pneumonia (0%)

 F. Pulmonary contusion (77%)

G. Tracheobronchial rupture (6%)

Incorrect 77%
Correct answer  Answered correctly  05 secs
Time Spent  08/01/2021
Last Updated
F

Explanation

This patient who sustained blunt thoracic trauma (eg, right chest wall bruising) several hours ago now has dyspnea, tachypnea, hypoxia, and
abnormal right lower lung findings (eg, bronchial breath sounds, irregular consolidation) most consistent with pulmonary contusion. Pulmonary
contusion occurs when kinetic energy is transmitted through the chest wall during trauma, bruising underlying lung parenchyma; alveolar
hemorrhage and edema result and impair oxygen diffusion. Although rib fractures are often present, pulmonary contusion can occur in their
absence.

Because significant alveolar edema may take up to 24 hours to accumulate, clinical symptoms (eg, dyspnea, hypoxia, chest discomfort) are often
delayed, and initial chest x-ray is often negative. CT scan or repeat chest x-ray may be necessary to reveal the irregular alveolar infiltrates
characteristic of pulmonary contusion. Because these infiltrates reflect alveolar damage in the lung tissue adjacent to the injury site (eg, right
chest in this patient), they are typically nonlobular (not restricted by anatomic landmarks). Management includes pain control, pulmonary hygiene
(eg, incentive spirometry), and supplemental oxygen until the contusion resolves.

(Choice A) Aortic rupture usually results in instantaneous death. Patients with contained ruptures may rarely survive but typically have severe
chest pain and findings of widened mediastinum and/or abnormal aortic contour on chest x-ray.

(Choice B) Aspiration pneumonitis can cause dyspnea with hypoxia; however, onset is typically rapid (eg, within 2 hours) following aspiration, and
findings are more severe (eg, cyanosis, diffuse infiltrates).

(Choice C) Diaphragmatic rupture can alter respiratory mechanics and cause respiratory difficulty; however, it typically causes loss of
diaphragmatic contour and/or visualization of abdominal organs within the thoracic cavity.

(Choice D) Hemothorax can cause dyspnea and a right lower lung opacity. However, hemothorax is commonly associated with chest wall (eg,
rib) fracture and typically presents more acutely. In addition, chest x-ray would show blunting of the costophrenic angle.

(Choice E) Hospital-acquired pneumonia can cause dyspnea with hypoxia and pulmonary infiltrates; however, it typically has accompanying
fever, cough, and purulent sputum and occurs ≥48 hours after admission.

(Choice G) Although tracheobronchial rupture can cause progressive dyspnea and hypoxemia, chest x-ray typically demonstrates evidence of air
leakage from the tracheobronchial tree (eg, pneumothorax, pneumomediastinum).

Educational objective:
Even without chest wall fracture, blunt thoracic trauma can cause pulmonary contusion. Common clinical features include delayed (ie, up to 24
hours postinjury) presentation of tachypnea and hypoxia and irregular (nonlobular) infiltrates on chest x-ray.

References
Lung contusion: a clinico-pathological entity with unpredictable clinical course.

(http://www.ncbi.nlm.nih.gov/pubmed/27162815)
Pulmonary contusion.

(http://www.ncbi.nlm.nih.gov/pubmed/30906578)

Surgery Pulmonary & Critical Care Pulmonary contusion


Subject System Topic

https://t.me/USMLEWorldStep2CK

REVIEW

https://www.uworld.com/ClientApp/v15/apps/qbanktestinterface/index.html#/launchtest/7281878/nbme/229400738/3/1 1/1
‫ م‬9:13 2021/‫‏‬9/‫‏‬11 https://t.me/USMLEWorldStep2CK UWorld STEP2 SIM Form 1

1 Item 13 of 40
2
Mark
Question Id: 6914 Previous Next
3

4 A 15-year-old boy is brought to the emergency department due to an episode of confusion. Early this morning, his parents found him lying on his
5 bed confused and disoriented. By the time they arrived at the hospital 30 minutes later, his symptoms had resolved. The patient cannot recall
6 what happened but says he has had frequent episodes of early morning limb jerking recently. He has also been staying up late preparing for
7 exams. The patient has had no headaches or vision changes. He has no medical conditions and takes no medications. Temperature is 37.1 C
8 (98.8 F). The patient is alert and fully oriented. Cardiac examination is unremarkable. Pupils are equal and reactive to light and accommodation,
9 and the remainder of the neurologic examination is unremarkable. The patient's clothes are wet and smell of urine. Which of the following is the
10 most likely diagnosis?
11
 A. Juvenile myoclonic epilepsy (82%)
12
13 B. Lennox-Gastaut syndrome (4%)
14
C. Psychogenic nonepileptic seizures (10%)
15
16 D. REM sleep behavior disorder (1%)
17 E. Sydenham chorea (0%)

18

19
20

21
Incorrect 82% 02 secs 08/07/2021
22
Correct answer
A
 Answered correctly  Time Spent  Last Updated

23

24
25 Explanation
26

27
Juvenile myoclonic epilepsy
28

29
Adolescents

30
Absence seizures
Clinical features
Morning myoclonus
31
Generalized tonic-clonic seizures
32

33 Diagnosis Electroencephalogram: bilateral polyspike & slow wave activity


34
Valproic acid
35 Management
Avoid triggers (eg, alcohol, sleep deprivation)
36

37 This patient's presentation is consistent with juvenile myoclonic epilepsy (JME), a genetic epilepsy syndrome that affects otherwise healthy
38 adolescents. JME can occur sporadically but is more common in individuals with a family history of seizures.
39 Almost all patients with JME have myoclonic (limb jerking) and generalized tonic-clonic seizures (GTCS), both of which typically occur in the
40 morning and may be triggered by sleep deprivation (eg, staying up late studying for examinations). A GTCS is often the event leading to
diagnosis, as with this patient whose episode of confusion (ie, postictal state) and urinary incontinence were most likely due to an unwitnessed
GTCS. About one-third of patients also have a history of absence seizures in childhood.

Bilateral polyspike and slow wave activity on interictal electroencephalogram supports the diagnosis, and valproic acid is first-line treatment to
control all JME seizure types. Lifelong therapy may be required to remain seizure-free.

(Choice B) Lennox-Gastaut syndrome typically presents in children age <5 with various, severe seizure types accompanied by intellectual
disability. This patient's age makes the diagnosis unlikely.

(Choice C) Psychogenic nonepileptic seizures mimic seizures but are not associated with abnormal neuronal activity. They are typically
witnessed events that occur during wakefulness and are followed by immediate return to baseline neurologic status. Urinary incontinence is
uncommon.

(Choice D) Patients with REM sleep behavior disorder act out dreams with purposeful physical movements (eg, punching) during REM sleep.
Brief disorientation may occur if a patient awakens during the enactment, but prolonged confusion and urinary incontinence are not associated
with this disorder.

(Choice E) Sydenham chorea, a manifestation of acute rheumatic fever, is characterized by rapid, abnormal movements of the face and
extremities after an untreated group A streptococcal infection. Movements are not limited to the morning and would not result in an episode of
confusion or urinary incontinence.

Educational objective:
Juvenile myoclonic epilepsy presents with both myoclonic and generalized tonic-clonic seizures in otherwise healthy adolescents. The seizures
most commonly occur in the morning and may be triggered by sleep deprivation. Patients may also have a history of childhood absence seizures.

References
Juvenile myoclonic epilepsy: challenges on its 60th anniversary.

(http://www.ncbi.nlm.nih.gov/pubmed/27665373)

Pediatrics Nervous System Seizures


Subject System Topic

https://t.me/USMLEWorldStep2CK

REVIEW

https://www.uworld.com/ClientApp/v15/apps/qbanktestinterface/index.html#/launchtest/7281878/nbme/229400738/3/1 1/1
‫ م‬9:13 2021/‫‏‬9/‫‏‬11 https://t.me/USMLEWorldStep2CK UWorld STEP2 SIM Form 1

1 Item 14 of 40
2
Mark
Question Id: 6917 Previous Next
3

4 A 39-year-old woman comes to the office due to a 4-month history of progressive muscle weakness. The patient has had difficulty combing her
5 hair because her arms feel weak when she holds them above her head. In addition, she has experienced a 9-kg (20-lb) weight gain as well as
6 irregular menses with vaginal dryness and low libido. Medical history is significant for hypertension, hyperlipidemia, and seasonal allergies.
7 Current medications include hydrochlorothiazide and simvastatin. She smokes a pack of cigarettes daily, drinks 3 or 4 beers every weekend, and
8 does not use illicit drugs. Blood pressure is 150/100 mm Hg and pulse is 90/min. BMI is 37 kg/m2. Physical examination shows oily facial skin
9 with prominent acne and scattered bruises on her arms and lower legs. The lungs are clear to auscultation and the abdomen is obese, soft, and
10 nontender. Deep tendon reflexes are normal and symmetric. Laboratory results are as follows:
11
Serum chemistry
12
13 Sodium 142 mEq/L
14 Potassium 3.4 mEq/L
15
Chloride 104 mEq/L
16

17 Bicarbonate 29 mEq/L
18
Blood urea nitrogen 8 mg/dL
19
Creatinine 0.9 mg/dL
20

21 Calcium 8.8 mg/dL


22
Glucose 166 mg/dL
23

24 Creatine kinase, serum 68 U/L


25

26 Which of the following is the most likely cause of this patient's symptoms?
27
 A. Cushing syndrome (71%)
28

29 B. Hypothyroidism (18%)
30
C. Lambert-Eaton myasthenic syndrome (2%)
31

32  D. Polycystic ovary syndrome (4%)


33 E. Primary hyperaldosteronism (1%)
34
F. Statin myopathy (1%)
35

36

37

38 Incorrect 71%
39
Correct answer  Answered correctly  02 secs
Time Spent  07/16/2021
Last Updated
A
40

Explanation

Features of Cushing syndrome

Central obesity
Skin atrophy & wide, purplish striae
Proximal muscle weakness
Clinical
Hypertension
manifestations
Glucose intolerance
Skin hyperpigmentation (if due to ACTH excess)
Depression, anxiety

24-hour urinary cortisol excretion


Diagnosis Late-night salivary cortisol assay
Low-dose dexamethasone suppression test

This patient has muscular weakness, weight gain, uncontrolled hypertension, and hyperglycemia, which are consistent with hypercortisolism
(Cushing syndrome). This disorder is often suspected due to the characteristic body habitus with enlargement of the dorsal and supraclavicular
fat pads, central obesity, and muscle wasting. In Cushing syndrome, proximal muscle weakness occurs due to the catabolic effects of
glucocorticoids on muscles. Catabolic effects on subcutaneous tissues can lead to skin fragility, stria, and easy bruising. Hyperglycemia is
common due to cortisol-induced gluconeogenesis, insulin resistance, and suppression of insulin release.

Hypercortisolism can be characterized as ACTH-dependent and ACTH-independent:

High levels of ACTH (eg, ACTH-secreting pituitary adenoma/Cushing disease, ectopic ACTH production) can cause hyperpigmentation due
to co-secretion of melanocyte-stimulating hormone (MSH) and direct stimulation of MSH receptors by ACTH. ACTH also increases
androgen production from the zona reticularis of the adrenal cortex, leading to androgenic symptoms (irregular menses, acne), as seen in
this patient.
ACTH-independent hypercortisolism (eg, exogenous glucocorticoid administration, adrenal adenoma) is not associated with
hyperpigmentation or androgen excess.

(Choices B and F) Hypothyroidism can cause a proximal myopathy associated with weight gain, hypertension, and menstrual irregularities. Most
patients have additional features of hypothyroidism (eg, cold intolerance, abnormal reflexes). Both hypothyroid myopathy and statin myopathy are
typically painful and cause significant elevations in creatine kinase.

(Choice C) Lambert-Eaton myasthenic syndrome is a paraneoplastic disorder associated with small cell lung cancer characterized by slowly
progressive weakness and loss of deep tendon reflexes. It would not cause weight gain, menstrual symptoms, or hyperglycemia.

(Choice D) Polycystic ovary syndrome presents with obesity, menstrual irregularities, and signs of androgen excess. Hyperglycemia is common.
However, myopathy and bruising are not typical features.

(Choice E) Primary hyperaldosteronism presents with hypertension and hypokalemic alkalosis (which can result in weakness), but it would not
cause features of glucocorticoid excess (eg, bruising, body habitus changes). Hypokalemia can occur in Cushing syndrome due to stimulation of
mineralocorticoid receptors by high levels of cortisol.

Educational objective:
Cushing syndrome presents with central obesity, proximal muscle weakness, hypertension, and easy bruising. Besides exogenous glucocorticoid
administration, the most common etiologies include increased pituitary secretion of ACTH (Cushing disease), an adrenal tumor, and ectopic ACTH
production.

References
Cushing's syndrome: update on signs, symptoms and biochemical screening.

(http://www.ncbi.nlm.nih.gov/pubmed/26156970)

Medicine Endocrine, Diabetes & Metabolism Cushing syndrome


Subject System Topic

https://t.me/USMLEWorldStep2CK

REVIEW

https://www.uworld.com/ClientApp/v15/apps/qbanktestinterface/index.html#/launchtest/7281878/nbme/229400738/3/1 1/1
‫ م‬9:13 2021/‫‏‬9/‫‏‬11 https://t.me/USMLEWorldStep2CK UWorld STEP2 SIM Form 1

1 Item 15 of 40
2
Mark
Question Id: 6924 Previous Next
3

4 A 65-year-old male presents to the ER with chest pain. He complains of a retrosternal, burning pain that started two hours ago and is radiating to
5 his left arm. The pain waxes and wanes in intensity, and is accompanied by nausea, sweating, and mild shortness of breath. His past medical
6 history includes hypertension, hypercholesterolemia, diabetes mellitus type 2, and osteoarthritis. He smokes one pack of cigarettes per day and
7 consumes alcohol occasionally. He denies any illegal drug use. His blood pressure is 120/56 mmHg and heart rate is 92/min. Physical exam
8 reveals bibasilar scattered crackles. An ECG done 10 minutes following his arrival to the ER shows ST segment depression in leads V2-V6.
9 Which of the following is the quickest way to decrease this patient’s pain?
10
A. Decrease in systolic blood pressure (1%)
11
12 B. Increase in diastolic blood pressure (2%)
13
 C. Venous dilation (63%)
14

15 D. Coronary artery dilation (28%)


16
 E. Platelet aggregation inhibition (4%)
17
18

19
20
Incorrect 63%
21
Correct answer
C
 Answered correctly  02 secs
Time Spent  07/30/2021
Last Updated

22

23

24 Explanation
25

26
The patient in this vignette is suffering from classic symptoms of myocardial ischemia, including retrosternal chest pain radiating to his left arm
accompanied by nausea, diaphoresis, and shortness of breath. He also has numerous risk factors for atherosclerotic coronary artery disease.
27
The presence of ST segment depression in contiguous anterior leads on his ECG further supports the suspicion for ongoing coronary ischemia. In
28
patients with acute myocardial ischemia, the appropriate initial treatment includes morphine, oxygen, nitrates, aspirin, beta-adrenergic antagonists,
29
low molecular weight heparin, and possibly statins.
30
31
While all of these medications as well as the possibility of reperfusion therapies are important considerations in the acute setting of myocardial
32
ischemia, the most important factor in reducing chest pain is to induce dilatation of the venous system. Venodilation decreases preload, thereby
33
reducing ventricular volume as well as ventricular wall stress. As a result, myocardial oxygen demand is decreased. Organic nitrates
34
preferentially cause venodilation and are therefore very effective in treating the pain caused by myocardial ischemia.
35

36 (Choice A) A decrease in the systolic blood pressure may result in decreased afterload, thereby reducing myocardial oxygen demand, and hence
37 chest pain. Venodilation with nitrates, however, results in a much quicker resolution of chest pain.
38

39 (Choice B) An increase in diastolic blood pressure (afterload) would be counterproductive in the treatment of myocardial ischemia because the
40 myocardium would need to contract more forcefully to maintain stroke volume, thereby resulting in increased myocardial oxygen demand.

(Choice D) Organic nitrates may cause some degree of coronary artery dilatation, but venodilation is the primary mechanism by which nitrates
reduce pain in acute myocardial ischemia.

(Choice E) The inhibition of platelet aggregation is accomplished by the administration of aspirin and glycoprotein IIb/IIIa inhibitors in patients with
acute myocardial infarction. This, however, does not acutely reduce the pain caused by myocardial ischemia.

Educational objective:
Venodilation decreases the pain associated with acute myocardial ischemia by reducing preload, which leads to a reduction in myocardial oxygen
demand. This effect is most commonly caused by the use of organic nitrates in the setting of acute myocardial ischemia.

Medicine Cardiovascular System Myocardial infarction


Subject System Topic

https://t.me/USMLEWorldStep2CK

REVIEW

https://www.uworld.com/ClientApp/v15/apps/qbanktestinterface/index.html#/launchtest/7281878/nbme/229400738/3/1 1/1
‫ م‬9:13 2021/‫‏‬9/‫‏‬11 https://t.me/USMLEWorldStep2CK UWorld STEP2 SIM Form 1

1 Item 16 of 40
2
Mark
Question Id: 6944 Previous Next
3

4 A 3-week-old boy is brought to the emergency department due to sudden-onset, recurrent, forceful bilious vomiting. He was feeding well until 8
5 hours ago when he developed vomiting, and since then he has refused to feed. The patient was born at term following an uncomplicated
6 pregnancy and vaginal delivery. He was discharged home in stable condition 48 hours after birth. Temperature is 37.2 C (99 F), blood pressure is
7 84/46 mm Hg, and pulse is 162/min and regular. Cardiopulmonary examination is unremarkable. The abdomen is distended. A 2 × 3 cm,
8 nontender, umbilical hernia is present. Upper gastrointestinal series is shown in the image below:
9

10
11
12
13

14

15
16

17
18

19
20

21

22

23

24
25

26

27

28

29
30
31

32

33
34

35

36

37

38

39
Which of the following is the most likely underlying cause of this patient's clinical presentation?

40
 A. Arrest of normal embryonic gut rotation (43%)

B. Failed recanalization of the duodenal lumen (22%)

C. Hypertrophic obstruction of the gastric outlet (15%)

D. Incarcerated umbilical hernia (7%)

E. Wide-based mesentery affixed to small bowel (11%)

43%
Correct  Answered correctly  04 secs
Time Spent  05/31/2021
Last Updated

Explanation

Gastrointestinal obstruction in the neonate


Key features Imaging

Delayed passage of meconium, abdominal


Contrast enema: transition zone between distal
distension
Hirschsprung disease narrow aganglionic segment & proximal dilated
Forceful expulsion of gas/stool on rectal
bowel
examination

Delayed passage of meconium, abdominal


Meconium ileus distension Contrast enema: microcolon
Associated with cystic fibrosis

Bilious emesis & abdominal distension within days


Intestinal atresia X-ray: double bubble sign (duodenal atresia)
of birth

Malrotation with Bilious emesis & abdominal distension (any age, Upper GI: right-sided ligament of Treitz,
volvulus usually age <4 weeks) corkscrew-shaped duodenum

Age 3-5 weeks


Pyloric stenosis Ultrasound: hypertrophy of pyloric muscle
Nonbilious projectile vomiting after feeds

GI = gastrointestinal.

During normal embryonic gut development, the intestinal loop rotates 270 degrees counterclockwise before fixating to the posterior abdominal
wall. This attachment results in a wide mesenteric base with the ligament of Treitz in the left upper quadrant and the ileocecal valve in the right
lower.

However, when there is arrest of normal embryonic gut rotation, malrotation (eg, incomplete or reverse rotation) occurs. The most common
malrotation results in the cecum malpositioned in the mid-to-upper abdomen (and held in place by fibrous adhesions, or Ladd bands) and the
ligament of Treitz abnormally located in the right abdomen. Malrotation also results in an unusually narrow mesenteric base, leaving the small
bowel beyond the proximal duodenum unaffixed to the posterior abdominal wall and abnormally mobile (Choice E).

Peristalsis of the unaffixed small intestine can lead to midgut volvulus, or duodenal twisting around the superior mesenteric artery, which is seen
as a corkscrew-shaped duodenum on this patient's upper gastrointestinal (GI) series. Neonates classically have signs of GI obstruction (eg,
bilious emesis, abdominal distension), as seen here. Bowel necrosis and perforation are potentially life-threatening complications, making
malrotation with midgut volvulus a surgical emergency.

(Choice B) Duodenal atresia is due to failed recanalization of the duodenal lumen and causes bilious emesis. However, presentation is typically
within the first 2 days of life, and GI series reveals a dilated stomach and proximal duodenum (double bubble sign) due to duodenal obstruction,
not a corkscrew appearance of the intestines.

(Choice C) Pyloric stenosis, or hypertrophic obstruction of the gastric outlet, classically presents at age 3-5 weeks. Because the obstruction is
proximal to the entrance of the common bile duct into the duodenum, patients have nonbilious vomiting and a narrow, elongated pylorus on upper
GI series.

(Choice D) Although umbilical hernias are common in infants due to a fascial opening around the umbilicus, incarceration (inability to reduce
abdominal contents within the hernia) is rare. Vomiting and feeding refusal may occur with bowel strangulation, but in contrast to this case, the
hernia would be erythematous and tender.

Educational objective:
Intestinal malrotation with midgut volvulus is due to arrest of normal embryonic gut rotation with subsequent twisting of the small bowel on a
narrow-based mesentery. Neonates have bilious emesis due to intestinal obstruction, and upper gastrointestinal series reveals a right-sided
ligament of Treitz and a corkscrew-shaped duodenum.

References
Intestinal malrotation—volvulus: imaging findings.

(http://www.ncbi.nlm.nih.gov/pubmed/25458122)
Malrotation and intestinal atresias.

(http://www.ncbi.nlm.nih.gov/pubmed/25448782)

Pediatrics Gastrointestinal & Nutrition Volvulus


Subject System Topic

https://t.me/USMLEWorldStep2CK

REVIEW

https://www.uworld.com/ClientApp/v15/apps/qbanktestinterface/index.html#/launchtest/7281878/nbme/229400738/3/1 1/1
‫ م‬9:13 2021/‫‏‬9/‫‏‬11 https://t.me/USMLEWorldStep2CK UWorld STEP2 SIM Form 1

1 Item 17 of 40
2
Mark
Question Id: 6949 Previous Next
3

4 A 64-year-old man is brought to the emergency department due to a day of fever, headache, nausea, and lethargy. Four weeks ago, the patient
5 had a temporal artery biopsy to evaluate headaches and blurred vision; he was diagnosed with giant cell arteritis and initiated on glucocorticoid
6 treatment. He also has a history of hypertension and osteoarthritis. Temperature is 38.3 C (101 F), blood pressure is 140/90 mm Hg, and pulse is
7 100/min. The patient is somnolent. Physical examination shows neck rigidity with passive flexion. Lumbar puncture is performed and he is
8 started on empiric vancomycin and a third-generation cephalosporin. This patient should receive additional antimicrobial coverage against which
9 of the following organisms?
10
A. Cryptococcus neoformans (8%)
11
12 B. Cytomegalovirus (3%)
13
 C. Listeria monocytogenes (75%)
14

15 D. Mycobacterium tuberculosis (1%)


16
E. Nocardia species (1%)
17
18  F. Pseudomonas aeruginosa (8%)

19 G. Toxoplasma gondii (0%)


20

21

22
Incorrect 75% 02 secs 09/04/2021
23

24
Correct answer
C
 Answered correctly  Time Spent  Last Updated

25

26
Explanation
27

28
This 64-year-old man on chronic glucocorticoids has acute fever, headache, neck rigidity, and somnolence suggesting community-acquired
29
bacterial meningitis. Bacterial meningitis is most commonly caused by Streptococcus pneumoniae (~70%), Neisseria meningitidis (~12%),
30
Group B Streptococcus, and Haemophilus influenzae. Third-generation cephalosporins (eg, ceftriaxone) have excellent bacteriocidal efficacy
31
against these organisms; vancomycin is added to cover the small amount of S pneumoniae resistant to beta-lactams. Patients who are
32
immunocompromised (eg, chronic glucocorticoids) or age >50 are also at risk for Listeria monocytogenes. Additional coverage with
33
ampicillin is required.
34

35
(Choice A) Cryptococcal meningitis is seen primarily in severely immunocompromised patients (eg, HIV with CD4 count <100/mm3). Symptoms
arise over weeks (not a single day) and include fever, malaise, and headache.
36

37 (Choice B) Cytomegalovirus (CMV) may rarely cause meningoencephalitis, primarily in severely immunocompromised patients (eg, HIV with CD4
38 count <50/mm3, post-transplant). Most patients have concurrent symptoms of CMV retinitis, gastroenteritis, pneumonitis, or other neurologic
39 manifestations (eg, cranial nerve palsies). This patient with a day of nuchal rigidity, fever, and altered mental status is more likely to have a
40 bacterial cause.

(Choice D) Tuberculosis meningitis usually manifests with subacute (not acute) symptoms of malaise, headache, low-grade fever, and mental
status change.

(Choice E) Nocardia may cause a brain abscess with symptoms of fever, headache, seizures, and/or focal neurologic changes. Meningitis is far
less common and tends to present with subacute or chronic (not acute) symptoms.

(Choice F) Pseudomonas aeruginosa is not a common cause of community-acquired meningitis but may cause central nervous system infections
in the nosocomial setting (eg, after neurosurgical procedures).

(Choice G) Toxoplasma gondii encephalitis is seen primarily in patients with advanced HIV (CD4 count <100/mm3). Symptoms usually include
the slow onset of headache, fever, and/or neurologic symptoms (eg, focal deficits, seizure). This patient with no known history of HIV and a day of
symptoms is far more likely to have a bacterial cause.

Educational objective:
Listeria monocytogenes is a common cause of bacterial meningitis in patients age >50 or who are immunocompromised (eg, chronic
glucocorticoids). These patients require empiric ampicillin in addition to the normal empiric treatment for bacterial meningitis (eg, third-generation
cephalosporin, vancomycin, dexamethasone).

References
Community-acquired Listeria monocytogenes meningitis in adults.

(http://www.ncbi.nlm.nih.gov/pubmed/17051485)

Medicine Infectious Diseases Meningitis


Subject System Topic

https://t.me/USMLEWorldStep2CK

REVIEW

https://www.uworld.com/ClientApp/v15/apps/qbanktestinterface/index.html#/launchtest/7281878/nbme/229400738/3/1 1/1
‫ م‬9:14 2021/‫‏‬9/‫‏‬11 https://t.me/USMLEWorldStep2CK UWorld STEP2 SIM Form 1

1 Item 18 of 40
2
Mark
Question Id: 6952 Previous Next
3

4 A 58-year-old man comes to the physician for a follow-up visit. The patient was recently discharged from the hospital after treatment for new-
5 onset acute gouty arthritis. He was treated with oral indomethacin and currently has no symptoms. The patient has a history of obesity, type 2
6 diabetes mellitus, hypercholesterolemia, and hypertension. His current medications include metformin, losartan, and rosuvastatin. The patient
7 does not use tobacco. He has consumed 3-4 cans of beer per day for the past 20 years. Blood pressure is 151/92 mm Hg and pulse is 85/min.
8 Laboratory results are as follows:
9
Triglycerides 300 mg/dL
10
11 Total cholesterol 170 mg/dL
12 HDL cholesterol 60 mg/dL
13
Blood urea nitrogen 15 mg/dL
14

15 Creatinine 0.8 mg/dL


16
Uric acid 7.3 mg/dL
17
18
HbA1c 7.2%

19
20
Which of the following is the best recommendation for this patient?

21
 A. Alcohol cessation (89%)
22

23 B. Colchicine (0%)
24
C. Febuxostat (1%)
25

26  D. Insulin therapy (1%)

27 E. Low-dose thiazide (1%)


28
F. Niacin (3%)
29
30 G. Probenecid (0%)
31
H. Protein-restricted diet (1%)
32

33
34

35 Incorrect 89%
36
Correct answer  Answered correctly  04 secs
Time Spent  05/05/2021
Last Updated
A
37

38

39 Explanation
40

Prevention of future gout attacks

Weight loss to achieve BMI <25 kg/m2


Low-fat diet
Decreased seafood & red meat intake
Protein intake preferably from vegetable & low-fat dairy products
Avoidance of organ-rich foods (eg, liver & sweetbreads)
Avoidance of beer & distilled spirits
Avoidance of diuretics when possible

This patient, with an acute first attack of gout, has responded to treatment and is now asymptomatic. Intercritical (between attacks) periods can
vary in duration, but many patients will have a recurrent gouty attack within 2 years of the first episode. For this reason, interventions should be
initiated to prevent recurrent attacks. Preventive measures for recurrent gout include lifestyle modifications and medications to lower uric acid
levels.

Medications for lowering serum urate are indicated for patients with the following factors:

Repeated and disabling attacks of gouty arthritis


Tophi suggesting chronic disease
X-ray evidence of chronic gouty joint disease
Uric acid kidney stones
Renal insufficiency

In the absence of these factors, lifestyle changes are considered first-line therapy. Recommended interventions include weight loss to achieve
BMI of <25 kg/m2, limiting alcohol consumption, and avoiding diuretics. Studies have shown that alcohol cessation or limiting alcohol intake to <2
drinks/day decreases the risk for gouty attacks.

(Choice B) Oral colchicine effectively treats acute gouty arthritis. It can also be used for short-term therapy (< 6 months duration) to prevent
gouty attacks while patients start urate lowering drugs (eg, allopurinol). Long-term colchicine can cause neuropathy or myopathy, which may
occur more often in patients taking statins.

(Choices C and G) Febuxostat and allopurinol are xanthine oxidase inhibitors that decrease uric acid production. Allopurinol is usually preferred,
and febuxostat is reserved for patients who cannot tolerate allopurinol. Probenecid is a uricosuric medication that increases uric acid excretion in
the kidneys. This is the patient's first attack, and so alcohol cessation is a more appropriate initial intervention.

(Choice D) This patient's HbA1C is only slightly above the goal level of 7%. Reinforcement of the patient's dietary and exercise habits along with
adjustment of his oral hypoglycemic regimen may be adequate to control his diabetes prior to considering insulin.

(Choice E) This patient requires more aggressive control of his blood pressure, which is higher than the goal of 140/90 mm Hg. However, a
thiazide diuretic may increase his serum uric acid level and potentially provoke another gout attack.

(Choice F) Niacin can treat hyperlipidemia, but it can also lead to hyperuricemia and should be avoided in this patient.

(Choice H) The preferred diet in gout patients is a calorie-restricted diet with decreased fat, increased complex carbohydrates, decreased intake
of meat and seafood, and increased proportion of protein from vegetable and low-fat dairy products. However, restriction in total protein intake
only minimally lowers serum uric acid and is not recommended.

Educational objective:
Lifestyle modifications, including alcohol cessation and weight loss, are considered first-line interventions following an initial gout attack.
Medications for lowering serum urate are indicated for patients with repeated and disabling attacks of gouty arthritis, tophi, x-ray evidence of
chronic gouty joint disease, uric acid kidney stones, or renal insufficiency.

References
Alcohol quantity and type on risk of recurrent gout attacks: an internet-based case-crossover study

(http://www.ncbi.nlm.nih.gov/pubmed/24440541)
Alcohol intake and risk of incident gout in men: a prospective study.

(http://www.ncbi.nlm.nih.gov/pubmed/15094272)
Purine-rich foods, dairy and protein intake, and the risk of gout in men.

(http://www.ncbi.nlm.nih.gov/pubmed/15014182)

Medicine Endocrine, Diabetes & Metabolism Gout


Subject System Topic

https://t.me/USMLEWorldStep2CK

REVIEW

https://www.uworld.com/ClientApp/v15/apps/qbanktestinterface/index.html#/launchtest/7281878/nbme/229400738/3/1 1/1
‫ م‬9:14 2021/‫‏‬9/‫‏‬11 https://t.me/USMLEWorldStep2CK UWorld STEP2 SIM Form 1

1 Item 19 of 40
2
Mark
Question Id: 6956 Previous Next
3

4 A 78-year-old woman is brought to the office due to multiple falls over the last 2 months. According to her family, she has repeatedly tripped and
5 fallen without losing consciousness. She has not had any broken bones, but has had multiple large bruises. Her past medical history is significant
6 for progressive dementia, occasional hallucinations, hypertension, and gout. Her current medications include memantine, haloperidol, quinapril,
7 metoprolol, and aspirin. On physical examination, her blood pressure is 140/90 mm Hg while supine and 138/87 mm Hg while standing. Her heart
8 rate is 57/min. Cardiac examination reveals a 2/6 systolic murmur at the apex. The lungs are clear to auscultation. She walks slowly taking small
9 steps. She speaks slowly and is poorly oriented to time, but she is oriented to place and person. She has a low-amplitude resting hand tremor
10 and exhibits resistance on wrist flexion bilaterally. Vibration sense is decreased over the ankles. Which of the following is the most likely cause of
11 this patient's falls?
12
A. Decreased postural sympathetic activity (5%)
13

14  B. Decreased dopaminergic activity in the brain (77%)


15
C. Advanced cortical atrophy (4%)
16

17 D. Sensory loss over her feet (12%)


18 E. Subclinical seizure activity (0%)

19
20

21
Incorrect 77%
22

23
Correct answer
B
 Answered correctly  02 secs
Time Spent  07/30/2021
Last Updated

24
25

26 Explanation

27
This patient has signs and symptoms of Parkinson disease (or the closely related Dementia with Lewy bodies [DLB]). These include dementia,
28
the classic "festinating" gait, slow speech, resting hand tremor, and "cogwheel" rigidity on passive range of motion. Frequent falls are a typical
29
feature of both Parkinson disease and DLB, due to gait disturbance and postural instability. Histologically, there is frontal lobe atrophy,
30
dopaminergic neuron degeneration in the substantia nigra pars compacta, and Lewy bodies in the affected neurons.
31

32 The pathology of Parkinson disease and DLB is similar, although there may be differences in clinical presentation.
33
34 (Choice A) Decreased postural sympathetic activity can lead to decreased central nervous system perfusion when shifting from supine to
35
standing, causing a loss of consciousness and fall. Although Parkinson disease can cause autonomic neuropathy, this patient has no orthostatic

36
changes in blood pressure, making decreased postural sympathetic activity an unlikely cause of her falls.

37
(Choice C) Advanced cortical atrophy occurs in Alzheimer disease, but resting tremor and falls are not characteristic of this disease.
38

39
(Choice D) Decreased vibration sense can be a normal finding in elderly patients. Sensory abnormalities are not seen with Parkinson disease.
40

(Choice E) The description of the patient’s falls is not consistent with subclinical seizure activity.

Educational objective:
Features of Parkinson disease and include dementia, falls, a festinating gait, slowed speech, a resting hand tremor and cogwheel rigidity on
passive range of motion. Parkinson disease is characterized by a loss of dopaminergic neurons in the substantia nigra.

Medicine Nervous System Parkinson disease


Subject System Topic

https://t.me/USMLEWorldStep2CK

REVIEW

https://www.uworld.com/ClientApp/v15/apps/qbanktestinterface/index.html#/launchtest/7281878/nbme/229400738/3/1 1/1
‫ م‬9:14 2021/‫‏‬9/‫‏‬11 https://t.me/USMLEWorldStep2CK UWorld STEP2 SIM Form 1

1 Item 20 of 40
2
Mark
Question Id: 6957 Previous Next
3

4 A 58-year-old man comes to the office due to progressive right-sided neck and arm pain. The pain started approximately 3 months ago, and the
5 patient saw a chiropractor. Despite neck manipulation, he continues to have pain. Lately, the pain has become constant and is associated with
6 numbness and tingling in his right forearm that extends to the tips of his fourth and fifth fingers. The patient's symptoms are becoming more
7 severe and awaken him at night. Acetaminophen no longer provides any relief. His chest x-ray is shown below.
8

10
11
12
13

14

15
16

17
18

19
20

21

22

23

24
25

26

27

28
This patient's current symptoms are most closely related to which of the following?
29
30
A. Alcohol consumption (2%)
31

32
B. Contact with tuberculosis patient (2%)

33 C. Family history (2%)


34
D. History of diabetes mellitus (1%)
35

36  E. Recent trauma (12%)


37
 F. Smoking history (78%)
38

39
G. Unsafe sexual habits (0%)

40

Incorrect 78% 03 secs 08/07/2021


Correct answer
F
 Answered correctly  Time Spent  Last Updated

Explanation

Common manifestations of superior pulmonary sulcus tumor

Shoulder pain
Horner syndrome (invasion of paravertebral sympathetic chain/stellate ganglion)
Ipsilateral ptosis, miosis, enophthalmos & anhidrosis
Neurologic symptoms in the arm (invasion of C8-T2 nerves)
Weakness/atrophy of intrinsic hand muscles
Pain/paresthesia of 4th/5th digits & medial arm/forearm
Supraclavicular lymphadenopathy
Weight loss

This patient's chest x-ray reveals a radio-opaque solid lesion occupying the apex of the right lung consistent with a superior pulmonary sulcus
(SPS) tumor (Pancoast tumor). The several-month history of right neck and arm pain progressing to numbness and tingling of the fourth and fifth
fingers is a common presentation due to tumor invasion of the inferior portion of the brachial plexus (symptoms typically occur in the ulnar nerve
distribution). Involvement of the phrenic nerve causing hemidiaphragm paralysis can also occur and is evidenced in this patient by the right
hemidiaphragm elevation on chest x-ray. Horner syndrome is also common, and superior vena cava syndrome can occur.

Primary lung malignancies such as squamous cell carcinoma and adenocarcinoma (small cell lung carcinoma accounts for only ~5%) are
overwhelmingly the most common etiology of SPS tumors, and a history of smoking is the strongest risk factor. Other rare etiologies include
lymphoma, mesothelioma, and some pulmonary infections.

(Choice A) Excessive alcohol consumption predisposes to aspiration pneumonia and pulmonary abscess. Chest x-ray would demonstrate
cavitation with an air-fluid level, and arm pain would be unusual.

(Choice B) Tuberculosis can cause cavitary apical lung lesions. These lesions are very unlikely to involve the brachial plexus and mimic an SPS
tumor, and the solid apical mass on this patient's chest x-ray is most consistent with an SPS tumor of pulmonary origin.

(Choice C) Family history plays a role in the development of some lung malignancies, but smoking is the strongest risk factor.

(Choice D) Poorly controlled diabetes mellitus can lead to peripheral neuropathy that typically occurs bilaterally in a "stocking and glove" pattern.
This patient's unilateral numbness and tingling in an ulnar nerve distribution are not consistent with diabetic neuropathy.

(Choice E) Recent trauma can cause nerve impingement, but this patient's progressive symptoms and apical mass on chest x-ray are more
consistent with an SPS tumor.

(Choice G) Unsafe sexual habits increase the risk for sexually transmitted infections such as syphilis. Tertiary syphilis can cause tabes dorsalis,
which is characterized by sensory ataxia and pupillary dysfunction. Unilateral numbness and tingling of the arm would be unusual.

Educational objective:
Superior pulmonary sulcus tumors are usually primary lung malignancies and can present with referred arm, neck, or shoulder pain and
paresthesia and weakness in the ulnar nerve distribution. Smoking is the strongest risk factor.

References
Pancoast tumors: characteristics and preoperative assessment.

(http://www.ncbi.nlm.nih.gov/pubmed/24672686)

Medicine Pulmonary & Critical Care Lung cancer


Subject System Topic

https://t.me/USMLEWorldStep2CK

REVIEW

https://www.uworld.com/ClientApp/v15/apps/qbanktestinterface/index.html#/launchtest/7281878/nbme/229400738/3/1 1/1
‫ م‬9:14 2021/‫‏‬9/‫‏‬11 https://t.me/USMLEWorldStep2CK UWorld STEP2 SIM Form 1

1 Item 21 of 40
2
Mark
Question Id: 6965 Previous Next
3

4 A 31-year-old man comes to the office due to worsening diarrhea over the past year. He has 3-4 large bowel movements a day associated with
5 abdominal distension and loud bowel sounds that often make him uncomfortable in public. The patient has never noted blood in his stool. His
6 appetite is good, and he consumes a broad variety of foods but has lost 15 pounds over the last year. Medical history is notable for a right wrist
7 fracture 6 months ago when the patient slipped and fell in a bathroom. Family history is significant for colon cancer in his father at age 50. The
8 patient does not use alcohol, tobacco, or illicit drugs. Laboratory results include the following:
9
Hemoglobin 10.8 g/dL
10
11 Mean corpuscular volume 70 μm3
12
13 Which of the following diagnoses is most likely?
14
 A. Colonic polyposis (10%)
15
16 B. Hyperthyroidism (2%)
17
C. Irritable bowel syndrome (4%)
18

19 D. Lactose intolerance (3%)


20
 E. Malabsorption (59%)
21

22
F. Ulcerative colitis (18%)

23

24
25 Incorrect 59%
26 Correct answer  Answered correctly  04 secs
Time Spent  08/28/2021
Last Updated
E
27

28

29 Explanation
30
31

32 Features of malabsorption in celiac disease


33 Nutrients Symptoms
34
General Bulky, foul-smelling, floating stools
35

36 Fat & protein Loss of muscle mass, loss of subcutaneous fat, fatigue
37
Iron Pallor (anemia), fatigue
38

39
Calcium & vitamin D Bone pain (osteomalacia), fracture (osteoporosis)

40 Vitamin K Easy bruising

Vitamin A Hyperkeratosis

This patient has several features to suggest malabsorption, including large-volume diarrhea, borborygmi, and weight loss. In addition, a fracture
following a ground level fall is unusual in a young man and reinforces the possibility of malabsorption as it may indicate osteopenia due to vitamin
D deficiency. Malabsorption can also lead to specific micronutrient deficiencies (eg, microcytic iron deficiency anemia), as well as steatorrhea,
presenting as unusually foul-smelling, floating stools. However, patients may have only mild, nonspecific symptoms at the time of initial
presentation.

The cause of malabsorption can often be deduced from historical features. This patient has no abdominal pain or heavy alcohol consumption to
suggest a pancreatic cause of malabsorption. The most likely etiology is celiac disease, an autoimmune disorder that causes malabsorption due
to villous atrophy. Even though celiac disease is triggered by gluten-containing wheat products, patients often do not recognize this association
due to the ubiquity of wheat-based foods. Because fat has the most complex digestive requirements of all macronutrients, assay of the stool for
fat is the most sensitive initial test for malabsorptive syndromes. Subsequent evaluation is dependent on individual clinical factors.

(Choice A) Villous adenomatous polyps may produce a secretory diarrhea, but absorption of nutrients is normal.

(Choice B) Hyperthyroidism can cause hyperdefecation and osteoporosis (due to increased bone resorption). However, large-volume bowel
movements would be unusual, and hyperthyroidism typically causes a normocytic, normochromic anemia (due to an increase in plasma volume).

(Choice C) Irritable bowel syndrome causes a variety of gastrointestinal symptoms. However, systemic symptoms (eg, weight loss) are not
common, and this patient's fragility fracture suggests a more significant alteration in nutrient uptake.

(Choice D) Lactose intolerance may present with diarrhea, but most patients note a correlation with dairy products. Malabsorption of iron or fat-
soluble vitamins should not occur.

(Choice F) With ulcerative colitis, diarrhea is a common presenting symptom but is typically bloody, episodic, and associated with abdominal pain.

Educational objective:
Malabsorption can cause steatorrhea, and assay of the stool for fat is the most sensitive initial test for malabsorptive syndromes. Celiac disease
causes malabsorption due to villous atrophy. Even though it is triggered by gluten-containing products, patients often do not recognize this
association due to the ubiquity of wheat-based foods.

References
Vitamin D deficiency in patients with intestinal malabsorption syndromes - think in and outside the gut.

(http://www.ncbi.nlm.nih.gov/pubmed/26316334)

Medicine Gastrointestinal & Nutrition Celiac disease


Subject System Topic

https://t.me/USMLEWorldStep2CK

REVIEW

https://www.uworld.com/ClientApp/v15/apps/qbanktestinterface/index.html#/launchtest/7281878/nbme/229400738/3/1 1/1
‫ م‬9:14 2021/‫‏‬9/‫‏‬11 https://t.me/USMLEWorldStep2CK UWorld STEP2 SIM Form 1

1 Item 22 of 40
2
Mark
Question Id: 6970 Previous Next
3

4 A 72-year-old man is undergoing medical evaluation prior to an elective inguinal herniorrhaphy. The patient's other medical problems include
5 hypertension, hyperlipidemia, diabetes mellitus type 2, and coronary artery disease status post right coronary artery stenting 2 years ago. His
6 surgical history is also significant for transurethral prostate resection 5 years ago. On physical examination, a soft murmur starts after the second
7 heart sound and declines in intensity until disappearing suddenly before the first heart sound. The murmur is heard along the left and right sternal
8 borders and is accentuated when the patient sits up, leans forward, and puts his hands behind his head. Which of the following is the most likely
9 cause of these physical examination findings?
10
 A. Aortic root dilation (71%)
11
12 B. Chordae tendinae degeneration (9%)
13
C. Coronary artery occlusion (0%)
14

15 D. Hyperdynamic circulation (8%)


16
E. Pericardial thickening (7%)
17
18  F. Pulmonary hypertension (2%)

19
20

21 Incorrect 71% 02 secs 03/21/2021


22 Correct answer
A
 Answered correctly  Time Spent  Last Updated
23

24
25
Explanation
26

27 The cardiac auscultation findings are consistent with aortic regurgitation (AR). The most common cause of AR in developing countries is
28 rheumatic heart disease. In developed countries, AR is most commonly due to aortic root dilation or congenital bicuspid valve.
29
AR leads to an early decrescendo diastolic murmur that begins immediately after A2 (aortic component of second heart sound [S2]) and is
30
accentuated with the patient sitting up and leaning forward while holding his breath in full expiration (bringing the aortic valve closer to the
31
stethoscope).
32

33
With valvular AR, the murmur is best heard along the left sternal border at the third and fourth intercostal spaces

34
With AR due to aortic root dilation, the murmur commonly radiates toward the right side and is best heard along the right sternal border

35 This patient has atherosclerosis and a diastolic murmur also heard on the right sternal border, suggesting that the likely cause of his AR is aortic
36 root dilation, possibly due to an ascending aortic aneurysm.
37
(Choice B) Chordae tendineae degeneration can lead to mitral valve prolapse (midsystolic click over cardiac apex) and mitral regurgitation (MR)
38
(mid to late systolic murmur).
39
(Choice C) Coronary artery occlusion by itself does not cause a murmur. Myocardial ischemia or infarction involving the dominant right coronary
40
artery or left circumflex artery can cause a holosystolic murmur due to papillary muscle dysfunction and MR.

(Choice D) Hyperdynamic circulation (eg, pregnancy, anemia, thyrotoxicosis) can occasionally cause an ejection or midsystolic murmur due to
increased blood flow across the semilunar (aortic or pulmonary) valves.

(Choice E) Acute pericardial inflammation (pericarditis) often causes a friction rub. Chronic pericardial inflammation and thickening (constrictive
pericarditis) can cause a pericardial knock, an accentuated diastolic heart sound heard after S2.

(Choice F) Pulmonary hypertension causes a loud P2 (pulmonic component of S2). In severe cases, there may be a diastolic pulmonary
regurgitation murmur (Graham-Steell murmur). Right-sided murmurs are accentuated with inspiration and leg raising in supine position.

Educational objective:
The murmur of aortic regurgitation (decrescendo diastolic murmur) due to congenital bicuspid valve is best heard along the left sternal border at
the third and fourth interspace with the patient sitting up, leaning forward, and holding the breath in full expiration. The murmur due to aortic root
dilation commonly radiates toward the right side and is best heard along the right sternal border.

References
Clinical practice. aortic regurgitation.

(http://www.ncbi.nlm.nih.gov/pubmed/15470217)
"Right-sided" murmurs of aortic insufficiency (diastolic murmurs better heard to the right of the sternum rather than to the left).

(http://www.ncbi.nlm.nih.gov/pubmed/13960803)

Medicine Cardiovascular System Aortic regurgitation


Subject System Topic

https://t.me/USMLEWorldStep2CK

REVIEW

https://www.uworld.com/ClientApp/v15/apps/qbanktestinterface/index.html#/launchtest/7281878/nbme/229400738/3/1 1/1
‫ م‬9:14 2021/‫‏‬9/‫‏‬11 https://t.me/USMLEWorldStep2CK UWorld STEP2 SIM Form 1

1 Item 23 of 40
2
Mark
Question Id: 6976 Previous Next
3

4 A 42-year-old man comes to the emergency department due to difficulty walking. He first noticed a tingling sensation in his toes a week ago, but
5 he now has unsteadiness on his feet and has stumbled while walking. The patient recently recovered from a diarrheal illness, during which he
6 recalls seeing frank blood in his stool. Temperature is 37.1 C (98.8 F). Physical examination reveals decreased muscle strength in his lower
7 extremities with diminished reflexes. There is no sensory loss. Cerebrospinal fluid analysis in this patient is most likely to show which of the
8 following?
9
A. Normal protein level (8%)
10
11  B. Normal leukocyte count (79%)
12
C. Low glucose content (7%)
13

14 D. Xanthochromia (3%)
15
 E. Acid-fast bacilli (0%)
16

17
18

19 Incorrect 79%
20
Correct answer  Answered correctly  01 sec
Time Spent  07/30/2021
Last Updated
B
21

22

23 Explanation
24
25
This patient most likely has Guillain-Barre Syndrome (GBS), which typically presents with ascending, symmetric weakness and loss of deep

26
tendon reflexes in the lower extremities. Sensory function is usually relatively preserved in GBS, but most patients have some form of distal
paresthesias. Autonomic symptoms are also commonly seen. GBS usually occurs following a respiratory or gastrointestinal infection, and is
27
thought to result from an abnormal immune response that targets the peripheral nerves. This patient’s bloody diarrhea was most likely caused by
28
an infection with Campylobacter jejuni, which is the most common precipitant of GBS.
29
30
GBS is usually slowly progressive over a period of 1-2 weeks, and can be fatal if the respiratory muscles are involved. The classic finding on
31
lumbar puncture in patients with GBS is an elevated protein content with a normal leukocyte count, referred to as albuminocytologic dissociation.
32

33
(Choice A) The protein content of the cerebrospinal fluid (CSF) is usually elevated in patients with GBS.
34

35
(Choice C) Low glucose concentration in the CSF may be seen in bacterial meningitis, but the glucose level should be normal in GBS.
36

37 (Choice D) Xanthochromia is seen in the CSF of patients with a subarachnoid hemorrhage, but not GBS.
38

39 (Choice E) Acid-fast bacilli can be seen in the CSF of patients with tuberculosis, which would not be consistent with this patient’s presentation.
40
Educational objective:
Guillain-Barre Syndrome (GBS) typically presents with symmetric, ascending weakness and loss of reflexes with relative preservation of sensory
function. It is often associated with a preceding respiratory or gastrointestinal infection. The classic finding on lumbar puncture in patients with
GBS is an elevated protein content with a normal leukocyte count, sometimes referred to as albuminocytologic dissociation.

Medicine Nervous System Guillain Barre syndrome


Subject System Topic

https://t.me/USMLEWorldStep2CK

REVIEW

https://www.uworld.com/ClientApp/v15/apps/qbanktestinterface/index.html#/launchtest/7281878/nbme/229400738/3/1 1/1
‫ م‬9:14 2021/‫‏‬9/‫‏‬11 https://t.me/USMLEWorldStep2CK UWorld STEP2 SIM Form 1

1 Item 24 of 40
2
Mark
Question Id: 7244 Previous Next
3

4 A 41-year-old man with type 1 diabetes mellitus comes to the emergency department with pain and redness of the left leg. The patient has been
5 admitted 3 times in the past 6 months due to similar presentations. Temperature is 38.3 C (100.9 F), blood pressure is 110/70 mm Hg, pulse is
6 96/min, and respirations are 12/min. Physical examination of the left calf shows cellulitis; the remainder of the examination is normal. The patient
7 is admitted to the hospital, and intravenous antibiotics are administered. The infection responds rapidly, and he is afebrile within 24 hours of
8 admission. The patient continues to recover without complications and is transitioned to oral antibiotics. On the day of discharge, he tells the
9 physician that his right leg hurts and that he is worried the medication is not working. Vital signs are within normal limits. A new puncture wound
10 surrounded by mild erythema is visualized on examination of the right lower leg. Which of the following is the most appropriate next step in
11 management of this patient?
12
A. Ask the patient if he intentionally infected his leg (9%)
13

14 B. Discharge the patient as planned with oral antibiotics (1%)


15
 C. Evaluate the patient for comorbid psychiatric conditions (73%)
16

17 D. Search the patient's room for used syringes (12%)


18 E. Tell the patient he will be placed on constant observation (3%)

19
20

21
Incorrect 73%
22

23
Correct answer
C
 Answered correctly  02 secs
Time Spent  09/10/2021
Last Updated

24
25

26 Explanation

27
Evidence of a new puncture wound with surrounding erythema on the day of discharge, after successful hospital treatment of cellulitis, is highly
28
suggestive of a self-induced injury. This patient's high health care use (ie, 3 hospitalizations over 6 months for similar presentations) is an
29
additional clue that he is attempting to prolong his hospitalization by infecting himself. Induction of injury or disease associated with deception
30
may be due to factitious disorder (ie, desire to assume the sick role without obvious external rewards) or malingering (eg, obtaining financial
31
benefits, drugs, time off work).
32

33 Management of these patients is difficult because they typically respond to confrontation with denial, leading them to sign out against medical
34 advice and seek treatment elsewhere. A confrontational approach also risks the patient becoming disruptive or physically threatening.
35
Recommended management includes:

36 Giving feedback in a supportive rather than a confrontational manner.


37
Evaluating the patient for comorbid psychiatric disorders, psychosocial stressors, and self-harm (ie, further self-injurious behavior and/or
38
suicidal ideation, intent, and plan).
39

40
Patients with factitious illness and malingering have high rates of psychiatric comorbidity, including depressive and anxiety disorders, substance
use disorders, and personality disorders (eg, borderline personality disorder, antisocial personality disorder). Psychiatric consultation can also be
obtained to assist in assessment and provide guidance on discussing the diagnosis and treatment recommendations with the patient.

(Choice A) Asking the patient if he intentionally infected his leg is confrontational and likely to be met with angry denial. Providing compassionate
feedback is more likely to keep the patient engaged in treatment.

(Choice B) It is premature to discharge the patient without further evaluation, and he would likely return to the emergency department shortly with
a similar presentation.

(Choices D and E) A room search and constant observation are intrusive and confrontational interventions that will likely anger the patient and
decrease his willingness to cooperate with further assessment and treatment. A compassionate discussion about the clinical findings and
recommendations for further evaluation should precede these interventions.

Educational objective:
Patients identified as falsifying or inducing illness typically respond to confrontation with denial. Recommended management includes providing
feedback in a supportive rather than a confrontational manner and evaluating for comorbid psychiatric conditions and contributing psychosocial
factors.

References
Factitious disorders and malingering: challenges for clinical assessment and management.

(http://www.ncbi.nlm.nih.gov/pubmed/24612861)

Psychiatry Psychiatric/Behavioral & Substance Abuse Factitious disorder


Subject System Topic

https://t.me/USMLEWorldStep2CK

REVIEW

https://www.uworld.com/ClientApp/v15/apps/qbanktestinterface/index.html#/launchtest/7281878/nbme/229400738/3/1 1/1
‫ م‬9:15 2021/‫‏‬9/‫‏‬11 https://t.me/USMLEWorldStep2CK UWorld STEP2 SIM Form 1

1 Item 25 of 40
2
Mark
Question Id: 6985 Previous Next
3

4 A 36-year-old man is brought to the emergency department due to bilateral lower extremity paralysis. The patient was unable to get out of bed
5 this morning and called his friend for help. He has had low back pain and malaise for several days. The patient has a history of daily injection
6 drug use. He has refused HIV testing in the past. He was recently released from prison after a short incarceration for drug possession.
7 Temperature is 38.3 C (100.9 F), blood pressure is 130/85 mm Hg, and pulse is 100/min. Lung auscultation is normal. No cardiac murmurs are
8 present. Neurologic examination shows loss of sensation and 0/5 strength in the bilateral lower extremities. Upper extremity neurologic
9 assessment is unremarkable. There are no meningeal signs. Which of the following is the best initial pharmacotherapy for this patient?
10
A. Antituberculosis treatment (0%)
11
12  B. Broad-spectrum antibacterial therapy (100%)
13
C. Equine antitoxin therapy (0%)
14

15 D. High-dose glucocorticoid (0%)


16
 E. Intravenous immunoglobulin (0%)
17
18

19
20
Incorrect
21
Correct answer
B
Collecting Statistics  02 secs
Time Spent  09/08/2021
Last Updated

22

23

24 Explanation
25

26
Spinal epidural abscess
27

28 Staphylococcus aureus (65%)


29
Immunosuppression (HIV, diabetes mellitus, alcohol use, old age)

30
Inoculating sources
Epidemiology
Distant infection (eg, cellulitis, joint/bone)
31
Spinal procedure (eg, epidural catheter)
32
Injection drug use
33
34 Classic triad
35 Fever (~50%)
Manifestations
36 Focal/severe back pain
37 Neurologic findings (eg, motor/sensory change, bowel/bladder dysfunction, paralysis)
38 ↑ ESR
39 Diagnosis Blood & aspirate cultures
40 MRI of the spine

Broad-spectrum antibiotics (eg, vancomycin plus ceftriaxone)


Treatment
Emergency aspiration/surgical decompression

ESR = erythrocyte sedimentation rate.

Spinal epidural abscess (SEA) is a rare infection of the epidural space that may threaten the spinal cord and result in paralysis. Most cases are
due to hematologic spread from a distant infection (eg, skin/soft tissue, bone/joint), intravenous drug use, or direct inoculation during a spinal
procedure. Staphylococcus aureus is the most common (65%) causative organism.

SEA classically results in the triad of fever, focal back pain, and neurologic deficits; however, all 3 symptoms are present in a minority of cases.
Most patients initially develop typically severe and focal back pain that progresses over days to nerve root pain ("shooting/electric") and more
severe neurologic manifestations (motor weakness, sensory changes, bowel/bladder dysfunction, paralysis). Laboratory studies, including
leukocyte count, may be normal, but erythrocyte sedimentation rate is almost always elevated.

Diagnosis is confirmed with MRI of the spine. The prompt initiation of broad-spectrum intravenous antibiotics (eg, vancomycin plus
ceftriaxone) is required, and urgent decompression of the epidural space with aspiration or surgery may prevent (or potentially reverse)
paralysis.

(Choice A) Pott disease (spinal tuberculosis) usually causes subacute or chronic back pain and alterations in gait. Paralysis may occur but is
more common after weeks or months, not days, of symptoms. Although this patient with a history of incarceration is at risk for tuberculosis, a
bacterial cause of SEA is much more common in injection drug users.

(Choice C) Equine antitoxin can be used to treat botulism, which usually causes descending bilateral paralysis beginning in the cranial nerves.
Fever is rare. This patient has fever and only lower extremity paralysis.

(Choice D) Although high-dose glucocorticoids are used to reduce compressive edema in symptomatic patients with neoplastic epidural spinal
cord compression (ie, epidural metastases), they are avoided in patients with SEA because of the risk of worsening infection. This patient's fever
and injection drug use make SEA much more likely than metastases.

(Choice E) Intravenous immunoglobulins can be used to treat Guillain-Barré syndrome (GBS), which usually causes an ascending bilateral motor
weakness that may progress to paralysis. This patient with intravenous drug use, low back pain, and days of fever and malaise is more likely to
have an epidural abscess. GBS would cause decreased/absent, not increased, deep tendon reflexes.

Educational objective:
Spinal epidural abscess most commonly arises with concurrent distant infection (eg, skin/soft tissue), injection drug use, or spinal procedure. The
classic triad of symptoms is fever, back pain, and neurologic manifestations. Treatment requires broad-spectrum antibiotics (eg, vancomycin plus
ceftriaxone) and aspiration or surgical decompression.

References
Spinal epidural abscess: a review with special emphasis on earlier diagnosis.

(http://www.ncbi.nlm.nih.gov/pubmed/28044125)

Medicine Nervous System Epidural abscess


Subject System Topic

https://t.me/USMLEWorldStep2CK

REVIEW

https://www.uworld.com/ClientApp/v15/apps/qbanktestinterface/index.html#/launchtest/7281878/nbme/229400738/3/1 1/1
‫ م‬9:15 2021/‫‏‬9/‫‏‬11 https://t.me/USMLEWorldStep2CK UWorld STEP2 SIM Form 1

1 Item 26 of 40
2
Mark
Question Id: 6988 Previous Next
3

4 A 30-month-old boy is brought to the clinic with a 2-day history of sore throat and decreased appetite. The patient has been crying when
5 swallowing food but is drinking water and milk without difficulty. He has no runny nose, cough, or trouble breathing. The patient was born full term
6 and has no chronic medical conditions. Temperature is 37.8 C (100 F). Tympanic membranes are clear bilaterally. The oropharynx is
7 erythematous with several small vesicles on the uvula, soft palate, and tonsillar pillars. The tonsils are slightly erythematous bilaterally and have
8 no exudates. No cervical lymphadenopathy is present. The skin is without rashes. Which of the following is the best next step in management of
9 this patient?
10
A. Herpes polymerase chain reaction (32%)
11
12 B. Monospot test (0%)
13
C. Rapid streptococcal antigen test (12%)
14

15  D. Reassurance and observation (50%)


16
E. Throat culture (3%)
17
18

19
20 50%
21
Correct  Answered correctly  02 secs
Time Spent  08/13/2021
Last Updated

22

23

24 Explanation
25

26

27

28

29
30
31

32

33
34

35

36

37

38

39

40

Viruses are the most common cause of pharyngitis in children. This patient has oral vesicles on the uvula, soft palate, and tonsillar pillars,
consistent with herpangina, an infection typically caused by Coxsackie A virus.

Herpangina is a clinical diagnosis based on the presentation of vesicular pharyngitis. Treatment is reassurance and supportive care (eg,
hydration, analgesia), as symptoms generally self-resolve within a week.

(Choice A) Herpes gingivostomatitis manifests as vesicular eruptions on the anterior oral mucosa (ie, buccal mucosa, hard palate, gingiva,
tongue) and perioral region. The posterior location of this patient's vesicles makes herpangina more likely.

(Choice B) A monospot test detects Epstein-Barr virus mononucleosis, which typically presents in adolescents with fever, pharyngitis, and
posterior cervical lymphadenopathy. Epstein-Barr virus does not cause oropharyngeal vesicles.

(Choices C and E) Routine testing for group A streptococcus with rapid streptococcal antigen or throat culture is not recommended. Clinical
scores to evaluate the need for microbiologic studies (eg, Centor criteria) are not useful in children and preadolescents. Streptococcal pharyngitis
is rare in individuals age <3 and does not progress to rheumatic fever. In addition, testing should be performed only in patients who have sore
throat with concomitant tonsillar exudates or swelling, palatal petechiae, and have no viral symptoms (eg, rhinorrhea).

Educational objective:
Herpangina is a viral infection characterized by oral ulcers or vesicles on the uvula, soft palate, and tonsillar pillars. Diagnosis is clinical, and
treatment is reassurance and supportive care.

References
Accuracy and precision of the signs and symptoms of streptococcal pharyngitis in children: a systematic review.

(http://www.ncbi.nlm.nih.gov/pubmed/22048053)
Clinical practice guideline for the diagnosis and management of group A streptococcal pharyngitis: 2012 update by the Infectious Diseases
Society of America.

(http://www.ncbi.nlm.nih.gov/pubmed/22965026)

Pediatrics Infectious Diseases Herpangina


Subject System Topic

https://t.me/USMLEWorldStep2CK

REVIEW

https://www.uworld.com/ClientApp/v15/apps/qbanktestinterface/index.html#/launchtest/7281878/nbme/229400738/3/1 1/1
‫ م‬9:15 2021/‫‏‬9/‫‏‬11 https://t.me/USMLEWorldStep2CK UWorld STEP2 SIM Form 1

1 Item 27 of 40
2
Mark
Question Id: 6990 Previous Next
3

4 A 62-year-old man is brought to the emergency department due to a 2-day history of redness and pain in the right foot. His medical history
5 includes hypertension and rheumatoid arthritis. The patient smokes half a pack of cigarettes daily and drinks alcohol occasionally. Blood pressure
6 is 80/52 mm Hg, pulse is 126/min, and respirations are 24/min. BMI is 28 kg/m2. The patient is drowsy but responds to verbal stimuli. Mucous
7 membranes are moist, and there is no jugular venous distension. The right foot is swollen, erythematous, tender, and warm to the touch.
8 Bounding peripheral pulses are palpable in all extremities. Laboratory results are as follows:
9
Complete blood count
10
11 Hemoglobin 10.5 g/dL
12 Platelets 130,000/mm3
13
Leukocytes 18,000/mm3
14

15 Serum chemistry
16
Blood urea nitrogen 22 mg/dL
17
18
Creatinine 2.0 mg/dL

19 Lactic acid (venous) 2.6 mEq/L (normal, 0.7-2.1)


20

21
Which of the following is most likely to be seen on hemodynamic monitoring in this patient?

22

23
Pulmonary Cardiac index Systemic vascular Mixed venous

24
capillary wedge resistance (SVR) oxygen saturation
pressure (PCWP) (MvO2)
25

26 A. ↑ ↓ ↑ ↓ (1%)
27
B. ↓ ↓ ↑ ↓ (5%)
28

29 C. Normal/↓ Normal/↓ ↑ ↑ (5%)


30
 D. Normal/↓ ↑ ↓ ↑ (72%)
31

32 E. Normal/↑ ↑ ↑ Normal/↓ (5%)


33 F. ↑ ↑ ↓ Normal/↓ (9%)
34

35

36

72%
37

38
Correct  Answered correctly  03 secs
Time Spent  05/09/2021
Last Updated

39

40
Explanation

This patient with an infected right foot, hypotension, tachycardia, leukocytosis, and elevated lactate with acute kidney injury likely has septic
shock, a form of distributive shock. Right heart catheterization of a patient with septic shock usually demonstrates:

Decreased systemic vascular resistance (SVR) due to peripheral vasodilation


Decreased (or low normal) pulmonary capillary wedge pressure (PCWP) due to capillary leakage and decreased preload
Elevated mixed venous oxygen (MvO2) saturation due to hyperdynamic circulation (the cardiac index increases due to reduced SVR to
maintain perfusion) with an inability of tissues to adequately extract oxygen

(Choice A) This patient likely has obstructive shock from cardiac tamponade. Such patients develop increased intrapericardial pressure that can
lead to increased right atrial pressure, right ventricular pressures, and PCWP (with equalization of these pressures at end-diastole) and a
decrease in the cardiac index (due to compression). This subsequent decrease causes a compensatory elevated SVR with a decreased MvO2
saturation as peripheral tissues extract more oxygen. Patients usually have distant heart sounds and jugular venous distension.

(Choice B) This patient likely has hypovolemic shock from low intravascular volume, which decreases PCWP with a compensatory increased
SVR. In early hypovolemic shock, the cardiac index can be normal, but with progression, it can decrease. Low volume causes decreased tissue
perfusion, signaling increased extraction of oxygen, thereby decreasing MvO2 saturation. These patients usually appear dehydrated (eg, dry
mucous membranes).

(Choice C) This patient likely has obstructive shock from pulmonary embolism, which can present with dyspnea and tachycardia and cause right
ventricular strain from the inability of the heart to pump against increased vascular resistance of the pulmonic system. Depending on embolic
burden, patients can have a normal to low PCWP and cardiac index with a compensatory increased SVR and an elevated MvO2 saturation due to
poor distribution of blood flow.

(Choice E) This patient has normal exercise physiology, including normal to elevated PCWP from increased blood flow, an elevated cardiac index
and SVR to compensate for increased oxygen demand by peripheral tissues, and normal to decreased MvO2 saturation due to increased
extraction of oxygen.

(Choice F) This patient may have high-output cardiac failure (eg, due to arteriovenous shunt), an uncommon, relatively hypermetabolic state
characterized by an elevated cardiac index ("high-output") with elevated PCWP (due to venous congestion), excessively low SVR (due to
vasodilation or shunting), and sometimes low MvO2 content (possibly reflecting oxygen consumption). Patients typically have manifestations of
heart failure (eg, dyspnea, edema) with bounding pulses and a systolic bruit.

Educational objective:
Systemic vascular resistance (SVR), pulmonary capillary wedge pressure (PCWP), and a cardiac index can be used to differentiate types of
shock. A patient with distributive shock (eg, septic shock) usually has an elevated cardiac index, normal to decreased PCWP, decreased SVR,
and increased mixed venous oxygen saturation.

References
The Third International Consensus Definitions for Sepsis and Septic Shock (Sepsis-3).

(http://www.ncbi.nlm.nih.gov/pubmed/26903338)
Circulatory shock.

(http://www.ncbi.nlm.nih.gov/pubmed/24171518)
High-output heart failure: a 15-year experience.

(http://www.ncbi.nlm.nih.gov/pubmed/27470455)

Medicine Cardiovascular System Sepsis


Subject System Topic

https://t.me/USMLEWorldStep2CK

REVIEW

https://www.uworld.com/ClientApp/v15/apps/qbanktestinterface/index.html#/launchtest/7281878/nbme/229400738/3/1 1/1
‫ م‬9:15 2021/‫‏‬9/‫‏‬11 https://t.me/USMLEWorldStep2CK UWorld STEP2 SIM Form 1

1 Item 28 of 40
2
Mark
Question Id: 6992 Previous Next
3

4 A 55-year-old white woman comes to the office for evaluation of a nonhealing, painful leg ulcer. The lesion started 3 weeks ago as a painful
5 nodule and later ulcerated. Two months ago, the patient was evaluated for right ear otalgia and hearing loss. She reports no recent travel history.
6 She underwent hysterectomy for uterine fibroids 10 years ago. Temperature is 38.3 C (101 F), blood pressure is 150/90 mm Hg, and pulse is
7 80/min. Pulse oxymetry shows 98% on room air. Physical examination shows an ulcer on the right leg. Cardiopulmonary examination is normal.
8 Laboratory results are as follows:
9
Complete blood count
10
11 Hemoglobin 10.4 g/dL
12 Platelets 380,000/mm3
13
Leukocytes 11,000/mm3
14

15
Serum chemistry
16

17 Sodium 140 mEq/L


18
Potassium 4.6 mEq/L
19
20
Bicarbonate 26 mEq/L

21 Blood urea nitrogen 26 mg/dL


22
Creatinine 1.9 mg/dL
23

24 Calcium 8.8 mg/dL


25 Glucose 100 mg/dL
26

27 Urinalysis
28
Protein +2
29
30 Blood trace
31
Nitrites negative
32

33
Bacteria none

34 White blood cells 1-2/hpf


35
Red blood cells 10-15/hpf
36

37
Which of the following is the most likely cause of this patient's nonhealing ulcer?
38

39 A. Bacterial infection (17%)



40
B. Hemoglobinopathy (2%)

C. Immunosuppression (16%)

D. Malnutrition (2%)

 E. Necrotizing vasculitis (44%)

F. Peripheral artery disease (16%)

Incorrect 44%
Correct answer  Answered correctly  09 secs
Time Spent  06/12/2021
Last Updated
E

Explanation

This patient likely has granulomatosis with polyangiitis (GPA) (formerly known as Wegener granulomatosis), a necrotizing vasculitis affecting
small- to medium-sized blood vessels. GPA most commonly occurs in white individuals age 30-50 and affects men and women with equal
frequency. It is characterized by:

Antineutrophil cytoplasmic antibody (ANCA) positivity


Upper respiratory tract (including ear, nose, and throat [ENT]) involvement: Manifested in this patient by the otalgia and hearing loss
Lower respiratory tract involvement
Kidney involvement: Manifested in this patient by the renal insufficiency and microscopic hematuria with proteinuria suggesting
glomerulonephritis.

Approximately 50% of patients with GPA develop cutaneous manifestations such as leukocytoclastic angiitis (purpura on the lower extremities
with ulceration), urticaria, livedo reticularis, or pyoderma gangrenosum (PG) (inflammatory papule/pustule progressing to painful ulcer). GPA can
be associated with classic PG, thought to be due to immune system dysregulation; or, as is likely in this patient, it can be associated with a PG-like
lesion caused by impaired nodular healing in the setting of vasculitis-induced blood vessel damage leading to vessel occlusion, localized
ischemia, and necrosis.

(Choices A, C, and D) In GPA, PG and PG-like lesions are due to immune dysregulation or ischemia, and no infection is present (fever is due to
vasculitic inflammation). Immunosuppression and malnutrition may impair wound healing by inhibiting the inflammatory response and increasing
the risk of infection. However, the constellation of a nonhealing ulcer, ENT symptoms, and possible glomerulonephritis is best explained by GPA
rather than by malnutrition, immunosuppression, or bacterial infection.

(Choice B) Sickle cell anemia can impair wound healing through local ischemia caused by physical obstruction of small arterioles by deformed
red blood cells. Sickle cell anemia is very unlikely in this adult, white patient with no history of vasoocclusive crises.

(Choice F) Peripheral artery disease (PAD) occurs due to atherosclerosis and can impair wound healing by local ischemia. PAD would be
unusual in this relatively young woman with no known atherosclerotic disease.

Educational objective:
Granulomatosis with polyangiitis is a necrotizing vasculitis typically affecting the upper and lower respiratory tracts and kidneys. Cutaneous
manifestations of the disease are common, and vasculitic inflammation may lead to localized ischemia and impaired wound healing.

Medicine Rheumatology/Orthopedics & Sports Granulomatosis with polyangiitis


Subject System Topic

https://t.me/USMLEWorldStep2CK

REVIEW

https://www.uworld.com/ClientApp/v15/apps/qbanktestinterface/index.html#/launchtest/7281878/nbme/229400738/3/1 1/1
‫ م‬9:15 2021/‫‏‬9/‫‏‬11 https://t.me/USMLEWorldStep2CK UWorld STEP2 SIM Form 1

1 Item 29 of 40
2
Mark
Question Id: 6999 Previous Next
3

4 A 76-year-old Caucasian male is found wandering in the street by police and is brought to the emergency department for evaluation. He denies
5 any complaints. His past medical history is significant for hypertension and nephrolithiasis. He states that he lives alone. He does not smoke
6 cigarettes or consume alcohol. His blood pressure is 140/90 mmHg and heart rate is 95/min. Examination reveals bilateral ecchymoses on his
7 extremities and trace pedal edema. His oral mucosa is moist and his gums appear to be inflamed. The lungs are clear to auscultation and his
8 abdomen is soft and non-distended. On mental status examination, he recalls 0/3 objects after 5 minutes. Laboratory studies show:
9
Hematocrit 30%
10
11 Leukocyte count 5,500/mm3
12 Platelet count 150,000/mm3
13

14 Which of the following best explains the physical findings in this patient?
15
16  A. Myelodysplastic syndrome (4%)
17
B. Adrenal insufficiency (0%)
18

19 C. Chronic liver disease (4%)


20 D. Vasculitis (2%)
21
 E. Malnutrition (84%)
22

23 F. Elder abuse (2%)


24
25

26
Incorrect 84%
27 Correct answer  Answered correctly  17 secs
Time Spent  07/30/2021
Last Updated
28 E
29
30
Explanation
31

32
This elderly patient most likely has underlying dementia as evidenced by his poor object recall. The various findings on examination and
33
laboratory evaluation can be attributed to malnutrition, which is a common finding in elderly patients with dementia. Poor nutrition can result in
34
hypoalbuminemia, which in turn can lead to pedal edema. Since this patient’s platelet level is within normal limits, the ecchymoses on his
35
extremities are most likely caused by a combination of vitamin C and vitamin K deficiencies. In addition, his inflamed gums could be secondary to
36
vitamin C deficiency. Although further workup is required for this patient’s anemia, folate deficiency is a likely etiology.
37

38 (Choice A) Myelodysplastic syndrome (MDS) is more common in the elderly and can lead to anemia, but pedal edema would be unusual and
39 leukopenia is often present. Although bleeding and ecchymoses may occur, it is usually due to decreased platelet levels.
40
(Choice B) Adrenal insufficiency can cause anorexia in many patients and result in malnutrition. However, most patients with adrenal insufficiency
will have a low blood pressure and electrolyte abnormalities.

(Choice C) Chronic liver disease can lead to anemia, hypoalbuminemia, and coagulopathy. However, this patient has no history of alcohol abuse
and does not have any other physical findings consistent with liver disease. His underlying dementia makes malnutrition more likely.

(Choice D) Vasculitis typically causes petechiae or purpura as opposed to the larger ecchymoses seen in this patient.

(Choice F) Elder abuse should be considered given this patient’s dementia and bruising, but his other physical findings make malnutrition more
likely.

Educational objective:
Malnutrition is a common problem in the elderly, particularly in patients with dementia. Certain nutrient and vitamin deficiencies can lead to
anemia, hypoalbuminemia, and coagulopathy.

Medicine Rheumatology/Orthopedics & Sports Malnutrition


Subject System Topic

https://t.me/USMLEWorldStep2CK

REVIEW

https://www.uworld.com/ClientApp/v15/apps/qbanktestinterface/index.html#/launchtest/7281878/nbme/229400738/3/1 1/1
‫ م‬9:15 2021/‫‏‬9/‫‏‬11 https://t.me/USMLEWorldStep2CK UWorld STEP2 SIM Form 1

1 Item 30 of 40
2
Mark
Question Id: 7011 Previous Next
3

4 A 66-year-old man comes to the office due to 6 months of progressive fatigue and decreased exercise tolerance. Over the past week, he has had
5 shortness of breath with minimal tasks such as household chores. The patient has a history of gastroesophageal reflux disease and benign
6 prostatic hyperplasia. He had an episode of chest pain a year ago, but his exercise stress test was normal. The patient has a history of tobacco
7 use and does not drink alcohol. He eats a balanced diet. Blood pressure is 130/80 mm Hg and pulse is 90/min. Conjunctiva are pale.
8 Cardiopulmonary auscultation is normal. The abdomen is soft and nontender with no organomegaly. There is no peripheral edema or sensory
9 loss in the lower extremities. Complete blood count results are as follows:
10
Hemoglobin 7.5 g/dL
11
12 Mean corpuscular volume 106 fL
13 Platelets 56,000/mm3
14
Leukocytes 4,000/mm3
15
16 Peripheral blood smear shows ovalomacrocytosis and neutrophils with reduced segmentation. Serum electrolytes, renal function, and vitamin B12
17 levels are normal. Which of the following is the best next step in management of this patient?
18

19  A. Anti-intrinsic factor antibody (5%)


20
B. Blood lead level (3%)
21

22  C. Bone marrow biopsy (65%)

23 D. Erythropoietin level (1%)


24
E. Methylmalonic acid level (17%)
25

26 F. Serum protein electrophoresis (5%)


27

28

29
Incorrect 65%
30 Correct answer  Answered correctly  03 secs
Time Spent  05/08/2021
Last Updated
31 C
32

33
Explanation
34

35

36 Myelodysplastic syndrome
37
Hematopoietic stem cell neoplasm
38
Epidemiology Risk ↑: Older age, previous chemotherapy/radiation
39 May transform to acute leukemia
40
Cytopenias
Anemia: Weakness, fatigue
Manifestations Leukopenia: Infections
Thrombocytopenia: Bruising/bleeding
Hepatosplenomegaly/lymphadenopathy are rare

Dysplastic red & white blood cells* on peripheral smear


Diagnosis
Bone marrow biopsy (hypercellular marrow)

Transfusions for symptomatic cytopenias


Treatment Chemotherapy
Hematopoietic stem cell transplantation

*Dysplastic changes include ovalomacrocytosis & neutrophil hyposegmentation/hypogranulation.

This patient has symptomatic anemia from myelodysplastic syndrome (MDS). MDS is a hematopoietic stem cell neoplasm that interferes
with the production of functional blood cell lines and leads to dysplasia and cytopenias. Risk is highest in those with advanced age or previous
chemotherapy or radiation therapy. Patients are often asymptomatic at diagnosis, but manifestations of anemia (weakness, fatigue, dyspnea on
exertion) or granulocytopenia (infection) are common. Physical examination usually shows minimal findings other than conjunctival pallor;
hepatosplenomegaly and lymphadenopathy are rarely present.

Almost all patients with MDS have normocytic or macrocytic anemia with insufficient reticulocytosis. Leukopenia with immature granulocytes and
thrombocytopenia are also common. Peripheral blood smear is notable for dysplastic erythrocytes (eg, ovalomacrocytosis) and granulocytes
(hyposegmentation, hypogranulation). Bone marrow biopsy is required for diagnosis and usually shows a hypercellular marrow.

(Choices A and E) Pernicious anemia is associated with anti-intrinsic factor antibodies and may lead to vitamin B12 deficiency. A common
laboratory finding in B12 deficiency is increased methylmalonic acid. Although vitamin B12 deficiency can cause symptomatic macrocytic anemia,
peripheral blood smear shows hypersegmented (not hyposegmented) neutrophils. In addition, this patient's vitamin B12 level is normal.

(Choice B) Lead interferes with hemoglobin production and may lead to hemolytic anemia (acute exposure) or normocytic/microcytic anemia
(chronic exposure); this patient has pancytopenia with macrocytic anemia, making lead toxicity unlikely.

(Choice D) Erythropoietin stimulates bone marrow erythropoiesis. Although erythropoietin is often used in the treatment of MDS-induced anemia,
patients first require a bone marrow biopsy for diagnosis.

(Choice F) Serum protein electrophoreses can identify monoclonal proteins in diseases such as multiple myeloma. Multiple myeloma often
causes normocytic anemia but is not typically associated with pancytopenia. In addition, peripheral smear usually shows rouleaux formation (not
dysplasia of erythrocytes and granulocytes).

Educational objective:
Myelodysplastic syndrome is a neoplastic stem cell disorder seen primarily in older adults and those who have had previous chemotherapy or
radiation therapy. Manifestations typically include macrocytic anemia, leukopenia, and thrombocytopenia. Peripheral blood smear shows signs of
dysplasia, including oval macrocytes and hyposegmented/hypogranulated neutrophils. Bone marrow biopsy is required for diagnosis.

Medicine Hematology & Oncology Myelodysplastic syndrome


Subject System Topic

https://t.me/USMLEWorldStep2CK

REVIEW

https://www.uworld.com/ClientApp/v15/apps/qbanktestinterface/index.html#/launchtest/7281878/nbme/229400738/3/1 1/1
‫ م‬9:16 2021/‫‏‬9/‫‏‬11 https://t.me/USMLEWorldStep2CK UWorld STEP2 SIM Form 1

1 Item 31 of 40
2
Mark
Question Id: 7016 Previous Next
3

4 A 46-year-old man with a history of seizure disorder is brought to the hospital after recurrent seizures. In the emergency department, the patient
5 opens his eyes to painful stimuli but does not significantly respond to questions. There is evidence of urinary incontinence. His home medications
6 include phenytoin and valproic acid. Blood pressure is 110/78 mm Hg, and pulse is 100/min. Laboratory results are as follows:
7
Sodium 146 mEq/L
8

9 Potassium 5.4 mEq/L


10
Chloride 105 mEq/L
11
12
Bicarbonate 20 mEq/L

13 Urea nitrogen 52 mg/dL


14
Creatinine 2.4 mg/dL
15
16 Calcium 8.6 mg/dL
17 Creatine phosphokinase 7,000 U/L
18

19 Total bilirubin 0.2 mg/dL


20
Alkaline phosphatase 110 U/L
21

22
Aspartate aminotransferase (SGOT) 230 U/L

23 Alanine aminotransferase (SGPT) 400 U/L


24
25
The patient's initial ECG is normal, and his mental status improves over the next hour. However, 4 hours later, he says he is dizzy and feels like
he is "going to pass out." Repeat blood pressure is 92/62 mm Hg, and pulse is 40/min. ECG shows low-amplitude P waves and high-degree
26
atrioventricular block. Which of the following is the most appropriate next step in management?
27

28
 A. Insertion of a transvenous pacemaker (29%)
29
30  B. Intravenous calcium gluconate (56%)
31 C. Intravenous dobutamine (8%)
32
D. Intravenous magnesium sulfate (4%)
33
34 E. Observation pending repeat laboratory studies (1%)
35

36

37
Incorrect 56% 03 secs 03/26/2021
38 Correct answer
B
 Answered correctly  Time Spent  Last Updated
39

40

Explanation

This patient's seizure likely triggered rhabdomyolysis as indicated by the presence of kidney injury, hyperkalemia, elevated creatine
phosphokinase, and increased aminotransferases. Although he transiently improved, the patient subsequently developed hypotension and
bradycardia from high-degree atrioventricular (AV) block. Because an ECG was normal 4 hours earlier, these new manifestations most likely
indicate hyperkalemic emergency due to continued release of potassium from damaged muscle tissue.

Hyperkalemic emergency occurs when increased levels of extracellular potassium (usually ≥6.5 mEq/L) alter the resting membrane potential of
cells, leading to reduced depolarization of muscle and subsequent weakness, paralysis, ECG changes, and/or cardiac arrhythmia (eg, bundle-
branch block, AV block). ECG abnormalities generally proceed in a stepwise manner as follows: tall, peaked T waves → progressive lengthening
of the PR interval → absent/flattened (ie, low amplitude) P waves → QRS complex widening → sine wave pattern.

Patients with hyperkalemic emergency require urgent treatment to prevent death. This typically includes administration of intravenous calcium
(eg, calcium gluconate), which counterbalances the membrane-depressive effect of hyperkalemia, and insulin plus glucose, which moves
potassium into cells due to increased activity of the Na+/K+-ATPase in skeletal muscle. These urgent treatments provide time for definitive removal
of potassium via the stool (eg, patiromer), urine (eg, furosemide), and/or blood (eg, hemodialysis).

(Choices A and C) Patients with complete heart block who are unstable (eg, hypotension) are initially managed with atropine. Transcutaneous
pacing is then initiated for those who remain unstable; transvenous pacing requires central venous access and is generally reserved for those who
require prolonged pacing until a permanent pacemaker can be placed. Patients who continue to have hypotension with transcutaneous pacing are
usually treated with dopamine; when cardiogenic shock (eg, severe heart failure) is present, dobutamine may be used. This patient would require
atropine and transcutaneous pacing first.

(Choice D) Magnesium sulfate is used for torsade de pointes, a polymorphic ventricular tachycardia. This patient has bradycardia from complete
heart block; magnesium sulfate would not be helpful.

(Choice E) This patient has serious ECG changes consistent with hyperkalemia and requires immediate intervention to prevent death.

Educational objective:
Hyperkalemic emergency is marked by weakness, paralysis, ECG changes, and/or cardiac arrhythmia. It requires urgent treatment with calcium
gluconate and/or insulin plus dextrose to stabilize the cardiac membrane and move potassium into cells prior to initiating therapy to definitively
remove potassium from the body.

Medicine Cardiovascular System Hyperkalemia


Subject System Topic

https://t.me/USMLEWorldStep2CK

REVIEW

https://www.uworld.com/ClientApp/v15/apps/qbanktestinterface/index.html#/launchtest/7281878/nbme/229400738/3/1 1/1
‫ م‬9:16 2021/‫‏‬9/‫‏‬11 https://t.me/USMLEWorldStep2CK UWorld STEP2 SIM Form 1

1 Item 32 of 40
2
Mark
Question Id: 7017 Previous Next
3

4 A 42-year-old woman comes to the emergency department due to severe headaches and sweating. For the last few months, she has had
5 intermittent migraine headaches that have not responded to a number of abortive medications. The patient was recently started on propranolol in
6 an attempt to prevent her headaches. Additional symptoms include a recent 7-kg (15-lb) weight loss. Medical history is notable for hypertension.
7 The patient drinks alcohol only on social occasions and does not use tobacco or illicit drugs. Temperature is 37 C (98.6 F), blood pressure is
8 240/140 mm Hg, and pulse is 110/min. She generally appears pale and diaphoretic. Cardiopulmonary examination shows a regular tachycardia
9 but is otherwise normal. Her abdomen is soft and nondistended with no palpable masses. There is no appreciable edema in the extremities.
10 Laboratory results are as follows:
11
Sodium 144 mEq/L
12
13 Potassium 3.8 mEq/L
14 Glucose (fasting) 127 mg/dL
15
Urea nitrogen 12 mg/dL
16

17 Creatinine 1.2 mg/dL


18
TSH 2.5 mIU/L
19
Urinalysis (automated) 1+ protein
20

21 Which of the following is the most likely cause of this patient's condition?
22

23  A. Adrenal cortical disease (9%)


24
 B. Adrenal medullary disease (80%)
25

26 C. Aortic disease (0%)


27 D. Parathyroid disease (0%)
28
E. Renal parenchymal disease (1%)
29
30 F. Renal vascular disease (6%)
31
G. Thyroid disease (2%)
32

33
34

35 Incorrect 80%
36
Correct answer  Answered correctly  03 secs
Time Spent  08/01/2021
Last Updated
B
37

38

39 Explanation
40

Pheochromocytoma
Classic triad: episodic headache, sweating & tachycardia
Indications for testing Resistant HTN or HTN accompanied by unexplained ↑ glucose
Family history or familial syndrome (eg, MEN2, NF1, VHL)

Urine or plasma metanephrine levels


Diagnostic approach
Confirmatory abdominal imaging for ↑ metanephrines

Notable features 10% bilateral, 10% extraadrenal, 10% malignant

Preoperative alpha blockade prior to beta blockade


Management
Laparoscopic or open surgical resection

HTN = hypertension; MEN2 = multiple endocrine neoplasia type 2; NF1 = neurofibromatosis type 1; VHL = von Hippel-Lindau syndrome.

This patient with severe hypertension and the classic triad of paroxysmal headaches, tachycardia, and sweating most likely has
pheochromocytoma. Pheochromocytomas are catecholamine-secreting tumors arising from the chromaffin cells of the adrenal medulla
(similar tumors in nonadrenal tissues are termed paraganglionomas). The diagnosis can be confirmed by demonstrating elevated urinary and
plasma catecholamine and metanephrine levels.

Pheochromocytomas can cause severe sustained or intermittent hypertension that can be controlled by combined alpha and beta adrenergic
blockade. However, alpha blockers (eg, phenoxybenzamine) should be initiated first because administration of beta blockers (eg, propranolol)
alone can cause unopposed alpha adrenergic effects, leading to severe peripheral vasoconstriction and a paradoxical rise in blood pressure.
Definitive treatment of pheochromocytoma in most patients is with laparoscopic adrenalectomy.

(Choice A) Hypertension due to excess mineralocorticoid or glucocorticoid production by the adrenal cortex is unlikely to present as severe
hypertension and sweating or to cause paroxysmal headaches. Additional findings related to excess corticosteroid production, including
hypokalemia (hyperaldosteronism) and Cushingoid features, would be expected.

(Choice C) Coarctation of the aorta is an uncommon cause of hypertension that usually presents in children or young adults. It causes differential
blood pressure measurements between the upper and lower extremities, not paroxysms of severe headache, diaphoresis, and hypertension.

(Choice D) Primary hyperparathyroidism may present with constipation, nausea, polyuria, and muscle weakness, but it would not explain severe
hypertension.

(Choices E) Renal parenchymal disease (eg, diabetic nephropathy) can lead to chronic kidney disease, often with secondary hypertension.
However, significant renal parenchymal disease is unlikely in the absence of elevated creatinine. This patient's mild proteinuria is likely due to
severe hypertension.

(Choice F) Renal artery stenosis (RAS) can cause secondary hypertension and may result from atherosclerotic disease or fibromuscular
dysplasia. Atherosclerotic RAS is unlikely in this relatively young woman and fibromuscular dysplasia would not explain paroxysmal headaches
and severe hypertension triggered by initiation of propranolol.

(Choice G) Elevated blood pressure can occur in thyrotoxicosis (as well as hypothyroidism), but thyroid dysfunction is unlikely given the normal
TSH. Weight loss can occur with both thyrotoxicosis and pheochromocytoma (due to an alteration in the basal metabolic rate).

Educational objective:
Pheochromocytomas are catecholamine-secreting tumors of the adrenal medulla. They can cause severe sustained or intermittent hypertension.
The diagnosis is confirmed with urinary and plasma catecholamine and metanephrine levels. Hypertension is controlled by combined alpha and
beta adrenergic blockade. Definitive treatment is with laparoscopic adrenalectomy.

References
Pheochromocytoma and paraganglioma: an endocrine society clinical practice guideline.

(http://www.ncbi.nlm.nih.gov/pubmed/24893135)

Medicine Endocrine, Diabetes & Metabolism Pheochromocytoma


Subject System Topic

https://t.me/USMLEWorldStep2CK

REVIEW

https://www.uworld.com/ClientApp/v15/apps/qbanktestinterface/index.html#/launchtest/7281878/nbme/229400738/3/1 1/1
‫ م‬9:16 2021/‫‏‬9/‫‏‬11 https://t.me/USMLEWorldStep2CK UWorld STEP2 SIM Form 1

1 Item 33 of 40
2
Mark
Question Id: 6879 Previous Next
3

4 A 29-year-old woman comes to the office due to a 2-month history of abdominal pain, diarrhea, and low-grade fever. The patient has 2 or 3
5 nonbloody bowel movements daily. Her abdominal pain is crampy and intermittent and occasionally causes her to wake up from sleep. For the
6 past week, she has been passing gas and malodorous discharge from the vagina. The patient has lost 4 kg (8.8 lb) over the past few months.
7 She was previously healthy. The patient works as a flight attendant and travels internationally. She has had a new male sexual partner for 3
8 months, and her only medication is oral contraceptives. The patient does not use tobacco, alcohol, or illicit drugs. Temperature is 38.3 C (100.9
9 F), blood pressure is 110/70 mm Hg, pulse is 90/min, and respirations are 14/min. Physical examination shows lower abdominal tenderness
10 without guarding or rebound. Which of the following is the most likely diagnosis for this patient?
11
 A. Crohn disease (68%)
12
13 B. Diverticulitis (1%)
14
C. Endometriosis (0%)
15
16 D. Ovarian cancer (0%)
17 E. Pelvic inflammatory disease (24%)
18
F. Tuberculosis (0%)
19
20 G. Ulcerative colitis (3%)
21

22

23
68%
24 Correct  Answered correctly  02 secs
Time Spent  09/06/2021
Last Updated
25

26

27
Explanation
28

29
30
31

32

33
34

35

36

37

38

39

40

This patient with chronic diarrhea, abdominal pain, fever, and symptoms of an enterovaginal fistula (eg, passing gas or feces from the vagina)
likely has Crohn disease (CD). The disease usually presents in patients age 15-40. Other frequent gastrointestinal manifestations include
weight loss, microscopic bleeding, stricture formation, and malabsorption. CD can involve any part of the gastrointestinal tract from mouth to
anus. Fistulas occur due to transmural bowel inflammation that predisposes to the development of sinus tracts, which then penetrate other hollow
organs (eg, urinary bladder, vagina). The process is slow and does not frequently result in peritonitis. Other frequent intraabdominal
complications include abscess and phlegmon formation.

Common extraintestinal symptoms include arthritis, uveitis, scleritis, and erythema nodosum. Laboratory findings are similar to those of other
chronic inflammatory diseases and can include leukocytosis, iron deficiency anemia, reactive thrombocytosis, and elevated inflammatory markers.
Diagnosis is confirmed with endoscopic or radiographic studies, and treatment involves immunosuppressive therapy.

(Choice B) Diverticulitis presents more acutely with abdominal pain and constipation or diarrhea. Although it can be complicated by fistula
formation, diverticulitis most commonly occurs in patients age >60. Weight loss is not a prominent feature.

(Choice C) Endometriosis usually affects women age 20-39 but is more frequently characterized by dysmenorrhea, pelvic pain, and dyspareunia.
Although endometriosis can cause diarrhea, it is not associated with weight loss, and enterovaginal fistulas are rare.

(Choice D) Ovarian cancer can lead to chronic abdominal pain and weight loss but is also generally accompanied by symptoms such as bloating,
early satiety, and urinary complications. Diarrhea and enterovaginal fistulas are not common presenting symptoms.

(Choice E) Pelvic inflammatory disease is characterized by lower abdominal pain, dyspareunia, and abnormal uterine bleeding. Diarrhea and
fistula formation are usually not seen.

(Choice F) Gastrointestinal tuberculosis can cause fever, weight loss, abdominal pain, and diarrhea; however, patients usually have either
concomitant or prior pulmonary tuberculosis. In addition, patients frequently have ascites (rare in CD) and a palpable abdominal mass.

(Choice G) Ulcerative colitis more commonly presents with bloody (rather than nonbloody) diarrhea, tenesmus, and fecal incontinence. Toxic
megacolon, but not fistula, is a common complication.

Educational objective:
Crohn disease is characterized by fevers, chronic diarrhea, abdominal pain, and weight loss. The formation of fistulas is a common complication;
this includes enterovaginal fistulas, which are characterized by the passage of gas and feces through the vagina.

References
Diagnosis and management of Crohn's disease.

(http://www.ncbi.nlm.nih.gov/pubmed/22230271)
Epidemiology and clinical course of Crohn's disease: results from observational studies.

(http://www.ncbi.nlm.nih.gov/pubmed/22553396)

Obstetrics & Gynecology Gastrointestinal & Nutrition Inflammatory bowel disease


Subject System Topic

https://t.me/USMLEWorldStep2CK

REVIEW

https://www.uworld.com/ClientApp/v15/apps/qbanktestinterface/index.html#/launchtest/7281878/nbme/229400738/3/1 1/1
‫ م‬9:16 2021/‫‏‬9/‫‏‬11 https://t.me/USMLEWorldStep2CK UWorld STEP2 SIM Form 1

1 Item 34 of 40
2
Mark
Question Id: 6885 Previous Next
3

4 A 3-year-old boy is brought to the office by his mother due to a rash. Three days ago the patient developed a few red papules on his chin; they
5 subsequently evolved into pustules and today developed an overlying crust. Several new pustules have appeared at the corner of his nose. He
6 has had no fever. There are several children with a similar rash at his day care facility. The patient has no chronic medical conditions and takes
7 no medications. Vaccinations are up to date. Vital signs are normal. On physical examination, there are 5 erythematous papules covered with a
8 thick, yellow crust on the chin and 2 pustules on the right nasal ala. The remainder of the examination is normal. This patient is at greatest risk of
9 developing which of the following complications?
10
A. Encephalitis (6%)
11
12  B. Glomerulonephritis (84%)
13
C. Myocarditis (5%)
14

15  D. Thyroiditis (0%)
16
E. Uveitis (3%)
17
18

19
20
Incorrect 84%
21
Correct answer
B
 Answered correctly  04 secs
Time Spent  06/15/2021
Last Updated

22

23

24 Explanation
25

26
Nonbullous impetigo
27

28 Staphylococcus aureus (most common)


Microbiology
29
Group A Streptococcus (S pyogenes)

30 Papules & pustules with honey-crusted lesions


31 Clinical features May be painful or itchy
32 Most commonly involves face or extremities in young children
33
Topical antibiotics (eg, mupirocin) if localized
34 Treatment
Oral antibiotics (eg, cephalexin) if extensive
35

36 Complications Poststreptococcal glomerulonephritis


37
Impetigo classically presents on the face or extremities of young children as erythematous papules covered by a thick, golden crust. The
38
papules may progress to pustules or vesicles, which then rupture, releasing a characteristic golden crust. The lesions are often painful but are
39
not typically pruritic. The most common cause of impetigo is Staphylococcus aureus, followed by group A Streptococcus. Diagnosis is usually
40 clinical, and treatment for localized disease is topical antibiotic therapy.

Though rare, a potential complication of group A streptococcal impetigo is acute poststreptococcal glomerulonephritis (APSGN), an immune
complex–mediated disease that develops 1-3 weeks after initial infection. Treatment of impetigo does not prevent the development of APSGN.
Symptoms of APSGN range from asymptomatic, microscopic hematuria to gross hematuria with edema and hypertension.

(Choice A) Patients with Coxsackie virus, which can present with hand-foot-mouth disease, can potentially develop encephalitis. Hand-foot-
mouth disease classically presents with papules or vesicles on the palms, soles, and posterior oropharynx, not seen in this patient.

(Choice C) Myocarditis is most commonly a complication of a viral infection, such as parvovirus. The typical rash of parvovirus (ie, erythema
infectiosum or fifth disease) includes an erythematous malar, or "slapped cheek," rash and a lacy, reticulated rash on the body. Acute rheumatic
fever may also present with myocarditis but is a postinfectious complication of group A Streptococcus pharyngitis, not impetigo; this may be due to
difference in M protein patterns between the streptococcocal strains that cause pharyngitis and those that cause skin infections (including
impetigo).

(Choice D) Dermatitis herpetiformis is a pruritic, vesicular rash classically found on the forearms and knees and is associated with celiac
disease. Risk of thyroiditis in celiac disease patients is increased. This patient has a classic rash of impetigo.

(Choice E) Uveitis is often associated with systemic disease (eg, juvenile idiopathic arthritis) or infections, such as herpes simplex or zoster. This
patient has no chronic medical conditions suggestive of an underlying disorder, and patients with herpes virus that are at risk of uveitis have a
painful, vesicular rash near the eye.

Educational objective:
Impetigo presents as erythematous papules and pustules covered with a honey-colored crust, typically on the face or upper extremities. A
potential sequelae of group A streptococcal impetigo is acute poststreptococcal glomerulonephritis.

References
Impetigo: diagnosis and treatment.

(http://www.ncbi.nlm.nih.gov/pubmed/25250996)

Pediatrics Infectious Diseases Poststreptococcal Glomerulonephritis


Subject System Topic

https://t.me/USMLEWorldStep2CK

REVIEW

https://www.uworld.com/ClientApp/v15/apps/qbanktestinterface/index.html#/launchtest/7281878/nbme/229400738/3/1 1/1
‫ م‬9:16 2021/‫‏‬9/‫‏‬11 https://t.me/USMLEWorldStep2CK UWorld STEP2 SIM Form 1

1 Item 35 of 40
2
Mark
Question Id: 6887 Previous Next
3

4 A 65-year-old man comes to the emergency department due to increasing shortness of breath for the last 2 days. He also has had a worsening
5 cough with yellowish sputum production. He can walk half a mile without becoming short of breath in his usual state of health. The patient's
6 regular medications include albuterol and ipratropium inhalers. He has a 40-pack-year smoking history. His blood pressure is 160/90 mm Hg and
7 pulse is 110/min and regular. Physical examination reveals decreased breath sounds bilaterally with prolonged expiration. Heart sounds are
8 distant. The abdomen is scaphoid, soft, and nontender. ECG shows no acute changes. His chest x-ray is shown below.
9

10
11
12
13

14

15
16

17
18

19
20

21

22

23

24
25

26 Which of the following would most likely decrease this patient's length of hospitalization?
27
A. ACE inhibitor (1%)
28

29 B. Alpha-adrenergic antagonist (0%)


30
C. Beta-adrenergic antagonist (1%)
31

32  D. Glucocorticoids (90%)
33 E. Leukotriene receptor antagonist (3%)
34
 F. Loop diuretics (1%)
35

36

37

38 Incorrect 90%
39
Correct answer
D
 Answered correctly  09 secs
Time Spent  08/26/2021
Last Updated

40

Explanation

Acute exacerbation of chronic obstructive pulmonary disease


Increased dyspnea
Cardinal
Increased cough (more frequent or severe)
symptoms
Sputum production (change in color or volume)

Diagnostic Chest x-ray: Hyperinflation


testing ABG: Hypoxia, CO2 retention (chronic &/or acute)

Oxygen (target SpO2 of 88%-92%)


Inhaled bronchodilators
Systemic glucocorticoids
Management Antibiotics if ≥2 cardinal symptoms
Oseltamivir if evidence of influenza
NPPV if ventilatory failure
Tracheal intubation if NPPV failed or contraindicated

ABG = arterial blood gas; NPPV = noninvasive positive-pressure ventilation; SpO2 = peripheral oxygen saturation.

Given this patient's extensive history of smoking, his increased pulmonary symptoms over the last 2 days likely represent an acute exacerbation
of chronic obstructive pulmonary disease (COPD). Characteristics of a COPD exacerbation include increased cough, increased sputum
production, and increased shortness of breath. His chest x-ray shows hyperinflation, as evidenced by flattening of his diaphragm.

The patient should continue to use his albuterol and ipratropium inhalers as they will help improve his bronchoconstriction. In addition, systemic
glucocorticoids should be administered as they have been shown to improve pulmonary function and decrease length of hospitalization in
patients with COPD exacerbation.

(Choice A) An ACE inhibitor could be considered in the treatment of this patient's hypertension, but it will not quicken recovery from his current
COPD exacerbation.

(Choice B) Alpha-adrenergic antagonists are used in the treatment of hypertension and benign prostatic hyperplasia, not COPD exacerbation.

(Choice C) Beta-adrenergic antagonists could further worsen his bronchospasm and therefore have no role in the treatment of COPD
exacerbation.

(Choice E) Leukotriene antagonists help to treat the allergic responses in asthma that lead to bronchospasm. They are not used in the treatment
of COPD exacerbation as bronchospasm associated with COPD is usually triggered by infection.

(Choice F) Loop diuretics would be beneficial if this patient's respiratory symptoms were due to pulmonary edema, but COPD is more likely given
his extensive smoking history, the lack of crackles on lung auscultation, and the hyperinflation with no evidence of vascular congestion seen on
chest x-ray.

Educational objective:
Chronic obstructive pulmonary disease (COPD) exacerbation should be diagnosed when a patient with underlying COPD presents with increased
shortness of breath, cough, or sputum production. Patients with a COPD exacerbation should be given systemic glucocorticoids as these have
been shown to improve lung function and promote a shorter hospital stay.

Medicine Pulmonary & Critical Care COPD


Subject System Topic

https://t.me/USMLEWorldStep2CK

REVIEW

https://www.uworld.com/ClientApp/v15/apps/qbanktestinterface/index.html#/launchtest/7281878/nbme/229400738/3/1 1/1
‫ م‬9:16 2021/‫‏‬9/‫‏‬11 https://t.me/USMLEWorldStep2CK UWorld STEP2 SIM Form 1

1 Item 36 of 40
2
Mark
Question Id: 6928 Previous Next
3

4 A 65-year-old obese female is brought to the hospital by her husband who reports a one day history of decreased oral intake and confusion. Her
5 medical history is significant for a uterine adenosarcoma for which she underwent debulking surgery one month ago. Bilateral ureteral stents were
6 also placed at that time because of obstructive uropathy. Last week she received chemotherapy and a packed RBC transfusion due to a falling
7 hematocrit. Today in the ER, her blood pressure is 120/80 mmHg, her heart rate is 120/min and regular, and her temperature is 38.8°C (102.0°F).
8 Laboratory findings are as follows:
9
Hemoglobin 9.1 g/dL
10
11 WBC count 600/mm3
12
Platelets 90,000/mm3
13

14
BUN 24 mg/dL

15 Creatinine 1.4 mg/dL


16

17 Which of the following is the most likely cause of her current mental status?
18

19  A. Tumor lysis syndrome (18%)


20 B. Tumor expansion (2%)
21
C. Delayed transfusion reaction (15%)
22

23  D. Bacterial infection (51%)


24
E. Drug fever (12%)
25

26

27

28 Incorrect 51% 02 secs 07/30/2021


29
Correct answer
D
 Answered correctly  Time Spent  Last Updated

30
31

32 Explanation
33
34 Infection is the most common complication of neutropenia, and bacteria are the most common cause of infection in newly neutropenic patients.
35 The patient described has just begun chemotherapy and now has a leukocyte count of 600/mm3. Patients with neutropenia do not manifest the
36 same signs and symptoms as immunocompetent patients in response to sepsis. For example, a neutropenic patient with sepsis may not mount a
37 fever because inflammatory cytokines are not produced in normal quantities due to leukopenia. This patient needs empiric antibiotics and a
38
thorough medical investigation for an infection source.

39
(Choice A) Tumor lysis syndrome is marked by hypocalcemia, hyperphosphatemia, hyperkalemia and elevated uric acid levels causing nausea
40
and vomiting, bowel disturbances, low urine output, acute renal insufficiency, seizures, tetany, and/or arrhythmias.

(Choice B) Tumor expansion would be unlikely to cause symptoms so soon after a debulking procedure. Additionally, tumor expansion alone
would not explain this patient’s fever, tachycardia and leukopenia.

(Choice C) Delayed transfusion reactions typically occur following repeated transfusions with blood that is not properly matched to a patient’s own
erythrocyte antigens. The result is hemolytic anemia with secondary jaundice, lactic acidosis and renal insufficiency. Confusion and decreased
mental status are unlikely.

(Choice E) Drug fever is typically a solitary finding without associated symptoms.

Educational objective:
Chemotherapy can cause neutropenia, putting patients at increased risk for systemic infections and sepsis. Neutropenic patients do not mount a
normal immune response to infection, thus, signs and symptoms of sepsis may be subtler than in an immunocompetent patient.

Medicine Hematology & Oncology Febrile neutropenia


Subject System Topic

https://t.me/USMLEWorldStep2CK

REVIEW

https://www.uworld.com/ClientApp/v15/apps/qbanktestinterface/index.html#/launchtest/7281878/nbme/229400738/3/1 1/1
‫ م‬9:16 2021/‫‏‬9/‫‏‬11 https://t.me/USMLEWorldStep2CK UWorld STEP2 SIM Form 1

1 Item 37 of 40
2
Mark
Question Id: 6901 Previous Next
3

5
The response options for the next 2 items are the same. Select one answer for each item in the set.

6
A 24-year-old woman is brought to the emergency department due to erratic behavior. She is accompanied by her boyfriend, who called the
7 police after the patient became physically aggressive during an argument. He says, "I told her that I didn't want to move in together and she got
8 really angry. At first, she just locked herself in her bedroom, but after 15 minutes she came running out, screaming about how I was trying to kill
9 her. Then she started hitting me." The patient's boyfriend states that they argue daily, and when arguments become especially prolonged, she
10 sometimes "freezes" and he has to repeat her name as many as 30 times before she "snaps out of it." He says, "In just a few minutes, she can go
11 from calling me the 'best boyfriend in the world' to 'the worst.'" He adds that after an argument with her parents during a family dinner a few
12 months ago, the patient thought her food had been poisoned; however, by the time they returned to her apartment later that night she had let go of
13 this belief. The patient appears angry and declines to answer most questions, although at one point she shouts, "I'm not the one that needs to be
14 on trial here. He's trying to get rid of me!" Physical examination is unremarkable.
15
Item 1 of 2
16

17
Which of the following is the most likely diagnosis for this patient with psychosis?

18
 A. Borderline personality disorder (83%)
19
20 B. Brief psychotic disorder (1%)
21
C. Cushing syndrome (0%)
22

23
D. Drug intoxication (4%)

24 E. Hyperthyroidism (0%)
25
F. Hypothyroidism (0%)
26

27 G. Paranoid personality disorder (3%)


28
H. Schizoid personality disorder (0%)
29
30 I. Schizophreniform disorder (5%)
31 J. Schizotypal personality disorder (0%)
32

33
34
83%
35

36
Correct  Answered correctly  04 secs
Time Spent  05/13/2021
Last Updated

37

38
Explanation
39

40

Borderline personality disorder

Pervasive pattern of behavior beginning at early adulthood; ≥5 of the following:


Unstable self-image
Feelings of emptiness
Unstable relationships
Abandonment fears
Clinical features
Mood instability (intense reactivity lasting hours to days)
Inappropriate anger
Transient paranoia or dissociation
Impulsivity (eg, substance abuse, binge eating)
Recurrent suicidality, self-injury (eg, cutting)

Psychotherapy
Treatment
Adjunctive pharmacotherapy for mood instability, transient psychosis, or comorbid disorders

This patient's chaotic relationships, unstable moods, intense anger, and impulsivity are characteristic of borderline personality disorder (BPD).
These patients often exhibit angry, threatening behaviors when they fear abandonment, which may include aggressive outbursts and suicidal
threats or attempts. This patient's rapid shifts in perception of her boyfriend between being the "best" and the "worst" is an example of splitting, a
defense mechanism commonly used by patients with BPD.

In addition to these features, patients with BPD may experience transient paranoid ideation during periods of severe stress (eg, patient briefly
believes boyfriend is trying to kill her or that she has been poisoned). These events have also been referred to as "micropsychotic episodes" to
emphasize their transient (minutes to hours) nature and to distinguish them from the firmly held, fixed delusional beliefs that characterize primary
psychotic disorders. Stress-related dissociative symptoms (eg, depersonalization) may also occur.

(Choice B) Brief psychotic disorder is characterized by sudden onset of psychotic symptoms (eg, delusions, hallucinations, disorganized
speech/behavior) lasting ≥1 day but <1 month. This patient's psychotic symptoms last <1 day and are better explained by BPD.

(Choice C) Patients with Cushing syndrome often become depressed, and a small percentage develop manic/psychotic symptoms. Classic
associated findings include abdominal obesity, striae, moon facies, and a dorsocervical fat pad (ie, "buffalo hump").

(Choice D) Many street drugs (eg, cocaine, amphetamines, phencyclidine) can cause psychotic symptoms. However, this patient's symptoms
consistently occur in the context of interpersonal conflict and relationship difficulties, making BPD more likely.

(Choices E and F) Psychiatric symptoms of hyperthyroidism can include anxiety, mania, and psychosis; and hypothyroidism can present with
depression and psychosis (in severe cases). This patient does not exhibit any physical manifestations of hyperthyroidism (eg, heat intolerance,
weight loss, tremor, hypertension, tachycardia, exophthalmos) or hypothyroidism (eg, fatigue, weight gain, cold intolerance, constipation, dry skin).

(Choice G) Paranoid personality disorder involves a pervasive pattern of suspiciousness and mistrust of others. This patient's paranoia is not
pervasive but rather fairly short-lived and triggered by intense, interpersonal conflict, making it more consistent with BPD.

(Choice H) Schizoid personality disorder is characterized by a lifelong pattern of detachment from social relationships and restricted range of
emotions. Patients with schizoid personality do not typically experience delusions or dissociative symptoms.

(Choice I) Schizophreniform disorder is characterized by psychotic symptoms that last 1-6 months.

(Choice J) Schizotypal personality disorder patients are typically "odd," holding magical beliefs and strange (but not overtly psychotic) thought
processes. This patient's chaotic relationships, mood lability, and impulsivity make BPD much more likely.

Educational objective:
Borderline personality disorder involves a pervasive pattern of unstable, chaotic relationships; mood instability; abandonment fears; excessive
anger; and impulsivity. Transient paranoia and psychosis may occur and last from minutes to hours.

References
Psychotic symptoms in patients with borderline personality disorder: prevalence and clinical management.

(http://www.ncbi.nlm.nih.gov/pubmed/23168909)

Psychiatry Psychiatric/Behavioral & Substance Abuse Personality disorders


Subject System Topic

https://t.me/USMLEWorldStep2CK

REVIEW

https://www.uworld.com/ClientApp/v15/apps/qbanktestinterface/index.html#/launchtest/7281878/nbme/229400738/3/1 1/1
‫ م‬9:17 2021/‫‏‬9/‫‏‬11 https://t.me/USMLEWorldStep2CK UWorld STEP2 SIM Form 1

1 Item 38 of 40
2
Mark
Question Id: 6878 Previous Next
3

4 Item 2 of 2
5
A 69-year-old man is brought to the office by his wife, who is concerned that he might be depressed. She reports that he has had little motivation
6
and poor energy for the last month. The patient has gained weight and sleeps 12 hours a day. His wife says that he has also become very
7
isolated and suspicious, refusing to let anyone into the apartment. The patient has accused the landlord of tampering with the thermostat and
8
purposely making him cold. His wife recently heard him shouting at the landlord even though the landlord was not there. The patient has no
9
recollection of this. He says that his muscles have been more stiff than usual and that he has been constipated recently. Which of the following is
10 the most likely diagnosis for this patient with psychosis?
11
12 A. Borderline personality disorder (0%)
13
B. Brief psychotic disorder (5%)
14

15 C. Cushing syndrome (1%)


16 D. Drug intoxication (6%)
17
E. Hyperthyroidism (0%)
18

19  F. Hypothyroidism (79%)
20
G. Paranoid personality disorder (1%)
21

22 H. Schizoid personality disorder (0%)


23
I. Schizophreniform disorder (4%)
24
25
J. Schizotypal personality disorder (0%)

26

27

28
79%
29 Correct  Answered correctly  02 secs
Time Spent  05/13/2021
Last Updated
30
31

32
Explanation
33
34 This patient has classic symptoms of hypothyroidism (eg, fatigue, weight gain, cold intolerance, myalgia, constipation). Depressed mood is
35 also a common finding. In severe cases, patients may become paranoid and experience hallucinations (ie, psychosis); this is sometimes referred
36 to as myxedema madness.
37
A diagnosis of primary hypothyroidism is made when TSH is high and free thyroxine (T4) is low; the condition is treated with levothyroxine, and
38
neuropsychiatric symptoms typically improve as the hypothyroid state resolves.
39
(Choice A) Borderline personality disorder is characterized by a persistent pattern of chaotic relationships, unstable moods, intense anger, and
40
impulsivity.

(Choices B and I) Brief psychotic disorder is characterized by psychotic symptoms lasting 1 day to 1 month, but this would not explain this
patient's mood and physical symptoms. Schizophreniform disorder is characterized by psychotic symptoms that last 1-6 months.

(Choice C) Patients with Cushing syndrome often become depressed, and a small percentage develop manic or psychotic symptoms. Classic
associated findings include abdominal obesity, striae, moon facies, and a dorsocervical fat pad (ie, "buffalo hump").

(Choice D) Drug intoxication would not explain this patient's month-long depression and physical symptoms.

(Choice E) Symptoms of hyperthyroidism include heat intolerance, weight loss, tremor, hypertension, tachycardia, and exophthalmos. Psychiatric
symptoms of hyperthyroidism can include anxiety, mania, and psychosis.

(Choices G, H, and J) Delusions and hallucinations are not characteristic of personality disorders. Paranoid personality disorder involves a
pervasive pattern of suspiciousness and mistrust of others. Patients with schizoid personality disorder are detached and socially withdrawn.
Schizotypal personality disorder is characterized by lifelong eccentric behavior and cognitive and perceptual distortions.

Educational objective:
New-onset psychosis in a patient with active physical symptoms requires a workup to determine the medical cause. Patients with hypothyroidism
are commonly diagnosed with depression and can also experience psychotic symptoms in severe cases.

References
Myxedema psychosis in a patient with undiagnosed Hashimoto thyroiditis.

(http://www.ncbi.nlm.nih.gov/pubmed/28055088)
Acute psychosis as an initial manifestation of hypothyroidism: a case report.

(http://www.ncbi.nlm.nih.gov/pubmed/26577152)

Psychiatry Psychiatric/Behavioral & Substance Abuse Hypothyroidism


Subject System Topic

https://t.me/USMLEWorldStep2CK

REVIEW

https://www.uworld.com/ClientApp/v15/apps/qbanktestinterface/index.html#/launchtest/7281878/nbme/229400738/3/1 1/1
‫ م‬9:17 2021/‫‏‬9/‫‏‬11 https://t.me/USMLEWorldStep2CK UWorld STEP2 SIM Form 1

1 Item 39 of 40
2
Mark
Question Id: 6927 Previous Next
3

5
The following vignette applies to the next 2 items. The items in the set must be answered in sequential order. Once you click Proceed to Next
Item, you will not be able to add or change an answer.
6

7 A 13-year-old boy is brought to the emergency department due to nonbloody, nonbilious vomiting that began yesterday. The patient has vomited
8 several times and has abdominal pain. He has had no diarrhea. Review of systems is positive for several weeks of increased thirst and urination.
9 He currently feels weak and dizzy. The patient was previously healthy. Immunizations are up to date. Temperature is 36.7 C (98 F), blood
10 pressure is 100/65 mm Hg, pulse is 124/min, respirations are 28/min, and oxygen saturation is 98% on room air. Examination indicates dry
11 mucous membranes and deep, rapid breathing. The abdomen is soft with diffuse tenderness to palpation but no rebound or guarding. The patient
12 is sleepy, but neurologic examination shows no focal deficits. Laboratory results are as follows:
13
Serum chemistry
14

15 Sodium 133 mEq/L


16
Potassium 5 mEq/L
17
18
Chloride 96 mEq/L

19 Blood urea nitrogen 20 mg/dL


20
Creatinine 0.9 mg/dL
21

22 Glucose 382 mg/dL


23

24 Venous blood gas


25 pH 7.2
26
Bicarbonate 10 mEq/L
27

28 Item 1 of 2
29
Which of the following processes is responsible for this patient's symptoms?
30
31
 A. Fatty acid breakdown in the liver (61%)
32

33
B. Glycogenolysis in the skeletal muscles (9%)

34 C. Impaired bicarbonate reabsorption in the proximal renal tubules (6%)


35
D. Impaired urine acidification in the distal renal tubules (1%)
36

37 E. Increased renal excretion of ketoacids (19%)


38
F. Peripheral lipolysis due to decreased catecholamine levels (1%)
39

40

61%
Correct  Answered correctly  05 secs
Time Spent  03/18/2021
Last Updated

Explanation

Diabetic ketoacidosis in children

Polyuria/nocturia
Polydipsia, polyphagia
Vomiting, abdominal pain
Clinical features
Weight loss, fatigue
Kussmaul respirations (deep, rapid breathing)
Dehydration

Glucose >200 mg/dL


Bicarbonate <15 mEq/L
Laboratory findings pH <7.3
Anion gap >14
Serum/urine ketones

10 mL/kg isotonic fluid bolus over 1 hour


Management
Insulin infusion + isotonic fluids with potassium

Complications Cerebral edema

This patient has hyperglycemia (glucose >200 mg/dL), an elevated anion gap (>14), and low serum bicarbonate (<15 mEq/L). These findings
are diagnostic for diabetic ketoacidosis (DKA), a common cause of anion gap metabolic acidosis in patients with type 1 diabetes mellitus.

DKA is caused by insulin deficiency that leads to increased lipolysis of peripheral fat stores secondary to high catecholamine levels (Choice F).
These fatty acids are delivered to the liver and broken down into ketones (eg, beta-hydroxybutyrate, acetoacetate). Acidosis from ketone
accumulation is responsible for many of the clinical manifestations of DKA (eg, vomiting, abdominal pain, Kussmaul respirations) and contributes
to the anion gap (Na+ − [Cl− + HCO3−]). In addition, poor aerobic metabolism causes lactic acid accumulation, which also increases the anion
gap. Other classic symptoms, such as polyuria and polydipsia, result from hyperglycemia-induced osmotic diuresis.

(Choice B) Insulin deficiency causes glycogenolysis in the skeletal muscles but not ketone formation. Although this patient likely has
glycogenolysis, it is not responsible for the clinical manifestations of DKA.

(Choice C) Impaired bicarbonate reabsorption in the proximal renal tubules occurs in renal tubular acidosis type 2, which causes a non-anion gap
metabolic acidosis. This patient's anion gap is high (133 − [96 + 10] = 27).

(Choice D) Impaired urine acidification in the distal renal tubules occurs in renal tubular acidosis type 1, which causes a non-anion gap metabolic
acidosis.

(Choice E) Increased renal excretion of ketoacids is a therapeutic result of rehydration with isotonic fluids. Excretion of these acids decreases
the anion gap, but the overall degree of acidosis does not improve until insulin is administered.

Educational objective:
High anion gap acidosis (low serum bicarbonate), ketosis, and hyperglycemia are diagnostic for diabetic ketoacidosis (DKA). Without insulin, fatty
acids break down to ketones in the liver, which leads to acidosis and the clinical manifestations of DKA (eg, vomiting, abdominal pain).

References
Diabetic ketoacidosis in the pediatric emergency department.

(http://www.ncbi.nlm.nih.gov/pubmed/23915602)

Pediatrics Endocrine, Diabetes & Metabolism Diabetic ketoacidosis


Subject System Topic

https://t.me/USMLEWorldStep2CK

REVIEW

https://www.uworld.com/ClientApp/v15/apps/qbanktestinterface/index.html#/launchtest/7281878/nbme/229400738/3/1 1/1
‫ م‬9:17 2021/‫‏‬9/‫‏‬11 https://t.me/USMLEWorldStep2CK UWorld STEP2 SIM Form 1

1 Item 40 of 40
2
Mark
Question Id: 6943 Previous Next
3

4 Item 2 of 2
5
The boy receives 10 mL/kg of normal saline over 1 hour. Currently, his blood pressure is 108/72 mm Hg, pulse is 100/min, and respirations are
6
24/min. Point-of-care urinalysis continues to show large ketones. Repeat laboratory results are as follows:
7
8 Serum chemistry
9
Sodium 134 mEq/L
10
11
Potassium 4.4 mEq/L

12 Chloride 96 mEq/L
13
Blood urea nitrogen 19 mg/dL
14

15 Creatinine 0.8 mg/dL


16 Glucose 360 mg/dL
17
18 Venous blood gas
19
pH 7.2
20

21 Bicarbonate 12 mEq/L
22
Which of the following is the best next step in management of this patient?
23

24
 A. Continuation of normal saline at a maintenance rate (4%)
25

26
B. Intravenous sodium bicarbonate (1%)

27 C. Regular insulin drip with dextrose-containing fluids (12%)


28
 D. Regular insulin drip with potassium-containing fluids (75%)
29
30 E. Subcutaneous long-acting insulin (0%)
31
F. Subcutaneous short-acting insulin with potassium supplementation (5%)
32

33
34

35 Incorrect 75%
36
Correct answer  Answered correctly  08 secs
Time Spent  03/18/2021
Last Updated
D
37

38

39 Explanation
40

Management of diabetic ketoacidosis


Rapid infusion of 0.9% normal saline
IV fluids
Add dextrose 5% when serum glucose is ≤200 mg/dL

Start continuous IV insulin infusion; hold if K <3.3 mEq/L


Switch to SQ (basal bolus) insulin for the following: able to eat, glucose <200 mg/dL, anion gap <12 mEq/L & serum
Insulin
HCO3− ≥15 mEq/L
Overlap SQ & IV insulin by 1-2 hr

Add IV K if serum K+ <5.3 mEq/L; hold if ≥5.3 mEq/L


Potassium
Nearly all patients' K+ depleted, even with hyperkalemia

Bicarbonate Consider for patients with pH ≤6.9

Consider for serum phosphate <1.0 mg/dL, cardiac dysfunction, or respiratory depression
Phosphate
Monitor serum calcium frequently

IV = intravenous; SQ = subcutaneous.

Patients with diabetic ketoacidosis (DKA) are dehydrated and require volume resuscitation with isotonic fluids (eg, normal saline, lactated
Ringer's solution). However, cerebral edema can develop if the volume is given in excess or too quickly. Therefore, a 10-mL/kg fluid bolus is first
given gradually over 1 hour. In addition, insulin decreases serum glucose and plasma osmolality, which promotes osmotic water movement into
the brain. For this reason, insulin administration is delayed until after the fluid bolus to further minimize the risk of cerebral edema.

Once the initial normal saline bolus is completed, patients are started on a regular intravenous insulin drip with isotonic fluids containing
potassium. Although this patient's serum potassium appears normal, his total body potassium is depleted as insulin deficiency causes potassium
to move extracellularly. Once insulin is administered, potassium shifts intracellularly, which can cause the patient to quickly become hypokalemic.
Potassium supplementation should be started concurrently with the insulin drip to prevent sequelae of hypokalemia (eg, cardiac arrhythmias).

(Choice A) This patient requires additional intravenous fluid repletion with isotonic fluids, but rehydration alone will not correct the metabolic
acidosis due to insulin deficiency. The fluid must also contain potassium to offset renal and gastrointestinal losses.

(Choice B) Sodium bicarbonate may be beneficial for severe acidosis (pH <6.9) but is otherwise ineffective and associated with increased side
effects (eg, cerebral edema). This patient has a pH of 7.2.

(Choice C) Patients with DKA and serum glucose <200 mg/dL are given dextrose-containing fluids to prevent serum glucose from dropping too
quickly or too low with insulin administration. However, dextrose-containing fluids are inappropriate in this patient with a serum glucose level of
360 mg/dL.

(Choices E and F) Short- and long-acting insulins are used as maintenance therapy for patients with type 1 diabetes mellitus. Subcutaneous
insulin is inappropriate for initial DKA treatment as it is slower-acting and more difficult to titrate compared with a regular insulin intravenous
infusion. This patient can be transitioned to a subcutaneous insulin regimen when his anion gap closes and the metabolic acidosis resolves.

Educational objective:
Treatment of diabetic ketoacidosis consists of gradual fluid resuscitation with a 10 mL/kg bolus of normal saline followed by an intravenous insulin
drip with potassium-containing isotonic fluids. Transition to subcutaneous insulin occurs when the anion gap closes and the metabolic acidosis
resolves.

References
Improving care for pediatric diabetic ketoacidosis.

(http://www.ncbi.nlm.nih.gov/pubmed/25092935)
Fluid management in pediatric patients with DKA and rates of suspected clinical cerebral edema.

(http://www.ncbi.nlm.nih.gov/pubmed/25800410)

Pediatrics Endocrine, Diabetes & Metabolism Diabetic ketoacidosis


Subject System Topic

https://t.me/USMLEWorldStep2CK

REVIEW

https://www.uworld.com/ClientApp/v15/apps/qbanktestinterface/index.html#/launchtest/7281878/nbme/229400738/3/1 1/1
‫ م‬9:18 2021/‫‏‬9/‫‏‬11 https://t.me/USMLEWorldStep2CK UWorld STEP2 SIM Form 1

1 Item 1 of 40
2
Mark
Question Id: 6837 Previous Next
3

4 A 43-year-old homeless man is hospitalized due to abdominal pain. He states that he has burning pain in the epigastric area and has had 2
5 episodes of vomiting. On admission, blood pressure is 100/70 mm Hg and heart rate is 90/min. Physical examination reveals dry mucous
6 membranes and prominent epigastric tenderness. Laboratory results are as follows:
7
Aspartate transaminase 320 U/L
8

9 Alanine transaminase 152 U/L


10
Albumin 3.7 mg/dL
11
12
INR 0.97

13
Urine toxicology screen is positive for opiates. On the night of hospitalization, the patient experiences insomnia and restlessness. Temperature is
14 37.8 C (100 F), blood pressure is 162/94 mm Hg, and heart rate is 120/min and regular. On examination, the pupils are slightly enlarged and
15 equal and reactive to light, and the hands are tremulous. Which of the following is the most appropriate initial pharmacotherapy for this patient?
16

17  A. Acamprosate (0%)
18
 B. Benzodiazepines (68%)
19
20 C. Buprenorphine (15%)
21 D. Lactulose (0%)
22
E. Naloxone (9%)
23

24 F. Naltrexone (4%)
25

26

27
Incorrect 68% 43 secs 07/16/2021
28 Correct answer
B
 Answered correctly  Time Spent  Last Updated
29
30
31
Explanation
32

33
This patient's epigastric pain, elevated liver enzymes, and development of sympathetic overactivity (ie, tachycardia, elevated blood pressure,
34
restlessness, tremulousness) within 12 hours of admission suggest alcohol withdrawal. Other symptoms of alcohol withdrawal may include
35
anxiety, insomnia, diaphoresis, nausea, and vomiting. Prompt treatment with benzodiazepines is required to prevent progression to severe
36
alcohol withdrawal that may include seizures and life-threatening delirium tremens.
37
Some patients may be withdrawing from both alcohol and opioids, and symptoms may overlap (eg, enlarged pupils and gastrointestinal symptoms
38
are common to both). However, significantly elevated blood pressure and tachycardia are more likely in alcohol withdrawal, whereas piloerection,
39
lacrimation, rhinorrhea, and increased yawning are more likely in opioid withdrawal. If both are present, the priority is treatment of alcohol
40
withdrawal with benzodiazepines, rather than the addition of opioid agonists (eg, buprenorphine) given that opioid withdrawal is generally not life-
threatening (Choice C).

(Choice A) Acamprosate, a modulator of glutamate neurotransmission, is FDA-approved for the treatment of alcohol dependence and may be
used as an aid for abstinence from alcohol use. It has no role in the treatment of acute withdrawal.

(Choice D) Lactulose is used in the treatment of hepatic encephalopathy, which is seen in patients with cirrhosis. This patient has a normal
albumin and INR, suggesting that he has not yet developed cirrhosis.

(Choices E and F) Naloxone and naltrexone are opiate antagonists that worsen opiate withdrawal but do not affect alcohol withdrawal.

Educational objective:
Alcohol withdrawal should be considered in any patient who develops sympathetic hyperactivity and altered mental status shortly after hospital
admission. Benzodiazepines should be started immediately to prevent severe alcohol withdrawal, which is potentially fatal.

References
Alcohol withdrawal syndrome: benzodiazepines and beyond.

(http://www.ncbi.nlm.nih.gov/pubmed/26500991)

Psychiatry Psychiatric/Behavioral & Substance Abuse Alcohol withdrawal


Subject System Topic

https://t.me/USMLEWorldStep2CK

REVIEW

https://www.uworld.com/ClientApp/v15/apps/qbanktestinterface/index.html#/launchtest/7281878/nbme/229400739/3/1 1/1
‫ م‬9:18 2021/‫‏‬9/‫‏‬11 https://t.me/USMLEWorldStep2CK UWorld STEP2 SIM Form 1

1 Item 2 of 40
2
Mark
Question Id: 6838 Previous Next
3

4 A 68-year-old man comes to the office with a cough productive of green sputum for the last week. The patient has had no fevers or chills. He was
5 treated for a viral upper respiratory infection at an urgent care center 2 weeks ago, and was prescribed a cough suppressant that does not seem
6 to be working. He has had no shortness of breath or dyspnea on exertion. The patient does not use tobacco, alcohol, or illicit drugs. Temperature
7 is 37.1 C (98.7 F), blood pressure is 148/92 mm Hg, and pulse is 92/min. Pulse oximetry is 97% on room air. Physical examination shows mild
8 mucosal pallor. There are several palpable, nontender lymph nodes in the cervical chain and axilla. His chest and abdominal examinations are
9 normal other than a slightly prominent spleen tip with deep exhalation. Laboratory results are as follows:
10
Hemoglobin 9.5 g/dL
11
12 Mean corpuscular volume 89 µm3
13 Platelets 127,000/mm3
14
Leukocytes 38,800/mm3
15
16 Neutrophils 7%
17
Lymphocytes 92%
18

19
Monocytes 1%

20

21
Which of the following is most likely to be seen on a peripheral blood smear?

22
 A. Hypersegmented neutrophils (0%)
23

24 B. Lymphoblasts (23%)
25
C. Neutrophils with Auer rods (2%)
26

27
D. Rouleaux formation (1%)

28 E. Smudge cells (69%)



29
F. Teardrop-shaped red blood cells (1%)
30
31

32

33 Incorrect 69% 02 secs 04/24/2021


34
Correct answer
E
 Answered correctly  Time Spent  Last Updated

35

36

37 Explanation
38

39

40

This elderly patient presents with likely post-viral bronchitis (recent viral upper respiratory infection, productive cough >5 days, afebrile, clear chest
examination) in the setting of multichain lymphadenopathy and splenomegaly. Laboratory results show normocytic anemia, lymphocytosis, and
mild thrombocytopenia. This is highly suggestive of chronic lymphocytic leukemia (CLL), one of the most common leukemias in the United
States.

CLL is primarily a disease of the elderly, with a median age of 70 at diagnosis. Although patients have dramatically elevated leukocyte counts,
there is a significant risk of infection due to hypogammaglobulinemia and irregularities in immune cell signaling and function. The characteristic
finding on peripheral blood smear is an abundance of mature-appearing lymphocytes, some of which may have a "smudged" appearance due to
their increased fragility. Diagnosis is confirmed by flow cytometry of the peripheral blood, which can identify circulating monoclonal B cell
lymphocytes.

(Choice A) Hypersegmented neutrophils may be seen in megaloblastic anemia due to vitamin B12 or folate deficiency. Megaloblastic anemia is
associated with an elevated mean corpuscular volume and does not typically cause splenomegaly or lead to an increased risk of infections.

(Choice B) Lymphoblasts suggest a more aggressive cancer such as acute lymphoblastic leukemia. The blood count differential would show
blasts rather than mature lymphocytes.

(Choice C) Auer rods may be seen in some subtypes of acute myeloid leukemia. Acute myeloid leukemia is a neutrophil-predominant cancer;
therefore, the lymphocyte count would be much lower.

(Choice D) Rouleaux formation is seen in patients with multiple myeloma, a disease that tends to cause leukopenia rather than dramatic
leukocytosis. Patients with multiple myeloma also tend to present with anemia and bone pain.

(Choice F) Teardrop-shaped red blood cells can be seen in myelofibrosis or beta thalassemia (especially after splenectomy). This is not a
common finding in CLL.

Educational objective:
Chronic lymphocytic leukemia is one of the most common leukemias in the United States. It is marked by dramatic lymphocytosis, often with
hepatosplenomegaly, anemia, and thrombocytopenia. Smudge cells, which are fragile but mature lymphocytes, are the pathognomonic finding on
peripheral blood smear.

Medicine Hematology & Oncology Chronic lymphocytic leukemia


Subject System Topic

https://t.me/USMLEWorldStep2CK

REVIEW

https://www.uworld.com/ClientApp/v15/apps/qbanktestinterface/index.html#/launchtest/7281878/nbme/229400739/3/1 1/1
‫ م‬9:19 2021/‫‏‬9/‫‏‬11 https://t.me/USMLEWorldStep2CK UWorld STEP2 SIM Form 1

1 Item 3 of 40
2
Mark
Question Id: 6839 Previous Next
3

4 A 64-year-old man undergoes elective left hemicolectomy for recurrent episodes of diverticulitis and diverticular bleeding. There are no immediate
5 intraoperative complications, but the patient receives 2 units of packed red blood cells on the first postoperative day due to anemia from
6 intraoperative blood loss. On the third postoperative day, he develops confusion. Temperature is 37 C (98.6 F), blood pressure is 90/50 mm Hg,
7 pulse is 110/min, and respirations are 20/min. Oxygen saturation is 96% on room air. The skin around the right subclavian line is normal. There
8 are scattered bilateral crackles at the lung bases that clear with coughing, and cardiac examination is unremarkable. Bowel sounds are present,
9 and the abdomen is soft. The laparotomy wound drains clear fluid that is not grossly erythematous. Neurologic examination is nonfocal.
10 Hemoglobin level is 9.2 g/dL, and leukocyte count is 21,000/mm3. Chest x-ray shows no infiltrates. Urinalysis reveals no bacteriuria, and
11 leukocytes are 7-10/hpf. What is the best next step in management of this patient?
12
 A. Blood cultures and antibiotics (54%)
13

14 B. Coombs test (17%)


15
C. Corticosteroids (6%)
16

17  D. CT pulmonary angiogram (11%)


18 E. CT scan of the head (1%)
19
F. Laparotomy (9%)
20

21

22

23 Incorrect 54%
24 Correct answer  Answered correctly  02 secs
Time Spent  09/09/2021
Last Updated
A
25

26

27 Explanation
28

29 Although this patient is afebrile, his confusion, hypotension, tachycardia, and significant leukocytosis indicate likely infection and sepsis. In the
30 absence of localizing symptoms and abnormal chest x-ray and urinalysis findings, the most likely source is the central catheter.
31
Catheter-related bloodstream infection (CRBSI) typically occurs >48 hours after catheter insertion. Most cases are caused by skin flora (eg,
32
coagulase-negative staphylococci [CoNS], Staphylococcus aureus, enterococci, Candida) migration into the bloodstream. Manifestations include
33
abrupt-onset sepsis (eg, hypotension, tachycardia, altered mental status) and fever (sometimes absent in CoNS bacteremia). Although most
34 patients with nontunneled CRBSI have purulence or erythema around the catheter site, these are not always present.
35
Patients with suspected CRBSI require 2 sets of blood cultures: 1 from the catheter and 1 from a peripheral site; blood cultures that grow faster
36
from catheter samples than from peripheral site samples drawn at the same time are strongly suggestive of CRBSI. Broad-spectrum empiric
37
antibiotics (eg, vancomycin) are required until culture results return. Catheter removal may be indicated to eliminate the nidus of infection.
38

39 (Choice B) A positive Coombs test would suggest acute hemolytic transfusion reaction (eg, due to incompatibility), which typically presents with
40
hypotension. However, cases often occur minutes to 24 hours after transfusion and present with fever, flank pain, and hemoglobinuria.

(Choice C) Adrenal crisis requiring emergent corticosteroid therapy can present with hypotension and confusion. However, leukocytosis makes
infection more likely. Blood cultures and antibiotics are required prior to evaluation of other diagnoses.

(Choice D) A large pulmonary embolus, diagnosed on CT angiogram, can cause hypotension and tachycardia. However, hypoxia would likely be
present, and confusion and leukocytosis would be atypical. Bibasilar crackles that clear with coughing suggest atelectasis.

(Choice E) Although confusion can indicate intracranial pathology (eg, bleeding, stroke), leukocytosis, tachycardia, and hypotension are more
suggestive of sepsis.

(Choice F) Laparotomy could treat a significant postoperative abdominal infection, bleeding, or necrosis. However, a soft abdomen (ie, no
peritoneal signs) with normal bowel sounds and clear (ie, nonbloody, nonpurulent) abdominal fluid findings with no obvious signs of bleeding make
this less likely. Furthermore, if there is suspicion for such processes in this patient, STAT imaging (eg, abdominal CT) would likely be obtained
prior to proceeding directly to laparotomy.

Educational objective:
Catheter-related bloodstream infections occur >48 hours after insertion. Patients usually have signs of sepsis, fever, and purulence/erythema at
the catheter site; however, fever and catheter site abnormalities are sometimes absent. Two sets of blood cultures (catheter and peripheral site)
followed by empiric broad-spectrum antibiotics are indicated.

Surgery Infectious Diseases Postoperative fever


Subject System Topic

https://t.me/USMLEWorldStep2CK

REVIEW

https://www.uworld.com/ClientApp/v15/apps/qbanktestinterface/index.html#/launchtest/7281878/nbme/229400739/3/1 1/1
‫ م‬9:19 2021/‫‏‬9/‫‏‬11 https://t.me/USMLEWorldStep2CK UWorld STEP2 SIM Form 1

1 Item 4 of 40
2
Mark
Question Id: 6852 Previous Next
3

4 A 33-year-old pregnant woman at 28 weeks gestation comes to the emergency department with acute-onset, right-side chest pain and shortness
5 of breath. She has no other medical problems. Temperature is 36.6 C (97.8 F), blood pressure is 110/70 mm Hg, pulse is 120/min, and
6 respirations are 26/min. She is diaphoretic and appears to be in distress. Cardiopulmonary examination shows no abnormalities. Extremities are
7 equal in size without swelling. Pelvic examination shows intact membranes and no evidence of vaginal bleeding or cervical dilation. Chest x-ray
8 is normal. Arterial blood gas analysis shows the following results:
9
pH 7.44
10
11 PO2 64 mm Hg
12 PCO2 32 mm Hg
13

14 Ventilation and perfusion scan shows low probability for pulmonary embolism. Which of the following is the most appropriate next step in
15 management of this patient?
16

17  A. CT angiogram of the chest (35%)


18
B. Discharge home with close follow-up (22%)
19
20 C. Heparin infusion (25%)
21 D. Inferior vena cava filter placement (0%)
22
E. Pulmonary function tests (15%)
23

24 F. Warfarin therapy (0%)


25

26

27
35%
28 Correct  Answered correctly  03 secs
Time Spent  06/21/2021
Last Updated
29
30
31
Explanation
32

33
34

35

36

37

38

39

40

Pregnancy increases risk of venous thromboembolism (VTE), with pulmonary embolism (PE) as a leading cause of mortality during pregnancy.
Wells criteria are generally not useful in pregnancy. However, this patient's acute right-side chest pain and dyspnea during pregnancy should raise
high suspicion for PE. Combined with the low PO2, unremarkable chest x-ray, and absence of a more likely explanation for her symptoms, this
patient should be considered to have high pretest probability of PE.

In pregnant patients with a normal chest x-ray and no manifestations of deep venous thrombosis, a ventilation/perfusion (V/Q) scan is the test of
choice because pregnancy changes (eg, increased blood volume) can lead to technical challenges with CT angiography (CTA) (eg, related to
contrast administration timing), resulting in lower diagnostic accuracy.

V/Q scan results are stratified into normal, low, moderate, and high probability, but they need to be combined with PE pretest probability to
evaluate for PE. In patients such as this one with high pretest probability for PE:

A high-probability V/Q scan result would confirm PE

Only a normal V/Q scan (not the same as a low-probability V/Q scan) rules out PE, regardless of pretest probability.

Any other combination (eg, low-probability V/Q scan result, as in this case) requires further testing with CTA.

The cumulative radiation exposure to the fetus from the combination of both a V/Q scan (which has a higher radiation exposure than CTA)
followed by CTA is considered well within acceptable levels, particularly given the relatively high mortality associated with untreated PE in
pregnancy.

(Choices B, C, D, and F) Discharge home would not be appropriate given PE has not been ruled out. PE should be diagnosed before treatment
is initiated in this patient who is hemodynamically stable. Warfarin is pregnancy category X due to teratogenicity; unfractionated heparin
(pregnancy category C) and low-molecular-weight heparin (pregnancy category B) are the anticoagulants of choice. Inferior vena cava filter
placement is appropriate in patients with contraindication to anticoagulation.

(Choice E) Pulmonary function tests help diagnose obstructive and restrictive lung disease, not acute PE.

Educational objective:
A ventilation/perfusion (V/Q) scan is preferred over CT angiography (CTA) in initial evaluation for pulmonary embolism (PE) in pregnant women
with a normal chest x-ray. In any patient (ie, pregnant, nonpregnant) with high pretest probability of PE, only a normal V/Q scan rules out PE; low-
or moderate-probability V/Q scan results require further testing with CTA despite the radiation exposure.

References
An official American Thoracic Society/Society of Thoracic Radiology clinical practice guideline: evaluation of suspected pulmonary embolism
in pregnancy.

(http://www.ncbi.nlm.nih.gov/pubmed/22086989)

Medicine Pulmonary & Critical Care Pulmonary embolism


Subject System Topic

https://t.me/USMLEWorldStep2CK

REVIEW

https://www.uworld.com/ClientApp/v15/apps/qbanktestinterface/index.html#/launchtest/7281878/nbme/229400739/3/1 1/1
‫ م‬9:19 2021/‫‏‬9/‫‏‬11 https://t.me/USMLEWorldStep2CK UWorld STEP2 SIM Form 1

1 Item 5 of 40
2
Mark
Question Id: 6854 Previous Next
3

4 A 24-year-old woman comes to the office for a routine checkup. She has no significant medical history. Blood pressure is 120/74 mm Hg and
5 pulse is 76/min. BMI is 23 kg/m2. The lungs are clear to auscultation. A normal S1 and S2 are heard on cardiac examination. The abdomen is
6 soft and nontender. The spleen tip is palpated in the left upper abdominal quadrant. There is no peripheral edema. A peripheral blood smear is
7 shown on the slide below.
8

10
11
12
13

14

15
16

17
18

19
20

21

22

23

24
25

26

27

28

29
30
This patient is at greatest risk for which of the following conditions?
31

32
 A. Acute leukemia (9%)
33
34 B. Atrophic gastritis (2%)
35 C. Gallstones (71%)

36
D. Osteoporotic fractures (1%)
37

38 E. Renal insufficiency (5%)


39
F. Venous thromboembolism (9%)
40

Incorrect 71%
Correct answer  Answered correctly  04 secs
Time Spent  07/29/2021
Last Updated
C

Explanation

This patient has a palpable spleen tip and spherocytes (small, hyperchromic red cells without central pallor) on peripheral blood smear consistent
with hereditary spherocytosis (HS). HS is caused by a genetic defect in red blood cell (RBC) scaffolding proteins (eg, spectrin, ankyrin).
Sphere-shaped RBCs, which are not as deformable as normal biconcave RBCs, get trapped in the spleen, leading to splenomegaly (ie, palpable
spleen tip) and hemolytic anemia.

HS is often asymptomatic; however, >50% of untreated patients develop pigment gallstones as a complication of chronic hemolysis. Increased
RBC turnover leads to excess bilirubin, which overwhelms conjugation and elimination from the body. The resulting hyperbilirubinemia manifests
as jaundice, dark urine, and pigment (calcium bilirubinate) gallstones.

Treatment of HS is primarily supportive with blood transfusions and folic acid supplementation. Splenectomy may be considered in some cases to
reduce hemolysis and gallstone formation.

(Choice A) Immature cells (eg, myeloblasts) on peripheral smear may suggest acute leukemia. No blast cells are seen on this patient's smear.

(Choice B) Atrophic gastritis can lead to a megaloblastic anemia due to B12 deficiency. Macrocytic RBCs and hypersegmented neutrophils would
be seen on peripheral blood smear.

(Choice D) Sickle cell disease can cause bone infarction and accelerated hematopoiesis, which can lead to osteoporosis; however, no sickle cells
are seen on this blood smear.

(Choice E) Peripheral smear of a patient with renal insufficiency may reveal Burr cells (ie, echinocytes), which have multiple small projections (not
seen on this patient's smear).

(Choice F) Venous thromboembolism risk is increased with microangiopathic hemolytic anemia, which is characterized by schistocytes on
peripheral smear (not present in this patient).

Educational objective:
Hereditary spherocytosis is characterized by hemolytic anemia; splenomegaly; and small, hyperchromic red cells without central pallor (ie,
spherocytes) on peripheral blood smear. Pigment gallstones can develop as a complication of chronic hemolysis.

References
Advances in laboratory diagnosis of hereditary spherocytosis.

(http://www.ncbi.nlm.nih.gov/pubmed/27837594)

Medicine Hematology & Oncology Hereditary spherocytosis


Subject System Topic

https://t.me/USMLEWorldStep2CK

REVIEW

https://www.uworld.com/ClientApp/v15/apps/qbanktestinterface/index.html#/launchtest/7281878/nbme/229400739/3/1 1/1
‫ م‬9:19 2021/‫‏‬9/‫‏‬11 https://t.me/USMLEWorldStep2CK UWorld STEP2 SIM Form 1

1 Item 6 of 40
2
Mark
Question Id: 18750 Previous Next
3

4 A 65-year-old woman with left index finger stenosing tenosynovitis (trigger finger) has a surgical procedure scheduled for release of the tendon
5 sheath. The procedure is scheduled as the surgeon's last case of the day in a large, busy hospital. While the patient waits to be taken to the
6 operating room, the surgeon and the assigned circulating nurse each independently verify the patient's identity and planned procedure, and obtain
7 informed consent. As per standard hospital policy, the patient's left arm is marked. The surgeon is then notified that, in an administrative effort to
8 accommodate more cases, the procedure will take place in a different operating room with different personnel. This results in a delay in the
9 procedure start time, and a brief time-out is conducted with the new team. During the surgery, the surgeon mistakenly performs surgical release of
10 the left carpal tunnel rather than the left index finger. Which of the following interventions would address a latent error that led to the incorrect
11 procedure being performed?
12
 A. Have patients verify the procedure to be performed rather than operating room teams (13%)
13

14 B. Implement disciplinary measures to strengthen organization's culture of safety (6%)


15
 C. Optimize scheduling to increase procedure spacing and avoid last-minute personnel changes (58%)
16

17 D. Require personnel not directly involved in cases to lead the time-out procedures (7%)
18 E. Require this particular surgeon to mark both exact site of incision and correct side in all future cases (14%)
19
20

21
Incorrect 58%
22

23
Correct answer
C
 Answered correctly  02 secs
Time Spent  07/07/2021
Last Updated

24
25

26 Explanation

27

28
Active versus latent errors
29
30
Active error Latent error

31 Proximal cause Distal cause


32
Involves actions of frontline Involves organizational vulnerabilities that
33
personnel influence actions of frontline personnel
34

35 Occurs at interface of provider


Occurs at level of system, process & policy
36 with patient or system
37 "Sharp end" of the scalpel, tip "Accident waiting to happen," "blunt end" of the
38 of the iceberg scalpel, below the surface
39

40
This scenario (based on a real case) illustrates wrong-surgery error, a rare but serious occurrence (~1:100,000 procedures). In complex
systems, serious adverse events reflect both active errors, describing frontline personnel's erroneous actions (ie, tip of iceberg), and latent
errors, describing system-level deficiencies increasing risk of unchecked human error (ie, below the surface). This surgeon's active error –
performing the wrong surgery – was enabled by multiple latent factors:

The scheduling protocol and possible excessive production demands (eg, last-minute room change to accommodate more cases)
increased the likelihood of fatigue and distractibility.

The last-minute change delayed the procedure start time, disrupting continuity (eg, assignment of a new support team).

The new team had less familiarity with the type of case; moreover, the original team's nurse (who had verified the procedure and site earlier)
could not provide oversight during time-out (ie, verification of patient surgery, site, side).

These latent factors reduced the likelihood of team-based awareness and prevention of the surgeon's active error. Optimizing scheduling to avoid
last-minute operating room, schedule, and personnel changes would address latent errors that precipitated this wrong-surgery error.

Requiring this surgeon to mark the incision site in future cases will minimize his future chances of wrong-surgery error. However, it singles out one
surgeon's individual, active error, rather than addressing latent, systemic factors (Choice E). Similarly, excessive use of disciplinary action targets
individual, active error (without acknowledging latent contributors) and promotes a culture of blame rather than safety (Choice B).

(Choice A) Patients are often included in the formal time-out process. However, relying only on patient input during this process (ie, excluding
the operating team) is not recommended.

(Choice D) As this case illustrates, time-out procedures should maintain care continuity between a treating team and the patient, rather than
involving personnel who are unfamiliar with the patient and procedure.

Educational objective:
To prevent serious adverse events (eg, wrong-surgery error), safety measures should address both latent errors, describing system-level safety
vulnerabilities (eg, operating room scheduling), and active errors, describing frontline personnel's actions.

References
Case records of the Massachusetts General Hospital: case 34-2010: a 65-year-old woman with an incorrect operation on the left hand.

(http://www.ncbi.nlm.nih.gov/pubmed/21067387)

Medicine Social Sciences (Ethics/Legal/Professional) Patient safety


Subject System Topic

https://t.me/USMLEWorldStep2CK

REVIEW

https://www.uworld.com/ClientApp/v15/apps/qbanktestinterface/index.html#/launchtest/7281878/nbme/229400739/3/1 1/1
‫ م‬9:20 2021/‫‏‬9/‫‏‬11 https://t.me/USMLEWorldStep2CK UWorld STEP2 SIM Form 1

1 Item 7 of 40
2
Mark
Question Id: 6863 Previous Next
3

4 A 66-year-old woman comes to the office due to frequent urine leakage that causes her to wear a pad. The leakage is preceded by an urge to
5 urinate, and most of the time she cannot make it to the restroom before urinating. She voids 10-12 times during the day and awakens 2-3 times
6 during the night to urinate. She is very embarrassed as loss of urine has occurred in public places. The patient has no urine leakage during
7 laughing or sneezing. She has no burning or difficulty with urination. The patient has no chronic medical conditions and underwent menopause at
8 age 54. Vital signs are normal. Pelvic examination reveals mild vaginal atrophy. Urinalysis is normal. Which of the following is the best next step
9 in management of this patient's symptoms?
10
 A. Alpha-adrenergic antagonist (23%)
11
12  B. Bladder training (61%)
13
C. Continence pessary (4%)
14

15 D. Fluid restriction (7%)


16
E. Midurethral sling (2%)
17
18

19
20
Incorrect 61%
21
Correct answer
B
 Answered correctly  02 secs
Time Spent  07/11/2021
Last Updated

22

23

24 Explanation
25

26
Urinary incontinence
27

28 Type Symptoms Treatment


29 Lifestyle modification
30 Leaking with Valsalva maneuver (coughing, Pelvic floor exercises
Stress
31 sneezing, laughing) Pessary
32 Pelvic floor surgery
33
Lifestyle modification
34
Urgency Sudden, overwhelming, or frequent need to void Bladder training
35
Antimuscarinic drugs
36

37 Mixed Features of stress & urgency incontinence Variable treatment depending on predominant symptoms
38 Identification and correction of underlying cause
Constant involuntary dribbling & incomplete
39 Overflow Cholinergic agonists
emptying
40 Intermittent self-catheterization

Urinary incontinence is underdiagnosed and undertreated and has a significant negative impact on patient quality of life (eg, daily pad use,
embarrassment). This patient's frequent urine leakage is due to urgency incontinence (overactive bladder), or urinary urgency followed by
immediate involuntary loss of urine. The cause is detrusor overactivity, or bladder spasms that do not allow adequate time between the
sensation to void and the loss of urine (eg, leaking prior to reaching the restroom). The volume of urine loss is variable, but urinary frequency
and nocturia are common.

First-line treatment for urgency incontinence is behavioral therapy with pelvic floor muscle exercises (eg, Kegels) and bladder training, which
involves delaying micturition and resisting the urge to void for progressively longer intervals to increase bladder capacity. If symptoms do not
improve, pharmacologic therapy (eg, antimuscarinics, beta-adrenergic agonists) may be prescribed.

(Choice A) Alpha-adrenergic antagonists (eg, terazosin, doxazosin, tamsulosin) are used for the treatment of bladder outlet obstruction due to
benign prostatic hyperplasia. Alpha-adrenergic antagonists decrease urethral tone and can exacerbate leakage in patients with urgency
incontinence.

(Choices C and E) Continence pessaries and midurethral slings provide urethral support for treatment of stress urinary incontinence (SUI).
Patients with SUI have leakage of urine with increased intraabdominal pressure (eg, laughing, sneezing), which is not seen in this patient. In
addition, pessaries and slings may worsen urgency incontinence due to bladder neck compression.

(Choice D) Avoidance of bladder irritants (eg, caffeine, alcohol) may improve urgency incontinence symptoms; however, fluid restriction is not
recommended in older patients due to the risks of dehydration (eg, orthostatic hypotension, electrolyte imbalances).

Educational objective:
Bladder training is first-line treatment for urgency incontinence, which typically presents with urinary urgency, frequency, and nocturia. If
symptoms do not improve, pharmacologic therapy (eg, antimuscarinics, beta adrenergic agonists) may be prescribed.

References
Practical aspects of lifestyle modifications and behavioural interventions in the treatment of overactive bladder and urgency urinary
incontinence.

(http://www.ncbi.nlm.nih.gov/pubmed/19575724)
Pharmacotherapy of overactive bladder in adults: a review of efficacy, tolerability, and quality of life.

(http://www.ncbi.nlm.nih.gov/pubmed/24400248)

Obstetrics & Gynecology Renal, Urinary Systems & Electrolytes Urinary incontinence
Subject System Topic

https://t.me/USMLEWorldStep2CK

REVIEW

https://www.uworld.com/ClientApp/v15/apps/qbanktestinterface/index.html#/launchtest/7281878/nbme/229400739/3/1 1/1
‫ م‬9:20 2021/‫‏‬9/‫‏‬11 https://t.me/USMLEWorldStep2CK UWorld STEP2 SIM Form 1

1 Item 8 of 40
2
Mark
Question Id: 6864 Previous Next
3

4 An 11-year-old girl is brought to the emergency department by her parents due to a rash that began this morning. She has otherwise been well
5 except for rhinorrhea, cough, and sore throat that developed a week ago and resolved within a few days. Temperature is 37.4 C (99.3 F), blood
6 pressure is 118/70 mm Hg, pulse is 100/min, and respirations are 20/min. Pulse oximetry is 98% on room air. The patient is awake and alert. The
7 oropharynx is clear, and the neck is supple. Cardiopulmonary examination is unremarkable. The abdomen is soft and nontender with no
8 organomegaly. The lower extremity joints have no swelling, erythema, or warmth, but passive range of motion of both knees elicits pain.
9 Extremities are warm and well perfused. Skin examination shows a nontender, nonblanching, deep red, raised rash over the buttocks and lower
10 extremities without a clear pattern. Laboratory results are as follows:
11
Complete blood count
12
13 Hemoglobin 12 g/dL
14 Platelets 220,000/mm3
15
Leukocytes 7,500/mm3
16

17
Serum chemistry
18

19
Sodium 136 mEq/L

20 Potassium 3.8 mEq/L


21
Chloride 104 mEq/L
22

23 Bicarbonate 24 mEq/L
24 Blood urea nitrogen 16 mg/dL
25
Creatinine 0.6 mg/dL
26

27 Glucose 100 mg/dL


28

29
Immunologic and rheumatologic studies

30 Erythrocyte sedimentation rate 10 mm/hr


31

32
Which of the following is the best next step in management of this patient?

33
 A. Antinuclear antibodies (7%)
34

35 B. Blood culture followed by antibiotics (7%)


36
C. Echocardiography (12%)
37
D. Intravenous immunoglobulin therapy (13%)
38

39  E. Urinalysis (58%)
40

Incorrect 58%
Correct answer  Answered correctly  02 secs
Time Spent  05/07/2021
Last Updated
E

Explanation

Henoch-Schönlein purpura (IgA vasculitis)


Perivenular leukocytoclastic (neutrophils & monocytes) vasculitis
Pathogenesis
Deposition of IgA, C3 & fibrin in small vessels

Classic findings*:
Palpable purpura/petechiae on lower extremities
Clinical Arthritis/arthralgia
manifestations Abdominal pain, intussusception
Renal disease (similar to IgA nephropathy)
Other findings: scrotal pain & swelling

Normal platelet count & coagulation studies


Laboratory
Normal to ↑ creatinine
findings
Hematuria ± RBC casts &/or proteinuria

Supportive care (hydration & NSAIDs) for most patients


Management
Hospitalization & systemic glucocorticoids for severe symptoms

*Clinical diagnosis requires purpuric rash plus ≥2 additional classic findings.


NSAIDs = nonsteroidal anti-inflammatory drugs; RBC = red blood cell.

This patient's rash and knee pain are concerning for Henoch-Schönlein purpura (HSP), or IgA vasculitis. The most common presenting
features include nonblanching, palpable purpura on the buttocks and lower extremities and arthralgia/arthritis of the hips, knees, and/or
ankles. Abdominal pain due to local vasculitis may also occur.

The diagnosis is usually clinical. Laboratory testing is nonspecific; leukocytes and erythrocyte sedimentation rate (ESR) may be normal but can
also be elevated, typically reflecting a preceding upper respiratory infection (eg, rhinorrhea, cough), which is common prior to the development of
HSP.

The next step in management is urinalysis to assess for renal involvement, which is typically mild and includes hematuria, red cell casts, and/or
nonnephrotic proteinuria. However, HSP can result in serious renal complications, such as acute kidney injury and nephrotic syndrome, in some
patients. In addition, because renal disease can develop days to months after HSP onset, urinalysis should be performed serially even if initially
negative and extrarenal manifestations have resolved. Most patients with mild renal involvement have spontaneous resolution of urinalysis
abnormalities.

(Choice A) Antinuclear antibodies are elevated in polyarticular juvenile idiopathic arthritis (JIA) and systemic lupus erythematosus, both of which
can cause joint pain but are often associated with elevated ESR. In addition, polyarticular JIA causes chronic joint pain and swelling, neither of
which are seen here, and dermatologic manifestations of lupus include a malar or discoid rash, not purpura.

(Choice B) Blood cultures and antibiotics are indicated for meningococcal infection. Although the initial presentation may be nonspecific (eg,
sore throat, myalgias), purpura develops with progressive infection, and signs of sepsis (eg, fever, cool extremities, leukocytosis) and
disseminated intravascular coagulation (eg, thrombocytopenia) would be expected. This alert patient has normal vital signs and laboratory
evaluation.

(Choice C) Echocardiography can be used to evaluate for cardiac manifestations of acute rheumatic fever following a recent sore throat. Skin
findings include erythema marginatum (annular, pink rash) and subcutaneous nodules, not palpable purpura. In addition, this patient's lack of
fever and normal ESR make this diagnosis less likely.

(Choice D) Intravenous immunoglobulin therapy is sometimes used for the management of idiopathic thrombocytopenic purpura, which can
present with petechiae/purpura after a viral illness. However, joint pain is not associated with this condition, and thrombocytopenia is required for
diagnosis.

Educational objective:
Henoch-Schönlein purpura, or IgA vasculitis, classically presents with nonblanching, palpable purpura and arthralgia/arthritis. Management
involves urinalysis to screen for renal involvement, which can occur days to months after symptom onset.

References
The diagnosis and classification of Henoch-Schönlein purpura: an updated review.

(http://www.ncbi.nlm.nih.gov/pubmed/24424188)

Pediatrics Rheumatology/Orthopedics & Sports IgA vasculitis


Subject System Topic

https://t.me/USMLEWorldStep2CK

REVIEW

https://www.uworld.com/ClientApp/v15/apps/qbanktestinterface/index.html#/launchtest/7281878/nbme/229400739/3/1 1/1
‫ م‬9:20 2021/‫‏‬9/‫‏‬11 https://t.me/USMLEWorldStep2CK UWorld STEP2 SIM Form 1

1 Item 9 of 40
2
Mark
Question Id: 6866 Previous Next
3

4 A 68-year-old man comes to the emergency department due to new-onset back pain, which started this morning. He describes the pain as
5 "intense" and not exacerbated by movement. Medical history is significant for hypertension and coronary artery disease. The patient underwent
6 coronary artery bypass grafting 3 years ago. He smokes a pack of cigarettes daily and consumes alcohol on social occasions. On physical
7 examination, there is a bruit over the right carotid artery. There is no tenderness to palpation of the vertebrae or paraspinal muscles. Lower
8 extremity deep tendon reflexes are symmetric. During ongoing evaluation, he begins to experience dizziness and weakness. His blood pressure
9 drops from 120/85 mm Hg to 90/60 mm Hg and his pulse increases from 78/min to 120/min. Which of the following is the most likely cause of this
10 patient's back pain?
11
 A. Epidural abscess (9%)
12
13 B. Lumbar disk disease (2%)
14
C. Multiple myeloma (6%)
15
16 D. Muscle sprain (0%)
17 E. Osteoarthritis (0%)
18
F. Osteoporosis (1%)
19
20  G. Referred pain (77%)
21
H. Spondyloarthritis (2%)
22

23

24
25 Incorrect 77%
26
Correct answer
G
 Answered correctly  02 secs
Time Spent  05/23/2021
Last Updated

27

28

29 Explanation
30
31

32

33
34

35

36

37

38

39

40

This patient with new-onset, intense back pain unaffected by movement and hypotension likely has a ruptured abdominal aortic aneurysm
(AAA). The classic presentation of ruptured AAA includes sudden onset of intense abdominal, back, or flank pain (referred from the aorta),
hypotension, and pulsatile abdominal pain. The position of the AAA can affect the location of the pain. However, the clinical manifestations of
ruptured AAA can be quite variable, and misdiagnosis occurs in up to one-third of patients. Ruptured AAA is a surgical emergency with a high
mortality rate, and rapid diagnosis and surgical intervention are necessary to prevent fatal complications.

Clinical risk factors associated with AAA development include advancing age (>60 years), Caucasian race, male sex, hypertension, smoking, and
systemic atherosclerosis (coronary or peripheral arterial disease).

(Choice A) Patients with epidural abscess present with fever, back pain, and/or neurologic deficits, and often have tenderness to palpation of the
vertebrae or paraspinal muscles.

(Choice B) Lumbar disk disease with lumbosacral radiculopathy causes acute back pain with radicular symptoms, varying degrees of motor or
sensory deficits, and abnormal deep tendon reflexes.

(Choice C) Bone pain due to multiple myeloma is typically exacerbated by movement or change in position and would not be accompanied by
acute hypotension.

(Choices D and E) The back pain associated with osteoarthritis and/or muscle sprain is typically worse with activity or movement and is relieved
with rest.

(Choice F) Vertebral compression fractures due to osteoporosis can cause significant back pain that is worse with movement or change in
position. The back pain would not be associated with hypotension.

(Choice H) Patients with spondyloarthritis involving the axial joints typically have inflammatory low back pain. The pain is chronic with insidious
onset and is often worse at night, improving during the day with activity and exercise.

Educational objective:
Ruptured abdominal aortic aneurysm (AAA) presents with a sudden onset of intense abdominal, back, or flank pain; hypotension; and pulsatile
abdominal mass. It is frequently misdiagnosed; therefore, a high degree of clinical suspicion should be maintained in patients with significant risk
factors for AAA who have new-onset abdominal or back pain.

References
Misdiagnosis of ruptured abdominal aortic aneurysms.

(http://www.ncbi.nlm.nih.gov/pubmed/1619721)
Ruptured abdominal aortic aneurysm: a surgical emergency with many clinical presentations.

(http://www.ncbi.nlm.nih.gov/pubmed/19520879)

Surgery Cardiovascular System Aortic aneurysm


Subject System Topic

https://t.me/USMLEWorldStep2CK

REVIEW

https://www.uworld.com/ClientApp/v15/apps/qbanktestinterface/index.html#/launchtest/7281878/nbme/229400739/3/1 1/1
‫ م‬9:20 2021/‫‏‬9/‫‏‬11 https://t.me/USMLEWorldStep2CK UWorld STEP2 SIM Form 1

1 Item 10 of 40
2
Mark
Question Id: 6867 Previous Next
3

4 A 74-year-old woman comes for follow-up due to persistent pain in her left knee. The patient was initially seen 4 weeks ago for left knee pain.
5 Left knee x-ray at that time revealed joint space narrowing and multiple osteophytes, and as-needed naproxen was initiated. Despite treatment,
6 her pain has worsened and is limiting her activities of daily living. The patient has no history of trauma or falls. Temperature is 36.5 C (97.7 F),
7 blood pressure is 130/80 mm Hg, and pulse is 78/min. BMI is 30 kg/m2. The left knee appears enlarged with mild to moderate effusion and
8 warmth but no erythema. Serum creatinine is 0.9 mg/dL. Which of the following is the best next step in management of this patient?
9
 A. Arthrocentesis (44%)
10
11 B. Continuous indomethacin (2%)
12
C. Intraarticular corticosteroid injection (41%)
13

14 D. Knee replacement (10%)


15
E. Oral colchicine (0%)
16

17
18

19 44%
20
Correct  Answered correctly  02 secs
Time Spent  08/10/2021
Last Updated

21

22

23 Explanation
24
25
The patient's x-ray confirms the diagnosis of osteoarthritis (OA) with joint space narrowing and osteophyte formation. However, her OA has not

26
responded to usual treatment (topical or oral nonsteroidal anti-inflammatory drugs [NSAIDs]). Given the warmth and effusion at the knee,
additional etiologies for her knee pain should be considered, including calcium pyrophosphate (CPP) crystal deposition.
27

28 CPP arthropathy is common in patients with underlying OA, especially in the knees, and can lead to progressive pain, stiffness, and joint effusion.
29 Although x-ray often shows chondrocalcinosis, mild changes may be missed, so joint aspiration with synovial fluid analysis (arthrocentesis) for
30 crystals is often warranted to clarify the diagnosis. If CPP crystal disease is present, joint aspiration and intraarticular glucocorticoid injection may
31 provide prompt relief.
32
(Choices B and E) Both indomethacin and colchicine are used for acute gout. In most patients, gout responds to NSAIDs (even with as-needed
33 dosing) and would be unlikely to worsen. Given this patient's effusion, other etiologies should be ruled out before trying additional empiric therapy.
34
(Choice C) Intraarticular corticosteroid injection can be considered for the treatment of uncomplicated knee OA, but the benefits are typically
35
small and transient. It may be more effective in patients with synovial inflammation due to CPP arthropathy, but this should first be confirmed with
36
synovial fluid analysis.
37

38 (Choice D) Knee replacement (ie, total knee arthroplasty) is commonly performed in patients with OA that limits functional capacity due to
39
intractable pain. However, it is typically reserved for when a more comprehensive and prolonged regimen of nonoperative management has

40
failed.

Educational objective:
Initial management of knee osteoarthritis includes exercise, weight loss, and nonsteroidal anti-inflammatory drugs. Joint aspiration with synovial
fluid analysis should be considered when treatment fails conservative measures, especially when significant warmth, joint tenderness, or effusion
is present.

Medicine Rheumatology/Orthopedics & Sports Osteoarthritis


Subject System Topic

https://t.me/USMLEWorldStep2CK

REVIEW

https://www.uworld.com/ClientApp/v15/apps/qbanktestinterface/index.html#/launchtest/7281878/nbme/229400739/3/1 1/1
‫ م‬9:20 2021/‫‏‬9/‫‏‬11 https://t.me/USMLEWorldStep2CK UWorld STEP2 SIM Form 1

1 Item 11 of 40
2
Mark
Question Id: 6872 Previous Next
3

4 A 36-year-old woman, gravida 7 para 3 aborta 3, at 32 weeks gestation comes to the hospital with vaginal bleeding. The patient awoke this
5 morning in a pool of bright red blood and has had some irregular contractions but no constant abdominal pain or leakage of fluid; fetal movement
6 is normal. She has had no vaginal bleeding prior to this morning and has had no prenatal care. She has used methamphetamines intermittently
7 throughout the pregnancy. Blood pressure is 136/78 mm Hg, pulse is 95/min, and respirations are 18/min. Speculum examination reveals a small
8 blood clot in the vagina but no active cervical bleeding. The fetal heart rate monitoring is shown in the exhibit. Blood type is AB positive and
9 hemoglobin is 9.8 g/dL. Which of the following is the best next step in management of this patient?
10
 A. Digital cervical examination (3%)
11
12 B. Emergency cesarean delivery (17%)
13
C. Fetal fibronectin testing (6%)
14

15 D. Kleihauer-Betke test (4%)


16
E. Tranexamic acid administration (0%)
17
18  F. Transabdominal ultrasound (66%)

19
20

21 Incorrect 66% 01 sec 04/01/2021


22 Correct answer
F
 Answered correctly  Time Spent  Last Updated
23

24
25
Explanation
26

27

28 Placenta previa
29 Prior placenta previa
30 Risk factors Prior cesarean delivery
31 Multiple gestation
32
Clinical
33 Painless vaginal bleeding >20 weeks gestation
features
34

35 Diagnosis Transabdominal followed by transvaginal sonogram


36
No intercourse
37 Management No digital cervical examination
38 Inpatient admission for bleeding episodes
39

40
Placenta previa occurs when the placenta covers the cervix and has the potential for massive hemorrhage. Bleeding is typically painless, as
physiologic cervical shortening, dynamic lower uterine segment changes, and nonpainful contractions can detach the placental bed where it
covers the cervix (ie, partial separation). Because most of the bleeding is maternal in origin (rather than fetal), reactive tracings are initially
seen. Risk factors include multiparity, advanced maternal age, and prior cesarean delivery.

In patients with vaginal bleeding at >20 weeks gestation, a transabdominal ultrasound is performed to evaluate placental location.
Transabdominal ultrasound has a high false-positive rate for detecting placenta previa; therefore, if transabdominal imaging reveals a possible
previa, transvaginal ultrasound is performed. Transvaginal ultrasound is safe in patients with suspected placenta previa as visualization is
optimal 2-3 cm away from the cervix, and the angle of the vagina prevents the ultrasound probe from entering the cervical canal.

(Choice A) Digital cervical examination is contraindicated as entry into the cervical canal can disrupt the placenta, leading to massive
hemorrhage. Digital examination can be used to assess cervical dilation only after placenta previa has been excluded with ultrasound.

(Choice B) Emergency cesarean delivery is indicated in patients with active vaginal bleeding, creating maternal hemodynamic instability and/or
fetal decelerations unresponsive to supportive measures. This patient's vital signs are normal and fetal heart rate tracing is reactive.

(Choice C) Fetal fibronectin (fFN) is found in the choriodecidual interface; its presence (ie, positive testing) in vaginal sampling at 24-34 weeks
gestation confers an elevated risk for preterm delivery. However, fFN testing cannot determine the cause of this patient's bleeding; in addition,
vaginal bleeding can cause a false-positive fFN result.

(Choice D) Kleihauer-Betke tests for fetomaternal hemorrhage in the setting of placental abruption to determine the necessary dose of anti-D
immunoglobulin after delivery of an Rh-positive fetus to an Rh-negative mother (unlike this patient with AB, Rh-positive blood). In addition,
placental abruption typically causes constant abdominal pain and fetal decelerations.

(Choice E) Tranexamic acid reduces postpartum hemorrhage (eg, uterine atony) by stabilizing fibrin, inhibiting fibrinolysis and plasmin activity. It
is typically not used prior to delivery as it crosses the placenta and has unknown fetal effects.

Educational objective:
Placenta previa causes painless vaginal bleeding after 20 weeks gestation due to the placenta covering the cervix. In patients with vaginal
bleeding and unknown placental location, the best next step is transabdominal ultrasound.

References
Abnormal placentation: placenta previa, vasa previa, and placenta accreta.

(http://www.ncbi.nlm.nih.gov/pubmed/26244528)
Risk factors and pregnancy outcomes: complete versus incomplete placenta previa in mid-pregnancy.

(http://www.ncbi.nlm.nih.gov/pubmed/30128867)

Obstetrics & Gynecology Pregnancy, Childbirth & Puerperium Antepartum bleeding


Subject System Topic

https://t.me/USMLEWorldStep2CK

REVIEW

https://www.uworld.com/ClientApp/v15/apps/qbanktestinterface/index.html#/launchtest/7281878/nbme/229400739/3/1 1/1
‫ م‬9:21 2021/‫‏‬9/‫‏‬11 https://t.me/USMLEWorldStep2CK UWorld STEP2 SIM Form 1

1 Item 12 of 40
2
Mark
Question Id: 6875 Previous Next
3

4 A 36-year-old man is brought to the emergency department following a high-speed motor vehicle collision. The patient was wearing a seat belt,
5 and front airbags deployed. There was no loss of consciousness. The patient is awake and alert but reports moderate chest pain. His medical
6 issues include obesity and gastroesophageal reflux. His last meal was 8 hours ago. Blood pressure is 140/84 mm Hg, pulse is 116/min,
7 respirations are 24/min, and pulse oximetry is 98% on 4 L oxygen. On examination, there is no obvious external bleeding. There is no cervical
8 spine point tenderness or jugular venous distension. An erythematous seat belt imprint is visible across the chest. There is mild tenderness to
9 palpation at the sternochondral and costochondral joints. Bilateral breath sounds are present and symmetric. Heart sounds are normal without
10 murmurs. The abdomen is soft and nontender to palpation. ECG shows sinus tachycardia. An upright chest x-ray reveals a widened
11 mediastinum but no sternal or rib fractures, pneumothorax, or pleural effusion. Which of the following is the best next step in management of this
12
patient?

13
 A. Analgesics and observation (2%)
14

15 B. Aortography (9%)
16
 C. CT angiography of the chest (67%)
17
18
D. Exploratory thoracotomy (7%)

19 E. Transthoracic echocardiography (13%)


20

21

22
Incorrect 67% 02 secs 05/16/2021
23

24
Correct answer
C
 Answered correctly  Time Spent  Last Updated

25

26

27
Explanation

28

29
30
31

32

33
34

35

36

37

38

39

40

This patient's rapid deceleration (eg, high-speed motor vehicle collision) puts him at risk for blunt thoracic aortic injury (BTAI), a life-
threatening condition. Even with increased use of seat belts and airbags, the incidence of BTAI remains unchanged as rapid deceleration still
occurs and exerts a combination of stretching, shearing, and torsional forces capable of rupturing the aorta. The aortic isthmus—the transition
zone between the relatively mobile ascending aorta/arch and the fixed descending aorta—is particularly vulnerable to these forces and is the most
common site of BTAI.

The presentation of BTAI depends on the extent of aortic injury; those with incomplete aortic rupture (ie, intact aortic adventitia preventing
exsanguination) may initially be normotensive or hypertensive (due to sympathetic response), as in this patient. Although there are no clinical
findings specific for aortic injury, concerning signs include external evidence of blunt chest trauma (eg, seat-belt imprint) or chest x-ray findings of
widened mediastinum, abnormal (eg, enlarged) aortic contour, or left-sided effusion (hemothorax).

BTAI typically requires emergent surgical repair. Ideally, confirmatory testing is performed prior to surgery to distinguish BTAI from other injuries
(eg, esophageal rupture that also causes widened mediastinum), but this depends on patient hemodynamic stability:

Hemodynamically unstable (eg, hypotension, active hemorrhage): proceed directly to thoracotomy ± transesophageal echocardiography
(TEE) in the operating room (Choice D)
Hemodynamically stable: obtain CT angiography to determine endovascular (vascular surgery) versus open (thoracic surgery) repair

(Choice A) Analgesics and observation are indicated for patients with costochondritis or musculoskeletal pain. This patient's widened
mediastinum warrants further testing to identify possible aortic injury.

(Choice B) Aortography (via femoral artery catheterization) is no longer routinely used to diagnose BTAI due to its invasiveness and lengthy setup
(which may delay diagnosis and treatment).

(Choice E) The thoracic aorta progressively becomes a more posterior anatomical structure (closer in proximity to the esophagus) as it arches
and transitions into the descending aorta. Transthoracic echocardiography (TTE) cannot adequately visualize the thoracic aorta (especially the
aortic isthmus) whereas transesophageal echocardiography (TEE) can.

Educational objective:
Rapid deceleration may cause blunt thoracic aortic injury. Clinical suspicion from mechanism of injury, signs of blunt chest trauma, and/or
abnormal chest x-ray (eg, widened mediastinum, abnormal aortic contour) should prompt CT angiography in hemodynamically stable patients.
Hemodynamically unstable patients should proceed directly to surgery.

References
Blunt trauma of the aorta, current guidelines.

(http://www.ncbi.nlm.nih.gov/pubmed/28683912)
Do patients with blunt thoracic aortic injury present to hospital with unstable vital signs? A systematic review and meta-analysis.

(http://www.ncbi.nlm.nih.gov/pubmed/29440235)

Surgery Cardiovascular System Blunt thoracic trauma


Subject System Topic

https://t.me/USMLEWorldStep2CK

REVIEW

https://www.uworld.com/ClientApp/v15/apps/qbanktestinterface/index.html#/launchtest/7281878/nbme/229400739/3/1 1/1
‫ م‬9:21 2021/‫‏‬9/‫‏‬11 https://t.me/USMLEWorldStep2CK UWorld STEP2 SIM Form 1

1 Item 13 of 40
2
Mark
Question Id: 6876 Previous Next
3

4 A 25-year-old woman comes to the office due to a painful bump in the genital area. The pain is especially bothersome when she sits, and she
5 cannot have sexual intercourse because of excruciating pain. The patient has multiple sexual partners and typically does not use barrier
6 contraception. She takes oral contraceptives and has no chronic medical conditions. The patient smokes a pack of cigarettes a day and drinks
7 alcohol on weekends. Temperature is 36.6 C (97.9 F), blood pressure is 122/78 mm Hg, and pulse is 90/min. Examination shows a tender, 3-cm,
8 fluctuant mass at the base of the right labium majus, bulging medially into the vagina. Speculum examination is limited by patient discomfort, but
9 no abnormal discharge is seen at the cervical os. The remainder of the pelvic examination is normal. Which of the following is the best treatment
10 for this patient?
11
 A. Acyclovir (0%)
12
13 B. Biopsy of the lesion (1%)
14
C. Ceftriaxone plus doxycycline (0%)
15
16  D. Incision and drainage (96%)
17 E. Wide local excision (0%)
18

19
20

21
Incorrect 96% 02 secs 03/29/2021
22
Correct answer
D
 Answered correctly  Time Spent  Last Updated

23

24
25 Explanation
26

27

28

29
30
31

32

33
34

35

36

37

38

39

40

The Bartholin glands are located at the posterior vaginal introitus and have ducts that drain into the vulvar vestibule at the 4 and 8 o'clock positions
for vulvovaginal lubrication. When a duct becomes obstructed, a Bartholin duct cyst can develop. Infection of the cyst results in a Bartholin
gland abscess, as in this patient. Infection is typically due to methicillin-resistant Staphylococcus aureus or Escherichia coli, and patients
typically develop a gradually enlarging labial mass that causes pain with sitting or sexual intercourse. Fever may be present but is not common.
Examination shows a tender, fluctuant mass at the base of the labium majus that protrudes into the vagina.

Initial treatment is with incision and drainage and fluid culture. A Word catheter is placed in the abscess cavity to form a fistulous tract, which
improves ductal drainage and reduces the risk of cyst/abscess recurrence. Antibiotics are typically not indicated but may be used for patients with
a high risk of complications (eg, systemic infection, pregnancy, immunosuppression). Marsupialization, a procedure that creates a permanent
opening for drainage of the Bartholin gland, is more invasive and is reserved for patients with recurrent abscesses.

(Choice A) Acyclovir is used to treat herpes simplex virus infections, which present as multiple vesicular and ulcerative lesions rather than a
single fluctuant mass.

(Choice B) Biopsy of the lesion is indicated if a Bartholin gland carcinoma is suspected. In contrast to this patient, those with Bartholin gland
carcinoma are typically older (age ~60) and have a solid, fixed lesion.

(Choice C) Ceftriaxone plus doxycycline is first-line, empiric therapy for acute cervicitis due to Neisseria gonorrhoeae or Chlamydia trachomatis
infection. Although this patient has risk factors that warrant testing for sexually transmitted infections, she does not have mucopurulent cervical
discharge, postcoital bleeding, or cervical friability that warrants empiric treatment.

(Choice E) Wide local excision is performed for patients with vulvar cancer, which typically causes a pruritic, friable ulcer or plaque rather than a
painful, fluctuant mass.

Educational objective:
A Bartholin gland abscess results from infection secondary to a blocked Bartholin gland duct. Patients typically have vulvar pain and a tender,
fluctuant mass at the base of the labium majus that protrudes into the vagina. Treatment is with incision and drainage; antibiotics are typically not
indicated.

References
Word balloon catheter for Bartholin's cyst and abscess as an office procedure: clinical time gained.

(http://www.ncbi.nlm.nih.gov/pubmed/26740004)
Clinical Pathology of Bartholin's glands: a review of the literature.

(http://www.ncbi.nlm.nih.gov/pubmed/26195958)

Obstetrics & Gynecology Female Reproductive System & Breast Bartholin duct cyst and abscess
Subject System Topic

https://t.me/USMLEWorldStep2CK

REVIEW

https://www.uworld.com/ClientApp/v15/apps/qbanktestinterface/index.html#/launchtest/7281878/nbme/229400739/3/1 1/1
‫ م‬9:21 2021/‫‏‬9/‫‏‬11 https://t.me/USMLEWorldStep2CK UWorld STEP2 SIM Form 1

1 Item 14 of 40
2
Mark
Question Id: 6882 Previous Next
3

4 A 26-year-old man is brought to the emergency department by police after being found shouting and behaving aggressively in the street. He is
5 wearing a suit and tie but is rather disheveled. During the interview, he says, "I'm great. They're just jealous. They know I can make more money
6 than they can. That's why they brought me here." During the physical examination, the patient is unable to stop moving and taps his feet
7 constantly. His temperature is 99.1 F (37.3 C), blood pressure is 168/97 mm Hg, and pulse is 121/min. Aside from dilated pupils, the remainder of
8 the examination is normal. He is kept overnight and the next morning his symptoms and vital sign abnormalities have resolved. The patient
9 apologizes for his behavior and asks to be released to go to work. He has had multiple similar emergency department visits over the past year.
10 He says he has no psychiatric history but adds that he always becomes severely depressed for 1-2 days after these episodes. Which of the
11 following would be the most appropriate long-term treatment for this patient?
12
 A. 12-step and individual therapy (64%)
13

14 B. Disulfiram (1%)
15
C. Methadone (5%)
16

17 D. Naltrexone (1%)
18 E. Olanzapine (5%)
19
F. Sertraline (2%)
20

21 G. Valproic acid (19%)


22

23

24
64% 02 secs 07/05/2021
25 Correct  Answered correctly  Time Spent  Last Updated
26

27

28
Explanation
29
30
This patient's euphoria, psychomotor agitation, dilated pupils, tachycardia, and hypertension suggest cocaine intoxication. His rapid improvement
31
without treatment and history of acute, short-lived depression following a binge (cocaine crash) are characteristic. Acute cocaine intoxication is
32
treated supportively and withdrawal is self-limited. Long-term treatment of cocaine use disorder consists of psychosocial interventions,
33
including individual psychotherapy and 12-step groups (eg, Cocaine Anonymous, Narcotics Anonymous). Although this is an active area of
34
research, no medication has shown consistent efficacy for cocaine use disorder.
35
(Choice B) Disulfiram is an aversion therapy for alcohol use disorders; it induces nausea and vomiting when alcohol is consumed concurrently.
36
Disulfiram is not useful for treatment of cocaine use disorder.
37

38 (Choice C) Methadone is used for maintenance treatment of opioid dependence. It has no utility in cocaine use disorder.
39 (Choice D) Naltrexone, an opioid receptor antagonist, is part of the treatment for opioid and alcohol dependence. It helps reduce cravings but
40 has not been shown to be effective for cocaine dependence.

(Choice E) Although the patient was delusional, his psychosis resolved rapidly as the effects of cocaine wore off. Although an antipsychotic such
as olanzapine may have been useful immediately to decrease agitation, it is not needed for long-term therapy.

(Choices F and G) Sertraline, a selective serotonin reuptake inhibitor, is used to treat depression; valproic acid is an anticonvulsant used as a
mood stabilizer. Although the patient exhibits some manic behaviors while intoxicated and acute depression during withdrawal, these symptoms
are self-limited. He has no symptoms independent of his drug use that would suggest a primary mood disorder.

Educational objective:
There is no pharmacologic treatment for cocaine use disorder. Individual and group psychotherapies are most effective.

References
Review of treatment for cocaine dependence.

(http://www.ncbi.nlm.nih.gov/pubmed/20088819)

Psychiatry Psychiatric/Behavioral & Substance Abuse Substance use disorders


Subject System Topic

https://t.me/USMLEWorldStep2CK

REVIEW

https://www.uworld.com/ClientApp/v15/apps/qbanktestinterface/index.html#/launchtest/7281878/nbme/229400739/3/1 1/1
‫ م‬9:21 2021/‫‏‬9/‫‏‬11 https://t.me/USMLEWorldStep2CK UWorld STEP2 SIM Form 1

1 Item 15 of 40
2
Mark
Question Id: 6883 Previous Next
3

4 A 45-year-old woman comes to the office due to fatigue and weight loss. She has lost 9 kg (20 lb) over the last 6 months. In addition, she has
5 had palpitations, diaphoresis, subjective dyspnea, and amenorrhea during that time. Medical history is notable for vitiligo and 2 uncomplicated
6 cesarean deliveries. The patient has a 25-pack-year smoking history but quit a month ago. Blood pressure is 150/90 mm Hg and pulse is 95/min
7 at rest and 130/min during ambulation. There are no orthostatic blood pressure changes. On physical examination, the sclera is seen above the
8 iris on downward gaze. Separation of the distal part of the fingernails from the nail beds is noted, along with bulbous enlargement of the
9 fingertips. Laboratory results are as follows:
10
Sodium 138 mEq/L
11
12 Potassium 4.1 mEq/L
13 Chloride 102 mEq/L
14
Bicarbonate 24 mEq/L
15
16 Blood urea nitrogen 10 mg/dL
17
Creatinine 0.9 mg/dL
18

19
Glucose 95 mg/dL

20 Calcium 10.3 mg/dL


21

22 Which of the following is the most likely location of this patient's pathology?
23

24  A. Adrenal cortex (0%)


25 B. Adrenal medulla (2%)
26
C. Lung (28%)
27

28 D. Pancreatic islets (0%)


29
E. Parathyroid gland (0%)
30
31  F. Thyroid gland (67%)
32

33
34
Incorrect 67%
35 Correct answer
F
 Answered correctly  02 secs
Time Spent  09/09/2021
Last Updated
36

37

38
Explanation
39

40

This patient has a number of features suggesting hyperthyroidism, including weight loss, palpitations, hypertension, menstrual irregularities,
onycholysis (separation of the nail from the nail bed), and lid lag (sclera seen above iris on downward gaze). Mild hypercalcemia is also common
in hyperthyroidism due to increased bone turnover.

The most common cause of hyperthyroidism is Graves disease, which characteristically presents with a diffuse goiter and ophthalmopathy
(proptosis, periorbital edema, extraocular muscle weakness). Nail clubbing (bulbous fingertip enlargement) in the setting of hyperthyroidism,
termed thyroid acropachy, is an uncommon but specific sign of Graves disease. This patient also has vitiligo, which is a visible marker of
autoimmune pathology and suggests Graves disease as the cause of her hyperthyroidism.

(Choice A) Adrenocortical tumors can cause hypertension due to overproduction of steroid hormones. However, this patient does not have the
other expected features of glucocorticoid (eg, weight gain, stria), androgen (eg, hirsutism, virilization), or mineralocorticoid (eg, hypokalemia,
metabolic alkalosis) excess.

(Choice B) Pheochromocytoma is a catecholamine-producing tumor of the adrenal medulla. It commonly presents with hypertension,
palpitations, diaphoresis, and weight loss. However, this patient's lid lag and nail changes are more consistent with Graves disease.

(Choice C) Digital clubbing is a characteristic feature of a number of smoking-related pulmonary diseases (eg, malignancies, bronchiectasis,
cavitary lung infections). However, lid lag is a more typical feature of hyperthyroidism, and vitiligo increases the likelihood of autoimmune thyroid
disease.

(Choice D) Pancreatic islet cell tumors can secrete insulin (hypoglycemia), glucagon (hyperglycemia), somatostatin (hyperglycemia, diarrhea),
gastrin (peptic ulcer), or vasoactive intestinal peptide (diarrhea). This patient does not have typical symptoms for any of these rare tumors.

(Choice E) Hyperparathyroidism can cause hypercalcemia but does not explain this patient's other symptoms (eg, lid lag, nail changes).

Educational objective:
Hyperthyroidism causes manifestations in multiple systems, including weight loss, palpitations, hypertension, menstrual irregularities, onycholysis,
lid lag, and hypercalcemia. Graves disease is the most common cause of hyperthyroidism and typically presents with a diffuse goiter and
ophthalmopathy.

References
Diagnosis and classification of Graves' disease.

(http://www.ncbi.nlm.nih.gov/pubmed/24424182)

Medicine Endocrine, Diabetes & Metabolism Hyperthyroidism


Subject System Topic

https://t.me/USMLEWorldStep2CK

REVIEW

https://www.uworld.com/ClientApp/v15/apps/qbanktestinterface/index.html#/launchtest/7281878/nbme/229400739/3/1 1/1
‫ م‬9:21 2021/‫‏‬9/‫‏‬11 https://t.me/USMLEWorldStep2CK UWorld STEP2 SIM Form 1

1 Item 16 of 40
2
Mark
Question Id: 20543 Previous Next
3

4 A phase I clinical trial is conducted to evaluate the nutritional status of patients with locally advanced pancreatic cancer undergoing
5 chemoradiotherapy who received baseline nutritional assessment and counseling. Fourteen newly diagnosed patients receiving
6 chemoradiotherapy are assessed for baseline clinical nutrition measures (BMI, albumin, weight loss, total energy, and protein intake). Participants
7 completed the self-administered Functional Assessment of Anorexia/Cachexia Treatment (FAACT) questionnaire Anorexia/Cachexia Subscale
8 (A/CS) schedule at baseline and at week 6 of treatment. Which of the following statistical tests is most appropriate to determine whether there is a
9 significant change in A/CS scores from baseline to week 6 of treatment?
10
 A. Analysis of variance (14%)
11
12 B. Chi-square test (18%)
13
C. Correlation analysis (12%)
14

15  D. Dependent samples t-test (38%)


16
E. Independent samples t-test (16%)
17
18

19
20
Incorrect 38%
21
Correct answer
D
 Answered correctly  02 secs
Time Spent  06/06/2021
Last Updated

22

23

24 Explanation
25

26
Dependent variable
27

28 Qualitative (categorical) Quantitative


29 Qualitative (categorical) Chi-square, logistic regression* t-test, ANOVA, linear regression
30 Independent variable
Quantitative Logistic regression* Correlation, linear regression
31

32 * Dependent variable must be dichotomous.


33 ANOVA = analysis of variance.
34
Variables are broadly classified as qualitative (ie, categorical) or quantitative (ie, continuous) based on their scale of measurement. Qualitative
35
variables (eg, type of treatment, blood type) represent categories or groups whereas quantitative variables (eg, temperature, glucose levels)
36
represent numerical values. The scale of measurement of the dependent (eg, outcome) and independent (eg, exposures, risk factors) variables in
37
a study determines the correct statistical test for any given situation.
38

39
A t-test compares the mean of 2 groups. It requires that a quantitative dependent variable (ie, outcome) be evaluated in 2 groups formed by the
levels of a categorical independent variable (ie, exposure). An independent samples t-test is used when groups are independent, and a paired t-
40
test is used when groups are dependent. Dependent (ie, matched, paired) groups result when each observation from one group is paired with
an observation from the other group. Examples include a study in which individuals have been assessed twice (eg, before and after an
intervention) and one in which 2 groups of individuals have been matched based on certain attributes (eg, age, severity of disease).

In this study:

The quantitative dependent variable was the Anorexia/Cachexia Subscale (A/CS) score.

The categorical independent variable was the assessment time (with categories "baseline" and "week 6").

A/CS scores are assessed twice for each patient (at baseline and at week 6), so the samples are dependent (Choice E).

A paired t-test will determine whether there is a statistically significant difference in mean A/CS scores between baseline and week 6 of treatment
(before and after treatment). A large, statistically significant difference in these mean scores indicates that the chemoradiotherapy treatment is
associated with changes in A/CS scores (ie, the null hypothesis is rejected).

(Choice A) The analysis of variance (ANOVA) test compares the mean of ≥3 independent groups, as in a study comparing serum ferritin
concentrations (ie, quantitative variable) in children (age 0-17), adults (age 18-59), and seniors (age ≥60).

(Choice B) The chi-square test evaluates the association between 2 categorical variables, as in a study evaluating the association between sex
(ie, male and female) and myocardial infarction (ie, presence or absence).

(Choice C) A correlation analysis uses the correlation coefficient to describe the linear relationship between 2 quantitative variables, as in a study
evaluating the linear relationship between stress level and irritability score.

Educational objective:
The paired t-test compares the mean of 2 related groups. The test requires that a quantitative dependent variable (ie, outcome) be evaluated in 2
related (ie, matched, paired) groups.

Medicine Biostatistics & Epidemiology Statistical tests


Subject System Topic

https://t.me/USMLEWorldStep2CK

REVIEW

https://www.uworld.com/ClientApp/v15/apps/qbanktestinterface/index.html#/launchtest/7281878/nbme/229400739/3/1 1/1
‫ م‬9:21 2021/‫‏‬9/‫‏‬11 https://t.me/USMLEWorldStep2CK UWorld STEP2 SIM Form 1

1 Item 17 of 40
2
Mark
Question Id: 6889 Previous Next
3

4 A 35-year-old homeless man is found unconscious on the street. His friends report that he was recently released from prison after serving time for
5 driving while intoxicated and vehicular homicide. The patient has a history of abusing alcohol and multiple illicit substances and a history of
6 previous drug overdoses. His blood pressure is 110/60 mm Hg and heart rate is 60/min. Pupils are equal and normal in size. There is no
7 evidence of bruising or trauma on physical examination. His arterial blood gases indicate respiratory acidosis. Which of the following are most
8 likely to be present in this patient?
9
 A. Respiratory rate <12/min, decreased tidal volume, hyperthermia, decreased bowel sounds (5%)
10
11  B. Respiratory rate <12/min, decreased tidal volume, hypothermia, decreased bowel sounds (81%)
12
C. Respiratory rate <12/min, increased tidal volume, hypothermia, decreased bowel sounds (8%)
13

14 D. Respiratory rate >12/min, decreased tidal volume, hyperthermia, decreased bowel sounds (1%)
15
E. Respiratory rate >12/min, decreased tidal volume, hypothermia, decreased bowel sounds (3%)
16

17
F. Respiratory rate >12/min, increased tidal volume, hypothermia, increased bowel sounds (0%)

18

19
20 Incorrect 81%
21 Correct answer  Answered correctly  02 secs
Time Spent  04/30/2021
Last Updated
B
22

23

24
Explanation
25

26 This patient's unconscious state, history of drug abuse and overdose, and respiratory acidosis are suggestive of opioid overdose. Opioid
27 intoxication results in a reduction in both respiratory rate and tidal volume, which can severely suppress minute ventilation and cause CO2
28 retention. On arterial blood gas sampling, the decreased ventilation will lead to findings of respiratory acidosis, which consist of a low pH and an
29 elevated pCO2. Patients who have had periods of abstinence (eg, during incarceration) are at increased risk given loss of previous tolerance to
30 the respiratory depressant effects of opioids.
31
Miosis is a classic sign of opioid intoxication. However, it is important to remember that normal pupil size does not exclude opioid
32
intoxication as patients frequently take other drugs (eg, sympathomimetics) in addition to opioids that may dilate the pupils. Hypotension,
33
hypothermia, and decreased bowel sounds may also be seen.
34
(Choices A, C, D, E, and F) Increased respiratory rate, increased tidal volume, hyperthermia, and increased bowel sounds are not seen in
35
patients with opioid intoxication or overdose.
36

37 Educational objective:
38 Opioid intoxication should be suspected in any unresponsive patient with a history of substance abuse and overdose. Both respiratory rate and
39 tidal volume are decreased in patients with an opioid overdose and lead to respiratory acidosis. Hypotension, hypothermia, miosis, and decreased
40
bowel sounds may also be present.

References
Acute respiratory failure from abused substances.

(http://www.ncbi.nlm.nih.gov/pubmed/15296619)

Psychiatry Psychiatric/Behavioral & Substance Abuse Opioids


Subject System Topic

https://t.me/USMLEWorldStep2CK

REVIEW

https://www.uworld.com/ClientApp/v15/apps/qbanktestinterface/index.html#/launchtest/7281878/nbme/229400739/3/1 1/1
‫ م‬9:21 2021/‫‏‬9/‫‏‬11 https://t.me/USMLEWorldStep2CK UWorld STEP2 SIM Form 1

1 Item 18 of 40
2
Mark
Question Id: 6893 Previous Next
3

4 A 26-year-old woman comes to the emergency department due to a severe, left-sided, throbbing headache for the past several hours. She also
5 has nausea and has vomited twice since the pain began. The patient has had similar headaches in the past but never this severe. While at
6 home, she took one of her sister's migraine medication without any relief. She has no other medical history. The patient's vital signs are within
7 normal limits. Physical examination is normal. Noncontrast CT scan of the head is normal. A diagnosis of migraine without aura is made, and the
8 patient receives a single dose of sumatriptan. Several hours later, she becomes disoriented and has a generalized tonic-clonic seizure. Blood
9 pressure is 220/110 mm Hg and pulse is 120/min. This patient's current condition is most likely due to an interaction of sumatriptan with which of
10 the following medications?
11
 A. Dexamethasone (0%)
12
13 B. Diclofenac (2%)
14
 C. Ergotamine (71%)
15
16 D. Metoclopramide (11%)
17 E. Promethazine (13%)
18

19
20

21
Incorrect 71% 03 secs 06/28/2021
22
Correct answer
C
 Answered correctly  Time Spent  Last Updated

23

24
25 Explanation
26

27
Migraine therapies
28

29
Triptans (eg, sumatriptan)

30
NSAIDs (eg, naproxen)
Abortives Acetaminophen
31
Antiemetics (eg, metoclopramide, prochlorperazine)
32
Ergotamines (eg, dihydroergotamine)
33
34 Topiramate
35 Divalproex sodium
Preventives
36 Tricyclic antidepressants
37 Beta blockers (eg, propranolol)
38 NSAIDs = nonsteroidal anti-inflammatory drugs.
39

40
This patient with recurrent episodes of severe, left-sided, throbbing headache associated with nausea and vomiting was diagnosed with migraine
headaches and given the migraine-abortive medication sumatriptan. It is a powerful vascular serotonin (5-hydroxytryptamine [5-HT]) agonist
that causes vasoconstriction and decreases neurogenic inflammation.

Patients who have already received a triptan or an ergot derivative (also a 5-HT agonist) within the past 24 hours (as this patient may have from
her sister) should not receive another triptan as this combination can cause severe, prolonged vasoconstriction due to the overactivation of
serotonin receptors, resulting in elevated blood pressure, myocardial infarction, or stroke.

Treatment of an acute migraine focuses on abortive therapy to decrease the severity and length of symptoms. In addition to triptans and ergots,
there are many other effective abortive medications, including acetaminophen, nonsteroidal anti-inflammatory drugs (NSAIDs), and antiemetics.
All are most effective when used at the onset of symptoms.

(Choice A) Dexamethasone is a glucocorticoid that can safely be given as a single dose with other abortive migraine medications to decrease the
risk of early headache recurrence without significant side effects or interaction with triptans.

(Choice B) Diclofenac is an NSAID that is most often used to treat pain associated with arthritis, but it can also be combined safely with triptans
to treat acute migraines.

(Choices D and E) Metoclopramide and promethazine are antiemetics effective in the treatment of acute migraines; they can be used either as
monotherapy or as adjuvants with NSAIDs or triptans. When combined with triptans, a single dose is usually sufficient, and unlikely to cause
serotonin syndrome. This therapy is generally safe and may be especially helpful in patients with significant nausea or vomiting.

Educational objective:
Coadministration of a triptan and an ergot derivative or an additional triptan after a first dose may result in prolonged vasospasm due to
overactivation of serotonin receptors, which can lead to severe elevations in blood pressure, myocardial infarction, or stroke.

References
Promethazine plus sumatriptan in the treatment of migraine: a randomized clinical trial.

(http://www.ncbi.nlm.nih.gov/pubmed/24182419)
Sumatriptan plus naproxen for acute migraine attacks in adults.

(http://www.ncbi.nlm.nih.gov/pubmed/24142431)

Medicine Nervous System Migraine


Subject System Topic

https://t.me/USMLEWorldStep2CK

REVIEW

https://www.uworld.com/ClientApp/v15/apps/qbanktestinterface/index.html#/launchtest/7281878/nbme/229400739/3/1 1/1
‫ م‬9:22 2021/‫‏‬9/‫‏‬11 https://t.me/USMLEWorldStep2CK UWorld STEP2 SIM Form 1

1 Item 19 of 40
2
Mark
Question Id: 6895 Previous Next
3

4 A 4-year-old boy is brought to the emergency department for evaluation of a skin rash that developed over the past 24 hours. He has had fevers
5 for 5 days. He has a history of epilepsy, and valproic acid therapy was started 4 months ago for seizure management. His parents have chosen
6 to delay his immunizations, and he has no known allergies to medication. Temperature is 40 C (104 F), blood pressure is 90/60 mm Hg, and pulse
7 is 138/min. Physical examination shows bilateral, bulbar conjunctival erythema with limbic sparing and no exudate. Mucous membranes are dry,
8 and the tongue is erythematous with prominent papillae. The posterior oropharynx appears normal. There is erythema of both palms and a faint,
9 erythematous rash on the extremities and trunk. There is prominent bilateral, anterior cervical lymphadenopathy. Laboratory results are as
10 follows:
11
Complete blood count
12
13 Hemoglobin 11.2 g/dL
14 Platelets 420,000/mm3
15
Leukocytes 14,000/mm3
16

17 Neutrophils 75%
18

19
Liver function studies

20 Aspartate aminotransferase (SGOT) 62 U/L


21
Alanine aminotransferase (SGPT) 74 U/L
22

23 Which of the following is the most important next step in management of this patient?
24
25  A. Discontinuation of valproic acid (5%)
26 B. Intravenous immunoglobulin (66%)

27
C. Oral doxycycline (1%)
28

29 D. Oral penicillin (8%)


30
E. Oral vitamin A (10%)
31

32 F. Reassurance and supportive care (7%)


33
34

35
Incorrect 66%
36 Correct answer  Answered correctly  03 secs
Time Spent  06/24/2021
Last Updated
37 B

38

39
Explanation
40

Kawasaki disease
Medium-vessel vasculitis
Pathophysiology & epidemiology Usually affects children age <5
↑ Incidence in East Asian ethnicity

Fever ≥5 days plus ≥4 of the following:


Conjunctivitis: bilateral, nonexudative
Mucositis: injected/fissured lips or pharynx, strawberry tongue
Diagnostic criteria
Cervical lymphadenopathy: ≥1 node >1.5 cm
Rash: perineal erythema & desquamation; polymorphous, generalized
Erythema & edema of the hands/feet, periungual desquamation

↑ Platelets & WBCs; ↓ hemoglobin


↑ Acute-phase reactants (eg, C-reactive protein)
Laboratory findings
↑ AST & ALT
Sterile pyuria

Treatment Intravenous immunoglobulin & aspirin

Coronary artery aneurysms


Complications
Ventricular dysfunction

ALT = alanine transaminase; AST = aspartate transaminase; WBCs = white blood cells.

This patient meets diagnostic criteria for Kawasaki disease (KD) with the following findings: fever for ≥5 days plus conjunctivitis, mucositis (eg,
erythematous tongue), rash, and palmar erythema. A >1.5-cm cervical lymph node may also be present but is not necessary for diagnosis given
the patient's other symptoms.

Initial laboratory findings (<7 days after symptom onset) include neutrophilia, normocytic anemia, and elevated transaminases, as seen here.
Markers of systemic inflammation, including thrombocytosis, leukocytosis with lymphocytosis, and elevated acute-phase reactants, typically
develop after a week of fever.

Although symptoms of KD are self-limiting and typically resolve within 2 weeks, associated coronary artery aneurysms can be life-threatening (ie,
cardiac ischemia, arterial thrombosis). A single dose of intravenous immunoglobulin, an anti-inflammatory therapy, administered ≤10 days of
fever onset significantly decreases the incidence of coronary artery aneurysms and should be administered to all patients with KD. Although
typically avoided in children due to the risk of Reye syndrome, aspirin is also given for its anti-inflammatory and antiplatelet effects.

(Choice A) In rare cases, valproic acid can cause Stevens-Johnson syndrome, which presents with fever, rash, and mucosal changes. Unlike in
this case, the rash consists of bullae and skin sloughing, and mucosal involvement includes exudative conjunctivitis and hemorrhagic oral lesions.
Moreover, symptoms usually occur <4 weeks after medication initiation, not months later.

(Choice C) Doxycycline treats Rocky Mountain spotted fever, a tick-borne illness that causes fever and rash. However, headache and
gastrointestinal symptoms (eg, abdominal pain, vomiting) are prominent initial features, and patients classically have thrombocytopenia.
Moreover, mucositis is not seen.

(Choice D) Penicillin treats scarlet fever, which, unlike in this case, presents with a sandpaper-textured rash and posterior pharyngeal erythema
with tonsillar exudate. In addition, elevated transaminases would not be seen.

(Choice E) Vitamin A decreases morbidity and mortality in hospitalized patients with measles. Although measles causes conjunctivitis, patients
also have cough and coryza. In addition, the measles rash spreads in a cephalocaudal pattern; this patient has no facial rash.

(Choice F) Supportive care is appropriate for pharyngoconjunctival fever, an adenoviral infection that often presents with prolonged fever and
conjunctivitis. However, pharyngitis is expected, and palmar erythema is not. Moreover, laboratory evaluation would show a lymphocytic
predominance.

Educational objective:
Early laboratory findings in Kawasaki disease include neutrophilia, anemia, and elevated transaminases. Prompt treatment with intravenous
immunoglobulin decreases the risk of coronary artery aneurysms.

References
Intravenous immunoglobulin for the treatment of Kawasaki disease.

(http://www.ncbi.nlm.nih.gov/pubmed/28079915)

Pediatrics Rheumatology/Orthopedics & Sports Kawasaki disease


Subject System Topic

https://t.me/USMLEWorldStep2CK

REVIEW

https://www.uworld.com/ClientApp/v15/apps/qbanktestinterface/index.html#/launchtest/7281878/nbme/229400739/3/1 1/1
‫ م‬9:22 2021/‫‏‬9/‫‏‬11 https://t.me/USMLEWorldStep2CK UWorld STEP2 SIM Form 1

1 Item 20 of 40
2
Mark
Question Id: 6897 Previous Next
3

4 A 67-year-old woman comes to the office due to difficulty driving at night for the past several months. She sees halos around streetlights and
5 excessive glare from the headlights of oncoming cars. The patient also has difficulty driving in bright sunlight. Interior daytime vision is fine, and
6 she has needed her reading glasses less often, but reading at night has been worse. Medical history is notable for hypertension and type 2
7 diabetes mellitus. She has a 20-pack-year smoking history but quit 10 years ago. Which of the following examination findings is most associated
8 with the cause of this patient's symptoms?
9
 A. Drusen deposits in the macula (19%)
10
11 B. Hemorrhage in the shape of a flame (1%)
12
C. Increased optic cup/disc ratio (23%)
13

14  D. Loss of the red reflex (50%)


15
E. Retinal microaneurysms (4%)
16

17
18

19 Incorrect 50%
20
Correct answer  Answered correctly  03 secs
Time Spent  06/20/2021
Last Updated
D
21

22

23 Explanation
24
25
Clinical features of age-related cataract
26

27 Age >60
28 Chronic sunlight exposure
29
Diabetes mellitus
Risk factors
30
Glucocorticoid use
Smoking
31
HIV infection
32

33 Gradual loss of visual acuity


34 Symptoms Excessive glare, halos around bright lights
35 Myopic shift
36
Opacification of lens
37 Signs
Loss of red reflex
38

39 This patient most likely has cataracts: progressive opacifications of the lens caused by oxidative damage (eg, from smoking, sunlight exposure).
40 Patients with diabetes mellitus are at especially high risk for cataracts due to the conversion of excess glucose into sorbitol, which accumulates
in the lens and increases cellular osmotic and oxidative stress.

Cataracts typically present with chronic, bilateral loss of visual acuity. Scattering of light within the lens leads to glare and halos around bright
lights, especially at night. Muting of color vision is common, although it is typically noted in retrospect after cataract surgery restores the normal
vibrancy of color. Cataracts can also increase the refractive power of the lens, which some patients may notice attenuates the difficulty in close
vision due to preexisting presbyopia (ie, myopic shift).

Examination typically reveals a visible opacity of the lens and reduced clarity of retinal detail. Advanced cases may also show loss of the red
reflex, the bright red color seen when the posterior retina is illuminated during ophthalmoscopic inspection. Testing shows decreased visual acuity
in both central and peripheral vision.

(Choice A) Drusen are lipoproteinaceous deposits that appear as white or yellow spots, typically clustered around the central retina. Large
numbers suggest age-related macular degeneration, which usually causes increasing difficulty with reading during both the day and the night and
does not cause excessive glare or halos.

(Choice B) Flame hemorrhages are a manifestation of hypertensive retinopathy, which causes acute monocular (or asymmetric) loss of visual
acuity and visual field defects. Headache and other neurologic symptoms are often present. This patient's chronic symptoms are more consistent
with cataracts.

(Choice C) An increased optic cup/disc ratio (ie, cupping) suggests atrophy of the optic nerve due to open-angle glaucoma, which causes gradual
loss of peripheral vision. Glare, halos, and difficulty with night driving are not characteristic.

(Choice E) Microaneurysms are a characteristic feature of diabetic retinopathy. Most patients are asymptomatic until late in the course of the
disease, when vitreous hemorrhage or retinal detachment causes acute vision loss.

Educational objective:
Cataracts present with chronic loss of visual acuity, excessive glare, and halos around bright lights. Ophthalmologic assessment shows a visible
opacity of the lens and reduced clarity of retinal detail. Advanced cases may also show loss of the red reflex.

References
Vision loss in older adults.

(http://www.ncbi.nlm.nih.gov/pubmed/27479624)

Surgery Ophthalmology Cataract


Subject System Topic

https://t.me/USMLEWorldStep2CK

REVIEW

https://www.uworld.com/ClientApp/v15/apps/qbanktestinterface/index.html#/launchtest/7281878/nbme/229400739/3/1 1/1
‫ م‬9:22 2021/‫‏‬9/‫‏‬11 https://t.me/USMLEWorldStep2CK UWorld STEP2 SIM Form 1

1 Item 21 of 40
2
Mark
Question Id: 6898 Previous Next
3

4 A 65-year-old man comes to the office for routine follow-up 3 months after a short hospitalization for pneumonia. He feels well and has no acute
5 problems. The patient has a history of hypertension, hyperlipidemia, and knee osteoarthritis. He underwent an emergency laparotomy 5 years
6 ago for blunt abdominal trauma from a motor vehicle collision. He has been on lisinopril and amlodipine for hypertension since his recent
7 hospitalization. The patient smokes a pack of cigarettes daily and drinks alcohol on weekends. Blood pressure is 160/90 mm Hg and pulse is
8 90/min. Cardiopulmonary auscultation reveals clear lung fields and normal first and second heart sounds. The abdomen is soft, without
9 organomegaly. A well-healed, midline abdominal surgical scar is present. Laboratory results are as follows:
10
Now 3 months ago
11
12 Complete blood count
13 Hemoglobin 14.4 g/dL 14.1 g/dL
14
Platelets 500,000/mm3 490,000/mm3
15
16 Leukocytes 7,000/mm3 14,200/mm3
17
Follow-up chest x-ray today shows complete resolution of the left lower lobe consolidation seen 3 months ago. Which of the following is the most
18
likely cause of this patient's thrombocytosis?
19
20 A. Alcoholic liver disease (0%)

21
B. Drug-induced (5%)
22

23 C. Essential thrombocythemia (46%)


24
 D. Prior surgery (22%)
25

26 E. Recent pneumonia (7%)


27
F. Smoking (15%)
28

29
G. Uncontrolled hypertension (2%)

30
31

32 Incorrect 22%
33 Correct answer  Answered correctly  03 secs
Time Spent  04/25/2021
Last Updated
D
34

35

36
Explanation
37

38 Reactive (secondary) thrombocytosis, caused by cytokines that promote platelet production, is usually driven by an inflammatory state (eg,
39 infection, recent surgery, malignancy); once the inflammatory response quells, reactive thrombocytosis should resolve. This patient had
40 pneumonia, which is sometimes associated with reactive thrombocytosis (Choice E). However, despite resolution in symptoms, lobar infiltrate,
and leukocytosis, his platelet count remains elevated 3 months later.

A thorough history and physical examination often reveal a likely source for thrombocytosis; in this patient's case, a history of blunt abdominal
trauma requiring urgent surgery suggests the underlying cause. Blunt abdominal trauma often injures the spleen, necessitating splenectomy.
Because the spleen removes senescent platelets, patients often have dramatic thrombocytosis (as high as 1,000,000/mm3) following
splenectomy. In most patients, platelet elevations resolve within weeks or months. However, a minority will have persistent thrombocytosis for
years. Although the mechanism is poorly understood, some studies suggest that this process is mediated by a cytokine-driven (reactive) process.

(Choice A) Patients with alcoholic liver disease often have thrombocytopenia (not thrombocytosis) due to decreased hepatic production of
thrombopoietin and/or direct suppressive effects of alcohol on the bone marrow.

(Choice B) Neither lisinopril nor amlodipine causes thrombocytosis.

(Choice C) Essential thrombocythemia is a form of autologous (primary) thrombocytosis, which is cytokine independent. Manifestations include
persistent elevation in platelets >600,000/mm3, thrombosis, and hemorrhage. This patient with mild thrombocytosis and a history of blunt force
abdominal trauma more likely has thrombocytosis from splenectomy.

(Choice F) Although patients who smoke cigarettes may have functional platelet abnormalities that increase risk of thrombosis, thrombocytosis is
uncommon. However, smoking does increase the risk of secondary erythrocytosis.

(Choice G) Hypertension is not typically a causative factor that drives thrombocytosis. Patients with certain forms of thrombocytosis (eg,
essential thrombocythemia) may be at increased risk for hypertension.

Educational objective:
Patients with splenectomy often have secondary thrombocytosis. Typically, platelet counts normalize after several weeks, but a minority of
patients may have thrombocytosis for months or years after surgery.

Surgery Hematology & Oncology Splenectomy


Subject System Topic

https://t.me/USMLEWorldStep2CK

REVIEW

https://www.uworld.com/ClientApp/v15/apps/qbanktestinterface/index.html#/launchtest/7281878/nbme/229400739/3/1 1/1
‫ م‬9:22 2021/‫‏‬9/‫‏‬11 https://t.me/USMLEWorldStep2CK UWorld STEP2 SIM Form 1

1 Item 22 of 40
2
Mark
Question Id: 6900 Previous Next
3

4 A 23-year-old nulligravid woman comes to the office for re-evaluation of continued burning with urination over the past 2 months. A month ago,
5 the patient completed a course of trimethoprim-sulfamethoxazole for similar symptoms but had no relief. She has not noticed any vulvar lesions.
6 The patient is concerned as she has a new sexual partner who she suspects has other partners. She uses combined estrogen/progestin pills for
7 contraception. Her last menstrual period was 10 days ago; menses occur every 26 days with heavy bleeding on the first cycle day. The patient
8 has no medical problems and has received all vaccinations. She has no travel history. She does not use tobacco, alcohol, or illicit drugs. BMI is
9 25 kg/m2. Vital signs are normal. Clean catch urinalysis results are as follows:
10
Specific gravity 1.022
11
12 Protein negative
13 Blood negative
14
White blood cells 50+/hpf
15
16 Nitrite negative
17
Urine culture shows no organisms. If untreated, this patient's condition will most likely result in which of the following?
18

19
 A. Cervical cancer (0%)
20

21  B. Infertility (66%)

22 C. Pyelonephritis (28%)
23
D. Urinary retention (3%)
24
25 E. Vertebral osteomyelitis (0%)
26

27

28
Incorrect 66%
29 Correct answer  Answered correctly  04 secs
Time Spent  03/30/2021
Last Updated
30 B
31

32
Explanation
33
34
Dysuria is a common symptom of urethritis and cystitis and should be evaluated with urinalysis and urine culture. This patient has sterile pyuria,
35
in which a urine specimen has white blood cells (eg, >3/hpf) in the absence of bacteriuria. The most common cause of sterile pyuria with
36
associated urethritis (eg, dysuria, urethral discharge) in a sexually active patient with non-monogamous sexual contacts is Chlamydia trachomatis.
37
Untreated C trachomatis infection can result in ascending infection of the upper genital tract (eg, pelvic inflammatory disease, tubo-ovarian
38
abscess), resulting in inflammation and adhesion formation. Scarring of the Fallopian tubes can lead to long-term consequences (eg, infertility,
39
ectopic pregnancy).
40
(Choice A) Human papillomavirus, the most common sexually transmitted infection, causes condyloma acuminata, cervical dysplasia, or cervical
cancer, not dysuria. Patients age <26 should be offered the human papillomavirus vaccination series to decrease these risks.

(Choice C) Pyelonephritis is a complication of cystitis. Patients with cystitis typically have dysuria and suprapubic pain. Although sterile pyuria
can result from recent antibiotic use, this patient received antibiotics a month ago. In addition, the lack of symptomatic improvement with
appropriate antibiotics for cystitis and the negative urine culture make this diagnosis unlikely.

(Choice D) Urinary retention is a potential complication of herpes simplex virus due to severe dysuria and autonomic nervous system
dysfunction. Although herpes simplex virus can also cause sterile pyuria, the typical painful, ulcerative/vesicular genital lesions are not present in
this patient.

(Choice E) Vertebral osteomyelitis is a complication of genitourinary tuberculosis. Genitourinary tuberculosis may present with sterile pyuria, but
patients typically have risk factors (eg, travel to endemic areas, HIV, incarceration), systemic symptoms (eg, fever, weight loss), and nephrotic-
range proteinuria.

Educational objective:
Chlamydia trachomatis urethritis should be suspected in a young patient with dysuria, sterile pyuria, and non-monogamous sexual contacts.
Untreated chlamydial infections can lead to infertility due to scarring of the Fallopian tubes.

References
Dysuria in the emergency department: missed diagnosis of Chlamydia trachomatis.

(http://www.ncbi.nlm.nih.gov/pubmed/24672617)
Sterile pyuria.

(http://www.ncbi.nlm.nih.gov/pubmed/25760357)

Obstetrics & Gynecology Female Reproductive System & Breast Pelvic inflammatory disease
Subject System Topic

https://t.me/USMLEWorldStep2CK

REVIEW

https://www.uworld.com/ClientApp/v15/apps/qbanktestinterface/index.html#/launchtest/7281878/nbme/229400739/3/1 1/1
‫ م‬9:22 2021/‫‏‬9/‫‏‬11 https://t.me/USMLEWorldStep2CK UWorld STEP2 SIM Form 1

1 Item 23 of 40
2
Mark
Question Id: 6948 Previous Next
3

4 A 63-year-old man comes to the office with shortness of breath on exertion. He can barely walk 1 block without becoming short of breath and has
5 to sleep with multiple pillows because he has been uncomfortable lying flat recently. The patient also has heaviness in his legs and gained 4.5 kg
6 (10 lb) over the last 2 months despite having a decreased appetite. He does not have any chest pain or wheezing but has occasional
7 palpitations. Past medical history is significant for type 2 diabetes mellitus, hypertension, and hyperlipidemia. He sustained an anterior wall
8 myocardial infarction 10 years ago but has had very poor medical follow-up since. He was taking metformin and aspirin in the past but stopped
9 more than 1 year ago. He is an ex-smoker with a 35-pack-year smoking history. He does not drink alcohol and is not allergic to any medications.
10 His blood pressure is 134/85 mm Hg and pulse is 75/min and regular. His body mass index is 30 kg/m2. He is not in acute respiratory distress.
11 Physical examination reveals moist mucous membranes and scattered bibasilar lung crackles. The cardiac point of maximal impulse is displaced
12
to the left and a third heart sound is heard. The abdomen is soft and nontender to palpation. He has 1+ bilateral pitting ankle edema. Fingerstick

13
glucose level is 144 mg/dL and serum creatinine is 0.9 mg/dL. After optimal diuresis with loop diuretics, which of the following medications should
be used for the best long-term outcomes?
14

15
 A. Amiodarone (0%)
16

17 B. Amlodipine (2%)
18 C. Atenolol (15%)
19
D. Chlorthalidone (5%)
20

21 E. Digoxin (4%)
22
F. Diltiazem (1%)
23

24  G. Metoprolol succinate (68%)


25
H. Propafenone (1%)
26

27

28

29 Incorrect 68%
30
Correct answer
G
 Answered correctly  02 secs
Time Spent  06/02/2021
Last Updated

31

32

33 Explanation
34

35
This patient's presentation – progressive dyspnea, orthopnea, lower extremity edema, and bibasilar lung crackles – is consistent with

36
decompensated congestive heart failure (CHF). It is most likely due to ischemic heart disease and prior myocardial infarction, leading to dilated
left ventricular (LV) cavity and systolic LV dysfunction. Medications that improve long-term survival in patients with LV systolic dysfunction
37
include angiotensin-converting enzyme (ACE) inhibitors, angiotensin II receptor blockers (ARBs), beta-blockers, aldosterone
38
antagonists, and a combination of hydralazine and nitrates in African-American patients.
39

40 Certain beta-blockers (metoprolol succinate, carvedilol, bisoprolol) have been shown to improve symptoms and overall long-term survival in
stable patients with heart failure and LV systolic dysfunction (LV ejection fraction <40%). Atenolol is occasionally used in this setting; however, it
has not been specifically evaluated in clinical trials (Choice C). Beta-blockers with intrinsic sympathomimetic activity (pindolol, acebutolol) should
be avoided. Beta-blockers are contraindicated in patients with bradycardia (heart rate <60/min), second- or third-degree AV block, symptomatic
hypotension, and history of asthma or reactive airway disease.

(Choice A) Amiodarone is the preferred antiarrhythmic drug to manage ventricular arrhythmias in patients with heart failure and/or systolic LV
dysfunction. However, there is no indication for its use at this point, and it does not improve any long-term outcomes.

(Choices B and F) Calcium channel blockers (amlodipine, diltiazem, verapamil) have not been shown to provide any short or long-term benefit in
patients with heart failure and are not recommended in patients with heart failure due to systolic LV dysfunction.

(Choice D) Chlorthalidone is a thiazide diuretic that is primarily used for hypertension. It may provide a synergistic effect with loop diuretics in
patients with persistent fluid retention, but does not provide any long-term mortality benefit.

(Choice E) Digoxin is used as an adjunctive therapy for symptom control in patients with heart failure and LV systolic dysfunction. It reduces the
rate of hospitalization for heart failure but has not been shown to improve overall survival.

(Choice H) Propafenone is a class IC antiarrhythmic drug that is occasionally used in patients with paroxysmal atrial fibrillation. It has a negative
inotropic effect and should be avoided in patients with heart failure. It is also contraindicated in patients with coronary heart disease due to
increased risk of proarrhythmia and death.

Educational objective:
Certain beta-blockers (metoprolol succinate, carvedilol, bisoprolol) have been shown to improve symptoms and overall long-term survival in
stable patients with heart failure and left ventricular (LV) systolic dysfunction (LV ejection fraction <40%).

References
2013 ACCF/AHA guideline for the management of heart failure: executive summary: a report of the American College of Cardiology
Foundation/American Heart Association Task Force on practice guidelines.

(http://www.ncbi.nlm.nih.gov/pubmed/23741057)
Beta-blockers in congestive heart failure. A Bayesian meta-analysis.

(http://www.ncbi.nlm.nih.gov/pubmed/11281737)

Medicine Cardiovascular System Acute heart failure


Subject System Topic

https://t.me/USMLEWorldStep2CK

REVIEW

https://www.uworld.com/ClientApp/v15/apps/qbanktestinterface/index.html#/launchtest/7281878/nbme/229400739/3/1 1/1
‫ م‬9:22 2021/‫‏‬9/‫‏‬11 https://t.me/USMLEWorldStep2CK UWorld STEP2 SIM Form 1

1 Item 24 of 40
2
Mark
Question Id: 6954 Previous Next
3

4 A 31-year-old woman comes to the clinic due to amenorrhea for the last 6 months after discontinuing oral contraceptive pills. She also has fatigue
5 and occasional headaches that respond to ibuprofen but has no vaginal discharge, abdominal pain, or vision changes. The patient has gained 3
6 kg (6.6 lb) over the last 4 months despite no changes in diet and exercise habits. She has no chronic medical conditions and has had no
7 surgeries. Family history is significant for breast cancer in her mother that was diagnosed at age 58. Blood pressure is 124/72 mm Hg and pulse
8 is 52/min. BMI is 21 kg/m2. The skin is dry and without lesions. Physical examination shows no masses on breast examination, but nipple
9 compression produces a scant, whitish discharge bilaterally. The remainder of the physical examination, including pelvic and neurologic
10 examination, is normal. Urine pregnancy test is negative. Which of the following is the best next step in management of this patient?
11
A. Diagnostic mammogram (0%)
12
13 B. Endometrial biopsy (0%)
14
C. MRI of the pituitary (34%)
15
16 D. Serum testosterone level (0%)
17 E. Serum TSH level (64%)

18

19
20

21 64% 02 secs 05/06/2021


22
Correct  Answered correctly  Time Spent  Last Updated

23

24
25 Explanation
26

27

28

29
30
31

32

33
34

35

36

37

38

39

40

This patient has secondary amenorrhea, the lack of menses for ≥3 months in women with previously regular menses (or ≥6 months in women with
previously irregular menses). Secondary amenorrhea may be due to pregnancy or hypothalamic, pituitary, ovarian, or uterine disorders. This
patient's amenorrhea, galactorrhea, and metabolic slowing (eg, fatigue, weight gain, bradycardia) are likely due to hypothyroidism.

In patients with hypothyroidism, low thyroxine levels stimulate the secretion of hypothalamic thyrotropin-releasing hormone, which induces
increased TSH and prolactin production in the pituitary; the concomitant hyperprolactinemia may cause headaches and galactorrhea, as in this
patient. In addition, elevated prolactin levels suppress FSH, LH, and estrogen levels (ie, anovulation), causing abnormal uterine bleeding (eg,
amenorrhea). Treatment is with thyroid hormone replacement, which normalizes both thyroxine and prolactin levels.

(Choice A) Breast cancer can cause unilateral serosanguineous nipple discharge. In contrast, bilateral, milky, nonbloody nipple discharge in the
absence of palpable abnormalities is typically benign and does not require further evaluation with diagnostic mammography.

(Choice B) Endometrial biopsy evaluates for intrauterine pathology (eg, polyp, hyperplasia) in patients who have abnormal uterine bleeding and
are at risk for endometrial cancer. Risk factors in women age <45 include obesity (ie, unopposed estrogen), Lynch syndrome, and tamoxifen use.
This patient has none of these risk factors.

(Choice C) A pituitary MRI evaluates sellar masses (eg, prolactinoma) that often present with headaches and galactorrhea. In this patient, the
symptoms of hyperprolactinemia are secondary to hypothyroidism, as indicated by the weight gain and skin changes; therefore, a TSH level is the
best next step.

(Choice D) Serum testosterone levels may be elevated in polycystic ovary syndrome, a common cause of irregular menses. Patients with this
syndrome are typically overweight, unlike this patient with a normal BMI, and there is no associated galactorrhea.

Educational objective:
Hypothyroidism is a common cause of hyperprolactinemia (eg, galactorrhea), abnormal uterine bleeding (eg, amenorrhea), and metabolic slowing
(eg, fatigue, bradycardia). Patients with these clinical features require further evaluation with a TSH level.

References
Evaluation of serum prolactin level in patients of subclinical and overt hypothyroidism.

(http://www.ncbi.nlm.nih.gov/pubmed/25737975)
Prevalence and predictors of hyperprolactinemia in subclinical hypothyroidism.

(http://www.ncbi.nlm.nih.gov/pubmed/27473607)
Primary hypothyroidism with markedly high prolactin.

(http://www.ncbi.nlm.nih.gov/pubmed/27199892)

Medicine Endocrine, Diabetes & Metabolism Amenorrhea


Subject System Topic

https://t.me/USMLEWorldStep2CK

REVIEW

https://www.uworld.com/ClientApp/v15/apps/qbanktestinterface/index.html#/launchtest/7281878/nbme/229400739/3/1 1/1
‫ م‬9:23 2021/‫‏‬9/‫‏‬11 https://t.me/USMLEWorldStep2CK UWorld STEP2 SIM Form 1

1 Item 25 of 40
2
Mark
Question Id: 6955 Previous Next
3

4 A 37-year-old woman comes to the office due to 6 weeks of pain and paresthesia in her hands. The symptoms are present bilaterally but more
5 severe on the right. There are no skin color changes, and the hands feel slightly better when placed under warm running water. The patient
6 works from home as a graphic designer. She has been using a wrist splint over the past 2 weeks with minimal relief of symptoms. Vital signs are
7 normal. BMI is 23 kg/m2. Neck is supple with normal range of motion. There is no lymphadenopathy or thyromegaly. Upper extremity sensation
8 and deep tendon reflexes are normal and symmetric. The patient is instructed to hold her wrists in full flexion with the dorsum of her hands
9 pressed together, and after 1 minute she reports reproduction of the pain and tingling. Which of the following is the most appropriate next step in
10 management of this patient?
11
 A. Local corticosteroid injection (42%)
12
13 B. Oral corticosteroids (1%)
14
C. Oral nonsteroidal anti-inflammatory drugs (36%)
15
16 D. Physical therapy (19%)
17 E. Vitamin B12 supplements (0%)
18

19
20

21
Omitted 42% 0 secs 07/10/2021
22
Correct answer
A
 Answered correctly  Time Spent  Last Updated

23

24
25 Explanation
26

27
Carpal tunnel syndrome
28

29
Obesity

30
Pregnancy
Diabetes
31 Risk factors
Hypothyroidism
32
Rheumatoid arthritis
33
End-stage renal disease/hemodialysis
34

35 Pain & paresthesia in median nerve distribution (first 3½ digits)


36 Positive Phalen test & Tinel sign
Clinical presentation
37 Severe disease: weakness of thumb abduction & opposition, atrophy of thenar
38 eminence

39 Confirmatory test Nerve conduction studies


40
Wrist splinting
Treatment Glucocorticoid injection
Surgery for severe or refractory symptoms

This patient has carpal tunnel syndrome (CTS) due to compression of the median nerve beneath the transverse carpal ligament in the wrist.
CTS causes pain and paresthesia in the first 3 digits and the radial aspect of the fourth; bilateral symptoms are common. CTS is associated with
occupations requiring frequent, strenuous use of the hand and wrist. Prolonged keyboard or mouse work is also a risk factor, although there is no
definitive evidence for a causative link between CTS and light-duty office work. The diagnosis is confirmed with provocative tests: percussion over
the median nerve at the wrist or holding the hands in extreme flexion (Tinel sign and Phalen test, respectively) can reproduce the symptoms.

CTS is often transiently relieved by shaking the hands or holding them under warm water. Initial home interventions include nighttime use of a
wrist splint that holds the wrist in a neutral position and reduces pressure within the tunnel. For patients (such as this one) who fail these initial
measures, local corticosteroid injection (eg, methylprednisolone) is often helpful and can provide rapid relief. A single injection is often
adequate; it may be repeated if necessary, but continued splinting in the interim can often lead to long-term improvement. Surgical
decompression is highly effective in CTS but is usually reserved for patients with severe symptoms or failure of less-invasive management.

(Choices B and C) Oral corticosteroids (eg, prednisone) can be considered for patients who decline injection but are less effective than
corticosteroid injections and carry significant systemic side effects. Oral nonsteroidal anti-inflammatory drugs are ineffective.

(Choice D) Physical/occupational therapy interventions (eg, carpal bone and soft tissue mobilization, ultrasound therapy, nerve gliding exercises)
are primarily used as adjunctive treatment or in recovery following carpal tunnel surgery. Evidence for these interventions as primary therapy is
somewhat limited.

(Choice E) Vitamin B12 deficiency can cause sensory disturbances in the extremities but typically manifests with diminished reflexes, loss of
position and vibratory sense, and neuropsychiatric symptoms.

Educational objective:
Carpal tunnel syndrome causes pain and paresthesia in the distribution of the median nerve in the hand. Initial treatment includes nocturnal wrist
splinting. If splinting fails, corticosteroid injections and surgical decompression are indicated; oral corticosteroids are also helpful but are less
effective and carry significant systemic side effects.

References
Carpal tunnel syndrome: diagnosis and management.

(http://www.ncbi.nlm.nih.gov/pubmed/28075090)

Medicine Rheumatology/Orthopedics & Sports Carpal tunnel syndrome


Subject System Topic

https://t.me/USMLEWorldStep2CK

REVIEW

https://www.uworld.com/ClientApp/v15/apps/qbanktestinterface/index.html#/launchtest/7281878/nbme/229400739/3/1 1/1
‫ م‬9:23 2021/‫‏‬9/‫‏‬11 https://t.me/USMLEWorldStep2CK UWorld STEP2 SIM Form 1

1 Item 26 of 40
2
Mark
Question Id: 6958 Previous Next
3

4 A 22-year-old primigravid woman at 33 weeks gestation comes to the emergency department due to lower abdominal pain occurring every 4-5
5 minutes. She had an episode of bloody vaginal discharge earlier in the morning. At 26 weeks gestation, the patient was diagnosed with
6 gestational diabetes mellitus that has not been well controlled. Temperature is 36.7 C (98.1 F), blood pressure is 130/80 mm Hg, pulse is 78/min,
7 and respirations are 18/min. The cervix is 3 cm dilated and 50% effaced. The patient is given a medication that significantly decreases
8 contractions. A few hours later, she has a headache and nausea and appears flushed on examination. Repeat blood pressure is 96/62 mm Hg
9 and pulse is 98/min. The patient's new symptoms are most likely related to which of the following?
10
A. Beta-adrenergic receptor stimulation (20%)
11
12  B. Calcium channel blockade (48%)
13
C. Circulation of synthetic glucocorticoids (1%)
14

15 D. Cyclooxygenase inhibition (23%)


16
E. Decreased acetylcholine release at the myoneural junction (6%)
17
18

19
20
Omitted 48%
21
Correct answer
B
 Answered correctly  01 sec
Time Spent  04/26/2021
Last Updated

22

23

24 Explanation
25

26
Tocolytics
27

28 Drug Mechanism Side effects


29 Maternal
30 Gastritis
31 Platelet dysfunction
Indomethacin Cyclooxygenase
32 Fetal
(<32 wk) inhibitor
33 Oligohydramnios
34 Closure of ductus
35 arteriosus
36
Maternal
37
Tachycardia/palpitations
38
Nifedipine Calcium channel
Nausea
(32-34 wk) blocker
39 Flushing
40 Headache

Maternal
Tachycardia/palpitations
Terbutaline β-Agonist
Hypotension
Pulmonary edema

This patient at 33 weeks gestation with regular, painful contractions resulting in cervical change is in preterm labor (ie, labor at <37 weeks
gestation). The goal of labor management in patients at <34 weeks gestation is to delay delivery until interventions can be performed for fetal
benefit (eg, antenatal corticosteroids, patient transfer to a tertiary care center). Therefore, medication is typically administered to decrease uterine
contractions (ie, tocolysis).

This patient's flushing, headache, and palpitations are due to nifedipine, the first-line tocolytic agent for patients in preterm labor at 32-34
weeks gestation. Nifedipine is a calcium channel blocker that inhibits calcium entry into muscle cells, which leads to myometrial muscle
relaxation. Nifedipine also causes profound peripheral vasodilation, which can lead to adverse effects such as flushing and headache.
Peripheral vasodilation also results in decreased systemic vascular resistance, which can cause hypotension, tachycardia, and palpitations.

(Choice A) Terbutaline, which stimulates beta-adrenergic receptors, is a short-term tocolytic used to achieve acute myometrial relaxation (eg,
uterine tachysystole during term labor). It is rarely administered for preterm labor (which requires 48 hours of tocolysis) due to high rates of
adverse reactions, including maternal arrhythmia and pulmonary edema. In addition, beta-agonists can worsen maternal hyperglycemia and are
contraindicated in patients with poorly controlled diabetes mellitus.

(Choice C) Glucocorticoids are administered to patients in preterm labor to stimulate fetal lung maturity and prevent neonatal respiratory distress
syndrome. Glucocorticoids do not decrease contractions or cause flushing or hypotension.

(Choice D) Cyclooxygenase inhibitors (eg, indomethacin) are effective tocolytic agents because they decrease prostaglandin synthesis. Adverse
effects include gastritis and platelet dysfunction. In addition, use is restricted to <32 weeks gestation due to increased risk for closure of the fetal
ductus arteriosus and oligohydramnios after 32 weeks gestation.

(Choice E) Magnesium sulfate decreases acetylcholine release at the myoneural junction. Maternal adverse effects include flushing and
headache; however, magnesium sulfate is not typically used for tocolysis because it is less effective than other agents. Instead, it is used primarily
in patients in preterm labor at <32 weeks gestation to decrease the risk of cerebral palsy. Excessive magnesium sulfate administration can also
cause somnolence, decreased deep tendon reflexes, respiratory depression, and cardiac arrest.

Educational objective:
Nifedipine, a calcium channel blocker, is the first-line tocolytic agent for preterm labor at 32-34 weeks gestation. Maternal adverse effects include
flushing, headache, hypotension, tachycardia, and palpitations due to peripheral vasodilation.

References
Calcium channel blockers as tocolytics: principles of their actions, adverse effects and therapeutic combinations.

(http://www.ncbi.nlm.nih.gov/pubmed/24276256)
Practice bulletin no. 159: management of preterm labor.

(http://www.ncbi.nlm.nih.gov/pubmed/26695585)

Obstetrics & Gynecology Pregnancy, Childbirth & Puerperium Preterm labor


Subject System Topic

https://t.me/USMLEWorldStep2CK

REVIEW

https://www.uworld.com/ClientApp/v15/apps/qbanktestinterface/index.html#/launchtest/7281878/nbme/229400739/3/1 1/1
‫ م‬9:23 2021/‫‏‬9/‫‏‬11 https://t.me/USMLEWorldStep2CK UWorld STEP2 SIM Form 1

1 Item 27 of 40
2
Mark
Question Id: 6968 Previous Next
3

4 A 6-week-old African American boy appears to breathe rapidly and becomes diaphoretic after breast feeding for 10-15 minutes. Examination
5 reveals bluish discoloration of his nail beds, lips, and tongue. His extremities are warm. Which of the following is the most likely cause of his
6 physical findings?
7
8
A. Peripheral vasoconstriction (5%)

9 B. High venous pressure (4%)


10
C. Sickling of red blood cells (13%)
11
12 D. Polycythemia (2%)
13
 E. Low arterial oxygen saturation (74%)
14

15
16

17 Omitted 74% 01 sec 07/30/2021


18
Correct answer  Answered correctly  Time Spent  Last Updated
E
19
20

21 Explanation
22

23 The bluish discoloration of this patient’s lips, tongue, and nail beds is suggestive of central cyanosis. Cyanosis occurs when there is a significant
24 amount of deoxygenated hemoglobin in the blood. Central cyanosis should be differentiated from peripheral cyanosis, in which bluish
25
discoloration is usually only seen in the distal extremities. Although central cyanosis typically indicates the presence of low arterial oxygen

26
saturation, patients with peripheral cyanosis have normal arterial oxygen saturation and increased oxygen extraction due to sluggish blood flow
through the capillaries. The extremities are usually cool or clammy in patients with peripheral cyanosis, which is not the case with this patient.
27

28 This patient’s history of tachypnea and diaphoresis while breastfeeding along with his findings of central cyanosis are highly suggestive of
29 congenital heart disease.
30
(Choice A) Peripheral vasoconstriction may lead to peripheral cyanosis, which is inconsistent with this patient’s warm extremities.
31

32
(Choice B) High venous pressure may lead to peripheral cyanosis by creating sluggish blood flow through the capillaries, resulting in increased
33 oxygen extraction.
34

35 (Choice C) Although sickle cell anemia could be considered in this African American patient, the symptoms do not usually start until later in life
36 due to the protective effects of fetal hemoglobin (HbF).
37

38 (Choice D) Polycythemia is typically associated with peripheral cyanosis. Neonatal polycythemia usually presents within the first few days of life.
39
Educational objective:
40 Cyanosis in highly vascularized tissues such as the lips and mucous membranes is consistent with central cyanosis, as opposed to peripheral
cyanosis which usually only involves the distal extremities. Central cyanosis is caused by low arterial oxygen saturation, whereas peripheral
cyanosis is due to increased oxygen extraction secondary to sluggish blood flow.

Pediatrics Pulmonary & Critical Care Congenital cardiac defects


Subject System Topic

https://t.me/USMLEWorldStep2CK

REVIEW

https://www.uworld.com/ClientApp/v15/apps/qbanktestinterface/index.html#/launchtest/7281878/nbme/229400739/3/1 1/1
‫ م‬9:23 2021/‫‏‬9/‫‏‬11 https://t.me/USMLEWorldStep2CK UWorld STEP2 SIM Form 1

1 Item 28 of 40
2
Mark
Question Id: 6972 Previous Next
3

4 A 23-year-old man comes to the emergency department due to severe epigastric pain, nausea and vomiting after a party. He recalls a similar but
5 less severe episode one year ago. The patient has no other medical problems. He takes no medications and does not have any allergies. He is
6 a graduate student. BMI is 32 kg/m2. Physical examination reveals yellowish streaks on his palms. Blood samples appear milky and opalescent.
7 Which of the following medications is most likely to be effective at preventing future episodes of abdominal pain in this patient?
8

9
A. Lovastatin (25%)

10 B. Fenofibrate (40%)

11
C. Cholestyramine (25%)
12
13 D. Ezetimibe (6%)
14
E. Psyllium (1%)
15
16

17
18 Omitted 40% 0 secs 07/30/2021
19
Correct answer
B
 Answered correctly  Time Spent  Last Updated

20

21

22 Explanation
23

24 This patient’s grossly lipemic serum and palmar xanthomas suggest that he has severe hypertriglyceridemia, possibly secondary to familial
25
dysbetalipoproteinemia. Alcohol consumption in the setting of severe hypertriglyceridemia may cause repeated bouts of pancreatitis. Fibric

26
acid derivatives such as fenofibrate can be used to treat hypertriglyceridemia.

27
(Choice A) Statins are competitive inhibitors of 3-hydroxy-3-methylglutaryl coenzyme A (HMG-CoA) reductase, which catalyzes the rate-limiting
28
step in cholesterol biosynthesis. Although statins reduce both LDL and triglyceride levels, fenofibrates are more potent at reducing triglyceride
29
level; therefore, in patients with pure, severe hypertriglyceridemia due to familial dysbetalipoproteinemia, statins are not considered first-line
30
therapy .
31

32
(Choice C) Cholestyramine is a bile acid-binding resin that helps reduce LDL levels. Triglyceride levels may actually be increased by this agent.
33
34
(Choice D) Ezetimibe prevents absorption of dietary cholesterol from the gut. It works best at reducing LDL when used in combination with a
35
statin.
36

37 (Choice E) Psyllium is a bulk-forming laxative useful in treating mild constipation.


38

39 Educational objective:
40 Patients with familial dysbetalipoproteinemia develop hypertriglyceridemia. A classic clinical finding is striate palmar xanthomas. The most
appropriate initial treatment is with a fibric acid derivative.

Medicine Endocrine, Diabetes & Metabolism Dyslipidemia


Subject System Topic

https://t.me/USMLEWorldStep2CK

REVIEW

https://www.uworld.com/ClientApp/v15/apps/qbanktestinterface/index.html#/launchtest/7281878/nbme/229400739/3/1 1/1
‫ م‬9:23 2021/‫‏‬9/‫‏‬11 https://t.me/USMLEWorldStep2CK UWorld STEP2 SIM Form 1

1 Item 29 of 40
2
Mark
Question Id: 6919 Previous Next
3

4 A 29-year-old woman, gravida 2 para 1, at 30 weeks gestation comes to the emergency department for pelvic pressure and contractions. This
5 morning, the patient began to have increased pelvic pressure and some clear vaginal discharge. Over the next few hours, she developed irregular
6 contractions that have now increased in intensity and frequency. The patient had preterm contractions with her last pregnancy but ultimately had a
7 spontaneous vaginal delivery at term. She has no chronic medical conditions, and her only medications are a prenatal vitamin and an iron
8 supplement for pregnancy-related iron deficiency anemia. Temperature is 36.7 C (98.1 F), blood pressure is 120/60 mm Hg, and pulse is 92/min.
9 Fetal heart rate tracing is normal. Uterine contractions are regular and occur every 5 minutes. Cervical examination shows 3 cm dilation, 80%
10 effacement, and a breech presentation at −3 station. On speculum examination, the membranes are intact. Which of the following is the best next
11 step in management of this patient?
12
A. Bed rest (11%)
13

14 B. Cerclage placement (5%)


15
C. Cesarean delivery (17%)
16

17 D. External cephalic version (11%)


18 E. Indomethacin (44%)

19
F. Vaginal progesterone (9%)
20

21

22

23 Omitted 44%
24 Correct answer
E
 Answered correctly  01 sec
Time Spent  04/06/2021
Last Updated
25

26

27 Explanation
28

29
30
Preterm labor
31 Gestational age
Management
32 (weeks)
33
Betamethasone
34
Penicillin if GBS positive or unknown
35 <32
Tocolysis (eg, indomethacin)
36
Magnesium sulfate
37

38
Betamethasone
32 0/7 to 33 6/7 Penicillin if GBS positive or unknown
39
Tocolysis (eg, nifedipine)
40
± Betamethasone
34 0/7 to 36 6/7
Penicillin if GBS positive or unknown

GBS = group B Streptococcus.

This patient with regular uterine contractions causing cervical change at <37 weeks gestation is in preterm labor. The management of preterm
labor varies by gestational age. Because delivery at an earlier gestational age is associated with greater perinatal morbidity and mortality, patients
at <32 weeks gestation (ie, very preterm) require the most intervention, which includes the following:

Antenatal corticosteroids (eg, betamethasone) to stimulate fetal lung maturity, which decreases the risk of neonatal respiratory distress
syndrome.

Penicillin to prevent vertical transmission of group B Streptococcus (GBS). Patients with either positive or unknown GBS status require
penicillin (even though not all are carriers) because the risk of neonatal GBS infection is higher in preterm infants, necessitating this risk-
based approach.

Tocolysis to inhibit contractions and delay delivery (up to 48 hours), which allows time for other interventions to take effect. Indomethacin is
a first-line treatment at <32 weeks gestation due to high efficacy and a reasonable maternal side effect profile. However, it poses greater
fetal risks (eg, premature closure of the ductus arteriosus) at increased gestational ages and therefore is avoided in patients ≥32 weeks
gestation.

Magnesium sulfate for fetal neuroprotection, which decreases the risk for cerebral palsy.

In addition to allowing other medications to take effect, tocolytic therapy allows for patient transfer to a higher level of care (if necessary).

(Choice A) Bed rest does not decrease the incidence of preterm delivery and is associated with maternal venous thromboembolism and bone
density loss; therefore, it is not recommended.

(Choices B and F) Cerclage can prevent preterm delivery in patients with cervical insufficiency (eg, prior painless, second-trimester losses).
Vaginal progesterone is used to prevent preterm delivery in patients with an incidental short cervix (ie, ≤2.5 cm on ultrasound). However, both
interventions are preventive rather than inhibitory for acute preterm labor (ie, neither has tocolytic effects).

(Choices C and D) Cesarean delivery is indicated if this patient undergoes rapid cervical change despite tocolysis and her fetus remains breech;
this avoids vaginal breech delivery, which is associated with increased morbidity and mortality. External cephalic version (ie, fetal rotation from
breech to cephalic presentation) is contraindicated in preterm patients because the risks (eg, fetal demise, placental abruption, emergency
cesarean delivery) outweigh the benefits.

Educational objective:
Patients in preterm labor at <32 weeks gestation (ie, very preterm) require antenatal corticosteroids, penicillin prophylaxis, tocolysis (eg,
indomethacin), and magnesium sulfate.

References
Clinical guidelines for prevention and management of preterm birth: a systematic review.

(http://www.ncbi.nlm.nih.gov/pubmed/29460323)

Obstetrics & Gynecology Pregnancy, Childbirth & Puerperium Preterm labor


Subject System Topic

https://t.me/USMLEWorldStep2CK

REVIEW

https://www.uworld.com/ClientApp/v15/apps/qbanktestinterface/index.html#/launchtest/7281878/nbme/229400739/3/1 1/1
‫ م‬9:23 2021/‫‏‬9/‫‏‬11 https://t.me/USMLEWorldStep2CK UWorld STEP2 SIM Form 1

1 Item 30 of 40
2
Mark
Question Id: 6921 Previous Next
3

4 A 54-year-old obese woman comes to the office with slowly progressive fatigue and lower extremity swelling. She has also gained 5 kg (11 lb)
5 over the last 6 months. The patient has type 2 diabetes mellitus diagnosed 20 years ago and has been poorly compliant with medications.
6 Medical history is notable for mixed hyperlipidemia, hypertension, and osteoarthritis. She smokes a pack of cigarettes a day and drinks 2 bottles
7 of beer each weekend. The patient's blood pressure is 165/90 mm Hg and pulse is 96/min. Her BMI is 38 kg/m2. Cardiopulmonary examination
8 is normal, although the patient has 2+ bilateral pitting edema at the ankles. Laboratory results are as follows:
9
Hemoglobin A1c 8.5%
10
11 Serum creatinine 1.5 mg/dL
12 Urine albumin-to-creatinine ratio 1200 mg/g
13

14 Which of the following would be most effective in slowing the progression of kidney disease in this patient?
15
16  A. Abstinence from alcohol (0%)
17
 B. Blood pressure control (76%)
18

19 C. Dietary protein restriction (0%)


20 D. Sensitizing peripheral tissues to insulin (13%)
21
E. Smoking cessation (3%)
22

23 F. Stimulating endogenous insulin production (4%)


24
G. Supplementing with omega-3 polyunsaturated fatty acids (0%)
25

26

27

28 Incorrect 76% 03 secs 07/27/2021


29
Correct answer
B
 Answered correctly  Time Spent  Last Updated

30
31

32 Explanation
33
34
Joint National Committee 8 recommendations for treating hypertension
35

36 Initiate Rx Goal blood pressure


37
>150 mm Hg systolic BP or
38 Age >60 <150/90 mm Hg
>90 mm Hg diastolic BP
39
Age <60,
40 >140 mm Hg systolic BP or
chronic kidney disease, <140/90 mm Hg
>90 mm Hg diastolic BP
diabetes

Thiazide diuretic or CCB,


Black alone or in combination
(ACEI/ARB, not first-line)

Thiazide diuretic, ACEI, ARB, or CCB,


Initial treatment choice Other ethnicities
alone or in combination

ACEI or ARB,
All ethnicities with
alone or in combination with
chronic kidney disease
other drug classes

ACEI = ACE inhibitor; ARB = angiotensin II receptor blocker; BP = blood pressure; CCB = calcium channel blocker.

This patient has long-standing poorly controlled diabetes (hemoglobin A1c >7%) with comorbid hypertension.

She has characteristic features of diabetic nephropathy, including peripheral edema, elevated creatinine, and severely increased
albuminuria (urine albumin-to-creatinine ratio >300 mg/g). Chronic hyperglycemia leads to impaired glomerular function due to glycation
of matrix and tissue proteins, increased matrix production, and mesangial cell apoptosis.
Hypertension is common in patients with diabetes and can be exacerbated by activation of the renin-angiotensin system in diabetic
nephropathy. The rising systemic blood pressures can subsequently contribute to the development of chronic kidney disease due to
increased glomerular pressures, especially in light of the impaired autoregulation in patients with long-standing diabetes.

The single most effective intervention for slowing the progression of diabetic nephropathy is tight blood pressure control. Current
recommendations from the American Diabetes Association advise drug therapy to lower blood pressure to <140/90 mm Hg, with additional
lifestyle measures (eg, weight loss, sodium restriction), to bring blood pressure to a target of 120/80 mm Hg. ACE inhibitors (eg, lisinopril) and
angiotensin receptor blockers (eg, losartan) are considered first-line medications.

(Choice A) Heavy alcohol use (>4 drinks/day) can raise blood pressure and contribute to nephropathy. However, this patient's light intake is
unlikely to affect blood pressure or renal function.

(Choice C) Very high protein intake (eg, >1 g/kg/day) may contribute to nephropathy. Otherwise, protein restriction is not advised as it would add
needless complexity to the patient's diet and increase risk for malnutrition.

(Choices D and F) Sulfonylureas (eg, glipizide) increase endogenous insulin production, whereas thiazolidinediones (eg, pioglitazone) increase
the insulin sensitivity of peripheral tissues. Tight glycemic control has been found to reduce the risk of diabetic nephropathy, but would not be as
effective as blood pressure control in preventing the progression of nephropathy.

(Choice E) Smoking cessation has been found to decrease the progression of diabetic nephropathy but has a smaller incremental benefit than
blood pressure control.

(Choice G) Omega-3 polyunsaturated fatty acids may help to improve this patient's hyperlipidemia, but they have not been shown to have a
beneficial effect on diabetic nephropathy.

Educational objective:
The most effective intervention for slowing the progression of diabetic nephropathy is tight blood pressure control. Medications are indicated to
lower blood pressure to <140/90 mm Hg, with additional lifestyle measures (eg, weight loss, sodium restriction), to bring blood pressure to a target
of 120/80 mm Hg. ACE inhibitors and angiotensin receptor blockers are the preferred medications.

References
Diabetic nephropathy: landmark clinical trials and tribulations.

(http://www.ncbi.nlm.nih.gov/pubmed/25637638)

Medicine Renal, Urinary Systems & Electrolytes Diabetic nephropathy


Subject System Topic

https://t.me/USMLEWorldStep2CK

REVIEW

https://www.uworld.com/ClientApp/v15/apps/qbanktestinterface/index.html#/launchtest/7281878/nbme/229400739/3/1 1/1
‫ م‬9:24 2021/‫‏‬9/‫‏‬11 https://t.me/USMLEWorldStep2CK UWorld STEP2 SIM Form 1

1 Item 31 of 40
2
Mark
Question Id: 6938 Previous Next
3

4 A 43-year-old man comes to the office due to an intermittent tingling sensation in his left fourth and fifth fingers and medial forearm. He has had
5 worsening symptoms over the last 6 weeks and has begun to experience weakness in his hand while lifting weights at a health club. The
6 symptoms are worse when he talks on his cellular phone during the day but they usually improve overnight. The patient works for a distribution
7 company and spends most of his day driving to visit clients in the area. His medical history is unremarkable. On examination, there is decreased
8 light touch sensation over the hypothenar eminence and medial dorsum of the hand. Which of the following is the most likely site of nerve
9 compression in this patient?
10
A. Cervical spine (2%)
11
12  B. Elbow (85%)
13
C. Forearm (2%)
14

15 D. Shoulder (1%)
16
 E. Wrist (8%)
17
18

19
20
Incorrect 85%
21
Correct answer
B
 Answered correctly  02 secs
Time Spent  04/22/2021
Last Updated

22

23

24 Explanation
25

26

27

28

29
30
31

32

33
34

35 This patient's paresthesias in the medial aspect of the forearm and fourth and fifth fingers appear to be in the distribution of the ulnar nerve and
36
are likely due to ulnar neuropathy at the elbow (UNE). Motor symptoms are less common than sensory symptoms in UNE, although patients
may have weakened grip due to motor involvement of the intrinsic muscles of the hand (and possibly flexor digitorum profundus).
37

38 UNE can be due to compression of the ulnar nerve at the epicondylar groove, cubital tunnel, or both. Compression in the epicondylar groove
39 often occurs from repeated leaning on the elbow (eg, on car window sill or armrest). This can be compounded by repeated or prolonged flexion
40 of the elbow (eg, cellular telephone use), which causes narrowing of the cubital tunnel and resultant compression of the nerve. Initial
management of UNE primarily entails identifying and avoiding the activities that cause compression. However, some cases require glucocorticoid
injection or surgical decompression.

(Choice A) The fifth digit lies in the C8 nerve root dermatome. Compression of the nerve root can cause paresthesias and pain in the lateral
hand and fifth digit but is much less common than other cervical radiculopathies and would usually also cause neck pain.

(Choices C and D) The ulnar nerve is not typically compressed in the forearm, shoulder region, or upper arm as there are no narrow or
vulnerable areas that it must traverse.

(Choice E) Ulnar nerve compression can occur in the wrist (UNW) as the nerve traverses the ulnar (Guyon) canal, typically due to prolonged
pressure over the heel of the hand (eg, bicycling) or acute hamate injury. However, UNW is unlikely to cause paresthesias along the medial
forearm and would be less likely than UNE to cause paresthesias in both the palmar and dorsal cutaneous nerves.

Educational objective:
The ulnar nerve can be compressed at the elbow, either in the epicondylar groove (due to direct pressure) or the cubital tunnel (due to repetitive or
prolonged flexion of the elbow). Typical symptoms include paresthesias in the fourth and fifth fingers and the medial forearm as well as weakness
of the intrinsic hand muscles.

References
Prolonged phone-call posture causes changes of ulnar motor nerve conduction across elbow.

(http://www.ncbi.nlm.nih.gov/pubmed/27417044)

Medicine Rheumatology/Orthopedics & Sports Ulnar nerve neuropathy


Subject System Topic

https://t.me/USMLEWorldStep2CK

REVIEW

https://www.uworld.com/ClientApp/v15/apps/qbanktestinterface/index.html#/launchtest/7281878/nbme/229400739/3/1 1/1
‫ م‬9:24 2021/‫‏‬9/‫‏‬11 https://t.me/USMLEWorldStep2CK UWorld STEP2 SIM Form 1

1 Item 32 of 40
2
Mark
Question Id: 6940 Previous Next
3

4 A 13-month-old boy is brought to the physician for evaluation of a painless, non-itchy rash that began this morning. The boy had a fever over the
5 past 3 days, but it resolved on its own today. He has not been exposed to new foods, clothing, lotions, or body washes. He has been more
6 "fussy" than usual but has maintained a good appetite and has no other symptoms. He takes no medications and his vaccinations are up to date.
7 Vital signs are normal. Examination shows a well-appearing infant with blanching, pink macules on his neck, back, abdomen, and chest. No
8 vesicles are present. The rest of his examination is normal. Which of the following organisms is the most likely cause of this patient's condition?
9
A. Coxsackie virus A (7%)
10
11  B. Human herpesvirus 6 (61%)
12
C. Measles virus (4%)
13

14 D. Parvovirus B19 infection (10%)


15
E. Rubella virus (12%)
16

17
F. Staphylococcus aureus (0%)

18 G. Streptococcus pyogenes (4%)



19
20

21
Incorrect 61%
22

23
Correct answer
B
 Answered correctly  03 secs
Time Spent  03/19/2021
Last Updated

24
25

26
Explanation

27

28
Roseola infantum
29
Microbiology HHV-6 most common
30
31 Epidemiology Age <2 years
32
Clinical 3-5 days of high fever followed by
33
features blanching maculopapular rash
34

35 Treatment Supportive care


36
HHV-6 = human herpesvirus 6.
37

38 This infant's 3-day history of fever followed by a blanching, erythematous macular rash is consistent with roseola infantum (also known as

39
exanthem subitum or sixth disease), which is generally caused by a human herpesvirus 6 (HHV-6) infection.

40 Affected infants typically have a very high fever that is often >40 C (104 F) and lasts 3–5 days (this period is a potential risk factor for developing
febrile seizures). The skin eruption associated with this condition classically occurs as the fever subsides; it is characterized by rose-colored
macules and papules that begin on the neck and trunk and spread to the face and extremities. Additional nonspecific manifestations include
fussiness, rhinorrhea, and palatal or pharyngeal erythema.

(Choice A) Coxsackie virus commonly causes hand-foot-mouth disease and herpangina, but the fever usually occurs with the exanthem or
enanthem.

(Choices C and E) Measles and rubella are vaccine-preventable illnesses, which are characterized by erythematous macules and papules that
start on the face and spread down to the the body. In both conditions, the rash and fever are concurrent. Measles presents as a darker "brick-red"
colored rash while the rubella rash is pink. However, these infections are unlikely as this patient is vaccinated and the fever resolved prior to rash
onset.

(Choice D) The rash caused by parvovirus B19 infection ("fifth disease") is most prominent on the cheeks (not seen in this patient).

(Choice F) Staphylococcus aureus is a bacterium that commonly causes skin and soft tissue infections, such as purulent cellulitis (focal area of
dermal inflammation/tenderness/warmth) and impetigo (nonbullous or bullous).

(Choice G) Streptococcus pyogenes is a common cause of bacterial skin and soft tissue infections; these include nonpurulent cellulitis, impetigo
(nonbullous), scarlet fever (confluent erythematous rash preceded by streptococcal pharyngitis), and erysipelas (raised, demarcated dermal
inflammation/tenderness/warmth).

Educational objective:
Roseola infantum is a viral infection caused by human herpesvirus 6. It is characterized by 3–5 days of a very high fever followed by a rose-
colored maculopapular exanthem.

References
Roseola infantum and its causal human herpesviruses.

(http://www.ncbi.nlm.nih.gov/pubmed/24673253)

Pediatrics Rheumatology/Orthopedics & Sports Human herpesvirus 6


Subject System Topic

https://t.me/USMLEWorldStep2CK

REVIEW

https://www.uworld.com/ClientApp/v15/apps/qbanktestinterface/index.html#/launchtest/7281878/nbme/229400739/3/1 1/1
‫ م‬9:24 2021/‫‏‬9/‫‏‬11 https://t.me/USMLEWorldStep2CK UWorld STEP2 SIM Form 1

1 Item 33 of 40
2
Mark
Question Id: 6946 Previous Next
3

4 A 42-year-old woman comes to the office due to difficulty swallowing. She states that food is "sticking" in her chest, and she has lost 5 kg (11 lbs)
5 over the last 2 months. She has no known medical problems. Her mother has breast cancer and her aunt died of ovarian cancer. Upon further
6 questioning, she says that she has extreme sensitivity to cold. She always wears leather gloves because even mild cold temperature can provoke
7 severe burning and paleness of her fingers. The latter finding can be treated with which of the following?
8

9
A. Metoprolol (3%)

10 B. Gabapentin (4%)
11
C. Tamoxifen (0%)
12
13  D. Nifedipine (85%)
14
 E. Prednisone (6%)
15
16

17
18 Incorrect 85%
19
Correct answer  Answered correctly  02 secs
Time Spent  06/08/2021
Last Updated
D
20

21

22 Explanation
23

24 This patient most likely has scleroderma. Her description of difficulty swallowing with food "sticking" in her throat is typical of patients with
25
esophageal dysmotility. The symptoms she describes in her fingers are consistent with Raynaud phenomenon, which is a cold-induced

26
vasospasm that leads to paleness of the fingers with even minimal cold exposure. Raynaud phenomenon may occur independently, but it is also
associated with connective tissue disorders such as scleroderma and lupus.
27

28 First-line therapy is aimed at avoiding situations in which the fingers become cold. If this is unsuccessful, pharmacologic treatment may be
29 necessary. The most commonly used medications are calcium channel blockers such as nifedipine, which promote vasodilatation and help to
30 counteract the vasospasm seen with Raynaud phenomenon.
31

32 (Choice A) Metoprolol is a beta-1 antagonist that can worsen Raynaud phenomenon due to unopposed alpha-activity in the peripheral
33 vasculature, which may promote vasoconstriction.
34

35
(Choice B) Gabapentin is used for neuropathic pain, not Raynaud phenomenon.

36
(Choice C) Tamoxifen is used in the treatment of breast cancer, not Raynaud phenomenon.
37

38
(Choice E) Prednisone may be beneficial in treating some manifestations of the patient’s scleroderma, but it will not directly improve her Raynaud
39
phenomenon.
40

Educational objective:
Raynaud phenomenon is a cold-induced vasospasm typically seen in the fingers, and is more common in patients with connective tissue disorders
such as scleroderma. Treatment consists of calcium channel blockers such as nifedipine.

Medicine Rheumatology/Orthopedics & Sports Raynaud phenomenon


Subject System Topic

https://t.me/USMLEWorldStep2CK

REVIEW

https://www.uworld.com/ClientApp/v15/apps/qbanktestinterface/index.html#/launchtest/7281878/nbme/229400739/3/1 1/1
‫ م‬9:24 2021/‫‏‬9/‫‏‬11 https://t.me/USMLEWorldStep2CK UWorld STEP2 SIM Form 1

1 Item 34 of 40
2
Mark
Question Id: 6982 Previous Next
3

4 A 24-year-old man is brought to the emergency department due to recurrent, generalized tonic-clonic seizures. The patient has a history of
5 seizure disorder and stopped taking his antiepileptic medication 2 months ago. Over the weekend, he consumed alcohol during a party and
6 subsequently experienced 3 episodes of generalized tonic-clonic seizure at home. The patient had 2 more seizures in the ambulance that were
7 not controlled by intravenous lorazepam. In the hospital, he has 1 more seizure before receiving intravenous fosphenytoin. On examination, the
8 patient is afebrile; blood pressure is 120/70 mm Hg, pulse is 90/min, and respirations are 16/min. He is confused and somnolent.
9 Cardiopulmonary examination is normal. The abdomen is soft and nontender. A Foley catheter is placed, which drains 100 mL of dark red urine.
10 The urinalysis and microscopy will most likely reveal which of the following?
11
12 Blood RBC/hpf WBC/hpf Casts
13
 A. 1+ 10-20 10-50 muddy brown (16%)
14

15  B. 2+ 0-5 1-2 pigmented (59%)


16 C. 2+ 10-20 1-2 red blood cell (10%)
17
D. none 0-5 1-2 none (13%)
18

19 E. none 0-5 10-20 white blood cell (0%)


20

21

22
Incorrect 59% 02 secs 08/27/2021
23 Correct answer
B
 Answered correctly  Time Spent  Last Updated
24
25

26
Explanation
27

28

29 Rhabdomyolysis
30
Skeletal muscle lysis/necrosis due to:
31
Crush injury or prolonged immobilization
32 Etiology
Intense muscle activity (eg, seizure, exertion)
33
Drug/medication toxicity (eg, statins)
34

35 Muscle pain & weakness


36 Dark urine (myoglobinuria/pigmenturia)

37
Clinical features + Blood on urinalysis & no RBCs on microscopy
↑ Serum K & PO4, ↓ serum Ca, ↑ AST > ALT
38
Acute kidney injury
39

40 Serum creatine kinase >1,000 U/L


Diagnosis
Consistent clinical features

Aggressive intravenous fluid resuscitation


Management
Sodium bicarbonate in some cases

ALT = alanine aminotransferase; AST = aspartate aminotransferase; RBCs = red blood cells.

This patient with dark red urine following multiple seizures likely has rhabdomyolysis. The intense and prolonged skeletal muscle contraction
that occurs with tonic-clonic seizures, particularly when combined with heavy consumption of alcohol (which has direct toxic effects on muscles),
can lead to skeletal muscle lysis with release of myoglobin into the bloodstream. Myoglobin, an O2-storage protein similar in structure to
hemoglobin, contains heme pigment, which is the major driver of the renal manifestations of rhabdomyolysis.

After being filtered by the renal glomeruli, myoglobin releases heme into the renal tubules where the pigment can exert direct toxicity,
leading to acute kidney injury (AKI) and eventual acute tubular necrosis (ATN).
The heme pigment (eg, pigmented cast) is also responsible for the dark red urine that appears grossly similar to hematuria; however, true
hematuria does not occur as no red blood cells (RBCs) cross the intact glomerular membrane.

Because the rapid urinalysis blood test detects heme rather than actual RBCs, this leads to the classic finding seen with rhabdomyolysis of
positive blood on urinalysis but no RBCs on urine microscopy. Hemolysis with transport of large amounts of hemoglobin to the kidneys can
also cause heme pigmenturia with the same urine study findings.

(Choice A) Urine studies in ATN typically show muddy brown casts as well as evidence of true hematuria (blood on urinalysis and RBCs on urine
microscopy) and renal parenchymal inflammation (white blood cells [WBCs] on urine microscopy). This patient may eventually develop ATN but it
is a later finding (eg, typically within days) in rhabdomyolysis.

(Choice C) Glomerulonephritis involves the passage of RBCs through the glomerular membrane and leads to true hematuria, characterized by
blood on urinalysis as well as RBCs and RBC casts on urine microscopy.

(Choice D) These findings represent unremarkable or bland urine testing results that are seen in a normal setting or with causes of AKI that do
not directly affect the renal parenchyma (eg, prerenal etiologies).

(Choice E) Acute interstitial nephritis involves renal parenchymal inflammation and is typically indicated by WBCs and WBC casts on urine
microscopy. Because WBCs do not contain heme, they do not trigger a positive blood result on urinalysis.

Educational objective:
Tonic-clonic seizures can cause rhabdomyolysis, which involves skeletal muscle necrosis with release of heme-containing myoglobin into the
bloodstream. The heme pigment is responsible for the major renal manifestations of rhabdomyolysis, including dark red urine, positive blood on
urinalysis in the absence of red blood cells on urine microscopy, and acute kidney injury.

References
Rhabdomyolysis: an evaluation of 475 hospitalized patients.

(http://www.ncbi.nlm.nih.gov/pubmed/16267412)

Medicine Renal, Urinary Systems & Electrolytes Rhabdomyolysis


Subject System Topic

https://t.me/USMLEWorldStep2CK

REVIEW

https://www.uworld.com/ClientApp/v15/apps/qbanktestinterface/index.html#/launchtest/7281878/nbme/229400739/3/1 1/1
‫ م‬9:24 2021/‫‏‬9/‫‏‬11 https://t.me/USMLEWorldStep2CK UWorld STEP2 SIM Form 1

1 Item 35 of 40
2
Mark
Question Id: 6984 Previous Next
3

4 A 22-year-old man comes to the emergency department due to diffuse swelling. The man has cystic fibrosis, and after being hospitalized for a
5 pulmonary exacerbation a few weeks ago, he noticed mild swelling around both ankles. The swelling has worsened in the last week, and the
6 patient's weight has increased by 5 kg (11 lb). He has also developed some difficulty breathing. Temperature is 37.2 C (99 F), blood pressure is
7 120/76 mm Hg, pulse is 120/min, and respirations are 30/min. Pulse oximetry is 92% on room air. Cardiac examination shows tachycardia.
8 Jugular venous distension is present. Lung sounds are coarse bilaterally. The abdomen has ascites but no tenderness to palpation. The liver is 3
9 cm below the costal margin. There is 3+ pretibial pitting edema. The results of liver function studies are as follows:
10
Albumin 3.3 g/dL
11
12 Total bilirubin 0.8 mg/dL
13 Aspartate aminotransferase (SGOT) 63 U/L
14
Alanine aminotransferase (SGPT) 74 U/L
15
16 Which of the following is the most likely cause for this patient's edema?
17
18  A. Decreased protein absorption from the intestine (15%)
19
B. Increased left ventricular end-diastolic pressure (13%)
20

21  C. Increased pulmonary artery resistance (54%)

22 D. Increased urinary albumin excretion (8%)


23
E. Portal vein compression by a fibrotic, nodular liver (6%)
24
25

26

27 Incorrect 54%
28
Correct answer  Answered correctly  02 secs
Time Spent  08/17/2021
Last Updated
C
29
30
31 Explanation
32

33
34
Cor pulmonale
35 Right-sided heart failure due to primary pulmonary disorder
Pathophysiology
36 Absence of left-sided heart disease
37
Parenchymal disease/chronic hypoxia
38
COPD (most common)
39 Common Cystic fibrosis
40 etiologies Interstitial lung disease
Obstructive sleep apnea
Pulmonary vascular disease: pulmonary embolism

Dyspnea & fatigue on exertion


Symptoms Exertional angina (due to ↓ cardiac demand)
Exertional syncope (due to ↑ cardiac output)

Jugular venous distension


Peripheral edema
Examination
Palpable RV heave, loud P2, tricuspid regurgitation murmur
Hepatomegaly with pulsatile liver

Electrocardiography
Incomplete/complete right bundle branch block
Right axis deviation, RV hypertrophy, RA enlargement
Diagnostic
Echocardiography
evaluation
Pulmonary hypertension, RV dilation/dysfunction, tricuspid regurgitation
Catheterization (gold standard)
Elevated filling pressures, decreased cardiac output, pulmonary hypertension

COPD = chronic obstructive pulmonary disease; RA = right atrial; RV = right ventricular.

This patient has jugular venous distension (JVD), hepatomegaly, and peripheral edema, findings concerning for right-sided heart failure (RHF).
For most patients with cystic fibrosis (CF), progressive lung disease is the major cause of morbidity and mortality. Given his history of CF and
associated pulmonary symptoms, this patient's RHF is most likely due to chronic lung disease (eg, cor pulmonale).

Mutations in the CFTR gene cause thick, inspissated secretions that obstruct airways, allowing bacterial colonization and recurrent respiratory
infections. Persistent airway inflammation eventually leads to bronchiectasis (enlarged, scarred airways) and chronic lung disease. Chronic
hypoxemia from mucus plugging and bronchiectasis causes diffuse vasoconstriction of the pulmonary arteries, resulting in increased
pulmonary vascular resistance (ie, pulmonary hypertension). Over time, pulmonary hypertension can lead to right ventricular hypertrophy
and dilation and cor pulmonale.

In addition to shortness of breath, peripheral edema, JVD, and tachycardia, other manifestations of RHF include a loud P2 (indicative of pulmonary
hypertension), a right-sided heave, or a tricuspid regurgitation murmur, as well as hepatic congestion, which can lead to ascites (causing rapid
weight gain), hepatomegaly, and elevated aminotransferases, as seen in this patient. Hypoalbuminemia can also occur but is usually due to
underlying malnutrition.

(Choice A) Decreased protein absorption from the intestine can occur in patients with CF due to pancreatic enzyme deficiencies. However,
significant hypoalbuminemia (<3 g/dL) would be expected, and JVD would not be present.

(Choice B) Increased left ventricular end-diastolic pressure can lead to pulmonary edema and subsequent RHF but is typically associated with
coronary artery disease and chronic hypertension, which are not seen in this patient. This patient's underlying CF makes a primary pulmonary
pathology a more likely cause of RHF.

(Choice D) Increased urinary albumin excretion is seen in nephrotic syndrome, which presents with edema, proteinuria, and significant
hypoalbuminemia (<3 g/dL). JVD does not occur.

(Choice E) Portal vein compression by a fibrotic, nodular liver can occur with CF-related liver disease and cause hepatomegaly, ascites, and
elevated transaminases, but it would not explain this patient's JVD.

Educational objective:
Patients with cystic fibrosis are at risk for cor pulmonale because chronic hypoxemia from bronchiectasis and mucus plugging causes
vasoconstriction and increased resistance of the pulmonary vasculature. Over time, pulmonary hypertension can lead to right-sided heart failure.

References
Prognostic significance of pulmonary hypertension in patients with cystic fibrosis: a systematic review and meta-analysis.

(http://www.ncbi.nlm.nih.gov/pubmed/29443734)
Pulmonary hypertension survival effects and treatment options in cystic fibrosis.

(http://www.ncbi.nlm.nih.gov/pubmed/24048083)

Pediatrics Pulmonary & Critical Care Cystic fibrosis


Subject System Topic

https://t.me/USMLEWorldStep2CK

REVIEW

https://www.uworld.com/ClientApp/v15/apps/qbanktestinterface/index.html#/launchtest/7281878/nbme/229400739/3/1 1/1
‫ م‬9:24 2021/‫‏‬9/‫‏‬11 https://t.me/USMLEWorldStep2CK UWorld STEP2 SIM Form 1

1 Item 36 of 40
2
Mark
Question Id: 6989 Previous Next
3

4 A 31-year-old woman, gravida 2 para 2, comes to the clinic for a routine examination and contraception counseling. Her medical history is
5 significant for renal calculi and pyelonephritis. She is moderately overweight and has recently started a weight loss program involving regular
6 exercise and a predominantly vegetarian diet. She does not use tobacco, alcohol, or illicit drugs. Her mother died of ovarian cancer at age 64 and
7 her sister had cervical cancer at age 37. Her father was obese and died of a stroke at age 54. Vital signs and physical examination are normal.
8 Her BMI is 33 kg/m2. The patient asks about the risks and benefits of various contraception methods. Regular use of combined oral
9 contraceptives decreases the risk of which of the following?
10
A. Breast cancer (13%)
11
12 B. Cerebral infarction (0%)
13
C. Cervical cancer (2%)
14

15 D. Hepatic adenoma (0%)


16
E. Myocardial infarction (2%)
17
18  F. Ovarian cancer (79%)

19 G. Venous thrombosis (0%)



20

21

22
Incorrect 79%
23

24
Correct answer  Answered correctly  02 secs
Time Spent  07/25/2021
Last Updated
F
25

26
Explanation
27

28

29 Estrogen-progestin contraceptives: Benefits & risks


30
Pregnancy prevention
31
Endometrial & ovarian cancer risk reduction
32 Benefits
Menstrual regulation (eg, anovulation, dysmenorrhea, anemia)
33
Hyperandrogenism treatment (eg, hirsutism, acne)
34

35 Venous thromboembolism
36 Hypertension
37 Risks Hepatic adenoma
38
Stroke, myocardial infarction (both very rare)
Cervical cancer
39

40 Combination estrogen-progestin oral contraceptive pills (OCPs) are an effective form of contraception that have the added benefit of regulating
menstrual cycles, decreasing symptoms of dysmenorrhea and hypermenorrhea, and being the first-line treatment for many common gynecologic
disorders.

OCP use has also been proven to reduce the risk of ovarian and endometrial cancer. Ovarian cancer risk reduction is due to chronic ovulation
suppression and directly correlates with the number of years of OCP use. Endometrial cancer is caused by continuous endometrial proliferation
from unopposed estrogen exposure (eg, anovulation, obesity, tamoxifen). OCPs decrease endometrial cancer risk because the progestin
component suppresses endometrial proliferation. This patient would benefit from OCP use as she is at increased risk of ovarian cancer due to
family history and endometrial cancer due to increased estrogen exposure from obesity.

(Choices A and C) Long-term use of combination OCPs is associated with a slight increase in the risk for cervical and breast cancer. The
increased risk returns to normal within 10 years of OCP discontinuance.

(Choices B, E, and G) Women age >35 who use tobacco and OCPs are at increased risk for venous thromboembolism, myocardial infarction,
and cerebral infarction. This is due to estrogen in OCPs, which has hypercoagulable properties. The thromboembolic risks are increased in OCP
users with hypertension.

(Choice D) Long-term OCP users have an increased risk of hepatic adenoma, a benign epithelial tumor. Active liver disease is an absolute
contraindication to OCP use.

Educational objective:
Use of combined oral contraceptive pills is associated with a decreased risk of ovarian and endometrial cancer due to suppression of ovulation
and endometrial proliferation, respectively. Potential risks include hepatic adenoma and venous thrombosis.

References
Oral contraceptive pills as primary prevention for ovarian cancer: a systematic review and meta-analysis.

(http://www.ncbi.nlm.nih.gov/pubmed/23743450)
Oral contraceptive use and risk of breast, cervical, colorectal, and endometrial cancers: a systematic review.

(http://www.ncbi.nlm.nih.gov/pubmed/24014598)

Obstetrics & Gynecology Female Reproductive System & Breast Contraception


Subject System Topic

https://t.me/USMLEWorldStep2CK

REVIEW

https://www.uworld.com/ClientApp/v15/apps/qbanktestinterface/index.html#/launchtest/7281878/nbme/229400739/3/1 1/1
‫ م‬9:25 2021/‫‏‬9/‫‏‬11 https://t.me/USMLEWorldStep2CK UWorld STEP2 SIM Form 1

1 Item 37 of 40
2
Mark
Question Id: 6904 Previous Next
3

5
The following vignette applies to the next 2 items. The items in the set must be answered in sequential order. Once you click Proceed to Next
Item, you will not be able to add or change an answer.
6

7 A 59-year-old man comes to the emergency department due to 2 episodes of syncope over a 12-hour period. The first episode occurred when he
8 was having his morning breakfast, and the second episode happened while he was watching evening television. He describes the episodes as
9 sudden blackouts lasting 1-2 minutes without any confusion afterwards. The patient has been feeling well otherwise, with the exception of mild
10 generalized weakness and a few recent bouts of diarrhea that he attributes to food poisoning. His past medical history is significant for coronary
11 artery disease and paroxysmal atrial fibrillation. He underwent right coronary artery stenting 3 years ago. His medications include simvastatin,
12 aspirin, clopidogrel, and lisinopril. He also recently started sotalol for maintenance of sinus rhythm. In the emergency department, his blood
13 pressure is 130/90 mm Hg and his pulse is 60/min with a regular rhythm. Chest examination reveals clear lungs and normal heart sounds.
14
Item 1 of 2
15
16
Which of the following most likely explains his current symptoms?

17
 A. Atrial fibrillation (21%)
18

19  B. Ventricular arrhythmias (34%)


20
C. Volume depletion (12%)
21

22
D. Vasovagal response (13%)

23 E. Myocardial ischemia (0%)


24
F. Vertebrobasilar insufficiency (14%)
25

26 G. Partial complex seizures (1%)


27
H. Conversion reaction (0%)
28

29
30
31 Incorrect 34%
32
Correct answer  Answered correctly  05 secs
Time Spent  07/30/2021
Last Updated
B
33
34

35 Explanation
36

37 This patient’s history of sudden-onset syncope without a prodrome is most consistent with arrhythmia. Torsades de pointes, a type of
38
polymorphic ventricular tachycardia, is most likely responsible. Sotalol, used to maintain sinus rhythm, has as a potential side effect prolongation
of the QT interval, which predisposes to torsades de pointes. The history of coronary artery disease, as well as electrolyte disturbances from
39
diarrhea, can also predispose to ventricular arrhythmias. Hypokalemia and hypomagnesemia from diarrhea were likely the "tipping point" that
40
provoked torsades de pointes in this patient.

(Choice A) Although atrial fibrillation can cause dizziness, sudden syncope is actually somewhat rare; in addition, the onset of these episodes
following the initiation of sotalol makes torsades de pointe more likely.

(Choice C) Volume depletion may predispose patients to orthostatic syncope, which typically occurs after rising from supine or sitting to standing.

(Choice D) Vasovagal syncope is classically associated with a prodrome of nausea, flushing, dizziness, and warmth as opposed to the sudden
syncope experienced by this patient.

(Choice E) Myocardial ischemia should be considered given this patient’s history of coronary artery disease, but he is not experiencing any chest
pain, making this diagnosis less likely.

(Choice F) Vertebrobasilar insufficiency is usually associated with neurologic manifestations.

(Choice G) Patients with partial complex seizures usually experience postictal confusion.

(Choice H) This patient has numerous risk factors for cardiac arrhythmia. Attributing his symptoms to conversion disorder without appropriate
medical work-up would be premature.

Educational objective:
Arrhythmia should be suspected as the cause of a patient’s syncope if there is a history of rapid-onset loss of consciousness without a preceding
prodrome. Usage of antiarrhythmic drugs, structural heart disease, and hypokalemia and/or hypomagnesemia are all potential predisposing
factors for torsades de pointes.

Medicine Cardiovascular System Ventricular tachycardia


Subject System Topic

https://t.me/USMLEWorldStep2CK

REVIEW

https://www.uworld.com/ClientApp/v15/apps/qbanktestinterface/index.html#/launchtest/7281878/nbme/229400739/3/1 1/1
‫ م‬9:25 2021/‫‏‬9/‫‏‬11 https://t.me/USMLEWorldStep2CK UWorld STEP2 SIM Form 1

1 Item 38 of 40
2
Mark
Question Id: 6947 Previous Next
3

4 Item 2 of 2
5
An ECG done in the emergency department shows normal sinus rhythm, PR interval prolongation to 0.21 sec, QT interval prolongation to 0.56
6
sec, and no significant ST segment or T wave changes. Laboratory results are as follows:
7
8 Sodium 133 mEq/L
9
Potassium 3.1 mEq/L
10
11
Blood urea nitrogen 32 mg/dL

12 Creatinine 1.3 mg/dL


13

14 Which of the following is the best initial treatment for this patient?
15
16  A. Amiodarone (19%)
17
B. Procainamide (10%)
18

19
C. Lidocaine (5%)

20 D. Adenosine (7%)
21
E. Digoxin (6%)
22

23  F. Magnesium sulfate (41%)


24
G. Verapamil (6%)
25

26 H. Propafenone (1%)
27

28

29 Incorrect 41%
30 Correct answer  Answered correctly  03 secs
Time Spent  07/30/2021
Last Updated
F
31

32

33
Explanation
34

35 This patient’s QT interval is prolonged at 0.56 seconds, with the normal corrected QT interval being <0.44 seconds for men and <0.46 for women.
36 The PR interval is also slightly prolonged at 0.21 seconds, with 0.20 seconds being the upper limit of normal. This patient’s laboratory values
37 reveal mild hyponatremia, hypokalemia, and elevated blood urea nitrogen-to-creatinine ratio, as would be expected given his recent diarrheal
38 illness and dehydration.
39
Hypomagnesemia is likely present as well; the magnesium level should always be drawn in patients with suspected electrolyte abnormalities.
40
This patient's ECG changes (particularly his prolonged QT) are likely due to his electrolyte abnormalities in combination with sotalol intake; he is at
risk of developing torsades de pointe again. The initial treatment of torsades de pointes is not antiarrhythmic medications, but rather intravenous
magnesium sulfate. Magnesium is also effective in preventing episodes of torsades de pointe even if the patient has a normal magnesium
level.

In patients with torsades de pointe, placement of a temporary pacemaker and/or intravenous isoproterenol are the next options if magnesium is
ineffective.

(Choice A) Amiodarone is used in the treatment of ventricular and atrial arrhythmias, but may prolong the QT interval and should be avoided in
this patient.

(Choice B) Procainamide can also prolong the QT interval and should be avoided.

(Choice C) Lidocaine may be used in the treatment of ventricular arrhythmias other than torsades de pointes.

(Choice D) Adenosine may be used in the treatment of supraventricular tachycardia, but this patient is currently in normal sinus rhythm and most
likely has a ventricular tachycardia.

(Choice E) Digoxin may be used for the treatment of heart failure or as a rate control agent in atrial fibrillation.

(Choice G) Verapamil is a calcium channel blocker that is used for rate control in supraventricular tachycardias.

(Choice H) Propafenone can also prolong the QT interval and should be avoided in this patient.

Educational objective:
Certain antiarrhythmic medications, structural heart disease, and hypokalemia and hypomagnesemia can predispose to QT interval prolongation
and possible development of torsades de pointes. The initial treatment of torsades de pointes in patients who are hemodynamically stable is
magnesium sulfate, even if the patient’s magnesium level is normal.

Medicine Cardiovascular System Ventricular tachycardia


Subject System Topic

https://t.me/USMLEWorldStep2CK

REVIEW

https://www.uworld.com/ClientApp/v15/apps/qbanktestinterface/index.html#/launchtest/7281878/nbme/229400739/3/1 1/1
‫ م‬9:25 2021/‫‏‬9/‫‏‬11 https://t.me/USMLEWorldStep2CK UWorld STEP2 SIM Form 1

1 Item 39 of 40
2
Mark
Question Id: 6996 Previous Next
3

5
The response options for the next 2 items are the same. Select one answer for each item in the set. Once you click Proceed to Next Item,
you will not be able to add or change an answer.
6

7 A 67-year-old woman comes to the emergency department due to an episode of vision loss in the right eye. The patient was at a movie theater
8 when her right eye vision suddenly became "dim" for approximately 10 minutes. Her vision has now recovered but she remains very concerned
9 about the episode. The patient has never had similar symptoms in the past but does report occasional headaches. She has had no focal
10 weakness or numbness. Her medical history is significant for coronary artery disease, hypertension, and type 2 diabetes mellitus. Blood pressure
11 is 130/85 mm Hg and pulse is 65/min and regular. There is a right-sided carotid bruit. Neurologic and funduscopic examinations are
12 unremarkable.
13
Item 1 of 2
14

15 Which of the following is the most likely cause of this patient's symptoms?
16
 A. Choroidal inflammation (0%)
17
18 B. Demyelinating disorder (0%)
19
C. Elevated intracranial pressure (0%)
20

21 D. Increased intraocular pressure (7%)


22 E. Large vessel vasculitis (5%)
23
F. Postictal cortical inhibition (0%)
24
25  G. Retinal embolism (80%)
26
H. Retinal hemorrhage (0%)
27

28 I. Vascular spasm (4%)


29
30
31
Incorrect 80%
32 Correct answer  Answered correctly  05 secs
Time Spent  08/10/2021
Last Updated
33 G

34

35
Explanation
36

37

38

39

40

This patient has amaurosis fugax, which is characterized by painless, rapid, and transient (<10 minutes) monocular vision loss. The most
common etiology is retinal ischemia due to atherosclerotic emboli originating from the ipsilateral carotid artery. Vision returns after the
embolus is broken up or displaced and retinal circulation is restored. This patient's right carotid bruit and multiple vascular risk factors (eg,
hypertension, diabetes mellitus, coronary artery disease) support the diagnosis. Funduscopic examination is often normal but may show embolic
plaques and retinal whitening (due to ischemia). Patients with amaurosis fugax and concomitant carotid artery disease have an increased annual
risk of stroke.

(Choice A) Choroidal inflammation is characteristic of posterior uveitis, which typically presents with painless vision loss and floaters. Uveitis is
often associated with systemic inflammatory disorders (eg, inflammatory bowel disease, ankylosing spondylitis) or infection (eg, cytomegalovirus).

(Choice B) Optic neuritis is frequently associated with multiple sclerosis, a demyelinating central nervous system disease that typically affects
women age 15-50. Optic neuritis causes monocular vision loss over several weeks with painful eye movement. Patients often have an afferent
pupillary defect with optic disc swelling on funduscopy.

(Choice C) Elevated intracranial pressure (eg, due to tumor, hemorrhage, infection) typically causes headache, nausea/vomiting, depressed
consciousness, and papilledema. Pseudotumor cerebri (idiopathic intracranial hypertension) presents in obese women of childbearing age with
headache, bilaterally symmetric papilledema, and transient visual obscurations that worsen with Valsalva.

(Choice D) An acute increase in intraocular pressure is characteristic of angle-closure glaucoma, which typically presents with painful monocular
vision loss, headache, and nausea/vomiting. Examination usually shows conjunctival injection with a poorly reactive mid-dilated pupil.

(Choice E) Temporal (giant cell) arteritis is a medium- to large-vessel vasculitis that affects adults age >50 and can present with monocular vision
loss; however, other features usually include unilateral headache, jaw claudication, constitutional symptoms (eg, fever, fatigue), and polymyalgia
rheumatica (proximal muscle weakness/pain).

(Choice F) Postictal vision loss can sometimes result from occipital lobe seizures. Patients usually report a preceding visual aura marked by
flashes of light and color.

(Choice H) Retinal hemorrhage often occurs with poorly controlled hypertension and causes painless, unilateral vision loss. Funduscopic
examination typically shows dot- or flame-shaped hemorrhages and other features of hypertensive retinopathy (eg, arteriovenous nicking, cotton-
wool spots).

(Choice I) Vascular spasm is characteristic of migraine, which typically presents in younger women with unilateral throbbing headache, nausea,
photophobia, and phonophobia. Patients may have a preceding visual aura consisting of fortification spectra or scintillating scotoma.

Educational objective:
Amaurosis fugax is painless, sudden, and transient (<10 minutes) monocular vision loss that most commonly results from retinal artery emboli
originating from an ipsilateral carotid artery atherosclerotic plaque.

References
A 65-year-old man with transient monocular vision loss.

(http://www.ncbi.nlm.nih.gov/pubmed/24847142)
Transient monocular blindness.

(http://www.ncbi.nlm.nih.gov/pubmed/16833032)

Medicine Nervous System Amaurosis fugax


Subject System Topic

https://t.me/USMLEWorldStep2CK

REVIEW

https://www.uworld.com/ClientApp/v15/apps/qbanktestinterface/index.html#/launchtest/7281878/nbme/229400739/3/1 1/1
‫ م‬9:25 2021/‫‏‬9/‫‏‬11 https://t.me/USMLEWorldStep2CK UWorld STEP2 SIM Form 1

1 Item 40 of 40
2
Mark
Question Id: 6981 Previous Next
3

4 A 32-year-old woman comes to the emergency department due to progressive vision impairment in her right eye. She has been having discomfort
5 in her right eye since last night, and her vision became blurry after taking a hot shower this morning. The patient has never experienced similar
6 symptoms in the past. She has occasional headaches but no focal weakness or numbness. She has also had several episodes of dizziness and
7 clumsiness over the past 6 months, which were attributed to benign positional vertigo and treated with meclizine. Blood pressure is 130/85 mm
8 Hg and pulse is 85/min and regular. Visual acuity is reduced in the right eye but funduscopic examination is unremarkable.
9
Item 2 of 2
10
Which of the following is the most likely cause of this patient's symptoms?
11
12
 A. Choroidal inflammation (1%)
13

14  B. Demyelinating disorder (74%)


15 C. Elevated intracranial pressure (4%)
16
D. Increased intraocular pressure (6%)
17
18 E. Large vessel vasculitis (5%)
19
F. Postictal cortical inhibition (0%)
20

21 G. Retinal embolism (1%)


22
H. Retinal hemorrhage (0%)
23

24
I. Vascular spasm (4%)

25

26

27 Incorrect 74%
28 Correct answer  Answered correctly  05 secs
Time Spent  08/10/2021
Last Updated
B
29
30
31
Explanation
32

33 This patient likely has multiple sclerosis, an inflammatory demyelinating disorder of the central nervous system that typically affects women age
34 15-50 and presents with neurologic deficits disseminated in space and time. Her relapsing/remitting episodes of dizziness and clumsiness were
35 the initial manifestations of her disease, likely caused by lesions in the cerebellum. She currently has features of optic neuritis characterized by
36 painful monocular vision loss that evolves over hours to days and improves after several weeks. Heat exposure (eg, hot shower) is known to
37 exacerbate symptoms (Uhthoff phenomenon). On examination, the affected eye typically has reduced visual acuity and an afferent pupillary
38 defect. Funduscopy is often normal but may show optic disc swelling.
39 (Choice A) Choroidal inflammation is characteristic of posterior uveitis, which typically presents with painless vision loss and floaters. Uveitis is
40 often associated with systemic inflammatory disorders (eg, inflammatory bowel disease, ankylosing spondylitis) or infection (eg, cytomegalovirus).

(Choice C) Elevated intracranial pressure (eg, due to tumor, hemorrhage, infection) typically causes headache, nausea/vomiting, depressed
consciousness, and papilledema. Pseudotumor cerebri (idiopathic intracranial hypertension) presents in obese women of childbearing age with
headache, bilaterally symmetric papilledema, and transient visual obscurations that worsen with Valsalva.

(Choice D) An acute increase in intraocular pressure is characteristic of angle-closure glaucoma, which typically affects older adults and presents
with painful monocular vision loss, headache, and nausea/vomiting. Examination usually shows conjunctival injection with a poorly reactive mid-
dilated pupil.

(Choice E) Temporal (giant cell) arteritis is a medium- to large-vessel vasculitis that can present with monocular vision loss; however, it typically
affects adults age >50 and is associated with unilateral headache, jaw claudication, constitutional symptoms (eg, fever, fatigue), and polymyalgia
rheumatica (proximal muscle weakness/pain).

(Choice F) Postictal vision loss can sometimes result from occipital lobe seizures. Patients usually report a preceding visual aura marked by
flashes of light and color.

(Choice G) Retinal ischemia is most often caused by emboli from carotid artery atherosclerosis and may result in retinal whitening on
funduscopy. This young patient has no vascular risk factors and her symptoms are progressive, which argue against an acute vascular ischemic
process.

(Choice H) Retinal hemorrhage frequently occurs in the setting of poorly controlled hypertension and causes painless monocular vision loss.
Funduscopic examination typically shows dot- or flame-shaped hemorrhages and other features or hypertensive retinopathy (eg, arteriovenous
nicking, cotton-wool spots).

(Choice I) Vascular spasm is characteristic of migraine, which typically presents with unilateral throbbing headache, nausea, photophobia, and
phonophobia. Patients may have a preceding visual aura consisting of fortification spectra or scintillating scotoma.

Educational objective:
Multiple sclerosis is an inflammatory demyelinating disorder that typically presents in women age 15-50 with neurologic deficits disseminated in
space and time. Patients frequently develop optic neuritis characterized by transient painful monocular vision loss that improves after several
weeks. Heat exposure may exacerbate symptoms (Uhthoff phenomenon).

References
Optic neuritis: a review.

(http://www.ncbi.nlm.nih.gov/pubmed/19878630)

Medicine Nervous System Multiple sclerosis


Subject System Topic

https://t.me/USMLEWorldStep2CK

REVIEW

https://www.uworld.com/ClientApp/v15/apps/qbanktestinterface/index.html#/launchtest/7281878/nbme/229400739/3/1 1/1

You might also like